ITE Flashcards

1
Q

What type of diabetes medications leads to hypoglycemia?
A. Biguanides
B. DPP-4 Inhibitors
C. SGLT-2 inhibitors
D. Sulfonylureas
E. Thiazolidinediones

A

D. Sulfonylureas like glipizide, glyburide and glimepiride

Biguanides= Metformin causeas diarrhea and lactic acidosis

DDP-4 Inhibitors: Saxagliptin, sitagliptin, linagliptin, alogliptin cause headache URI, elevated liver enzymes

SGLT2 inhibitors: canagliflozin, dapagliflozin, empagliflozin can cause UTI, candidiasis

Thiazolidinediones: Pioglitazone and rosiglitazone, weight gain, salt retention, edema

How well did you know this?
1
Not at all
2
3
4
5
Perfectly
2
Q

Vit D in new born

A

400 IU of vitamin D in infants

If breast feeding or partial formula

Exclusive formula may not need vit d

How well did you know this?
1
Not at all
2
3
4
5
Perfectly
3
Q

Scolosis progression

A

Magnitude of curve at presentation
Potential of future growth
Female sex

Cobb angle of 20

How well did you know this?
1
Not at all
2
3
4
5
Perfectly
4
Q

When titrating the dosage of opioids, the CDC recommends that you should consider prescribing naloxone when the opioid dosage reaches what morphine milligram equivalent (MME) per day. threshold?
A. 30
B. 50
C. 80
D. 90
E. 100

A

B. 50

How well did you know this?
1
Not at all
2
3
4
5
Perfectly
5
Q

A 62 y.o female with stage 3 chronic kidney disease and an estimated glomerular filtration rate of 37 mL/min/ 1.73 m2 is found to have a mildly low ionized calcium level. Which of the followign would you expect to see if her hypocalcemia is secondary to her chronic kidney disease?
A. Elevated PTH and Elevated phosphrous
B. Elevated PTH and low phosphorus
C. Low PTH and elevated phosphorus
D. Low PTH and low phosphorus

A

A. Elevated PTH and elevated phosphorus

In patients with CKD, phosphate is not appropriately excreted and the subsequent hyperphosphatemia leads to secondary hyperparathyroidism and binding of calcium.

Decreased production of calcitriol in patients with CKD also leads to hypocalcemic hyperparathyroidism.

How well did you know this?
1
Not at all
2
3
4
5
Perfectly
6
Q

When to evaluate sacral dimple

A

If small at anal verge without other skin or hair findings dont need evaluation to rule out spinal dysraphism (tethered cord)

> 0.5 cm diameter
within 2.5 cm of anal verge
Needs imaging

How well did you know this?
1
Not at all
2
3
4
5
Perfectly
7
Q

A 38 y.o female with a 6 month history of mild shortness of breath associated with some intermittent wheezing during upper respiratory infection presents for follow up. You previously prescribed albuterol (Proventil, Ventolin) via metered-dose inhaler, which she says helps her symptoms. You suspect asthma. PFT reveals a normal FEV1/ FVC ratio for her age. Which of the following would be the most appropriate next step?

A. Consider an alternative diagnosis
B. Assess her bronchodilator response
C. Perform methacholine challenge
D. Prescribe an inhaled corticosteroid
E. Proceed with treatment for COPD

A

C. Perform a methacholine challenge

Spirometry is central to confirming diagnosis of asthma. Normal FEV1/FVC, which doesnt rule in/out asthma.

A methacholine challenge is recommended in this scenario to assess for the airway hyper responsiveness that is the hallmark of asthma.

Bronchoconstriction (reduction in FEV1) observed at low levels of methacholine administration (<4 mg/mL) is consistent with asthma.

How well did you know this?
1
Not at all
2
3
4
5
Perfectly
8
Q

According to the most recent american college of cardiology/ American heart association guidelines, hypertension is defined as a blood pressure reading greater than

A. 120/80
B. 130/80
C. 135/85
D. 140/90
E. 150/90

A

B. 130/80

How well did you know this?
1
Not at all
2
3
4
5
Perfectly
9
Q

A 26-year-old G2P1001 at 30 weeks gestation was recently diagnosed with gestational diabetes and is ready to start testing her blood glucose at home. Which one of the following is the recommended goal for fasting blood glucose in this patient? (check one)
<75 mg/dL
<95 mg/dL
<120 mg/dL
<150 mg/dL
<180 mg/dL

A

The goal fasting blood glucose level in patients with gestational diabetes is <95 mg/dL. A fasting glucose
level <80 mg/dL is associated with increased maternal and fetal complications. The goal 2-hour
postprandial glucose level is <120 mg/dL and the goal 1-hour postprandial glucose level is <140 mg/dL.

How well did you know this?
1
Not at all
2
3
4
5
Perfectly
10
Q

A 75-year-old male sees you for evaluation of a unilateral resting tremor of his right hand. The tremor resolves if he is touched on the hand by someone. His wife notes that he seems to drag his feet now, but he has no history of falls.

Which one of the following has been shown to delay progression of his disease? (check one)
A. No currently available pharmacologic agents
B. Amantadine
C.Carbidopa/levodopa (Sinemet)
D. Rasagiline (Azilect)
E. Ropinirole

A

No treatment to delay only help symptoms

There are no currently available medications that have been shown to delay progression of Parkinson’s disease. However, guidelines recommend initiating the treatment of motor symptoms when they begin to affect the functions of daily life or decrease the quality of life. The first-line treatment for motor symptoms is carbidopa/levodopa due to its effectiveness for tremors, rigidity, and bradykinesia. It is a myth that delaying the use of levodopa will prevent a lack of efficacy later in the course of the illness, as what appears to be a lack of efficacy actually represents progression of the disease.

Amantadine can be used for patients under 65 years of age who are only experiencing tremors. Monoamine oxidase inhibitors such as rasagiline and non-ergot dopamine agonists such as ropinirole are not as effective as carbidopa/levodopa for motor symptoms, but they do not cause the dyskinesias and motor fluctuations seen with levodopa. Monoamine oxidase inhibitors are considered first-line therapy for patients under age 65 with mild motor symptoms.

How well did you know this?
1
Not at all
2
3
4
5
Perfectly
11
Q

Which one of the following is necessary to make the diagnosis of a functional gastrointestinal disorder? (check one)

A. Symptom-based clinical criteria
B. Noninvasive testing for Helicobacter pylori infection
C. Celiac serology
D. Gastric emptying studies
E. Esophagogastroduodenoscopy

A

In the absence of red-flag symptoms such as nocturnal defecation, weight loss, or gastrointestinal bleeding, functional gastrointestinal disorders can be diagnosed using symptom-based clinical criteria. Symptoms such as recurrent abdominal pain related to defecation, pain related to a change in the frequency of defecation, abdominal bloating and distension, and loose and watery or lumpy and hard stools are used to diagnose functional bowel disorders. Noninvasive testing for Helicobacter pylori, celiac serology, gastric emptying studies, and esophagogastroduodenoscopy are not required in order to make a diagnosis.

How well did you know this?
1
Not at all
2
3
4
5
Perfectly
12
Q

A 67-year-old male diagnosed with polymyalgia rheumatica is started on long-term prednisone therapy. Which one of the following is the recommended first-line agent to prevent steroid-induced osteoporosis? (check one)
Alendronate (Fosamax)
High-dose vitamin D
Raloxifene (Evista)
Teriparatide (Forteo)

A

Alendronate (Fosamax)

Supplementation of calcium (800–1000 mg) and vitamin D (400–800 IU) is also recommended.

How well did you know this?
1
Not at all
2
3
4
5
Perfectly
13
Q

A 54-year-old male develops chest pain while running. He is rushed to the emergency department of a hospital equipped for percutaneous coronary intervention. An EKG shows 3 mm of ST elevation in the anterior leads. He is diaphoretic and cool with ongoing chest pain. His blood pressure is 80/50 mm Hg, his pulse rate is 116 beats/min, and his oxygen saturation is 98% on room air.

You would immediately administer (check one)
A. a β-blocker
B. dual antiplatelet therapy and an anticoagulant
C. intravenous fibrinolytic therapy
D. an intravenous vasopressor

A

B. dual antiplatelet therapy and an anticoagulant

This patient is likely experiencing an acute anterior wall myocardial infarction with possible incipient
cardiogenic shock. Along with initiating the hospital’s protocol for myocardial infarction, immediate
treatment should include dual antiplatelet therapy with a 325-mg dose of nonenteric aspirin, a P2Y12
inhibitor (clopidogrel, prasugrel, or ticagrelor), and an anticoagulant (unfractionated heparin or
bivalirudin). Given the possibility of cardiogenic shock, -blockers should not be used. Unless more than
a 2-hour delay in percutaneous coronary intervention is expected, fibrinolytics should not be administered.
An intravenous vasopressor is not indicated.

How well did you know this?
1
Not at all
2
3
4
5
Perfectly
14
Q

A 28-year-old white female comes to your office at 37 weeks gestation with a 24-hour history of painful vesicles on the vulva. She does not have a past history of similar lesions. You make a presumptive diagnosis of genital herpes.

Of the following, the most sensitive and specific test is (check one)
exfoliative cytology (Tzanck test)
a polymerase chain reaction (PCR) test
an enzyme-linked immunosorbent assay (ELISA)
HSV serology (IgG/IgM)

A

PCR

When genital herpes occurs during pregnancy, the best method of diagnosis is either a tissue culture or a
polymerase chain reaction (PCR) test, which is more sensitive. Enzyme-linked immunosorbent assays are
sensitive, but not as sensitive or specific as PCR.

How well did you know this?
1
Not at all
2
3
4
5
Perfectly
15
Q

A 36-year-old male went skiing last year for the first time and when he made it to the top of the mountain he developed a headache, nausea, and dizziness, but no respiratory difficulty. That night he had difficulty sleeping. He asks for your recommendation on preventing a recurrence of the problem when he goes skiing again this year.

Which one of the following medications would you recommend he start the day before his ascent and continue until his descent is complete? (check one)
Acetazolamide (Diamox Sequels)
Aspirin
Dexamethasone (Decadron)
Tadalafil (Adcirca)
Zolpidem (Ambien)

A

Acetazolamide is the preferred agent for preventing acute mountain sickness (AMS). Multiple trials have
demonstrated its efficacy in preventing AMS. Dexamethasone is a first-line treatment for acute mountain
sickness of any severity but is a second-line drug for prevention because of its side-effect profile. Tadalafil
is advised as a second-line treatment after nifedipine for the prevention and treatment of high-altitude
pulmonary edema. Zolpidem may help with sleep but not AMS, and aspirin is not recommended for
prevention of AMS.

How well did you know this?
1
Not at all
2
3
4
5
Perfectly
16
Q

A 62-year-old male with diabetes mellitus recently underwent angioplasty with placement of a drug-eluting stent for the treatment of left main coronary artery disease and acute coronary syndrome. The patient is not considered at high risk for bleeding and you initiate dual antiplatelet therapy with aspirin and clopidogrel (Plavix).

For how long should this patient continue dual antiplatelet therapy? (check one)
1 month
3 months
6 months
9 months
At least 12 months

A

AT least 12 months

How well did you know this?
1
Not at all
2
3
4
5
Perfectly
17
Q

A 90-year-old male presents to the emergency department with chest pain, dyspnea, and diaphoresis. He has experienced these symptoms intermittently since his wife died last week. An EKG shows ST elevation in the anterior leads, and cardiac enzymes are elevated. An echocardiogram shows apical ballooning of the left ventricle. Cardiac catheterization does not reveal coronary vascular disease. You plan to discharge the patient after observation overnight.
Which one of the following would be the most appropriate management of this patient’s stress-induced (Takotsubo) cardiomyopathy after discharge? (check one)
Home medications only
A cardiac event monitor to detect any rhythm abnormalities
A diuretic, ACE inhibitor, and β-blocker until his symptoms and the abnormalities seen on the echocardiogram resolve
A statin, diuretic, ACE inhibitor, and β-blocker to be continued indefinitely
Pacemaker placement

A

Takotsubo cardiomyopathy, also known as stress-induced cardiomyopathy, can develop following
emotional distress and is characterized by the abrupt onset of dysfunction of the left ventricle. The clinical
presentation and laboratory studies can mirror acute coronary syndrome and should be treated similarly.
Once symptoms and cardiac abnormalities resolve, treatment is no longer indicated and may be withdrawn
if there are no signs of coronary disease. Because this patient currently has cardiomyopathic abnormalities,
a diuretic, ACE inhibitor, and -blocker are indicated. Ambulatory cardiac monitors are not indicated for
this patient with a known diagnosis of Takotsubo cardiomyopathy. A pacemaker is not indicated in the
absence of arrhythmias caused by conduction abnormalities.

How well did you know this?
1
Not at all
2
3
4
5
Perfectly
18
Q

A 55-year-old patient with a history of alcoholism is admitted through the emergency department with acute pancreatitis. Which one of the following tests performed at the time of admission can best predict the severity of pancreatitis? (check one)
Hematocrit
C-reactive protein
Serum amylase
Serum lipase
CT of the abdomen

A

Hematocrit

Knowing the severity of pancreatitis helps predict how aggressive management should be. Hematocrit,
BUN, and creatinine levels are the most useful predictors of the severity of pancreatitis, reflecting the
degree of intravascular volume depletion. C-reactive protein is often elevated, but it is not as useful as
hematocrit for predicting severity. Serum amylase and lipase have no prognostic value. CT evidence of
severe pancreatitis lags behind clinical and laboratory evidence, and early CT underestimates the severity
of the acute process.

How well did you know this?
1
Not at all
2
3
4
5
Perfectly
19
Q

A 30-year-old gravida 3 para 2 sees you for prenatal care at 13 weeks gestation. During her previous pregnancies she became hypertensive and had bilateral leg edema and proteinuria. These conditions resolved after delivery. Her only current medication is a prenatal vitamin.

In order to prevent this condition, which one of the following should be started today? (check one)
No new medications
Aspirin
Fish oil
Magnesium
Vitamin C

A

Preeclampsia take Aspirin

Aspirin, 81 mg daily, is recommended for high-risk pregnant patients to prevent preeclampsia. Prophylaxis should begin after 12 weeks gestation and continue until delivery. Fish oil, magnesium, and vitamin C are not beneficial in the prevention of preeclampsia.

How well did you know this?
1
Not at all
2
3
4
5
Perfectly
20
Q

A 34-year-old male with sickle cell disease has a new onset of mild to moderate thirst and polyuria. He ate a large meal about 2 hours ago.

An examination reveals a BMI of 32 kg/m2. Results of a urinalysis performed by your staff include 3+ glucose and no ketones. His blood glucose level is 288 mg/dL and his hemoglobin A1c is 5.2%.

Which one of the following would be most appropriate at this point to help diagnose and monitor this patient’s glycemic control? (check one)
A serum fructosamine level
A repeat hemoglobin A1c
A 2-hour glucose tolerance test
Hemoglobin electrophoresis
Referral to an endocrinologist

A

This patient with sickle cell disease has a new onset of diabetes mellitus. Hemoglobinopathies falsely lower
hemoglobin A1c as a result of hemolysis and abnormal glycation. Fructosamine correlates well with
hemoglobin A1c levels and is recommended instead of hemoglobin A1c for monitoring glucose control in
patients with diabetes and hemoglobinopathies. A 2-hour glucose tolerance test or hemoglobin
electrophoresis would not provide useful information. Referral to an endocrinologist is not indicated at this
point because the patient has not failed primary care management.

How well did you know this?
1
Not at all
2
3
4
5
Perfectly
21
Q

Premature adrenache

A

Laboratory studies and radiography warrant consideration if the
patient develops secondary sex characteristics before the age of 8, or if her height velocity increases
rapidly during the surveillance period.

How well did you know this?
1
Not at all
2
3
4
5
Perfectly
22
Q

An otherwise asymptomatic 7-year-old male has a blood pressure above the 95th percentile for gender, age, and height on serial measurements. Which one of the following studies would be most appropriate at this time? (check one)
Renin and aldosterone levels
24-hour urinary fractionated metanephrines and normetanephrines
Renal ultrasonography
Doppler ultrasonography of the renal arteries
A sleep study

A

Renal US

Renal parenchymal diseases such as glomerulonephritis, congenital abnormalities, and reflux nephropathy
are the most common cause of hypertension in preadolescent children. Preadolescent children with
hypertension should be evaluated for possible secondary causes and renal ultrasonography should be the
first choice of imaging in this age group.
Renin and aldosterone levels are indicated if there is a reason to suspect primary hyperaldosteronism, such
as unexplained hypokalemia. Measurement of 24-hour urinary fractionated metanephrines and
normetanephrines is used to diagnose pheochromocytomas, which are rare and usually present with a triad
of symptoms including headache, palpitations, and sweating. Doppler ultrasonography of the renal arteries
is useful for diagnosing renal artery stenosis, which should be suspected in patients with coronary or
peripheral atherosclerosis or young adults, especially women 19–39 years of age, who are more at risk for
renal artery stenosis due to fibromuscular dysplasia. Sleep studies are indicated in patients who are obese
or have signs or symptoms of obstructive sleep apnea.

How well did you know this?
1
Not at all
2
3
4
5
Perfectly
23
Q

A 30-year-old female presents with pain over the proximal fifth metatarsal after twisting her ankle. Radiographs reveal a nondisplaced tuberosity avulsion fracture of the fifth metatarsal.

Which one of the following would be the most appropriate initial management? (check one)
A short leg walking boot
A compressive dressing with weight bearing and range-of-motion exercises as tolerated
A posterior splint with no weight bearing, and follow-up in 3–5 days
A short leg cast with no weight bearing
Surgical fixation

A

The fifth metatarsal has the least cortical thickness of all of the metatarsals. There are strong ligaments and
capsular attachments on the proximal fifth metatarsal that can put significant stress on this area of the bone,
leading to fractures. Nondisplaced tuberosity fractures can generally be treated with compressive dressings
such as an Aircast or Ace bandage, with weight bearing and range-of-motion exercises as tolerated.
Minimally displaced (<3 mm) avulsion fractures of the fifth metatarsal tuberosity can be treated with a
short leg walking boot. If the displacement is >3 mm, an orthopedic referral is warranted.

How well did you know this?
1
Not at all
2
3
4
5
Perfectly
24
Q

Montelukast (Singulair) has an FDA boxed warning related to an increased risk of: (check one)
delirium
myocardial infarction
suicidality
venous thromboembolism

A

In March 2020, the FDA upgraded its warning label for montelukast to a boxed warning (black box warning) based on the trends for all neuropsychiatric adverse events, including suicidality, associated with montelukast use reported in the FDA Adverse Event Reporting System database from the date of FDA approval in February 1998 through May 2019 (SOR B). The boxed warning does not indicate an increased risk of delirium, myocardial infarction, or venous thromboembolism

How well did you know this?
1
Not at all
2
3
4
5
Perfectly
25
Q

A 25-year-old female who is 3 months post partum presents with multiple complaints, including
increasing weakness and fatigue, intolerance to warm environments, a weight loss of 30 lb
despite an increased appetite, difficulty sleeping, awareness that her heart is beating faster and
“pounding” in her chest, increasing restlessness and difficulty concentrating, increased
tremulousness, and a significant swelling in her neck. She takes no medication, has experienced
no recent trauma, and has not ingested large amounts of iodine.

When you examine her you find no exophthalmos or lid lag and no pretibial edema, but her skin
is warm, smooth, and moist. You also find a smooth, non-nodular, nontender, enlarged thyroid
gland, clear lungs, a resting tremor, and hyperactive reflexes.

Laboratory testing reveals a low TSH level, elevated free T3 and free T4, and high uptake on a
radioactive iodine uptake scan.

Which one of the following is the most likely diagnosis? (check one)
Postpartum thyroiditis
Silent thyroiditis
Subacute thyroiditis
Graves disease
Exogenous thyroid ingestion

A

This patient has symptoms consistent with hyperthyroidism, which could be caused by any of the options
listed. TSH is suppressed and free T4 and free T3 are elevated in all of these conditions.

**Only Graves
disease, however, will cause high radioactive iodine uptake on a thyroid scan. **

Uptake will be low in the
other conditions.

How well did you know this?
1
Not at all
2
3
4
5
Perfectly
26
Q

Which one of the following medications for the treatment of type 2 diabetes has been associated with ketoacidosis? (check one)
Dapagliflozin (Farxiga)
Liraglutide (Victoza)
Metformin
Pioglitazone (Actos)
Sitagliptin (Januvia)

A

Dapagliflozin (Farxiga)

SGLT2 inhibitors such as dapagliflozin have increasingly been shown to be associated with diabetic ketoacidosis under certain circumstances. Liraglutide, metformin, pioglitazone, and sitagliptin are not associated with diabetic ketoacidosis.

How well did you know this?
1
Not at all
2
3
4
5
Perfectly
27
Q

Three weeks after he had knee surgery, a 64-year-old male presents for follow-up of an emergency department visit for a pulmonary embolism. He has no previous history of pulmonary embolism and is otherwise in good health. He is being treated with apixaban (Eliquis).

The recommended duration of anticoagulation therapy for this patient is
(check one)
1 month
3 months
6 months
9 months
12 months

A

3 months

Patients who have a venous thromboembolism (VTE) require anticoagulation therapy for treatment and
prevention of recurrence. The risk of recurrence is greatest in the first year after the event and remains
elevated indefinitely. The risk for VTE recurrence is dependent on patient factors, such as active cancers
and thrombophilia. Current guidelines recommend treatment for at least 3 months. In patients who have
a reversible provoking factor such as surgery, anticoagulation beyond 3 months is not recommended.
Ref: Wilbur J, Shian B: Deep venous thrombosis and pulmonary embolism: Current therapy. Am Fam Physician
2017;95(5):295-302.

How well did you know this?
1
Not at all
2
3
4
5
Perfectly
28
Q

A 30-year-old gravida 2 para 1 in her second trimester is evaluated for hypothyroidism. The normal TSH range in pregnancy is (check one)
lower than in the nonpregnant state
higher than in the nonpregnant state
the same as in the nonpregnant state
not useful for evaluating hypothyroidism after the first trimester

A

Lower

The TSH reference range is lower during pregnancy because of the cross-reactivity of the -subunit of
hCG. Levels of hCG peak during weeks 7–13 of pregnancy, and hCG has mild TSH-like activity, leading
to slightly high free T4 levels in early pregnancy. This leads to a feedback decrease in TSH.

How well did you know this?
1
Not at all
2
3
4
5
Perfectly
29
Q

Ottawa Knee rules

A

Several decision support tools can help guide the decision to order imaging of an injured knee, such as the
Ottawa Knee Rule, the Pittsburgh Knee Rule, and American College of Radiology (ACR) criteria.

The inability to take four or more steps immediately after an injury or in the emergency setting is an indication
for radiography in all three rules.
Age is an indication for radiography in acute knee pain in patients over 55 years of age according to the
Ottawa rule, or under 12 or over 50 years of age according to the Pittsburgh rule. The patient’s sex does
not factor into the criteria for imaging.
Bony tenderness is an indication for imaging according to the ACR and Ottawa rules, but only if isolated
over the proximal fibula or over the patella without other bony tenderness. The inability to flex the knee
to 90° is also an indication for imaging according to the ACR and Ottawa rules.

How well did you know this?
1
Not at all
2
3
4
5
Perfectly
30
Q

A 32-year-old female presents with heat intolerance, excessive weight loss, and anxiety. She gave birth 6 months ago and recently stopped breastfeeding. On examination her thyroid gland is slightly diffusely enlarged and nontender. Laboratory studies reveal a decreased TSH level and elevated free T3 and T4 levels. You suspect that she has postpartum thyroiditis.

Which one of the following tests would be most useful to confirm the diagnosis? (check one)
Radioactive iodine uptake
Thyroid peroxidase antibody levels
Thyroid ultrasonography
Thyrotropin receptor antibody levels

A

Radioactive iodine uptake

Postpartum thyroiditis is defined as a transient or persistent thyroid dysfunction that occurs within 1 year
of childbirth, miscarriage, or medical abortion. Release of preformed thyroid hormone in the bloodstream
initially results in hyperthyroidism. During the hyperthyroid phase, radioactive iodine uptake will be low,
which can help to confirm the diagnosis. Pregnancy and breastfeeding are contraindications to radionuclide
imaging. Thyroid peroxidase antibody levels are elevated with chronic autoimmune thyroiditis
(Hashimoto’s thyroiditis), and patients present with symptoms of hypothyroidism. The Endocrine Society and American Association of Clinical Endocrinologists do not recommend routine thyroid ultrasonography
in patients with abnormal thyroid function tests if there is no palpable abnormality of the thyroid gland.
Thyrotropin receptor antibody levels are positive in Graves disease.

How well did you know this?
1
Not at all
2
3
4
5
Perfectly
31
Q

A 15-year-old male presents with a 2-day history of dark-colored urine, lower extremity edema, and fatigue. Approximately 2 weeks ago he said he had a “bad sore throat” that was treated empirically with amoxicillin. On examination his blood pressure is 144/92 mm Hg, his pulse rate is 76 beats/min, and his other vital signs are normal. Other than mild dependent edema there are no additional significant physical examination findings. A urinalysis dipstick shows 3+ hematuria.

Which one of the following findings on microscopic evaluation of the urine sediment would help to confirm the diagnosis in this patient? (check one)
Gram-positive cocci in chains
RBC casts
WBC casts
Eosinophils
Oxalate crystals

A

This is a classic presentation for acute poststreptococcal glomerulonephritis (APSGN), with the onset of
gross hematuria associated with hypertension and systemic edema. This is most commonly seen in
school-age children, usually 1–2 weeks after an episode of pharyngitis or 3–4 weeks after an episode of
impetigo, caused by so-called nephritogenic strains of Group A -hemolytic Streptococcus. The hematuria
is caused by immune complex–mediated glomerular injury.

Bacteriuria may be seen in both upper and lower urinary tract infections, but may also be a spurious
finding, especially with the combined presence of epithelial cells. The classic finding on microscopic
urinalysis for acute glomerulonephritis is the presence of RBC casts. WBC casts are seen with acute
pyelonephritis. The presence of urinary eosinophils indicates acute interstitial nephritis. Calcium oxalate
makes up the most common type of kidney stones.
Antibiotics prescribed for antecedent pharyngitis do not prevent APSGN. Treatment is supportive,
controlling blood pressure and edema with a thiazide or a loop diuretic. The prognosis for resolution and
full recovery of the vast majority of patients with APSGN is excellent, especially in the pediatric age
group.

How well did you know this?
1
Not at all
2
3
4
5
Perfectly
32
Q

A patient comes to your outpatient clinic with a persistent migraine that she has been unable to treat effectively at home. The symptoms began several hours ago and are typical for her. She has already tried her usual treatments of ibuprofen, 800 mg, and rizatriptan (Maxalt), 10 mg, but they have not provided any relief. She took a second dose of rizatriptan 2 hours later without benefit. She is in significant pain, which is causing mild nausea, and she has photophobia and phonophobia.

Which one of the following would be most appropriate at this point? (check one)
Oral butalbital/acetaminophen/caffeine (Fioricet)
Oral ergotamine/caffeine (Cafergot)
Subcutaneous sumatriptan (Imitrex)
Intramuscular morphine
Intramuscular prochlorperazine

A

Multiple studies have determined that parenteral antiemetics have benefits for the treatment of acute
migraine beyond their effect on nausea. Most outpatient clinics do not have the ability to administer
intravenous metoclopramide, which is the preferred treatment. However, most clinics do have the ability
to administer intramuscular prochlorperazine or promethazine. Due to concerns about oversedation,
misuse, and rebound, treatment with parenteral opiates is discouraged but may be an option if other
treatments fail. Oral butalbital/acetaminophen/caffeine and oral ergotamine/caffeine have less evidence of
success in the treatment of acute migraine. Sumatriptan is contraindicated within 24 hours of the use of
rizatriptan.

How well did you know this?
1
Not at all
2
3
4
5
Perfectly
33
Q

A 72-year-old female presents for a routine health maintenance visit. Which one of the following medications in her current regimen places her at risk for osteoporosis? (check one)
Atorvastatin (Lipitor)
Hydrochlorothiazide
Metformin (Glucophage)
Phenytoin (Dilantin)
Ranitidine (Zantac)

A

Medications reported to be associated with osteoporosis and increased fracture risk include antiepileptic
drugs, long-term heparin, cyclosporine, tacrolimus, aromatase inhibitors, glucocorticoids,
gonadotropin-releasing hormone agonists, thiazolidinediones, excessive doses of levothyroxine, proton
pump inhibitors, SSRIs, parenteral nutrients, medroxyprogesterone contraceptives, methotrexate, and
aluminum antacids. Atorvastatin, hydrochlorothiazide, metformin, and ranitidine are not associated with
osteoporosis.

How well did you know this?
1
Not at all
2
3
4
5
Perfectly
34
Q

A 42-year-old bricklayer was diagnosed with acute bronchitis at an urgent care center 6–7 weeks ago. A chest radiograph was negative for pneumonia but revealed a solitary pulmonary nodule. No previous chest radiograph was available, so a follow-up chest radiograph was ordered for 4–6 weeks after the initial one. He is following up with you today to review those results.

The patient’s bronchitis has since resolved, and he feels well. He has no significant past medical history and does not take any medications. He has no constitutional symptoms and a physical examination today is within normal limits. He has a 10-pack-year cigarette smoking history and quit 15 years ago.

If a lesion is noted on the chest radiograph, which one of the following characteristics would be most suspicious for malignancy? (check one)
A diameter of 5 mm
Concentric calcifications
Doubling in size in less than 1 month
A nonsolid “ground glass” appearance
Smooth borders

A

Characteristics that are more commonly associated with malignant lesions include a nonsolid “ground glass” appearance, a size >6 mm, noncalcified lesions, a lesion size or volume doubling time between 1 month and 1 year, and irregular or spiculated borders. Findings on a chest radiograph that are more commonly associated with benign lesions include a lesion size <6 mm, concentric or “popcorn-like”
calcifications, doubling times of <1 month or >2–4 years, dense solid-appearing lesions, and lesions with smooth regular borders. Other diagnostic imaging modalities are also utilized, including CT and PET, and a biopsy is sometimes necessary to establish the diagnosis. Chest radiographs are still useful for monitoring
patients with multiple findings that correlate most often with benign lesions. Informed decision making by the patient and family physician can sensibly guide the follow-up of patients with solitary pulmonary nodules without automatically referring them to specialists or ordering the most sophisticated imaging

How well did you know this?
1
Not at all
2
3
4
5
Perfectly
35
Q

You are the team physician for the local high school track team. During a meet one of the athletes inadvertently steps off the edge of the track and inverts her right foot forcefully. She is able to bear weight but with significant pain. She reports pain across her right midfoot. An examination reveals edema over the lateral malleolus and diffuse tenderness, but she does not have any pain with palpation of the navicular, the base of the fifth metatarsal, or the posterior distal lateral and medial malleoli.
Which one of the following would be most appropriate at this time?
A) Radiographs of the right ankle only
B) Radiographs of the right foot only
C) Radiographs of the right foot and ankle
D) Lace-up ankle support, ice, compression, and clinical follow-up
E) Crutches and no weight bearing for 2 weeks, followed by a slow return to weight
bearing

A

ANSWER: D
The Ottawa foot and ankle rules should be used to determine the need for radiographs in foot and ankle injuries. A radiograph of the ankle is recommended if there is pain in the malleolar zone along with the inability to bear weight for at least four steps immediately after the injury and in the physician’s office or emergency department (ED), or tenderness at the tip of the posterior medial or lateral malleolus. A radiograph of the foot is recommended if there is pain in the midfoot zone along with the inability to bear weight for four steps immediately after the injury and in the physician’s office or ED, or tenderness at the base of the fifth metatarsal or over the navicular bone. The Ottawa foot and ankle rules are up to 99% sensitive for detecting fractures, although they are not highly specific. In this case there are no findings that would require radiographs, so treatment for the ankle sprain would be recommended. Compression combined with lace-up ankle support or an air cast, along with cryotherapy, is recommended and can increase mobility. Early mobilization, including weight bearing as tolerated for daily activities, is associated with better long-term outcomes than prolonged rest.

How well did you know this?
1
Not at all
2
3
4
5
Perfectly
36
Q

Increases risk of afib

A

Alcohol

How well did you know this?
1
Not at all
2
3
4
5
Perfectly
37
Q

A 10-year-old male has an 8-mm induration 2 days after a tuberculin skin test. He shares a bedroom with his 18-year-old brother who was recently diagnosed with tuberculosis. There are no other historical or physical examination findings to suggest active tuberculosis infection and a chest radiograph is normal.
Which one of the following would be most appropriate at this point?
A) Monitoring with annual tuberculin skin testing
B) Observation and repeat tuberculin skin testing in 3 weeks
C) Rifampin (Rifadin) daily for 4 months
D) Isoniazid daily for 9 months
E) Once-weekly isoniazid and rifampin for 3 months

A

D. Isoniazid for 9 months

For children age 2–11 years, treatment with isoniazid, 10–20 mg/kg daily or 20–40 mg/kg twice weekly for 9 months, is the preferred and most efficacious treatment regimen. The shorter 6-month treatment course is considered an acceptable option for adults, but it is not recommended for children. The use of rifampin alone or in combination with isoniazid is also an acceptable option for adults but not for children under the age of 12.

> 5 mm for exposed
10 for risk factors
15 mm for no risks

How well did you know this?
1
Not at all
2
3
4
5
Perfectly
38
Q

A 28-year-old female presents with a 3-month history of fatigue and postural lightheadedness. On examination she is diffusely hyperpigmented, especially her skin creases and areolae. A CBC and basic metabolic panel are normal except for an elevated potassium level. You order a corticotropin stimulation test.
Prior to the corticotropin injection, you should order which one of the following tests to confirm that this patient has a primary insufficiency and not a secondary (pituitary) disorder?
A) ACTH
B) Aldosterone
C) Melanocyte-stimulating hormone
D) Renin
E) TSH

A

A
A plasma ACTH level is recommended to establish primary adrenal insufficiency. The sample can be obtained at the same time as the baseline sample in the corticotropin test. A plasma ACTH greater than twice the upper limit of the reference range is consistent with primary adrenal insufficiency. Aldosterone and renin levels should be obtained to establish the presence of adrenocortical insufficiency, but these do not differentiate primary from secondary adrenal insufficiency. The hyperpigmentation of Addison’s disease is caused by the melanocyte-stimulating hormone (MSH)–like effect of the elevated plasma levels of ACTH. ACTH shares some amino acids with MSH and also produces an increase in MSH in the blood. TSH is not part of the feedback loop of adrenal insufficiency.

How well did you know this?
1
Not at all
2
3
4
5
Perfectly
39
Q

A 32-year-old female requests a physical examination prior to participating in an adult soccer league. Her blood pressure is 118/70 mm Hg and her pulse rate is 68 beats/min. The examination is otherwise normal except for a systolic murmur that intensifies with Valsalva maneuvers. She says that she has recently been experiencing mild exertional dyspnea and moderate chest pain. The chest pain has been atypical and is not necessarily related to exertion. Echocardiography reveals hypertrophic cardiomyopathy.
In addition to referring the patient to a cardiologist, you recommended initiating therapy with
A) amiodarone (Cordarone)
B) amlodipine (Norvasc)
C) furosemide (Lasix)
D) lisinopril (Prinivil, Zestril)
E) metoprolol

A

E
Hypertrophic cardiomyopathy is the most common primary cardiomyopathy, with a prevalence of 1:500 persons. Many patients with hypertrophic cardiomyopathy are asymptomatic and are diagnosed during family screening, by auscultation of a heart murmur, or incidentally after an abnormal result on electrocardiography. On examination physicians may hear a systolic murmur that increases in intensity during Valsalva maneuvers. The main goals of therapy are to decrease exertional dyspnea and chest pain and prevent sudden cardiac death. -Blockers are the initial therapy for patients with symptomatic hypertrophic cardiomyopathy. Nondihydropyridine calcium channel blockers such as verapamil can be used if -blockers are not well tolerated.

How well did you know this?
1
Not at all
2
3
4
5
Perfectly
40
Q

High intensity statin

A

high-intensity statin regimens if tolerated. These regimens include atorvastatin, 40–80 mg/day, and rosuvastatin, 20–40 mg/day.

How well did you know this?
1
Not at all
2
3
4
5
Perfectly
41
Q

A 62-year-old female who is a new patient requests a thyroid evaluation because she has a history of abnormal thyroid test results. You obtain a copy of her records, which include a TSH level of 0.2 U/mL (N 0.4–4.2) and a free T4 level of 2.0 ng/dL (N 0.8–2.7) from 3 years ago. She reports feeling well and has no other health conditions. She does not take any medications.
A physical examination reveals normal vital signs, a BMI of 23.0 kg/m2, no neck masses, a normal thyroid size, and normal heart sounds. Laboratory studies reveal a TSH level of 0.1 U/mL, a free T4 level of 2.5 ng/dL, and a free T3 level of 3.1 pg/mL (N 2.3–4.2).
Treatment for this condition would be indicated if the patient has an abnormal
A) calcium level
B) DXA scan
C) glucose level
D) lipid level
E) thyroid ultrasonography study

A

ANSWER: B
This patient has subclinical hyperthyroidism as evidenced by her low TSH level with normal free T4 and free T3 levels. Common causes of subclinical hyperthyroidism include Graves disease, autonomous functioning thyroid adenoma, and multinodular toxic goiter. Subclinical hyperthyroidism may progress to overt hyperthyroidism; this is more likely in patients with TSH levels <0.1 U/mL. Even in the absence of overt hyperthyroidism these patients are at higher risk for several health conditions, including atrial fibrillation, heart failure, and osteoporosis. For this reason it is important to assess for these conditions and consider treating the underlying thyroid condition, as well as the complication. The American Thyroid Association recommends treating patients with complications who are either over age 65 or have a TSH level <0.1 U/mL.
Lipid and glucose abnormalities are not known to be related to subclinical hyperthyroidism. Calcium levels may be abnormal in hyperparathyroidism but not hyperthyroidism. Thyroid ultrasonography may be helpful to determine the cause of hyperthyroidism but is not used to help decide when to treat subclinical hyperthyroidism.

How well did you know this?
1
Not at all
2
3
4
5
Perfectly
42
Q

A 7-year-old female with asthma is brought to your office because of her fourth episode of wheezing in the last 3 months. She has also had to use her short-acting -agonist rescue inhaler more frequently.
Which one of the following should be added to reduce the frequency of asthma exacerbations?
A) A leukotriene receptor antagonist
B) A long-acting -agonist
C) An inhaled corticosteroid
D) Inhaled cromolyn via nebulizer

A

ANSWER: C
Pediatric asthma is the most commonly encountered chronic illness, occurring in nearly one out of seven individuals. Short-acting -agonists in the form of metered-dose inhalers are clearly favored for acute exacerbations, as well as for intermittent asthma. Treatment for persistent asthma requires the use of inhaled corticosteroids, with short-acting -agonists used for exacerbations. For patients not well controlled with those options, either a long-acting -agonist or a leukotriene receptor antagonist may be added. While both cromolyn and nedocromil are fairly devoid of adverse effects, their use is limited because of a lack of efficacy in the prevention of acute asthma exacerbations.

How well did you know this?
1
Not at all
2
3
4
5
Perfectly
43
Q

Woods lamp colors

A

Corynebacterium minutissimum infection, and use of an ultraviolet light would reveal a coral pink color. Pale blue fluorescence occurs with Pseudomonas infections, yellow with tinea infections, and totally white with vitiligo. A lime green fluorescence is not characteristic of a particular skin condition.

How well did you know this?
1
Not at all
2
3
4
5
Perfectly
44
Q

Hypertensive emergency

A

> 180 or > 110

How well did you know this?
1
Not at all
2
3
4
5
Perfectly
45
Q

when to add insulin

A

A1C >9

How well did you know this?
1
Not at all
2
3
4
5
Perfectly
46
Q

Which one of the following antihypertensive drugs may reduce the severity of sleep apnea?
A) Amlodipine (Norvasc)
B) Hydralazine
C) Lisinopril (Prinivil, Zestril)
D) Metoprolol
E) Spironolactone (Aldactone)

A

Spironolactone

Diuretics lessen the severity of obstructive sleep apnea and reduce blood pressure. Aldosterone antagonists offer further benefit beyond that of traditional diuretics. Resistant hypertension is common in patients with obstructive sleep apnea. Resistant hypertension is also associated with higher levels of aldosterone, which can lead to secondary pharyngeal edema, increasing upper airway obstruction.

How well did you know this?
1
Not at all
2
3
4
5
Perfectly
47
Q

Which one of the following malignancies is associated with hereditary hemochromatosis?
A) Biliary carcinoma
B) Chronic myeloid leukemia
C) Hepatocellular carcinoma
D) Multiple myeloma
E) Pancreatic cancer

A

C
Hereditary hemochromatosis is a genetic disorder of iron regulation and subsequent iron overload. Possible end-organ damage includes cardiomyopathy, cirrhosis of the liver, and hepatocellular carcinoma. Symptoms are often nonspecific early on, but manifestations of iron overload eventually occur. The diagnosis should be suspected in patients with liver disease or abnormal iron studies indicative of iron overload. A liver biopsy can confirm the diagnosis and the degree of fibrosis. Identification of such patients and proper ongoing treatment with phlebotomy may prevent the development of hepatocellular carcinoma and other complications of this disease. There is some data that suggests an association of breast cancer with hereditary hemochromatosis but not with any of the other malignancies listed.

How well did you know this?
1
Not at all
2
3
4
5
Perfectly
48
Q

You admit a previously healthy 62-year-old female to the hospital for intractable nausea and vomiting with intravascular volume depletion and hypotension. She lives in rural northern New Mexico. Prior to the onset of her symptoms she had been gardening and cleaning out a chicken coop, where she encountered several rodents. She is febrile and you obtain blood and urine cultures. Two out of four blood culture bottles are positive for gram-negative rods.
Which one of the following is the most likely pathogen?
A) Brucella melitensis
B) Coxiella burnetii
C) Escherichia coli
D) Listeria monocytogenes
E) Yersinia pestis

A

Yersinia pestis is an aerobic fermentative gram-negative rod. It causes a zoonotic infection with humans as the accidental host. The disease is spread by a bite from a flea vector, direct contact with infected tissue, or inhalation of infectious aerosols from a person with pulmonary plague. Plague occurs in two regions in the western United States. One region includes northern New Mexico, northern Arizona, and southern Colorado, and the other region includes California, southern Oregon, and far western Nevada.
Escherichia coli is also an aerobic fermentative gram-negative rod but it generally causes symptoms of gastroenteritis, hemolytic-uremic syndrome, urinary tract infection, intra-abdominal infection, and meningitis. E. coli infection does not have a specific regional distribution. Listeria monocytogenes is a gram-positive rod and causes an influenza-like illness with or without gastroenteritis in adults. Infection occurs through ingestion of contaminated food products such as milk, cheese, processed meats, and raw vegetables. Outbreaks can occur in any geographic distribution.
25
Coxiella burnetii is a gram-negative intracellular bacterium that causes Q fever. Human infections are associated with contact with infected cattle, sheep, goats, dogs, and cats. Brucella melitensis is a gram-negative coccobacilli that causes brucellosis. Humans are accidental hosts who can develop the disease from contact with tissues rich in erythritol, and from shedding of organisms in milk, urine, and birth products from goats and sheep.

How well did you know this?
1
Not at all
2
3
4
5
Perfectly
49
Q

A 21-year-old female is being evaluated for secondary causes of refractory hypertension. Which one of the following would be most specific for fibromuscular dysplasia?
A) A serum creatinine level
B) An aldosterone:renin ratio
C) 24-hour urine for metanephrines
D) Renal ultrasonography
E) Magnetic resonance angiography of the renal arteries

A

ANSWER: E
In young adults diagnosed with secondary hypertension, evaluation for fibromuscular dysplasia of the renal arteries with MR angiography or CT angiography is indicated (SOR C). The aldosterone/renin ratio is the most sensitive test to diagnose primary hyperaldosteronism. Renal ultrasonography is an indirect test that is not as sensitive or specific for fibromuscular dysplasia. Serum creatinine elevation shows renal involvement but does not identify the cause. Testing for metanephrines is indicated only if a pheochromocytoma is suspected.

How well did you know this?
1
Not at all
2
3
4
5
Perfectly
50
Q

Which one of the following diabetes mellitus medications is MOST likely to cause weight gain?
A) Empagliflozin (Jardiance)
B) Glimepiride (Amaryl)
C) Liraglutide (Victoza)
D) Metformin (Glucophage)
E) Sitagliptin (Januvia)

A

NSWER: B
Since many patients with diabetes mellitus are obese, the impact of medications on the patient’s weight is important to consider. Treatment with sulfonylureas, including glimepiride, is associated with weight gain. Empagliflozin, liraglutide, metformin, and sitagliptin are not associated with weight gain. In particular, the SGLT2 inhibitors such as empagliflozin and the GLP1 agonists such as liraglutide are associated with clinically significant weight loss.

How well did you know this?
1
Not at all
2
3
4
5
Perfectly
51
Q

A 45-year-old male presents to your office with a 2-month history of a nonproductive cough, mild shortness of breath, fatigue, and a 5-lb weight loss. On examination his lungs are clear. A PPD skin test is negative. A chest radiograph shows bilateral hilar adenopathy and his angiotensin converting enzyme level is elevated. A biopsy of the lymph node shows a noncaseating granuloma.
Which one of the following would be the most appropriate initial treatment?
A) Azathioprine (Imuran)
B) Fluconazole (Diflucan)
C) Isoniazid
D) Levofloxacin (Levaquin)
E) Prednisone

A

ANSWER: E
This patient has sarcoidosis that has been confirmed by a biopsy. He is symptomatic so treatment would be indicated. The recommended initial treatment for sarcoidosis is oral corticosteroids. Anti-infective agents are not appropriate treatment for sarcoidosis. Immunosuppressants are second- and third-line therapy for sarcoidosis and would not be recommended as first-line treatment.

How well did you know this?
1
Not at all
2
3
4
5
Perfectly
52
Q

Tdap at what week

A

27-36

How well did you know this?
1
Not at all
2
3
4
5
Perfectly
53
Q

A 25-year-old gravida 1 para 0 at 24 weeks gestation comes to your office with right lower extremity swelling and pain. Her pregnancy has been uncomplicated so far and her only medication is a prenatal vitamin. She does not have chest pain, shortness of breath, or fever. She recently started feeling the baby move, and an anatomy scan at 20 weeks gestation was normal.
Lower extremity Doppler ultrasonography confirms a right lower extremity deep vein thrombosis (DVT). Laboratory studies including a CBC, coagulation studies, and renal function are normal.
Which one of the following would be the most appropriate initial treatment of her DVT?
A) Oral apixaban (Eliquis)
B) Oral aspirin
C) Oral warfarin (Coumadin)
D) Subcutaneous enoxaparin (Lovenox)
E) Subcutaneous heparin

A

ANSWER: D
Enoxaparin is the most appropriate pharmacologic therapy for anticoagulation in patients who are pregnant. Aspirin is not used as treatment for deep vein thrombosis. Apixaban, warfarin, and heparin either have not been studied for use in pregnancy or there is data indicating potential fetal harm.

How well did you know this?
1
Not at all
2
3
4
5
Perfectly
54
Q

A 67-year-old female who was recently diagnosed with colon cancer presented to the emergency department 2 days ago with acute shortness of breath and was diagnosed with a pulmonary embolism. She was started on enoxaparin (Lovenox) and was hemodynamically stable during her stay in the hospital. Her shortness of breath has improved and her oxygen saturation is currently 95% on room air.
Which one of the following would be most appropriate for this patient?
A) Continue enoxaparin upon discharge
B) Discontinue enoxaparin and start rivaroxaban (Xarelto)
C) Discontinue enoxaparin and start warfarin (Coumadin)
D) Start warfarin and continue enoxaparin until the INR is 2.0

A

ANSWER: A
Enoxaparin and other low molecular weight heparins are effective and are the preferred agents for acute and long-term anticoagulation in patients with an active malignancy (SOR B). Warfarin has been shown to be less effective in cancer patients and is not recommended to treat venous thromboembolic disease in this setting (SOR B). The novel oral anticoagulants including rivaroxaban have not been studied in the setting of malignancy and are not recommended.

How well did you know this?
1
Not at all
2
3
4
5
Perfectly
55
Q

A 67-year-old female with hypertension and atrial fibrillation has been taking warfarin (Coumadin) for the past 10 years. She has been hemodynamically stable for many years with no complications from her atrial fibrillation. She is scheduled to undergo elective bladder sling surgery for urinary incontinence. She does not have any other significant past medical history.
Which one of the following would be the most appropriate perioperative management of her warfarin?
A) Continue warfarin without interruption
B) Discontinue warfarin the day prior to surgery and provide bridge therapy with low
molecular weight heparin
C) Discontinue warfarin 2 days prior to surgery and restart it 2 days postoperatively
unless there is a bleeding complication
D) Discontinue warfarin 2 days prior to surgery and restart it 5 days postoperatively
unless there is a bleeding complication
E) Discontinue warfarin 5 days prior to surgery and restart it 12–24 hours
postoperatively unless there is a bleeding complication

A

ANSWER: E
Perioperative management of chronic anticoagulation requires an assessment of the patient’s risk for thromboembolism and the risk of bleeding from the surgical procedure. High-risk patients include those with mechanical heart valves, a stroke or TIA within the past 3 months, venous thromboembolism within the past 3 months, or coronary stenting within the previous 12 months. High-risk patients require bridging therapy with low molecular weight heparin, while patients at low risk do not require bridging anticoagulation. For low-risk patients, it is recommended that warfarin be discontinued 5 days prior to surgery and restarted 12–24 hours postoperatively. This patient is at low risk for thromboembolism because her CHA2DS2-VASc score is 3. A patient with atrial fibrillation should receive bridging therapy with a CHA2DS2-VASc score 6. This patient’s surgery is associated with a high risk for bleeding, so it is preferable to stop her warfarin 5 days before the operation.

How well did you know this?
1
Not at all
2
3
4
5
Perfectly
56
Q

A 27-year-old female with a past medical history of polycystic ovary syndrome (PCOS) would like to become pregnant. Which one of the following treatments for PCOS is associated with greater live-birth and ovulation rates?
A) Finasteride (Proscar)
B) Letrozole (Femara)
C) Metformin (Glucophage)
D) Spironolactone (Aldactone)

A

ANSWER: B
In a double-blind randomized trial, letrozole was associated with greater live-birth and ovulation rates compared to clomiphene (SOR A). A Cochrane review indicated that metformin does not increase fertility in patients diagnosed with polycystic ovary syndrome (PCOS). Spironolactone and finasteride are both used to treat PCOS in women who do not desire pregnancy.

How well did you know this?
1
Not at all
2
3
4
5
Perfectly
57
Q

A 58-year-old male with a history of tobacco and alcohol abuse presents with the sudden onset of many well circumscribed brown, oval, rough papules with a “stuck-on” appearance on his trunk and proximal extremities. On examination you also note an unintentional 6-kg (13-lb) weight loss over the last 3 months and conjunctival pallor. A review of systems is positive for more frequent stomachaches, decreased appetite, and mild fatigue.
You order a laboratory workup. Which one of the following would be most appropriate at this point?
A) Reassurance that the skin lesions are benign
B) A skin biopsy
C) Referral to a dermatologist
D) CT of the abdomen and pelvis
E) Upper and lower endoscopy

A

ANSWER: E
This patient’s age, risk factors, red-flag symptoms, and other clinical findings indicate the need for endoscopy. The Leser-Trélat sign may be defined as the abrupt onset of multiple seborrheic keratoses, which is an unusual finding that often indicates an underlying malignancy, most commonly an adenocarcinoma of the stomach. CT is not an initial approach for diagnosing a suspected malignancy of the stomach or colon. Further skin evaluation and lifestyle changes, which are indicated, will not address the need for evaluation of weight loss and other abnormal symptoms and findings.

How well did you know this?
1
Not at all
2
3
4
5
Perfectly
58
Q

A patient with hypercalcemia has low parathyroid hormone levels and elevated parathyroid hormon related peptid levels.

A. Familial hypocalciuric hypercalcemia
B. Pseudohypoparathyroidism
C. Vitamin D deficiency
D. Humoral hypercalcemia of malignancy

A

D. Humoral hypercalcemia of malignancy

How well did you know this?
1
Not at all
2
3
4
5
Perfectly
59
Q

Medication for reducing tension headaches

A

Amitryptyline

How well did you know this?
1
Not at all
2
3
4
5
Perfectly
60
Q

Antibiotic assocaited with c d.iff

A

clindamycin

How well did you know this?
1
Not at all
2
3
4
5
Perfectly
61
Q

What medications increased risk of ventilator associated pneumonia

A

HIstamine H2 blockers (famotidine)
PPI

How well did you know this?
1
Not at all
2
3
4
5
Perfectly
62
Q

Hb goal level in thalessemia

A

9-10.5

How well did you know this?
1
Not at all
2
3
4
5
Perfectly
63
Q

Pertussis prophylaxis

A

azithromycin within 21 days of exposure

How well did you know this?
1
Not at all
2
3
4
5
Perfectly
64
Q

Bishop score

A

Dilation
0 closed
1 1-2 cm
2 3-4 cm
3 >= 5 cm

Effacement
0 0-30%
1 40-50%
2 60-70%
3 >=80%

Fetal station
0 -3
1 -2
2 -1,0
3 +1, +2

> 5 = successful vaginal delivery

How well did you know this?
1
Not at all
2
3
4
5
Perfectly
65
Q

Weight in pregnancy

A

underweight 28-40 lb
normal weight 25-35
over weight 15-25
obese 11-20

How well did you know this?
1
Not at all
2
3
4
5
Perfectly
66
Q

New alzheimer drug

A

Aducanumab (aduhelm)

How well did you know this?
1
Not at all
2
3
4
5
Perfectly
67
Q

Triptans are contraindicated in who

A

cardiovascualr disease
cerbrovascular disease
uncontrolled htn

How well did you know this?
1
Not at all
2
3
4
5
Perfectly
68
Q

Cluter headache prophylaxis

A

Verapamil

How well did you know this?
1
Not at all
2
3
4
5
Perfectly
69
Q

Orthostatic hypotension definity

A

change in blood pressure of 20 mm Hg or more systolic of 10 mm Hg or more diastolic

How well did you know this?
1
Not at all
2
3
4
5
Perfectly
70
Q

Transietn global amnesia due to

A

confusion for up to 24 hours due to impairment of venous drainage from hippocampus

How well did you know this?
1
Not at all
2
3
4
5
Perfectly
71
Q

A 49-year-old male is concerned about lesions on his penis that he has noticed over the past 6 months. He was circumcised as a child and has had the same female sexual partner for 5 years. He does not have any pain, itching, or dysuria. On examination you note multiple reddish-blue papules on the scrotum and a few similar lesions on the shaft of the penis.
The most likely diagnosis is
A) pearly penile papules
B) lichen nitidus
C) lichen sclerosus
D) angiokeratomas
E) squamous cell carcinoma in situ (Bowen’s disease)

A

ANSWER: D
Penile lesions are usually easily diagnosed from clinical findings. Pearly penile papules are common and benign, and present as small, skin-colored, dome-shaped papules in a circular pattern around the coronal sulcus.
Lichen nitidus is benign but uncommon. It presents as discrete, pinhead-sized hypopigmented papules that are asymptomatic. Papules are often found scattered all over the penis, as well as on the abdomen and upper extremities.
Lichen sclerosus is more common and appears as hypopigmented lesions with the texture of cellophane. The lesions are usually located on the glans or prepuce. Atrophy, erosions, and bullae are common, and patients often present with itching, pain, bleeding, and possibly phimosis or obstructed voiding. Lichen sclerosus is associated with squamous cell cancer in a small percentage of cases.
Carcinoma in situ is a premalignant condition that is more common in uncircumcised males over age 60. Lesions are typically beefy red, raised, irregular plaques and can be found on the glans, meatus, frenulum, coronal sulcus, and prepuce. Lesions can be ulcerated or crusted. Pruritus and pain are common. A biopsy is important for making the diagnosis.
Angiokeratomas are lesions that are usually asymptomatic, circumscribed, red or bluish papules. They may appear solely on the glans of the penis, but are also found on the scrotum, abdomen, thighs, groin, and extremities. They may be misdiagnosed as pearly papules or carcinoma. Treatment is not necessary unless the lesions are bleeding or extensive. It is important to realize that angiokeratomas on the shaft of the penis, the suprapubic region, or the sacral region can be associated with Fabry disease. Patients with this finding should be promptly referred.

How well did you know this?
1
Not at all
2
3
4
5
Perfectly
72
Q

You see a patient who is being treated for opioid use disorder with buprenorphine. Which one of the following can be used as adjuvant treatment to reduce stress-related opioid cravings and increase abstinence?
A) Clonidine (Catapres)
B) Methadone
C) Naloxone
D) Naltrexone (Vivitrol)
E) Nifedipine (Procardia)

A

ANSWER: A
Clonidine (0.1–0.3 mg every 6–8 hours) is a useful adjunct to buprenorphine in the treatment of opioid use disorder to help increase the rates of abstinence and decrease stress-related opioid cravings (SOR C). Naloxone is an opioid antagonist used to treat overdoses. Nifedipine is a common antihypertensive like clonidine but it has no role in the treatment of opioid use disorder. Methadone and naltrexone are used to treat opioid use disorder but neither of these agents would be used simultaneously with buprenorphine.

How well did you know this?
1
Not at all
2
3
4
5
Perfectly
73
Q

In a 60-year-old patient who has not previously received pneumococcal vaccine, which one of the following would be an indication for both 13-valent pneumococcal conjugate vaccine (PCV13, Prevnar 13) and 23-valent pneumococcal polysaccharide vaccine (PPSV23, Pneumovax 23)?
A) Alcoholism
B) Chronic renal failure
C) Cigarette smoking
D) COPD
E) Diabetes mellitus

A

ANSWER: B
Both 13-valent pneumococcal conjugate vaccine (PCV13) and 23-valent pneumococcal polysaccharide vaccine (PPSV23) are recommended for patients with chronic renal failure. Indications for PPSV23 alone in immunocompetent persons younger than 65 include chronic lung disease, diabetes mellitus, chronic heart disease, smoking, and alcoholism.

How well did you know this?
1
Not at all
2
3
4
5
Perfectly
74
Q

A 36-year-old male presents with a 2-day history of painless right-sided facial droop. There are no associated symptoms and his medical history is otherwise unremarkable. An examination is remarkable for an unfurrowed right brow, mouth droop, a sagging right lower eyelid, and a complete inability to move the muscles of the right face and forehead. No other weakness is elicited and no rash is seen.
Which one of the following would be the most appropriate management at this point?
A) Reassurance only
B) Valacyclovir (Valtrex) alone
C) A tapering dose of prednisone alone
D) Valacyclovir and a tapering dose of prednisone
E) Immediate transfer to the emergency department

A

ANSWER: D
Early recognition and effective treatment of acute Bell’s palsy (idiopathic facial paralysis) has been shown to decrease the risk of chronic partial paralysis and pain. Corticosteroids have been shown in a meta-analysis to decrease chronic symptoms, but a Cochrane meta-analysis of 10 studies concluded that antiviral medication along with corticosteroids is significantly more effective than corticosteroids alone. The medications are most effective if started within 72 hours of symptom onset. The same analysis showed that antiviral medications alone were less effective than corticosteroids alone. This patient’s presentation is not consistent with stroke or another emergency. Because supranuclear input to the facial nerves comes from both cerebral hemispheres, strokes and other central pathologies affecting the facial nerves typically spare the forehead, which is not the case in this patient.

How well did you know this?
1
Not at all
2
3
4
5
Perfectly
75
Q

In a patient presenting with truncal obesity, hypertension, type 2 diabetes mellitus, hirsutism, osteopenia, and skin fragility, which one of the following tests is needed to confirm the diagnosis of Cushing syndrome?
A) A dexamethasone suppression test
B) Inferior petrosal sinus sampling
C) Plasma corticotropin
D) Plasma free cortisol
E) Urinary free cortisol

A

ANSWER: E
In a patient presenting with obesity, hypertension, type 2 diabetes mellitus, and hirsutism, who also has thin skin and osteopenia, an elevated 24-hour collection showing high urinary free cortisol confirms the presence of Cushing syndrome. The dexamethasone suppression test, though still commonly used, no longer has a place in the diagnosis and treatment of patients with Cushing syndrome. Corticotropin-dependent and corticotropin-independent causes of Cushing syndrome can be separated by measuring plasma corticotropin. Plasma free cortisol measurements should be obtained only to determine the success or failure of transsphenoidal microadenomectomy or adrenalectomy. Inferior petrosal sinus sampling is used to confirm the source of corticotropin secretion before surgical intervention.

How well did you know this?
1
Not at all
2
3
4
5
Perfectly
76
Q

A 50-year-old gravida 2 para 2 who is 3 years post menopausal presents with fatigue, headache, galactorrhea, and loss of libido. Your evaluation reveals elevated serum prolactin and a pituitary adenoma of 5–6 mm.
You recommend
A) bromocriptine (Parlodel)
B) estrogens
C) haloperidol
D) testosterone
E) neurosurgical consultation

A

ANSWER: A
This patient has a pituitary microadenoma. Microadenomas <10 mm in size that are secreting prolactin may be treated with a dopaminergic agent such as bromocriptine. This will lower the prolactin level and shrink the adenoma. Nonprolactin-secreting adenomas, especially those >10 mm in size (macroadenomas), require neurosurgical evaluation.

> 10 mm go to surgery!!

How well did you know this?
1
Not at all
2
3
4
5
Perfectly
77
Q

Coordination of the eyes in newborns improved by what age

A

6 months

How well did you know this?
1
Not at all
2
3
4
5
Perfectly
78
Q

A 47-year-old male presents with a 3-day history of fever, chills, low back pain, and urinary frequency. He does not have any nausea, vomiting, or abdominal pain. There is no significant past medical history.
The patient’s vital signs include a temperature of 38.1°C (100.6°F), a pulse rate of 88 beats/min, and a respiratory rate of 14/min. The examination reveals a mildly tender lower abdomen with no guarding or rebound tenderness; no costovertebral angle tenderness; and an enlarged, homogeneous, exquisitely tender prostate.
Which one of the following is indicated to help guide this patient’s treatment?
A) A serum prostate-specific antigen level
B) A culture of prostate secretions after massage of the prostate
C) A culture of midstream voided urine
D) CT of the abdomen and pelvis with intravenous and oral contrast
E) An ultrasound-guided prostate biopsy

A

ANSWER: C
This patient has clinically diagnosable acute bacterial prostatitis, and no further testing, including imaging, is required to establish the diagnosis. Culture of a midstream voided urine may aid in identifying the pathogen, but prostate massage should be avoided because it may increase the risk of bacteremia. A prostate biopsy is not indicated in the presence of acute infection, and a prostate-specific antigen level is not indicated because it is likely to be elevated in the presence of infection.

How well did you know this?
1
Not at all
2
3
4
5
Perfectly
79
Q

A 57-year-old female is admitted to the hospital with lower lobe pneumonia. She has no history of diabetes mellitus. She has not met sepsis criteria but had a blood glucose level of 172 mg/dL in the emergency department.
Insulin should be started if this patient has a persistent blood glucose level greater than or equal to
A) 120 mg/dL
B) 140 mg/dL
C) 160 mg/dL
D) 180 mg/dL

A

ANSWER: D
Insulin therapy should be initiated in hospitalized patients with persistent hyperglycemia, starting at a threshold of 180 mg/dL. Once insulin therapy is started, a target glucose range of 140–180 mg/dL is recommended for the majority of hospitalized patients, regardless of whether they have a critical illness.

How well did you know this?
1
Not at all
2
3
4
5
Perfectly
80
Q

A 21-year-old gravida 1 para 0 is diagnosed with overt hyperthyroidism early in the first trimester. The most appropriate management at this time is
A) observation only
B) methimazole (Tapazole)
C) propylthiouracil
D) radioactive iodine
E) thyroidectomy

A

ANSWER: C
Overt hyperthyroidism during pregnancy is associated with adverse effects to the mother and fetus, so treatment is required. Since methimazole is associated with birth defects when used in the first trimester, propylthiouracil is preferred. Methimazole should be considered after the first trimester because the risk of congenital anomalies is less than the risk of liver failure associated with propylthiouracil. Surgery and radioactive iodine should only be used if there is a clear indication, and radioactive iodine would not be appropriate during pregnancy.

How well did you know this?
1
Not at all
2
3
4
5
Perfectly
81
Q

A 45-year-old male with a 30-pack-year smoking history reports a chronic cough with a small amount of phlegm production and dyspnea with strenuous exercise. You order spirometry, which shows a pre- and postbronchodilator FEV1/FVC ratio of 0.6 and an FEV1 of 85% of predicted.
Which one of the following agents would be the best initial pharmacologic management?
A) An inhaled corticosteroid
B) A short-acting anticholinergic
C) A long-acting anticholinergic
D) A long-acting beta 2-agonist
E) Theophylline

A

ANSWER: B
This patient has COPD and is in a risk category of A (low risk, fewer symptoms) based on the Global Initiative for Chronic Obstructive Lung Disease (GOLD) combined assessment of COPD. As a result, either a short-acting anticholinergic or a short-acting 2-agonist should be selected as the initial pharmacologic management. Long-acting 2-agonists or long-acting anticholinergics are indicated for patients with a GOLD combined assessment category of B or worse. Long-acting inhaled corticosteroids are indicated for patients with a GOLD combined assessment category of C or worse. Due to its narrow therapeutic window, modest benefit, and need for monitoring, theophylline is not recommended as an initial agent and should be considered as an alternative only for patients with severe refractory symptoms.

How well did you know this?
1
Not at all
2
3
4
5
Perfectly
82
Q

A 30-year-old white male presents to the emergency department with a 4-day history of fever to 101°F, a sore throat, rhinorrhea, and cough. An examination reveals rhinorrhea and a boggy nasal mucosa, but is otherwise unremarkable. A chest radiograph shows a questionable infiltrate.
Which one of the following would help determine if antibiotic treatment would be appropriate?
A) A C-reactive protein level
B) A procalcitonin level
C) A WBC count with differential
D) An erythrocyte sedimentation rate
E) CT of the chest

A

ANSWER: B
Using a procalcitonin-guided therapy algorithm reduces antibiotic use by 3.47 days without increasing either morbidity or mortality in adults with acute respiratory infections. If the procalcitonin level is <0.10 mg/dL, a bacterial infection is highly unlikely and it is strongly recommended that antibiotics not be prescribed. If the procalcitonin level is 0.10–0.24 mg/dL a bacterial infection is still unlikely and it is recommended that antibiotics not be used. If the level is 0.25–0.50 mg/dL a bacterial infection is likely and antibiotics are recommended. It is strongly recommended that antibiotics be given if the level is >0.50 mg/dL, because a bacterial infection is very likely.

How well did you know this?
1
Not at all
2
3
4
5
Perfectly
83
Q

An 80-year-old male sees you for the first time. He is asymptomatic except for some fatigue. His pulse rate is 50 beats/min. An EKG shows a prolonged PR interval.
Which one of the following medications in his current regimen is the most likely explanation for these findings?
A) Donepezil (Aricept)
B) Escitalopram (Lexapro)
C) Lisinopril (Prinivil, Zestril)
D) Memantine (Namenda)
E) Zolpidem (Ambien)

A

ANSWER: A
The 2015 American Geriatrics Society Beers Criteria for potentially inappropriate medication use in older adults 65 years of age states that donepezil use should be avoided in patients with syncope, due to an increased risk of bradycardia (Moderate Evidence Level; Strong Strength of Recommendation). Donepezil is a cholinesterase inhibitor. Due to their cholinergic effect, these medications have a vagotonic effect on the sinoatrial and atrioventricular nodes. This can cause bradycardia or heart block in patients with or without underlying cardiac conduction abnormalities. Syncope has been reported with these medications.

How well did you know this?
1
Not at all
2
3
4
5
Perfectly
84
Q

A 14-year-old female is brought to your office for an annual well child check and sports preparticipation physical examination. She says she does a lot of running during basketball practices and games but has trouble controlling her weight. Most of her family is overweight. She does not have any difficulty participating in sports, and has no symptoms such as chest pain, shortness of breath, or headaches. She has no significant past medical history.
On examination the patient’s height is 154 cm (61 in) and she weighs 63 kg (139 lb). Her BMI is 26.4 kg/m2, which places her in the 90th percentile for her age. Her blood pressure is 130/85 mm Hg, which places her between the 95th and 99th percentile for her age, height, and sex. Her chart reveals that her blood pressure was at this level at the last two visits. The physical examination is otherwise normal.
In addition to counseling and support for weight loss, which one of the following would be most appropriate at this point?
A) Informing the patient and her parents that she is prehypertensive and having her return for a blood pressure check in 3 months
B) Plasma renin and catecholamine levels
C) An imaging study of the renal arteries
D) A fasting basic metabolic panel, a lipid profile, and a urinalysis
E) Antihypertensive drug therapy

A

ANSWER: D
In a pediatric patient, blood pressure should be evaluated using comparisons based on age, sex, and height. Although this adolescent’s blood pressure is prehypertensive for an adult according to JNC 8 guidelines, it is stage 1 hypertension (between 95% and 99%) for her age, sex, and height. All pediatric patients with confirmed hypertension should have further evaluation to check for renal dysfunction as well as other cardiac risk factors. Additionally, renal ultrasonography is recommended to evaluate for renal disease and echocardiography to evaluate for end-organ damage that would affect treatment goals. Additional studies, such as plasma renin and catecholamine levels or renovascular imaging, may be indicated in children with abnormalities on initial evaluation that suggest secondary causes of hypertension.
Pharmacologic therapy is usually recommended for pediatric patients with symptomatic hypertension, secondary hypertension, target organ damage, diabetes mellitus, or persistent hypertension in spite of nonpharmacologic treatment. A low-sodium diet may be helpful for decreasing blood pressure, and given this patient’s obesity, intensive counseling about lifestyle changes is appropriate.

How well did you know this?
1
Not at all
2
3
4
5
Perfectly
85
Q

An otherwise healthy 57-year-old male presents with mild fatigue, decreased libido, and erectile dysfunction. A subsequent evaluation of serum testosterone reveals hypogonadism.
Which one of the following would you recommend at this time?
A) No further diagnostic testing
B) A prolactin level
C) A serum iron level and total iron binding capacity
D) FSH and LH levels
E) Karyotyping

A

ANSWER: D
In men who are diagnosed with hypogonadism with symptoms of testosterone deficiency and unequivocally and consistently low serum testosterone concentrations, further evaluation with FSH and LH levels is advised as the initial workup to distinguish between primary and secondary hypogonadism. If secondary hypogonadism is indicated by low or inappropriately normal FSH and LH levels, prolactin and serum iron levels and measurement of total iron binding capacity are recommended to determine secondary causes of hypogonadism, with possible further evaluation to include other pituitary hormone levels and MRI of the pituitary. If primary hypogonadism is found, karyotyping may be indicated for Klinefelter’s syndrome.

How well did you know this?
1
Not at all
2
3
4
5
Perfectly
86
Q

A 35-year-old female comes to your office for evaluation of a tremor. During the interview you note jerking movements first in one hand and then the other, but when the patient is distracted the symptom resolves. Aside from the intermittent tremor the neurologic examination is unremarkable. She does not drink caffeinated beverages and takes no medications.
Which one of the following is the most likely diagnosis?
A) Parkinson’s disease
B) Cerebellar tremor
C) Essential tremor
D) Physiologic tremor
E) Psychogenic tremor

A

ANSWER: E
Psychogenic tremor is characterized by an abrupt onset, spontaneous remission, changing characteristics, and extinction with distraction. Cerebellar tremor is an intention tremor with ipsilateral involvement on the side of the lesion. Neurologic testing will reveal past-pointing on finger-to-nose testing. CT or MRI of the head is the diagnostic test of choice. Parkinsonian tremor is noted at rest, is asymmetric, and decreases with voluntary movement. Bradykinesia, rigidity, and postural instability are generally noted. For atypical presentations a single-photon emission CT or positron emission tomography may help with the diagnosis. One of the treatment options is carbidopa/levodopa.
Patients who have essential tremor have symmetric, fine tremors that may involve the hands, wrists, head, voice, or lower extremities. This may improve with ingestion of small amounts of alcohol. There is no specific diagnostic test but the tremor is treated with propranolol or primidone. Enhanced physiologic tremor is a postural tremor of low amplitude exacerbated by medication. There is usually a history of caffeine use or anxiety.

How well did you know this?
1
Not at all
2
3
4
5
Perfectly
87
Q

A nulliparous 34-year-old female comes to your office for evaluation of fatigue, hair loss, and anterior neck pain. These symptoms have been gradually worsening for the past few months. Her past medical history is unremarkable. She has gained 5 kg (11 lb) since her last office visit 18 months ago. Examination of the thyroid gland reveals tenderness but no discrete nodules. Her TSH level is 7.5 U/mL (N 0.4–4.2), her T4 level is low, and her thyroid peroxidase antibodies are elevated.
Which one of the following would be the most appropriate next step?
A) Continue monitoring TSH every 6 months
B) Begin thyroid hormone replacement and repeat the TSH level in 6–8 weeks
C) Begin thyroid hormone replacement and repeat the TSH level along with a T3 level in
6–8 weeks
D) Order ultrasonography of the thyroid
E) Order fine-needle aspiration of the thyroid

A

ANSWER: B
This patient has thyroiditis with biochemical evidence for autoimmune (Hashimoto’s) thyroiditis. The most appropriate plan of care is to begin thyroid hormone replacement and monitor with a repeat TSH level 6–8 weeks later. It is not necessary to include a T3 level when assessing the levothyroxine dose. There is no need to routinely order thyroid ultrasonography when there are no palpable nodules on a thyroid examination. Fine-needle aspiration may be necessary to rule out infectious thyroiditis when a patient presents with severe thyroid pain and systemic symptoms.

How well did you know this?
1
Not at all
2
3
4
5
Perfectly
88
Q

thyroid peroxidase antibodies

A

Hashimoto

How well did you know this?
1
Not at all
2
3
4
5
Perfectly
89
Q

Mobitz type 1 vs mobitz type 2

A

Mobitz type 1 cardiac monitoring
- prolonging P

Mobitz type 2 need pace maker
- Same p wave

How well did you know this?
1
Not at all
2
3
4
5
Perfectly
90
Q

antibiotic for travelers diarrhea

A

azithromycin

How well did you know this?
1
Not at all
2
3
4
5
Perfectly
91
Q

A 30-year-old gravida 1 para 0 develops erythematous patches with slightly elevated scaly borders during her first trimester. There was a 2-cm herald patch 2 weeks before multiple smaller patches appeared. The rash on the back has a “Christmas tree” pattern. She has not had any prenatal laboratory work.
This condition is associated with
A) no additional pregnancy risk
B) a small-for-gestational-age newborn
C) congenital cataracts
D) multiple birth defects
E) spontaneous abortion

A

ANSWER: E
This patient has classic pityriasis rosea. This is generally a benign disease except in pregnancy. The epidemiology and clinical course suggest an infectious etiology. Pregnant women are more susceptible to pityriasis rosea because of decreased immunity. Pityriasis rosea is associated with an increased rate of spontaneous abortion in the first 15 weeks of gestation. It is not associated with an increased risk for a small-for-gestational-age newborn, congenital cataracts, or multiple birth defects.

How well did you know this?
1
Not at all
2
3
4
5
Perfectly
92
Q

herald patch

A

pityrasis rosea

How well did you know this?
1
Not at all
2
3
4
5
Perfectly
93
Q

cough, coryza, conjunctivitis, and Koplik spots.

A

Measles

How well did you know this?
1
Not at all
2
3
4
5
Perfectly
94
Q

Mumps

A

myalgia, fatigue, loss of appetite, fever, and headache

orchitiis

How well did you know this?
1
Not at all
2
3
4
5
Perfectly
95
Q

A 5-year-old female is brought to your office with a progressive rash on her legs (shown below) and buttocks. No rash is noted above the level of the mid-torso. Her mother also reports that the child had two episodes of bloody diarrhea 3 days ago. She also has abdominal pain and on examination she has abdominal tenderness with no rigidity but some voluntary guarding. You also note swelling and tenderness in her left wrist and right knee. A CBC, platelets, prothrombin time, and partial thromboplastin time are normal. A urinalysis reveals mild proteinuria and 5–10 RBCs/hpf.
Which one of the following is the most likely diagnosis?
A) Erythema infectiosum (fifth disease)
B) Gianotti-Crosti syndrome
C) Hemolytic uremic syndrome
D) Henoch-Schönlein purpura
E) Thrombotic thrombocytopenic purpura

A

ANSWER: D
Henoch-Schönlein purpura is an IgA vasculitis that is usually diagnosed clinically. It presents as palpable purpura of the lower extremities without thrombocytopenia or coagulopathy. It is often associated with arthralgias and arthritis, abdominal pain, and renal dysfunction. It is self-limited and treatment is supportive only. Erythema infectiosum (fifth disease) can be identified by an erythematous rash on the cheeks and a lacy reticular rash on the extremities. Gianotti-Crosti syndrome is a sudden papular or papulovesicular eruption on the extensor surfaces of the arms, legs, buttocks, and face, and it is not purpuric. Hemolytic uremic syndrome presents with the classic triad of hemolytic anemia, thrombocytopenia, and kidney injury. Thrombotic thrombocytopenic purpura is rare in the pediatric age group.

How well did you know this?
1
Not at all
2
3
4
5
Perfectly
96
Q

LDCT start at what age

A

50-80

How well did you know this?
1
Not at all
2
3
4
5
Perfectly
97
Q

A 32-year-old female presents with heat intolerance, excessive weight loss, and anxiety. She gave birth 6 months ago and recently stopped breastfeeding. On examination her thyroid gland is slightly diffusely enlarged and nontender. Laboratory studies reveal a decreased TSH level and elevated free T3 and T4 levels. You suspect that she has postpartum thyroiditis.
Which one of the following tests would be most useful to confirm the diagnosis?
A) Radioactive iodine uptake
B) Thyroid peroxidase antibody levels
C) Thyroid ultrasonography
D) Thyrotropin receptor antibody levels

A

ANSWER: A
Postpartum thyroiditis is defined as a transient or persistent thyroid dysfunction that occurs within 1 year of childbirth, miscarriage, or medical abortion. Release of preformed thyroid hormone in the bloodstream initially results in hyperthyroidism. During the hyperthyroid phase, radioactive iodine uptake will be low, which can help to confirm the diagnosis. Pregnancy and breastfeeding are contraindications to radionuclide imaging. Thyroid peroxidase antibody levels are elevated with chronic autoimmune thyroiditis (Hashimoto’s thyroiditis), and patients present with symptoms of hypothyroidism. The Endocrine Society and American Association of Clinical Endocrinologists do not recommend routine thyroid ultrasonography in patients with abnormal thyroid function tests if there is no palpable abnormality of the thyroid gland. Thyrotropin receptor antibody levels are positive in Graves disease.

How well did you know this?
1
Not at all
2
3
4
5
Perfectly
98
Q

A 13-year-old female is brought to your office by her father for evaluation of left knee pain. It has been slowly worsening over the past several weeks and she does not recall any inciting injury or event. She plays soccer and participates in practice 3 days a week with games on the weekends. She is finding it increasingly difficult to participate fully due to pain. She notes some swelling and tenderness just below her kneecap. She has tried icing the area after activity and has recently started taking ibuprofen with moderate relief of the pain.
On examination you note prominence of the tibial tubercle with tenderness to palpation. The remainder of the knee examination is unremarkable.
Which one of the following is indicated at this time to further assess this condition?
A) No imaging
B) Radiography
C) Ultrasonography
D) CT
E) MRI

A

ANSWER: A
This patient has Osgood-Schlatter disease, a common cause of knee pain in active children with immature skeletons. It occurs as a result of abnormal development, injury, or overuse of the growth plate and the surrounding ossification centers. Osteochondrosis is a more general term for this condition, which can occur at growth plates around other joints, including the hip, foot, elbow, and back. In Osgood-Schlatter disease repetitive traction of the patellar tendon on the tibial tubercle ossification center leads to inflammation and pain. Imaging is not required to make the diagnosis when patients present with typical symptoms and physical examination findings. Radiographs may be obtained if there is uncertainty about the diagnosis. Radiographic findings in Osgood-Schlatter disease include soft-tissue swelling and fragmentation of the tibial tubercle.
This condition is self-limited and treatment consists of activity modification and the use of acetaminophen or NSAIDs. An orthopedic referral is indicated if symptoms persist in a patient with a mature skeleton.

How well did you know this?
1
Not at all
2
3
4
5
Perfectly
99
Q

A 75-year-old male nursing home resident is brought to the emergency department with a cough and fever. His past medical history is significant for coronary artery disease, COPD, hypertension, and osteoarthritis. On examination he has a blood pressure of 145/90 mm Hg, a pulse rate of 84 beats/min, and an oxygen saturation of 89% on room air. A physical examination is remarkable for mildly labored breathing and crackles in his left lower lung field. A chest radiograph confirms left lower lobe pneumonia. He is admitted to the hospital for intravenous antibiotics.
Which one of the following would be the most appropriate antibiotic treatment?
A) Cefdinir only
B) Piperacillin/tazobactam (Zosyn) only
C) Ceftriaxone and azithromycin (Zithromax)
D) Cefixime (Suprax) and vancomycin (Vancocin)
E) Piperacillin/tazobactam, vancomycin, and ciprofloxacin (Cipro)

A

ANSWER: C
Current recommendations state that nursing home–acquired pneumonia should be treated as community-acquired pneumonia unless patients have severe illness, chronic wounds, foreign bodies in the airway, a history of antibiotic use in the last 90 days or recent hospitalization, colonization with multidrug-resistant pathogens, or very low functional status, or reside in a facility with a high prevalence of multidrug-resistant pathogens. Community-acquired pneumonia should be treated with either a respiratory fluoroquinolone or an advanced macrolide plus a -lactam antibiotic. Doxycycline could also be used in place of the macrolide.

How well did you know this?
1
Not at all
2
3
4
5
Perfectly
100
Q

A 56-year-old male with a history of hepatitis C cirrhosis is admitted to the hospital with gastrointestinal (GI) bleeding. The patient has been stable, taking only furosemide (Lasix) and spironolactone (Aldactone). Upper GI endoscopy confirms variceal bleeding and the gastroenterologist performs appropriate variceal banding.
A nurse calls you because laboratory studies that were ordered in the emergency department reveal a serum ammonia level of 120 g/dL (N 39–90). The patient has no signs of confusion, insomnia, or decreased mental alertness. A physical examination reveals mild ascites but no other abnormalities.
Which one of the following would be most appropriate for addressing the elevated ammonia level?
A) No additional treatment
B) Lactulose (Kristalose)
C) Methotrexate
D) Neomycin
E) Prednisone

A

ANSWER: A
Elevated ammonia levels may occur with urea cycle disorders, portosystemic shunting, urinary tract infection from urease-producing organisms, gastrointestinal bleeding, shock, renal disease, heavy exercise, smoking, parenteral nutrition, salicylate intoxication, use of medications including diuretics, and alcohol use. In patients with chronic liver disease, hepatic encephalopathy is diagnosed based on the overall clinical presentation and not on ammonia levels. A normal ammonia level does not exclude the diagnosis of hepatic encephalopathy, nor does an elevated ammonia level establish the diagnosis. This patient had an elevated serum ammonia level that was found incidentally during his hospital admission for gastrointestinal bleeding. Because there was no clinically significant encephalopathy, treatment based on ammonia levels is not indicated. The patient’s elevated ammonia level was probably from diuretic use. Lactulose, methotrexate, neomycin, or prednisone would not be appropriate.

How well did you know this?
1
Not at all
2
3
4
5
Perfectly
101
Q

A 37-year-old female presents with a 3-day history of left thumb pain. She first noted diffuse pain in her left thumb after a fall while skiing. She does not recall the mechanism of injury. The pain is greatest at the medial metacarpophalangeal joint, but there is no mass or instability. A radiograph does not show any fractures.
An examination reveals mild laxity in her ulnar collateral ligament with 30° of joint opening when abduction stress is applied to the distal thumb while stabilizing the metacarpal. The right side has only 15° of joint opening.
Which one of the following would be most appropriate at this point?
A) Reassurance and follow-up if symptoms do not improve
B) Anti-inflammatory medication for 7–10 days
C) Immobilization of the left thumb in a thumb spica cast or brace for 6 weeks
D) Immediate referral to an orthopedic surgeon for surgical repair

A

ANSWER: C
Ulnar collateral ligament (UCL) disruption, or “skier’s thumb,” should be suspected in traumatic thumb injuries. It is important to recognize and treat this injury because it can lead to joint instability and a weak pincer grip if untreated. Initial treatment of UCL disruption involves immobilization of the affected thumb in a thumb spica cast or brace for 6 weeks. In the absence of an avulsion fracture, indications for referral to an orthopedic surgeon would include 35°–40° of joint opening or no end point on stress abduction testing. A Stener lesion (entrapment of the UCL outside of the adductor aponeurosis) would usually present with joint instability and a tender mass and would necessitate an orthopedic referral.

How well did you know this?
1
Not at all
2
3
4
5
Perfectly
102
Q

A 65-year-old female with hypertension, osteoporosis, and GERD presents to your office for a well woman visit. She reports no new symptoms or concerns. A review of laboratory work performed prior to her visit reveals lipid levels at goal, normal glucose and sodium levels, a calcium level of 10.6 mg/dL (N 8.6–10.3), an albumin level of 4.1 g/dL (N 3.6–5.1), and a 25-hydroxyvitamin D level of 35 ng/mL (N 20–50). Her calcium level was 10.5 mg/dL on a basic metabolic panel 6 months ago.
The patient’s medications include hydrochlorothiazide, 12.5 mg daily; lisinopril (Prinivil, Zestril), 10 mg daily; alendronate (Fosamax), 70 mg weekly; omeprazole (Prilosec), 20 mg daily as needed; and vitamin D, 2000 IU daily. The patient’s blood pressure is 110/60 mm Hg. An examination is normal.
In addition to ordering follow-up laboratory studies and scheduling a follow-up visit in 1 month, which one of the following would be most appropriate?
A) Discontinue alendronate
B) Discontinue hydrochlorothiazide
C) Discontinue lisinopril
D) Discontinue omeprazole and begin ranitidine (Zantac)
E) Increase vitamin D to 5000 IU daily

A

ANSWER: B
This patient has hypercalcemia with a normal albumin level. Hydrochlorothiazide can cause drug-induced hypercalcemia. Alendronate, lisinopril, and omeprazole do not cause hypercalcemia. A high vitamin D level can cause hypercalcemia, so increasing vitamin D is not appropriate at this point. A laboratory evaluation can help differentiate between PTH- and non–PTH-mediated hypercalcemia.

How well did you know this?
1
Not at all
2
3
4
5
Perfectly
103
Q

A 48-year-old male is brought to the medical tent of a marathon after he became confused and dizzy in the middle of the race. On examination his rectal temperature is 41.1°C (106.0°F).
Which one of the following should be initiated immediately?
A) Passive cooling in an air-conditioned space
B) Gradual cooling with a cold water spray and a fan
C) Ice packs applied to the groin, neck, and axilla
D) Full-body immersion in an ice bath
E) Cold intravenous fluids

A

ANSWER: D
Heatstroke can be nonexertional from prolonged exposure to a high heat index, or it can be exertional, as in this case. A core temperature >40°C (104°F) is consistent with heatstroke. In treating patients with either clinical variant of heatstroke, cold or ice-water immersion is the most effective treatment and should be initiated as soon as possible, without delaying for transfer to the hospital setting (SOR A). Treatment should continue until the core body temperature is <39°C (102°F). If cold water immersion is not possible other forms of cooling such as cold intravenous fluids, ice packs, cold water immersion of the extremities, and evaporative cooling have been shown to have some benefit. Once the body temperature is decreased patients should be transferred to a hospital for evaluation for known complications of heatstroke, including coagulopathy, renal and hepatic dysfunction, hypoglycemia, electrolyte disturbance, and rhabdomyolysis.

How well did you know this?
1
Not at all
2
3
4
5
Perfectly
104
Q

basophilic stippling

A

anemia associated with lead toxicity

How well did you know this?
1
Not at all
2
3
4
5
Perfectly
105
Q

Schistocytes

A

hemolytic anemia.

How well did you know this?
1
Not at all
2
3
4
5
Perfectly
106
Q

You are administering a mental status examination to a 92-year-old male with suspected dementia. You give the patient a pencil and ask him to show how it is used. He gives you a bewildered look and eventually puts the pencil in his mouth and demonstrates using it as a toothbrush.
This task assesses
A) executive functioning
B) gnosia
C) orientation
D) praxis
E) visuospatial proficiency

A

ANSWER: D
Praxis is the ability to carry out intentional motor acts and is commonly assessed by giving the patient a common object such as a hairbrush or pencil and asking the patient to show how it is used. A patient unable to carry out such motor acts is referred to as having apraxia (SOR C). Several other common components of the cognitive assessment will be impaired in persons with dementia. Executive functioning is the ordering and implementation of cognitive functions necessary to engage in appropriate behavior and is often assessed by asking a patient to draw a clock with the hands set at a certain time.
Gnosia is the ability to name objects and their function and is often assessed by showing a patient a common object such as a pen, watch, or stethoscope and asking whether he or she can identify it and describe how it is used. Orientation is the ability of the patient to recognize his or her place in time and space. Orientation is commonly assessed by asking a patient the date, the current location, his or her name, and his or her place of birth. Visuospatial proficiency is the ability to perceive and manipulate objects and shapes in space. It is often assessed by asking the patient to copy intersecting pentagons or a three-dimensional cube on paper.

How well did you know this?
1
Not at all
2
3
4
5
Perfectly
107
Q

A 58-year-old male with well controlled type 2 diabetes presents for follow-up of ongoing exertional chest pain that has been present for months with no significant change. He has a remote history of myocardial infarction and recently underwent cardiac catheterization, which showed non-obstructing coronary artery disease with a left ventricular ejection fraction of 60%. His current medications include aspirin, 81 mg; atorvastatin (Lipitor), 80 mg; lisinopril (Prinivil, Zestril), 40 mg; metoprolol succinate (Toprol-XL), 100 mg; and metformin (Glucophage), 1000 mg twice daily. Home blood pressure monitoring shows an average blood pressure of 142/92 mm Hg and a pulse rate of 58 beats/min. A physical examination, including a cardiovascular examination, is unremarkable.
Which one of the following medications would be the best addition to his current regimen?
A) Amlodipine (Norvasc), 2.5 mg daily
B) Diltiazem extended release (Cardizem LA), 120 mg daily
C) Isosorbide mononitrate extended release, 30 mg daily
D) Nifedipine (Procardia), 30 mg 3 times daily
E) Ranolazine (Ranexa), 500 mg twice daily

A

ANSWER: A
This patient has a blood pressure that is elevated according to all major current guidelines. Amlodipine, a long-acting dihydropyridine calcium channel blocker (CCB), is the best pharmacologic choice because it will lower blood pressure and treat angina without the risk of heart block. Short-acting CCBs such as non–extended-release nifedipine may cause reflex tachycardia and are not recommended. Nondihydropyridine CCBs such as diltiazem would put this patient at risk for heart block because he is already taking long-acting metoprolol and his heart rate is in the 50s. Long-acting nitrates and ranolazine are options to treat stable angina but would not be effective blood pressure medications.

How well did you know this?
1
Not at all
2
3
4
5
Perfectly
108
Q

An 8-year-old male was brought to your office 7 months ago because of frequent diarrhea and abdominal discomfort that had been present for 11⁄2–2 years. An examination revealed no abnormalities. A CBC, a serum iron level, and a metabolic panel were normal. Serology was consistent with celiac disease and a duodenal biopsy confirmed the diagnosis. The patient’s symptoms resolved after his parents took him to a nutritionist who recommended a gluten-free diet. Today you see the child for preventive care and he remains asymptomatic.
In addition to normal well child care, which one of the following would you recommend?
A) No further testing
B) Gradual reintroduction of gluten into the diet
C) IgG antigliadin antibody
D) IgA tissue transglutaminase antibody
E) A duodenal mucosal biopsy to ensure healing

A

ANSWER: D
Celiac disease affects approximately 1% of the U.S. population and can affect all ages. Individuals with northern European ancestry are most commonly affected. The condition is caused by autoimmunity induced by gluten-containing foods in susceptible individuals. Untreated celiac disease is associated with anemia, malabsorption, osteoporosis, weight loss, and gastrointestinal lymphomas. In children, growth stunting and delayed puberty are also common. With strict adherence to a gluten-free diet most complications from celiac disease are preventable and, in children, growth and development return to normal. The World Gastroenterology Organisation recommends annual monitoring of children and adolescents with celiac disease by anthropometry, pubertal development, and celiac serology. The preferred serology is IgA antibody to tissue transglutaminase (IgA anti-tTG) due to its balance of good performance and low cost. Failure of IgA anti-tTG titers to decrease in 6 months suggests continued ingestion of gluten. Repeat duodenoscopy with a biopsy would be unnecessary and would subject the child to potential harm.

How well did you know this?
1
Not at all
2
3
4
5
Perfectly
109
Q

An unimmunized child is brought to the local health department and diagnosed with measles. All individuals at the day care facility that the child attends were exposed to this child about 48 hours prior to the diagnosis.
Which one of the following should be given the MMR vaccine as postexposure prophylaxis?
A) An otherwise healthy 9-month-old child who is up to date on all age-appropriate immunizations
B) A 5-year-old child with asthma who received a second dose of MMR 2 weeks ago
C) A pregnant 24-year-old day care staff member who received two doses of MMR as a
child
D) A pregnant 36-year-old day care staff member who had one dose of MMR vaccine last
year when she started working at the day care facility
E) A 52-year-old nurse who works part-time at the day care facility and received two doses
of MMR as an adult

A

ANSWER: A
Measles outbreaks are becoming more common and the CDC has outlined who should receive postexposure prophylaxis with the MMR vaccine. To be effective as postexposure prophylaxis the vaccine must be administered within 72 hours of exposure.
Infants <12 months of age are considered to be at high risk for complications from measles and should receive postexposure MMR vaccine, although intramuscular immunoglobulin is also an option. Children who are otherwise fully vaccinated do not need additional prophylaxis. Pregnant women cannot receive the MMR vaccine due to fetal risk, but they should receive intravenous immunoglobulin if they do not have evidence of immunity. Health care workers only need to be given the MMR vaccine as prophylaxis if they did not receive two doses previously.

How well did you know this?
1
Not at all
2
3
4
5
Perfectly
110
Q

A 58-year-old female presents to your office to discuss a new diagnosis of COPD. She has a 40-pack-year smoking history, and she quit using tobacco 18 months ago. Spirometry performed last week showed an FEV1/FVC ratio of 0.62 with an FEV1 that is 75% of predicted. She was first treated for an exacerbation last month and has never required hospitalization for any respiratory illness. You administer a COPD assessment test and she rates herself at 16/40, which is a moderately high score.
Which one of the following inhaled medications would be appropriate to initiate today?
A) Albuterol (Proventil, Ventolin)
B) Fluticasone (Flovent)
C) Fluticasone/salmeterol (Advair)
D) Ipratropium (Atrovent)
E) Tiotropium (Spiriva)

A

ANSWER: E
This patient has airflow obstruction consistent with a diagnosis of COPD, evidenced by an FEV1/FVC ratio <0.7. An evidence-based standard approach to COPD is found in the annual guidelines published by the Global Initiative for Chronic Obstructive Lung Disease (GOLD). According to the 2019 report, this patient’s FEV1 of 75% of predicted puts her in the GOLD grade 2 (moderate) category of airflow limitation. Her symptoms and risk of exacerbations places her in GOLD group B. Patients in this category have symptoms that bother them regularly without having frequent COPD exacerbations. Patients in this category benefit from daily use of long-acting bronchodilators, either long-acting -agonists (LABAs) or long-acting muscarinic agents (LAMAs). Long-acting agents such as tiotropium (a LAMA) or salmeterol (a LABA) are preferred over the short-acting agents ipratropium and albuterol for patients in this category of disease severity. Patients with persistent symptoms while using one of these agents may benefit from a combination of a LABA and a LAMA.

How well did you know this?
1
Not at all
2
3
4
5
Perfectly
111
Q

An otherwise healthy 55-year-old male who is visiting from Arizona presents to your office with a 4-week history of intermittent fevers, night sweats, dry cough, weight loss, and myalgia. The patient has no other recent history of travel.
Of the following, the most likely cause of his symptoms is
A) blastomycosis
B) coccidioidomycosis C) cryptococcosis
D) histoplasmosis E) mucormycosis

A

ANSWER: B
Knowledge of endemic fungi capable of causing infection in otherwise healthy patients can be very helpful in ensuring an appropriate evaluation. Coccidioidomycosis is a common infection in the southwestern United States. In addition to the symptoms in this patient, coccidioidomycosis can also present with a rash such as erythema nodosum. Histoplasmosis is most common in the Midwest and with low-level exposure symptoms are usually mild or absent. Blastomycosis is also present in the Midwest, as well as in the Atlantic and southeastern states. Symptoms include an abrupt onset of fever, chills, pleuritic chest pain, arthralgias, and myalgias. The cough is initially nonproductive but frequently becomes purulent. Cryptococcosis and mucormycosis are more opportunistic infections occurring in immunocompromised hosts.

How well did you know this?
1
Not at all
2
3
4
5
Perfectly
112
Q

A 25-year-old male presents with a 4-month history of crampy abdominal pain, diarrhea, and fatigue. His symptoms began gradually but have become more severe and he is now experiencing rectal bleeding. He says that his abdominal pain seems to temporarily improve after eating. He has smoked five cigarettes per day for the past 8 years. He is surprised to learn that he has lost 7 kg (15 lb) when he is weighed today.
His vital signs include a blood pressure of 116/70 mm Hg, a heart rate of 76 beats/min, a respiratory rate of 12/min, and a temperature of 37.7°C (99.9°F). A physical examination reveals abdominal tenderness and mild distention. An anorectal examination is significant for a perianal fistula. A laboratory evaluation is notable for mild anemia. His kidney and liver function are normal.
Which one of the following is the most likely diagnosis?
A) Celiac disease
B) Chronic pancreatitis
C) Crohn’s disease
D) Irritable bowel syndrome
E) Ulcerative colitis

A

ANSWER: C
Crohn’s disease may present insidiously with diarrhea, abdominal pain, rectal bleeding, fever, weight loss, and fatigue. Red-flag symptoms include perianal lesions, a first degree relative with inflammatory bowel disease, weight loss of 5% of the patient’s usual weight, abdominal pain for more than 3 months, nocturnal diarrhea, fever, the absence of abdominal pain for 30–45 minutes after eating, and the absence of rectal urgency. This patient exhibits symptoms consistent with Crohn’s disease. While anemia is also common in celiac disease, rectal bleeding is not. Chronic pancreatitis does not generally present with improved pain after eating. Irritable bowel syndrome is not associated with fever, rectal bleeding, anemia, or perianal fistulas. Ulcerative colitis is not associated with perianal lesions.

How well did you know this?
1
Not at all
2
3
4
5
Perfectly
113
Q

An 18-month-old female is brought to your office in January for evaluation of a cough and fever. She has no chronic medical conditions. She abruptly developed a barking cough and hoarseness with a low-grade fever 2 days ago. The cough is worse at night. She has been drinking normally but is not interested in eating. On examination she is alert and resists the examination. Her respiratory rate and effort are normal. She has no stridor or wheezing.
Which one of the following would be most appropriate at this point?
A) A nasal swab for influenza testing
B) A chest radiograph
C) A single dose of oral dexamethasone
D) Azithromycin (Zithromax)
E) Oseltamivir (Tamiflu)

A

ANSWER: C
This patient has symptoms consistent with croup, a lower respiratory infection that is common in the winter months in children ages 6 months to 3 years. The diagnosis is clinical and should be suspected in children with a history of a sudden onset of a deep cough, hoarseness, and a low-grade fever. Randomized studies have shown that even with mild croup (an occasional barking cough with no stridor at rest), oral corticosteroids provide some benefit.
A Cochrane review of two randomized trials with a total of 2024 patients found that chest radiographs did not change the outcome of ambulatory children with lower respiratory tract infections. A patient such as this would not need antiviral treatment for influenza.

How well did you know this?
1
Not at all
2
3
4
5
Perfectly
114
Q

34-year-old female at 32 weeks gestation presents with a right-sided, pounding headache that began 8 hours ago and is similar to headaches she has had in the past. She is sensitive to light and sound, and has vomited several times since the onset of pain. She has taken acetaminophen without relief. She takes prenatal vitamins but no other routine medications. On examination her blood pressure is normal.
Which one of the following would be the most appropriate treatment for this patient?
A) Dihydroergotamine
B) Metoclopramide (Reglan)
C) Naproxen
D) Oxycodone (OxyContin)
E) Sumatriptan (Imitrex)

A

ANSWER: B
Metoclopramide and acetaminophen are the only two medications considered safe for abortive migraine treatment during pregnancy (SOR B). The dopamine antagonist antiemetics are considered second-line abortive treatments in the general population. Dihydroergotamine should not be used during pregnancy due to its oxytocic properties and the potential risk of intrauterine growth restriction with its use. NSAIDs are not considered safe during pregnancy, particularly in the first and third trimesters. Opioids are only moderately useful for migraine treatment and should be avoided during pregnancy due to their abuse potential. Triptans are generally considered safe during the first trimester but not in the second and third trimesters. Their use has been associated with uterine atony, increased risk of bleeding during delivery, and increased risk of preterm birth.

How well did you know this?
1
Not at all
2
3
4
5
Perfectly
115
Q

Which one of the following U-100 insulin products has the longest duration of action?
A) Degludec (Tresiba)
B) Glargine (Lantus)
C) Isophane NPH (Humulin N)
D) Lispro (Humalog)
E) Regular (Humulin R)

A

ANSWER: A
Among the available U-100 insulin products, the one with the longest duration of action is ultralong-acting degludec, which lasts 42 hours. The duration of action of rapid-acting lispro is 3–6.5 hours, short-acting regular is 5–8 hours, intermediate-acting isophane is 12–16 hours, and long-acting glargine is 11–24 hours.

How well did you know this?
1
Not at all
2
3
4
5
Perfectly
116
Q

A 9-year-old male with a history of moderate persistent asthma is brought to the emergency department with an acute exacerbation. His symptoms began with a runny nose and nasal congestion 2 days ago. His parents state that he has not had any fevers or chills and he was eating and drinking well until a few hours ago when his breathing started to appear more labored. After multiple treatments with inhaled albuterol (Proventil, Ventolin) and oral prednisolone he remains tachypneic and wheezy.
Which one of the following intravenous medications should be added to the patient’s current treatment to reduce the likelihood of hospital admission?
A) Ketorolac
B) Magnesium sulfate
C) Methylprednisolone
D) Omalizumab (Xolair)
E) Theophylline

A

ANSWER: B
Children who present to the emergency department with an asthma exacerbation and fail to improve adequately with inhaled short-acting bronchodilators and corticosteroids may benefit from treatment with intravenous (IV) magnesium sulfate. A 2016 Cochrane review of three randomized, controlled trials found that this reduced hospital admissions by 68%. Ketorolac is not known to have any benefit in the treatment of asthma. Oral administration of corticosteroids is as effective as IV administration, so there is no reason to give IV methylprednisolone. Omalizumab may be used to prevent exacerbations in patients with severe asthma who do not achieve adequate control with high-dose inhaled corticosteroids, but it has no role in the management of acute exacerbations. IV theophylline is not recommended for asthma exacerbations given its safety profile and poor efficacy compared to short-acting bronchodilators.

How well did you know this?
1
Not at all
2
3
4
5
Perfectly
117
Q

corticotropin stimulation

A

For cushings

How well did you know this?
1
Not at all
2
3
4
5
Perfectly
118
Q

Antitrypsin organs effect

A

Lungs and liver

How well did you know this?
1
Not at all
2
3
4
5
Perfectly
119
Q

Good pasture vasculits affects

A

Lungs and kidneys

How well did you know this?
1
Not at all
2
3
4
5
Perfectly
120
Q

Wellbutrin false positive on drug test for what

A

amphetamines

How well did you know this?
1
Not at all
2
3
4
5
Perfectly
121
Q

False positive for marijuana on drug screen

A

PPI

How well did you know this?
1
Not at all
2
3
4
5
Perfectly
122
Q

False positive for opioids

A

Levofloxacin

How well did you know this?
1
Not at all
2
3
4
5
Perfectly
123
Q

Falso positive for benzo

A

Sertraline

How well did you know this?
1
Not at all
2
3
4
5
Perfectly
124
Q

For patients with atrial fibrillation, which one of the following comorbid conditions represents the strongest indication for thromboprophylaxis with warfarin (Coumadin), rather than a direct oral anticoagulant?
A) A CHA2DS2-VASc score 3
B) End-stage chronic kidney disease
C) A mechanical heart valve
D) Mild mitral stenosis
E) Severe mitral regurgitation

A

ANSWER: C
According to 2019 guidelines from the American College of Cardiology, American Heart Association, and Heart Rhythm Society, patients with nonvalvular atrial fibrillation and an elevated CHA2DS2-VASc score (2 in men and 3 in women) should receive anticoagulation, preferably with a direct-acting oral anticoagulant (DOAC), rather than warfarin (level of evidence A). Recent evidence has shown that DOAC options are not inferior to, and in some studies are superior to, warfarin for preventing strokes and systemic embolic events, with a lower risk of serious bleeding. However, warfarin is still recommended over a DOAC for valvular atrial fibrillation that occurs in the presence of moderate to severe mitral stenosis or a mechanical heart valve. For atrial fibrillation in patients with other forms of valvular heart disease, including mitral regurgitation and mild mitral stenosis, DOAC therapy is preferred over warfarin. For patients with atrial fibrillation and end-stage chronic kidney disease, both apixaban (a direct factor Xa inhibitor) and warfarin are comparable options.

How well did you know this?
1
Not at all
2
3
4
5
Perfectly
125
Q

You are examining a 65-year-old male from Central America with a history of rheumatic valvular disease. Which one of the following is the principal auscultatory finding of aortic regurgitation?
A) An S3 gallop heard best at the cardiac apex with the patient supine
B) A triphasic pericardial friction rub with the patient seated and leaning forward
C) A low-pitched decrescendo diastolic murmur that is loudest at the lower left sternal
border with the patient seated and leaning forward
D) A high-pitched crescendo/decrescendo midsystolic murmur that is loudest at the right
upper sternal border and radiates to the carotid arteries with the patient sitting upright
E) A harsh holosystolic murmur that is loudest at the lower left sternal border and radiates
to the left lateral chest wall with the patient in the left lateral decubitus position

A

ANSWER: C
Aortic regurgitation in an older adult may be due to a congenital bicuspid aortic valve, which often is accompanied by aortic stenosis. Rheumatic aortic valvular disease may also cause aortic regurgitation, which is the most common cause in the developing world but less common in the United States.
The hallmark murmur of aortic regurgitation in either case is a “blowing” decrescendo diastolic murmur along the tract from the aortic valve (upper right sternal border) down to the lower left sternal border, where it is loudest. It is best heard with the patient sitting, leaning forward, and holding his or her breath in expiration.
Both bicuspid aortic valve and rheumatic valve disease may also be associated with aortic stenosis. The typical murmur of aortic stenosis is a mid- to long crescendo/decrescendo systolic murmur, loudest at the right upper sternal border, and often radiating to the carotid arteries. An S3 gallop may also be present in decompensating aortic regurgitation, due to the associated left ventricular dilatation, but this is a secondary finding in later stages.

How well did you know this?
1
Not at all
2
3
4
5
Perfectly
126
Q

A 50-year-old male with no significant past medical history presents with a 5-day history of fever to 101°F, chills, and mild diffuse joint and muscle pains. He also reports a mild headache but has not had any sore throat, rhinorrhea, cough, shortness of breath, nausea, vomiting, or change in bowel habits. He noticed a round red rash (shown below) a few days ago on his leg that has grown in size since then. It is minimally pruritic but not painful. He has no other rashes. He recently traveled to Vermont for his family’s annual fall hiking trip but does not recall any insect bites. He does not take any medications and has no drug allergies. He has a blood pressure of 120/74 mm Hg, a pulse rate of 84 beats/min, and a temperature of 37.8°C (100.0°F). Cardiac, pulmonary, musculoskeletal, and abdominal examinations are normal.
Which one of the following is the most likely diagnosis?
A) Adenovirus
B) Ehrlichiosis
C) Influenza
D) Lyme disease
E) Rocky Mountain spotted fever

A

ANSWER: D
This patient has a classic presentation for Lyme disease. Lyme disease is transmitted by a tick bite, but not all patients remember being bitten by a tick. The classic erythema migrans lesion appears a couple of weeks after the tick bite. The first-line treatment for Lyme disease is either amoxicillin or doxycycline. Macrolides can be used if patients have true allergies to -lactams and doxycycline, but they are less effective. This patient is not exhibiting the respiratory symptoms typically associated with adenovirus. Ehrlichiosis and Rocky Mountain spotted fever typically present with headaches and fevers but not with an erythema migrans rash. Although influenza should be considered in the differential diagnosis, it would not present with an erythema migrans rash.

How well did you know this?
1
Not at all
2
3
4
5
Perfectly
127
Q

One of your patients will turn 65 in 2 weeks and your practice manager routinely encourages scheduling a Welcome to Medicare preventive visit soon after patients’ 65th birthdays. This patient continues to work full time and is currently insured through his employer-sponsored health insurance.
In order to bill for a Welcome to Medicare visit after this patient turns 65, which one of the following is true?
A) The patient must be enrolled in Medicare Part A
B) The patient must be enrolled in Medicare Part B
C) The patient must be enrolled in Medicare Part D
D) The patient must be over age 65 and the specific type of Medicare enrollment is not relevant

A

ANSWER: B
The Welcome to Medicare preventive visit, also known as an Initial Preventive Physical Examination (IPPE), is a one-time service that can be provided within the first year of a patient’s enrollment in Medicare Part B. Medicare Part B covers provider visits and outpatient services such as laboratory testing. Beneficiaries are automatically enrolled in Part A when they apply to Medicare, which provides coverage for hospital-based and hospice care. Because Part A does not typically carry a monthly premium, some working older adults who continue to have insurance through their employer may opt to obtain Part A only, and wait on Part B coverage, which does have a monthly premium. Medicare Part D is prescription drug coverage.

How well did you know this?
1
Not at all
2
3
4
5
Perfectly
128
Q

Elevated potassium give

A

Calcium gluconate

How well did you know this?
1
Not at all
2
3
4
5
Perfectly
129
Q

Ottawa knee get xray base on

A

1) > 55
2) Isolated tenderness of patella
3) Tenderness at head of fibula
4) Inability to flex to 90
5) Inability to bear weight 4 steps

How well did you know this?
1
Not at all
2
3
4
5
Perfectly
130
Q

Routine follow-up blood tests for colorectal cancer survivors should include
A) carcinoembryonic antigen (CEA) levels only
B) liver function tests only
C) CBCs and CEA levels only
D) CBCs and liver function tests only
E) CBCs, CEA levels, and liver function tests

A

ANSWER: A
The Choosing Wisely campaign recommends checking only carcinoembryonic antigen (CEA) levels following curative treatment for colorectal cancer (SOR C). No routine laboratory studies such as a CBC or liver function tests should be ordered for follow-up.

How well did you know this?
1
Not at all
2
3
4
5
Perfectly
131
Q

Coccidioides immitis i

A

present in the desert regions of the U.S. southwest

muscle and joint pain,

How well did you know this?
1
Not at all
2
3
4
5
Perfectly
132
Q

Histoplasma

A

Mississippi and Ohio River valleys of the Midwest and the South.
Muscle and joint pain

How well did you know this?
1
Not at all
2
3
4
5
Perfectly
133
Q

A 57-year-old female with a history of diabetes mellitus, hypertension, and depression sees you for a routine follow-up visit. Her vital signs include a heart rate of 88 beats/min, a blood pressure of 162/84 mm Hg, and a BMI of 32 kg/m2.
The recommended antihypertensive regimen for reducing cardiovascular events in this patient is an ACE inhibitor plus
A) an -blocker
B) an angiotensin receptor blocker
C) a calcium channel blocker
D) a loop diuretic

A

ANSWER: C
The ACCOMPLISH trial demonstrated that an ACE inhibitor (ACEI) in combination with a calcium channel blocker (CCB) reduced both fatal and nonfatal cardiovascular events in patients with diabetes mellitus and hypertension. The benefit of an ACEI and a CCB for reducing cardiovascular events was greater than that of an ACEI and a thiazide diuretic. Evidence has shown that combination therapy for most patients should include a CCB, an ACEI or angiotensin receptor blocker (ARB), or a thiazide diuretic (SOR A). The American College of Cardiology/American Heart Association guidelines recommend against centrally acting medications such as -blockers for first-line therapy. Combining ACEIs and ARBs is not recommended, as the risk of side effects such as hyperkalemia outweighs the benefits. Loop diuretics are not considered first-line antihypertensive agents.

How well did you know this?
1
Not at all
2
3
4
5
Perfectly
134
Q

An elderly homeless male is brought to the emergency department. He is clearly hypothermic due to cold exposure and has superficial frostbite of his extremities. He is still conscious and shivering.
In addition to rewarming him, which one of the following should you administer?
A) Acetazolamide (Diamox Sequels)
B) Amitriptyline
C) Ceftriaxone
D) Ibuprofen
E) tPA

A

ANSWER: D
Frostbite is a freezing injury that occurs when initial cooling causes vasoconstriction and localized ischemia. Continued cold exposure leads to ice crystal formation, which causes cellular lysis, electrolyte abnormalities, and microvascular occlusion. Rewarming creates an inflammatory response. Ibuprofen is the most appropriate agent for the treatment of frostbite until the wounds heal or surgery is performed (SOR C). Acetazolamide can cause frostbite at high altitudes. Amitriptyline is used to treat the pain of immersion foot (also called trench foot), which is a nonfreezing injury that happens when the foot is exposed to prolonged wet conditions above 0°C (32°F). Antibiotics are indicated if open or dirty wounds are present (SOR B). tPA has a role in treating patients with frostbite, but it is used only to decrease the risk of amputation when rewarming patients with grade 3, grade 4, or deep frostbite (SOR B).

How well did you know this?
1
Not at all
2
3
4
5
Perfectly
135
Q

positive anti–thyrotropin-receptor (thyroid-stimulating immunoglobulin) antibody

A

Graves disease

How well did you know this?
1
Not at all
2
3
4
5
Perfectly
136
Q

Graves disease

A

positive anti–thyrotropin-receptor (thyroid-stimulating immunoglobulin) antibody

How well did you know this?
1
Not at all
2
3
4
5
Perfectly
137
Q

Medication that can elevated calcium

A

Lithium

How well did you know this?
1
Not at all
2
3
4
5
Perfectly
138
Q

What medication to not use if you have gout

A

Thiazides (chlorthalidone)

How well did you know this?
1
Not at all
2
3
4
5
Perfectly
139
Q

aphthous stomatitis

A

canker sores

How well did you know this?
1
Not at all
2
3
4
5
Perfectly
140
Q

Dose of ceftriaxone for gonorrhea

A

500 mg IM (< 150 kg)

How well did you know this?
1
Not at all
2
3
4
5
Perfectly
141
Q

Testing for ulcerative colitis

A

Fecal calprotectin in children rules out UC

How well did you know this?
1
Not at all
2
3
4
5
Perfectly
142
Q

Diagnosis of alcoholic hepatitis

A

Acute onset jaundice

How well did you know this?
1
Not at all
2
3
4
5
Perfectly
143
Q

Medication to stop drinking

A

Acamprosate and baclofen

How well did you know this?
1
Not at all
2
3
4
5
Perfectly
144
Q

Tx hereditary hemochromoatosis

A

Lifelong phlebotomy with goal to keep ferritin level around 50 ng per mL

How well did you know this?
1
Not at all
2
3
4
5
Perfectly
145
Q

Medication for PAD

A

Cilostazol

Improved initial and absolute walking distances

How well did you know this?
1
Not at all
2
3
4
5
Perfectly
146
Q

Geographic tongue associated with

A

Benign migratory glossitis

associated with psoriasis

How well did you know this?
1
Not at all
2
3
4
5
Perfectly
147
Q

VTE anticoagulation

A

VTE include PE and DVT

3 months, then start aspirin

Unprovoked VTE should continue anticoagulation indefinitely

How well did you know this?
1
Not at all
2
3
4
5
Perfectly
148
Q

Pregnancy and VTE prophylaxis

A

Postpartum prophylaxis for 6 weeks

How well did you know this?
1
Not at all
2
3
4
5
Perfectly
149
Q

Chlamydia tx

A

Doxycycline 100 mg 2x daily for 7 days

How well did you know this?
1
Not at all
2
3
4
5
Perfectly
150
Q

REM sleep behavior disorders are linked to

A

Antidepressant use and early indicator of Parkinson disease, Lewy body dementia and multiple system atrophy

How well did you know this?
1
Not at all
2
3
4
5
Perfectly
151
Q

Medication that can cause RLS

A

Dopamine antagonists (metoclopramide (Reglan), prochlorperazine (Compazine), droperidol (Inapsine), and promethazine (Phenergan))
Antipsychotics
Serotonergic antidepressants (SSRI)

How well did you know this?
1
Not at all
2
3
4
5
Perfectly
152
Q

Ulcerative colitis seen with

A

Salmonella or Campylobacter infection

How well did you know this?
1
Not at all
2
3
4
5
Perfectly
153
Q

First line therapy for UC

A

Biologics

How well did you know this?
1
Not at all
2
3
4
5
Perfectly
154
Q

Tx for severe alcoholic hepatits

A

Corticosteroids

How well did you know this?
1
Not at all
2
3
4
5
Perfectly
155
Q

Tests for alcohol hepatitis

A

Maddrey Discriminant function tool (at least 32)

Model for end stage liver disease (> 21)

How well did you know this?
1
Not at all
2
3
4
5
Perfectly
156
Q

Tx psorasis rash

A

Triamcinolone acetonide 0.1% cream

How well did you know this?
1
Not at all
2
3
4
5
Perfectly
157
Q

Constipation maintenance therapy

A

for at least 2 months

How well did you know this?
1
Not at all
2
3
4
5
Perfectly
158
Q

Leading cause of nonsmoking lung cancer

A

Radon exposure

How well did you know this?
1
Not at all
2
3
4
5
Perfectly
159
Q

Tx NSCLC

A

Immunotherapy PD-L1 (programmed death ligand 1)

How well did you know this?
1
Not at all
2
3
4
5
Perfectly
160
Q

SCLC tx

A

etoposide (etopopphos) plus cisplatin chemotherapy

How well did you know this?
1
Not at all
2
3
4
5
Perfectly
161
Q

Epilepsy criteria

A

2 unprovoked seizures occurring at least 24 hours apart

How well did you know this?
1
Not at all
2
3
4
5
Perfectly
162
Q

Medications that can increase risk of seizure

A

Wellbutrin
Benadryl (diphenhydramine)
Tramadol

How well did you know this?
1
Not at all
2
3
4
5
Perfectly
163
Q

HIV preexposure prophylaxis (PrEP)

A

emtricitabine and tenofovir alafenamide

How well did you know this?
1
Not at all
2
3
4
5
Perfectly
164
Q

Expediated partner therapy for gonorrhea

A

Cefixime 800 mg

How well did you know this?
1
Not at all
2
3
4
5
Perfectly
165
Q

Rapidly enlarging painful nodules and weight loss

A

Diffuse large B cell lymphoma

How well did you know this?
1
Not at all
2
3
4
5
Perfectly
166
Q

Blistering skin lesions, itchy, rapidly progressive

A

Bullous pemphigoid

How well did you know this?
1
Not at all
2
3
4
5
Perfectly
167
Q

Purple reddish blue lesions to legs, genitals, face and oral mucosa

A

Kaposi sarcoma

How well did you know this?
1
Not at all
2
3
4
5
Perfectly
168
Q

Boutonniere deformity aka

A

Central slip injury, FOced flexion of extended finger at PIP joint

How well did you know this?
1
Not at all
2
3
4
5
Perfectly
169
Q

Forced extension of partially flexed finger, inability to flex finger

A

Jersey finger

How well did you know this?
1
Not at all
2
3
4
5
Perfectly
170
Q

Jersey finger

A

Forced extension of partially flexed finger, inability to flex finger

How well did you know this?
1
Not at all
2
3
4
5
Perfectly
171
Q

Mallet finger

A

Sudden blow to tip of extended finger, drooping or flexed posture at the DIP joint

How well did you know this?
1
Not at all
2
3
4
5
Perfectly
172
Q

Mammogram screening

A

biannual 50-75

How well did you know this?
1
Not at all
2
3
4
5
Perfectly
173
Q

Multiple patchy areas of consolidation of CT chest

A

Interstital lung disease

How well did you know this?
1
Not at all
2
3
4
5
Perfectly
174
Q

Medication for sickle cell

A

hydroxyurea

How well did you know this?
1
Not at all
2
3
4
5
Perfectly
175
Q

Positive ANA, next for lupus testing

A

Low Complement levels

How well did you know this?
1
Not at all
2
3
4
5
Perfectly
176
Q

Anticyclic citrullinated peptide antibodies

A

RA

How well did you know this?
1
Not at all
2
3
4
5
Perfectly
177
Q

Anti-smooth muscle antibodies

A

Autoimmune hepatitis

How well did you know this?
1
Not at all
2
3
4
5
Perfectly
178
Q

Anti-centromere antibodies

A

Systemic sclerosis

How well did you know this?
1
Not at all
2
3
4
5
Perfectly
179
Q

HLA-B27

A

Psoratric arthritis

How well did you know this?
1
Not at all
2
3
4
5
Perfectly
180
Q

What to check for presence of severe bacterial infection

A

Procalcitonin

How well did you know this?
1
Not at all
2
3
4
5
Perfectly
181
Q

Weightloss, Low blood pressure, patches of nonpigmented skin on hands, Low sodium, High potassium

A

Addison’s disease or adrenal insufficency

Patches are vitiligo

Get ACTH suppresion test (low morning cortisol)

How well did you know this?
1
Not at all
2
3
4
5
Perfectly
182
Q

17- hydroxyprogesterone deficiency

A

Congenital adrenal hyperplasia

How well did you know this?
1
Not at all
2
3
4
5
Perfectly
183
Q

Late night salivary cortisol

A

Initial test for cushing syndrome

Dexamethasone confirms disorder

How well did you know this?
1
Not at all
2
3
4
5
Perfectly
184
Q

Antibiotic taht increases INR

A

Trimethoprim/sulfame (Bactrim)

How well did you know this?
1
Not at all
2
3
4
5
Perfectly
185
Q

Coral red on woods lamp, TX

A

Corynebacteria
Red brown amcules
Erythromycin gel

How well did you know this?
1
Not at all
2
3
4
5
Perfectly
186
Q

Female hair loss tx

A

Minoxidil 5% foam indefinitely

How well did you know this?
1
Not at all
2
3
4
5
Perfectly
187
Q

Medication for refractory cough not improving

A

Gabapentin

How well did you know this?
1
Not at all
2
3
4
5
Perfectly
188
Q

Medication that can decrease libido

A

Opioids
SSRi

How well did you know this?
1
Not at all
2
3
4
5
Perfectly
189
Q

Measure vit D levels only in

A

Decreased kidney function
Skeletal disease
Hypercalcemia

How well did you know this?
1
Not at all
2
3
4
5
Perfectly
190
Q

Tx Bullous impetigo

A

Oral trimethoprim/sulfamethoxazole (Bactrim)

How well did you know this?
1
Not at all
2
3
4
5
Perfectly
191
Q

diabetes values

A

Fasting glucose >= 126
2 hour >= 200
Random glucose >= 200

How well did you know this?
1
Not at all
2
3
4
5
Perfectly
192
Q

Tumor lysis syndrome

A

Hyperphosphatemia
Hyperkalemia
Hyperuricemia

Calcium decreased due to binding with free phosphosus

How well did you know this?
1
Not at all
2
3
4
5
Perfectly
193
Q

Thumb and index finger pinch nerve

A

Ulnar

How well did you know this?
1
Not at all
2
3
4
5
Perfectly
194
Q

Radial nerve injury

A

Weakness of wrist extension

How well did you know this?
1
Not at all
2
3
4
5
Perfectly
195
Q

Carpal tunnel nerve

A

Median nerve

How well did you know this?
1
Not at all
2
3
4
5
Perfectly
196
Q

Weakness of supination of the forearm nerve

A

Musculocutanoeus nerve

How well did you know this?
1
Not at all
2
3
4
5
Perfectly
197
Q

Most common complication of gastric sleeve

A

GERD

How well did you know this?
1
Not at all
2
3
4
5
Perfectly
198
Q

A 52-year-old male with known hypertension and hyperlipidemia comes to your office for a follow-up visit. His last visit was more than a year ago. He was unemployed for several months and lost his health insurance. Two months ago he ran out of his medications, which included amlodipine (Norvasc), hydrochlorothiazide, and atorvastatin (Lipitor). He says that he feels fine and has not had any chest pain, changes in vision, difficulty breathing, or lower extremity edema. He is a nonsmoker, and he does not drink alcohol or use illicit drugs. He drinks one cup of coffee daily. He does not take any over-the-counter medications.
On examination his vital signs include a blood pressure of 190/120 mm Hg, a pulse rate of 80 beats/min, and an oxygen saturation of 96% on room air. You recheck his blood pressure after he sits quietly for 30 minutes and there is no significant change. A physical examination, including fundoscopy, is normal.
Which one of the following would be the most appropriate next step?
A) Administer clonidine (Catapres), 0.1 mg orally, and recheck his blood pressure in 30 minutes
B) Administer nifedipine (Procardia), 60 mg orally, and recheck his blood pressure in 30 minutes
C) Order laboratory studies to look for end-organ damage, and tell him to restart his previous medications
D) Admit him to the intensive-care unit for intravenous treatment to lower his blood pressure
E) Call 911 and have him transported to the emergency department

A

ANSWER: C
A hypertensive urgency is defined as a confirmed blood pressure >180/110–120 mm Hg without symptoms or signs of end-organ damage. Patients without symptoms in the setting of severe hypertension rarely have end-organ damage. The most common cause of hypertensive urgency in patients with known hypertension is nonadherence to the use of antihypertensive medications.
When a significantly elevated blood pressure is measured, it should be repeated after 20–30 minutes of quiet rest. Blood pressures should be taken in both arms and a thigh to confirm elevation. One-third of patients with an initially elevated blood pressure will have significantly lower pressure after rest.
Patients who are asymptomatic with persistently elevated blood pressures can be safely treated with oral antihypertensives with close follow-up (SOR C). There is no standard workup for patients with hypertensive urgencies, but common practice includes obtaining a basic metabolic panel, CBC, urinalysis, EKG, and troponin to rule out end-organ damage.
Oral medications to lower blood pressure in a patient with a hypertensive urgency are not indicated unless the patient is symptomatic. Symptoms such as headache or epistaxis warrant acute lowering of blood pressure. Preferred medications include clonidine, labetalol, and captopril, among others. Oral nifedipine is not recommended due to unpredictable blood pressure responses.
Patients with physical or laboratory evidence of end-organ damage should be admitted to the intensive-care unit for intravenous treatment of blood pressure. Without symptoms of end-organ damage there is no need to transport patients to the emergency department, as hypertensive urgencies can be managed with outpatient care.

How well did you know this?
1
Not at all
2
3
4
5
Perfectly
199
Q

A 67-year-old male sees you for a Medicare annual wellness visit. He tells you that his best friend had a stroke and he asks about his risk for stroke. He has no history of stroke, TIA, or neurologic symptoms. He has a family history of cardiovascular disease in his father, who had a myocardial infarction at age 65 and died from a thrombotic stroke at age 71. The patient exercises regularly and has a BMI of 27 kg/m2. His only current medical condition is hyperlipidemia, and his cholesterol level is at goal on rosuvastatin (Crestor), 10 mg daily. He also takes aspirin, 81 mg daily. His blood pressure is 125/78 mm Hg.
Based on U.S. Preventive Services Task Force guidelines, which one of the following would be most appropriate at this time?
A) No additional testing for stroke risk
B) Auscultation for carotid bruits
C) Carotid duplex ultrasonography
D) Magnetic resonance angiography
E) CT angiography of the carotid arteries

A

ANSWER: A
Carotid artery disease affects extracranial carotid arteries and is caused by atherosclerosis. This patient is asymptomatic and has no history of an ischemic stroke, neurologic symptoms referable to the carotid arteries such as amaurosis fugax, or TIA. He has risk factors for cardiovascular disease (age, male sex, hyperlipidemia), but the U.S. Preventive Services Task Force recommends against specific screening for asymptomatic carotid artery stenosis (D recommendation), which has a low prevalence in the general adult population. Stroke is a leading cause of disability and death in the United States, but asymptomatic carotid artery stenosis causes a relatively small proportion of strokes. Auscultation of the carotid arteries for bruits has been found to have poor accuracy for detecting carotid stenosis and is not a reasonable screening approach. Appropriate modalities for detecting carotid stenosis include carotid duplex ultrasonography, magnetic resonance angiography, and computed tomography, but these are not recommended for screening asymptomatic patients.

How well did you know this?
1
Not at all
2
3
4
5
Perfectly
200
Q

A 57-year-old female with diabetes mellitus comes to your office for a routine follow-up. Her current medications include metformin (Glucophage), 1000 mg twice daily. She tells you that she does not exercise regularly and finds it difficult to follow a healthy diet. A hemoglobin A1c today is 7.5%. She does not want to add medications at this time, but she does want to get her hemoglobin A1c below 7%, which is the goal that was previously discussed.
Which one of the following would be the most effective way to improve glucose control for this patient?
A) Discuss the components of a healthy diabetic diet and encourage her to follow it more closely
B) Discuss the importance of regular exercise and encourage her to exercise 30–45 minutes daily
C) Recommend that she check her glucose level 1–3 times daily to help determine what adjustments need to be made
D) Start her on an additional medication
E) Refer her to a diabetes educator for medical nutrition therapy

A

ANSWER: E
Counseling by a diabetic educator or team of educators for medical nutrition therapy lowers hemoglobin A1c by 0.2–0.8 percentage points in patients with type 2 diabetes. While a healthy diabetic diet and regular exercise are important, simply reminding the patient of that fact is not likely to be as successful as comprehensive diabetic education. According to the Society of General Internal Medicine in the Choosing Wisely campaign, patients with type 2 diabetes who are not on insulin therapy should not check their blood glucose level daily. An additional medication will likely decrease the hemoglobin A1c, but this patient has expressed a desire to avoid additional medication, is near goal, and is not currently managing her diabetes with adequate lifestyle changes, so it would be appropriate to respect her wishes and pursue proven interventions that do not require medication.

How well did you know this?
1
Not at all
2
3
4
5
Perfectly
201
Q

A 3-year-old male has developed multiple large areas of bullous impetigo on the legs, buttocks, and trunk after being bitten numerous times by ants. Which one of the following would be the most appropriate treatment?
A) Topical mupirocin ointment
B) Oral azithromycin (Zithromax)
C) Oral tetracycline
D) Oral trimethoprim/sulfamethoxazole (Bactrim)
E) Intramuscular penicillin G benzathine (Bicillin L-A)

A

ANSWER: D
Impetigo may be caused by Streptococcus pyogenes or Staphylococcus aureus, but bullous impetigo is caused exclusively by S. aureus. Oral trimethoprim/sulfamethoxazole is an appropriate treatment for skin infections caused by S. aureus, including susceptible cases of methicillin-resistant S. aureus (MRSA). Topical mupirocin ointment is not practical in very widespread cases or in cases with large bullae. Neither azithromycin nor penicillin is a preferred treatment for impetigo, due to a high rate of treatment failure. Tetracycline should be avoided in children under 8 years of age due to a propensity to cause permanent staining of the teeth.

How well did you know this?
1
Not at all
2
3
4
5
Perfectly
202
Q

A 57-year-old male recently diagnosed with acute lymphoblastic leukemia presents to the emergency department with intractable nausea, vomiting, and myalgias. His first chemotherapy infusion was administered earlier in the day.
Which one of the following electrolyte disturbances would be consistent with tumor lysis syndrome?
A) Hypocalcemia B) Hypokalemia C) Hyponatremia
D) Hypophosphatemia E) Hypouricemia

A

ANSWER: A
Tumor lysis syndrome is a common complication of chemotherapy in hematologic malignancies, such as acute leukemia. Homeostasis is overwhelmed with phosphorus, potassium, calcium, and uric acid released into the bloodstream due to acute cell lysis. Hyperphosphatemia, hyperkalemia, and hyperuricemia are indicative of tumor lysis syndrome. Calcium levels are decreased due to binding with free phosphorus and a depletion of calcium in the bloodstream. Sodium electrolyte levels are not as likely to be affected.

How well did you know this?
1
Not at all
2
3
4
5
Perfectly
203
Q

A 30-year-old male is diagnosed with hereditary hemochromatosis. Periodic therapeutic phlebotomy may be appropriate to prevent
A) chronic liver disease
B) chronic renal disease
C) encephalopathy
D) myelofibrosis
E) Wilson disease

A

ANSWER: A
Hereditary hemochromatosis is a common inherited disorder of iron metabolism. Iron deposits in the liver may lead to chronic liver disease and hepatocellular cancer. Screening for hereditary hemochromatosis includes serum ferritin levels, a family history, and genetic testing. Chronic renal disease, encephalopathy, myelofibrosis, and Wilson disease (disorder of copper transport) do not result from iron overload.

How well did you know this?
1
Not at all
2
3
4
5
Perfectly
204
Q

After a thorough history and examination you determine that a 30-year-old male has an upper respiratory infection with a persistent cough. He is afebrile and is otherwise healthy.
The best treatment for symptomatic relief of his persistent cough would be intranasal
A) antibiotics
B) antihistamines
C) corticosteroids
D) ipratropium (Atrovent)
E) saline

A

ANSWER: D
Upper respiratory tract infections are the most common acute illness in the United States. Symptoms are self-limited and can include nasal congestion, rhinorrhea, sore throat, cough, general malaise, and a low-grade fever. According to a Cochrane review of 10 trials without a meta-analysis, antitussives and expectorants are no more effective than placebo for cough. Intranasal ipratropium is the only medication that improves persistent cough related to upper respiratory infection in adults. Intranasal antibiotics, antihistamines, corticosteroids, and saline would not improve this patient’s cough.

How well did you know this?
1
Not at all
2
3
4
5
Perfectly
205
Q

A 30-year-old female presents for evaluation of chronic abdominal bloating, cramping, diarrhea, and recent weight loss. An abdominal examination is unremarkable, and stool guaiac testing is negative. She requests testing for celiac disease.
Which one of the following would be most likely to cause a false-negative result on serologic testing for celiac disease?
A) A recent increase in dietary wheat consumption
B) Recent use of loperamide (Imodium A-D)
C) A skin rash consistent with dermatitis herpetiformis
D) IgA deficiency
E) Iron deficiency anemia

A

ANSWER: D
Celiac disease is a chronic malabsorptive disorder with an estimated worldwide prevalence of 1.4%. The preferred initial diagnostic test includes a serum IgA transglutaminase-2 (TG2) antibody level, which has a 98% sensitivity and 98% specificity for the diagnosis of celiac disease. False-negative serologic results may occur in patients with an IgA deficiency, which includes up to 3% of patients with celiac disease. Therefore, when a diagnosis of celiac disease is strongly suspected despite a negative IgA TG2 antibody test, a total IgA level should be obtained. Diagnostic confirmation for patients with positive serologic testing is accomplished with endoscopic mucosal biopsy.

Dietary elimination of gluten, not an increase in gluten intake, prior to serologic testing may lead to false-negative results. Recent use of medications, including loperamide, would not be expected to interfere with the accuracy of serologic testing for celiac disease. Dermatitis herpetiformis is a widespread pruritic papulovesicular rash that occurs in less than 10% of patients with celiac disease, although is essentially pathognomonic for the condition, as nearly all patients with this rash have evidence of celiac disease on an intestinal biopsy. Iron deficiency anemia often occurs in patients with celiac disease due to poor iron absorption, although the presence of iron deficiency anemia does not decrease the sensitivity of serologic testing.

How well did you know this?
1
Not at all
2
3
4
5
Perfectly
206
Q

A 20-year-old male presents with a painful second finger after his right hand was stepped on 3 days ago while he was playing basketball. He has marked pain as well as numbness of the distal finger. There are no open wounds and the skin color and nail appear normal other than moderate edema of the fingertip. A radiograph reveals a distal phalanx fracture.
Which one of the following would be the most appropriate next step?
A) Treat symptomatically with ice and an anti-inflammatory medication
B) Tape the first and second digits together until symptoms resolve
C) Splint the affected digit for 2–4 weeks
D) Remove the nail to evaluate for a nail bed injury
E) Refer to a hand surgeon

A

ANSWER: C
Tuft fractures are the most common type of distal phalanx fracture. They rarely require orthopedic referral but often result in up to 6 months of hyperesthesia, pain, and numbness. Treatment involves splinting the affected digit for 2–4 weeks, followed by range of motion and strengthening exercises. Symptomatic treatment may also be involved, but splinting is needed. Taping digits would likely not provide enough stability for the second digit distal phalanx, which extends beyond the first digit. Patients with distal finger injuries need careful physical examination to evaluate for a nail bed injury, but in this case there is no evidence of nail bed damage or laceration.

How well did you know this?
1
Not at all
2
3
4
5
Perfectly
207
Q

A 72-year-old female with a history of type 2 diabetes and hypertension presents to your clinic because of fatigue and depression for the last 5–6 months. She has gained about 7 kg (15 lb) and now has a BMI of 32 kg/m2. A physical examination is otherwise unremarkable. Laboratory studies reveal a TSH level of 8.2 U/mL (N 0.4–4.0). A repeat test 1 month later shows a TSH level of 7.4 U/mL and a free T4 level of 1.6 ng/dL (N 0.8–2.8).
Treatment of this patient with L-thyroxine
A) has no proven benefit
B) can increase grip strength
C) can increase her energy level
D) can help improve depression symptoms
E) can reduce her BMI

A

ANSWER: A
Subclinical hypothyroidism (SCH) is defined as an elevation in TSH level with a normal free T4 level. It is relatively common in adults over the age of 65, with a prevalence of 20%. The TRUST (Thyroid Hormone Replacement for Subclinical Hypothyroidism) trial and subsequent meta-analyses of randomized, controlled trials demonstrate that there is no benefit in treating SCH. Symptoms such as muscle strength, fatigue or tiredness, depression, and BMI do not improve with L-thyroxine treatment (SOR A), and up to 60% of cases resolve within 5 years without intervention in older adults.
Appropriate management of an elevated TSH level includes repeat testing in 1–3 months along with a free T4 level. If SCH is diagnosed, levels should be monitored yearly. Only 2%–4% of patients with SCH develop overt hypothyroidism.

How well did you know this?
1
Not at all
2
3
4
5
Perfectly
208
Q

A 72-year-old male with a history of hypertension, heart failure, and chronic kidney disease sees you for evaluation of gradually worsening lumbar pain. The pain worsens with walking but improves when he sits. He says that the pain radiates to the buttocks and down the right leg, especially with activity. He has not had any fevers, chills, or new urinary symptoms. MRI indicates severe degenerative changes resulting in moderate to severe canal stenosis at the L4-L5 level.
Which one of the following would be most appropriate at this point?
A) Oral diclofenac, 75 mg twice daily
B) Oral pregabalin (Lyrica), 75 mg twice daily
C) Physical therapy
D) Referral to an orthopedic surgeon for elective surgical resolution
E) Referral to a neurosurgeon for urgent surgical resolution

A

ANSWER: C
Lumbar spinal stenosis is a common cause of low back pain in older adults, with varying reports of prevalence but at least 10% in most studies. It is the most common reason for lumbar spinal surgery in the United States. Management of this condition is delayed due to the lack of strong evidence for definitively efficacious nonsurgical approaches, and by high rates of major complications with surgical approaches. Focused physical therapy has the best evidence for initial management. Given this patient’s cardiac and renal comorbidities, chronic use of oral NSAIDs is likely to cause significant harm. While some oral pain medications may be considered, pregabalin has not been found to be any more effective than placebo. Both orthopedic and neurosurgical subspecialists perform lumbar spinal surgeries across the United States. In this case, there is no indication for urgent or emergent surgical management. Given the high complication rate, elective surgical management should be considered only after more conservative options have been found ineffective.

How well did you know this?
1
Not at all
2
3
4
5
Perfectly
209
Q

A 42-year-old female presents to your office to discuss bariatric surgery and its potential complications. Her BMI is 40 kg/m2 and she has hypertension, type 2 diabetes, and osteoarthritis of both knees.
If she opts to have a sleeve gastrectomy, which one of the following complications is most likely in the first 6 months?
A) Cholelithiasis
B) Dumping syndrome
C) GERD
D) Leaking at the surgical site
E) Small bowel obstruction

A

ANSWER: C
Sleeve gastrectomy is currently the most common bariatric procedure. The most common complication is development of GERD, which occurs in 20% of patients. Since this procedure does not produce a malabsorption component, complications such as cholelithiasis, dumping syndrome, and small bowel obstruction are not as likely as with other available procedures. A postoperative leak develops in <2% of cases.
A sleeve gastrectomy involves removing the majority of the greater curvature of the stomach, which creates a tubular stomach. Roux-en-Y gastric bypass and biliopancreatic diversion with duodenal switch both combine volume restriction and nutrient malabsorption.

How well did you know this?
1
Not at all
2
3
4
5
Perfectly
210
Q

A 14-year-old female is brought to your office for a well child check and a sports physical examination. During the substance abuse screening she says that she does not drink alcohol or smoke marijuana or traditional cigarettes, but occasionally uses e-cigarettes with her friends.
Which one of the following statements describing e-cigarettes is true?
A) E-cigarettes do not contain heavy metals
B) The nicotine in e-cigarettes is not addictive
C) Teens who use e-cigarettes are less likely to use marijuana
D) More teens use traditional cigarettes than e-cigarettes
E) E-cigarette use among teens increases the likelihood of cigarette smoking

A

ANSWER: E
E-cigarette use has become quite popular among youth in the United States, with rates surpassing traditional cigarette use in 2014. Among teens who have never smoked, the odds of cigarette smoking are 3–6 times higher in those who have used e-cigarettes within the last year. Nicotine is highly addictive regardless of the source, and heavy metal toxicants are still present when using e-cigarettes, although less than with traditional cigarettes. E-cigarette use is associated with an increased risk of future marijuana use.

How well did you know this?
1
Not at all
2
3
4
5
Perfectly
211
Q

A 52-year-old male with known hypertension and hyperlipidemia comes to your office for a follow-up visit. His last visit was more than a year ago. He was unemployed for several months and lost his health insurance. Two months ago he ran out of his medications, which included amlodipine (Norvasc), hydrochlorothiazide, and atorvastatin (Lipitor). He says that he feels fine and has not had any chest pain, changes in vision, difficulty breathing, or lower extremity edema. He is a nonsmoker, and he does not drink alcohol or use illicit drugs. He drinks one cup of coffee daily. He does not take any over-the-counter medications.
On examination his vital signs include a blood pressure of 190/120 mm Hg, a pulse rate of 80 beats/min, and an oxygen saturation of 96% on room air. You recheck his blood pressure after he sits quietly for 30 minutes and there is no significant change. A physical examination, including fundoscopy, is normal.
Which one of the following would be the most appropriate next step?
A) Administer clonidine (Catapres), 0.1 mg orally, and recheck his blood pressure in 30 minutes
B) Administer nifedipine (Procardia), 60 mg orally, and recheck his blood pressure in 30 minutes
C) Order laboratory studies to look for end-organ damage, and tell him to restart his previous medications
D) Admit him to the intensive-care unit for intravenous treatment to lower his blood pressure
E) Call 911 and have him transported to the emergency department

A

ANSWER: C
A hypertensive urgency is defined as a confirmed blood pressure >180/110–120 mm Hg without symptoms or signs of end-organ damage. Patients without symptoms in the setting of severe hypertension rarely have end-organ damage. The most common cause of hypertensive urgency in patients with known hypertension is nonadherence to the use of antihypertensive medications.
When a significantly elevated blood pressure is measured, it should be repeated after 20–30 minutes of quiet rest. Blood pressures should be taken in both arms and a thigh to confirm elevation. One-third of patients with an initially elevated blood pressure will have significantly lower pressure after rest.
Patients who are asymptomatic with persistently elevated blood pressures can be safely treated with oral antihypertensives with close follow-up (SOR C). There is no standard workup for patients with hypertensive urgencies, but common practice includes obtaining a basic metabolic panel, CBC, urinalysis, EKG, and troponin to rule out end-organ damage.
Oral medications to lower blood pressure in a patient with a hypertensive urgency are not indicated unless the patient is symptomatic. Symptoms such as headache or epistaxis warrant acute lowering of blood pressure. Preferred medications include clonidine, labetalol, and captopril, among others. Oral nifedipine is not recommended due to unpredictable blood pressure responses.
Patients with physical or laboratory evidence of end-organ damage should be admitted to the intensive-care unit for intravenous treatment of blood pressure. Without symptoms of end-organ damage there is no need to transport patients to the emergency department, as hypertensive urgencies can be managed with outpatient care.

How well did you know this?
1
Not at all
2
3
4
5
Perfectly
212
Q

A 28-year-old female presents with a depressed mood and sleep disturbance. She tells you that this has occurred for the past 4 years but only during the winter months. Her past medical history and a physical examination are unremarkable.
Which one of the following interventions has the strongest evidence for preventing recurrence of her condition?
A) Exercise
B) Light therapy
C) Cognitive-behavioral therapy
D) Bupropion (Wellbutrin XL)
E) Fluoxetine (Prozac)

A

ANSWER: D
This patient has seasonal affective disorder (SAD) that has recurred and is likely to continue to recur. Bupropion is the only medication beneficial for prevention of SAD. Light therapy and SSRIs are helpful for treating this disorder but do not prevent it. Exercise and cognitive-behavioral therapy are beneficial adjuncts to treatment but would not prevent recurrence.

How well did you know this?
1
Not at all
2
3
4
5
Perfectly
213
Q

An 85-year-old female with a previous history of diabetes mellitus, hypertension, dementia, and peptic ulcer disease has been in a skilled nursing facility for 4 weeks for rehabilitation after a hip fracture repair secondary to a fall during an ischemic stroke. She is transported to the emergency department today when she develops confusion, shortness of breath, and diaphoresis. Her blood pressure is 172/98 mm Hg, her heart rate is 122 beats/min with an irregular rhythm, and her respiratory rate is 22/min. An EKG demonstrates atrial fibrillation and 0.2 mV ST-segment elevation compared to previous EKGs. Her first troponin level is elevated.
Which one of the following conditions in this patient is considered an ABSOLUTE contraindication to fibrinolytic therapy?
A) Poorly controlled hypertension
B) Peptic ulcer disease
C) Alzheimer’s dementia
D) Hip fracture repair
E) Ischemic stroke

A

ANSWER: E
A history of an ischemic stroke within the past 3 months is an absolute contraindication to fibrinolytic therapy in patients with an ST-elevation myocardial infarction (STEMI), unless the stroke is diagnosed within 41⁄2 hours. Poorly controlled hypertension, dementia, peptic ulcer disease, and major surgery less than 3 weeks before the STEMI are relative contraindications that should be considered on an individual basis.

How well did you know this?
1
Not at all
2
3
4
5
Perfectly
214
Q

You are the team physician for the local high school track team. During a meet one of the athletes inadvertently steps off the edge of the track and inverts her right foot forcefully. She is able to bear weight but with significant pain. She reports pain across her right midfoot. An examination reveals edema over the lateral malleolus and diffuse tenderness, but she does not have any pain with palpation of the navicular, the base of the fifth metatarsal, or the posterior distal lateral and medial malleoli.
Which one of the following would be most appropriate at this time?
A) Radiographs of the right ankle only
B) Radiographs of the right foot only
C) Radiographs of the right foot and ankle
D) Lace-up ankle support, ice, compression, and clinical follow-up
E) Crutches and no weight bearing for 2 weeks, followed by a slow return to weight
bearing

A

ANSWER: D
The Ottawa foot and ankle rules should be used to determine the need for radiographs in foot and ankle injuries. A radiograph of the ankle is recommended if there is pain in the malleolar zone along with the inability to bear weight for at least four steps immediately after the injury and in the physician’s office or emergency department (ED), or tenderness at the tip of the posterior medial or lateral malleolus. A radiograph of the foot is recommended if there is pain in the midfoot zone along with the inability to bear weight for four steps immediately after the injury and in the physician’s office or ED, or tenderness at the base of the fifth metatarsal or over the navicular bone. The Ottawa foot and ankle rules are up to 99% sensitive for detecting fractures, although they are not highly specific. In this case there are no findings that would require radiographs, so treatment for the ankle sprain would be recommended. Compression combined with lace-up ankle support or an air cast, along with cryotherapy, is recommended and can increase mobility. Early mobilization, including weight bearing as tolerated for daily activities, is associated with better long-term outcomes than prolonged rest.

How well did you know this?
1
Not at all
2
3
4
5
Perfectly
215
Q

A 65-year-old male with type 2 diabetes mellitus, hypertension, and obstructive sleep apnea sees you for follow-up. He does not use tobacco or other drugs, and his alcohol consumption consists of two drinks per day. His BMI is 31.0 kg/m2, and he just started a fitness program. The patient tells you that his brother was recently diagnosed with atrial fibrillation and he asks you if this increases his own risk.
Which one of the following factors would increase the risk of atrial fibrillation in this patient?
A) Alcohol use
B) Treatment with lisinopril (Prinivil, Zestril)
C) Treatment with pioglitazone (Actos)
D) Use of a continuous positive airway pressure (CPAP) device
E) Physical stress

A

ANSWER: A
Alcohol consumption greater than one drink/day has been associated with atrial fibrillation. While not recommended to prevent atrial fibrillation, pioglitazone and lisinopril have both been associated with lower rates of atrial fibrillation compared to alternative therapies. Treatment of obstructive sleep apnea, along with a regular fitness regimen, has been associated with a decrease in the recurrence of atrial fibrillation.

How well did you know this?
1
Not at all
2
3
4
5
Perfectly
216
Q

You see an adult patient who has chronic urticaria and no other known chronic conditions. He continues to experience hives after a 3-month course of daily loratadine (Claritin).
Which one of the following would be the most appropriate addition to his treatment regimen at this time?
A) A short course of oral corticosteroids
B) Cyclosporine
C) Ranitidine (Zantac)
D) Narrow-band UV light treatment

A

ANSWER: C
First- and second-generation H1 antihistamine receptor antagonists are generally considered first-line treatment for chronic urticaria, and approximately 60% of patients experience a satisfactory result. Second-generation options such as loratadine have the added benefit of a lower likelihood of side effects such as drowsiness. For those who fail to achieve the desired result with monotherapy using an H1 antihistamine receptor antagonist, the addition of an H2 antihistamine receptor antagonist such as cimetidine or ranitidine is often beneficial. The tricyclic antidepressant doxepin has strong H1 and H2 antihistamine receptor antagonist effects and has been used as an off-label treatment option in some studies. A short course of oral corticosteroids, narrow-band UV light treatment, or cyclosporine can be used in the management of recalcitrant chronic urticaria, but these are considered second- or third-line adjunctive options.

How well did you know this?
1
Not at all
2
3
4
5
Perfectly
217
Q

A 62-year-old female with diabetes mellitus presents to your office with left lower quadrant pain and guarding. She has a previous history of a shellfish allergy that caused hives and swelling.
Further evaluation of this patient should include which one of the following?
A) Ultrasonography of the abdomen
B) CT of the abdomen and pelvis with oral and intravenous (IV) contrast
C) Oral corticosteroids and antihistamines, then CT of the abdomen and pelvis with oral
and IV contrast
D) Intravenous corticosteroids and antihistamines, then CT of the abdomen and pelvis
with oral and IV contrast
E) Laparotomy

A

ANSWER: B
Evaluation of this patient should include CT of the abdomen and pelvis with oral and intravenous (IV) contrast. There is no reason to inquire about shellfish allergies prior to CT with IV contrast, because premedication is not needed. There is no correlation between shellfish allergies and allergic reactions to contrast. Patients with moderately severe to severe reactions to IV contrast in the past would need pretreatment with corticosteroids.

How well did you know this?
1
Not at all
2
3
4
5
Perfectly
218
Q

A 15-year-old female presents with a 3-month history of intermittent abdominal pain and headaches. She does not have any associated weight loss, fever, nausea, change in bowel habits, or other worrisome features. An examination is unremarkable. She does report being stressed at school and has a PHQ-2 score of 4.
Which one of the following would be most appropriate at this point?
A) Further evaluation for depression
B) Laboratory studies
C) Abdominal imaging
D) Citalopram (Celexa)
E) Fluoxetine (Prozac)

A

ANSWER: A
The U.S. Preventive Services Task Force recommends depression screening for all adolescents 12–18 years of age. Although this patient has abdominal pain, the history and physical examination suggest that depression may be playing a role in her somatic complaints. She had a positive initial depression screen on her PHQ-2. This is a brief screening tool, and a positive result merits further evaluation. The evaluation should include a full PHQ-A or a discussion with a qualified clinician. If the patient meets the criteria for major depressive disorder then she should receive treatment for her depression, which could include medication. Both fluoxetine and citalopram have been approved by the FDA to treat depression in this age group. She could also be referred for psychotherapy. Further laboratory studies and imaging may be appropriate at some point, but the most urgent need is to evaluate her positive depression screen.

How well did you know this?
1
Not at all
2
3
4
5
Perfectly
219
Q

A 2-year-old African-American male with a history of sickle cell disease is brought to your office for a well child check. Which one of the following would be most appropriate for screening at this time?
A) A chest radiograph
B) A DXA scan
C) Abdominal ultrasonography
D) Renal Doppler ultrasonography
E) Transcranial Doppler ultrasonography

A

ANSWER: E
Individuals with sickle cell disease are at increased risk for vascular disease, especially stroke. All sickle cell patients 2–16 years of age should be screened with transcranial Doppler ultrasonography (SOR A). A chest radiograph, abdominal ultrasonography, a DXA scan, and renal Doppler ultrasonography are not recommended for screening patients with sickle cell disease.

How well did you know this?
1
Not at all
2
3
4
5
Perfectly
220
Q

A 32-year-old female requests a physical examination prior to participating in an adult soccer league. Her blood pressure is 118/70 mm Hg and her pulse rate is 68 beats/min. The examination is otherwise normal except for a systolic murmur that intensifies with Valsalva maneuvers. She says that she has recently been experiencing mild exertional dyspnea and moderate chest pain. The chest pain has been atypical and is not necessarily related to exertion. Echocardiography reveals hypertrophic cardiomyopathy.
In addition to referring the patient to a cardiologist, you recommended initiating therapy with
A) amiodarone (Cordarone)
B) amlodipine (Norvasc)
C) furosemide (Lasix)
D) lisinopril (Prinivil, Zestril)
E) metoprolol

A

ANSWER: E
Hypertrophic cardiomyopathy is the most common primary cardiomyopathy, with a prevalence of 1:500 persons. Many patients with hypertrophic cardiomyopathy are asymptomatic and are diagnosed during family screening, by auscultation of a heart murmur, or incidentally after an abnormal result on electrocardiography. On examination physicians may hear a systolic murmur that increases in intensity during Valsalva maneuvers. The main goals of therapy are to decrease exertional dyspnea and chest pain and prevent sudden cardiac death. -Blockers are the initial therapy for patients with symptomatic hypertrophic cardiomyopathy. Nondihydropyridine calcium channel blockers such as verapamil can be used if -blockers are not well tolerated.

How well did you know this?
1
Not at all
2
3
4
5
Perfectly
221
Q

A 62-year-old female who is a new patient requests a thyroid evaluation because she has a history of abnormal thyroid test results. You obtain a copy of her records, which include a TSH level of 0.2 U/mL (N 0.4–4.2) and a free T4 level of 2.0 ng/dL (N 0.8–2.7) from 3 years ago. She reports feeling well and has no other health conditions. She does not take any medications.
A physical examination reveals normal vital signs, a BMI of 23.0 kg/m2, no neck masses, a normal thyroid size, and normal heart sounds. Laboratory studies reveal a TSH level of 0.1 U/mL, a free T4 level of 2.5 ng/dL, and a free T3 level of 3.1 pg/mL (N 2.3–4.2).
Treatment for this condition would be indicated if the patient has an abnormal
A) calcium level
B) DXA scan
C) glucose level
D) lipid level
E) thyroid ultrasonography study

A

ANSWER: B
This patient has subclinical hyperthyroidism as evidenced by her low TSH level with normal free T4 and free T3 levels. Common causes of subclinical hyperthyroidism include Graves disease, autonomous functioning thyroid adenoma, and multinodular toxic goiter. Subclinical hyperthyroidism may progress to overt hyperthyroidism; this is more likely in patients with TSH levels <0.1 U/mL. Even in the absence of overt hyperthyroidism these patients are at higher risk for several health conditions, including atrial fibrillation, heart failure, and osteoporosis. For this reason it is important to assess for these conditions and consider treating the underlying thyroid condition, as well as the complication. The American Thyroid Association recommends treating patients with complications who are either over age 65 or have a TSH level <0.1 U/mL.
Lipid and glucose abnormalities are not known to be related to subclinical hyperthyroidism. Calcium levels may be abnormal in hyperparathyroidism but not hyperthyroidism. Thyroid ultrasonography may be helpful to determine the cause of hyperthyroidism but is not used to help decide when to treat subclinical hyperthyroidism.

How well did you know this?
1
Not at all
2
3
4
5
Perfectly
222
Q

Intensive behavioral intervention has more benefit than other treatment modalities in treating children who have been diagnosed with
A) attention-deficit/hyperactivity disorder
B) autism
C) depression
D) obsessive-compulsive disorder
E) posttraumatic stress disorder

A

ANSWER: B
The only evidence-based treatment that confers significant benefits to children with autism is intensive behavioral interventions, which should be initiated before 3 years of age. Attention-deficit/hyperactivity disorder can be treated with cognitive-behavioral therapy (CBT) but medication is often required. CBT is as effective, if not more effective, than medication for treating anxiety, depression, and trauma-related disorders.

How well did you know this?
1
Not at all
2
3
4
5
Perfectly
223
Q

You are notified by the nurse that a 66-year-old female who was admitted for pain control for her bone metastases is still having breakthrough pain. You gave her 10 mg of immediate-release oxycodone (Roxicodone) 15 minutes ago.
You are hoping to optimize pain control and minimize sedation, so you advise the nurse that the last dose will have its peak effect
A) now
B) 1 hour after it was given
C) 2 hours after it was given
D) 4 hours after it was given

A

ANSWER: B
Most orally administered immediate-release opioids such as morphine, oxycodone, and hydromorphone reach their peak effect at about 1 hour, at which time additional medication can be given if the patient is still in pain. Intravenous opioids reach their peak effect at about 10 minutes and intramuscular and subcutaneous opioids at about 20–30 minutes. Additional medication may therefore be given at those intervals if additional pain relief is required.

How well did you know this?
1
Not at all
2
3
4
5
Perfectly
224
Q

A 62-year-old female has a history of COPD graded as moderate on pulmonary function testing, with an FEV1 of 65% of predicted and a PaO2 of 57 mm Hg. Because her symptoms of dyspnea on exertion and fatigue seem out of proportion to her pulmonary function tests, you order echocardiography, which shows a pulmonary artery systolic pressure of 50 mm Hg, indicating pulmonary hypertension.
Which one of the following would be most effective for decreasing mortality in this situation?
A) Supplemental oxygen
B) An endothelin receptor antagonist such as bosentan (Tracleer)
C) A calcium channel blocker such as nifedipine (Procardia)
D) A phosphodiesterase 5 inhibitor such as sildenafil (Revatio)
E) Referral for pulmonary artery endarterectomy

A

ANSWER: A
The only proven therapy for pulmonary hypertension related to COPD is supplemental oxygen. Supplemental oxygen should be recommended when the PaO2 is <60 mm Hg, because it has been shown to improve mortality by lowering pulmonary arterial pressures. Treatments effective for pulmonary artery hypertension should not be used. Pulmonary vasodilators such as nifedipine, sildenafil, and bosentan may cause a ventilation-perfusion mismatch. Pulmonary endarterectomy may be indicated for pulmonary hypertension caused by chronic thromboembolic disease.

How well did you know this?
1
Not at all
2
3
4
5
Perfectly
225
Q

A 13-year-old male sees you because of pain in his throwing arm. He is a very dedicated football quarterback and has been practicing throws and playing games every day for 2 months. The pain started gradually over the season, and there is no history of acute injury. The patient is right-hand dominant, and on examination he has pain when he raises his right arm above his shoulder. There is also tenderness to palpation of the proximal and lateral humerus.
Which one of the following would be most appropriate at this point?
A) Injection of 10 mL of lidocaine into the subacromial space
B) Plain radiographs of the shoulder
C) Ultrasonography of the supraspinatus muscle
D) MRI of the shoulder
E) A bone scan of the shoulder

A

ANSWER: B
Pain in the shoulder of a young athlete can be caused by many problems, including acromioclavicular strain, biceps tendinitis, glenohumeral instability, and rotator cuff pathology. Although rotator cuff pathologies are the most frequent cause of shoulder pain in adults, they are uncommon in children. Unique to children, however, is a repetitive use injury causing disruption at the proximal growth plate of the humerus. This condition is referred to as Little League shoulder and can be seen on plain radiographs as widening, demineralization, or sclerosis at the growth plate. If the radiograph is normal but suspicion for this condition is high, a bone scan or MRI can be ordered.

How well did you know this?
1
Not at all
2
3
4
5
Perfectly
226
Q

You admit a previously healthy 62-year-old female to the hospital for intractable nausea and vomiting with intravascular volume depletion and hypotension. She lives in rural northern New Mexico. Prior to the onset of her symptoms she had been gardening and cleaning out a chicken coop, where she encountered several rodents. She is febrile and you obtain blood and urine cultures. Two out of four blood culture bottles are positive for gram-negative rods.
Which one of the following is the most likely pathogen?
A) Brucella melitensis
B) Coxiella burnetii
C) Escherichia coli
D) Listeria monocytogenes
E) Yersinia pestis

A

Yersinia pestis is an aerobic fermentative gram-negative rod. It causes a zoonotic infection with humans as the accidental host. The disease is spread by a bite from a flea vector, direct contact with infected tissue, or inhalation of infectious aerosols from a person with pulmonary plague. Plague occurs in two regions in the western United States. One region includes northern New Mexico, northern Arizona, and southern Colorado, and the other region includes California, southern Oregon, and far western Nevada.
Escherichia coli is also an aerobic fermentative gram-negative rod but it generally causes symptoms of gastroenteritis, hemolytic-uremic syndrome, urinary tract infection, intra-abdominal infection, and meningitis. E. coli infection does not have a specific regional distribution. Listeria monocytogenes is a gram-positive rod and causes an influenza-like illness with or without gastroenteritis in adults. Infection occurs through ingestion of contaminated food products such as milk, cheese, processed meats, and raw vegetables. Outbreaks can occur in any geographic distribution.

Coxiella burnetii is a gram-negative intracellular bacterium that causes Q fever. Human infections are associated with contact with infected cattle, sheep, goats, dogs, and cats. Brucella melitensis is a gram-negative coccobacilli that causes brucellosis. Humans are accidental hosts who can develop the disease from contact with tissues rich in erythritol, and from shedding of organisms in milk, urine, and birth products from goats and sheep.

How well did you know this?
1
Not at all
2
3
4
5
Perfectly
227
Q

A 21-year-old female is being evaluated for secondary causes of refractory hypertension. Which one of the following would be most specific for fibromuscular dysplasia?
A) A serum creatinine level
B) An aldosterone:renin ratio
C) 24-hour urine for metanephrines
D) Renal ultrasonography
E) Magnetic resonance angiography of the renal arteries

A

ANSWER: E
In young adults diagnosed with secondary hypertension, evaluation for fibromuscular dysplasia of the renal arteries with MR angiography or CT angiography is indicated (SOR C). The aldosterone/renin ratio is the most sensitive test to diagnose primary hyperaldosteronism. Renal ultrasonography is an indirect test that is not as sensitive or specific for fibromuscular dysplasia. Serum creatinine elevation shows renal involvement but does not identify the cause. Testing for metanephrines is indicated only if a pheochromocytoma is suspected.

How well did you know this?
1
Not at all
2
3
4
5
Perfectly
228
Q

The novel anticoagulants (NOACs) include apixaban (Eliquis), dabigatran (Pradaxa), edoxaban (Savaysa), and rivaroxaban (Xarelto). Which one of the following should be considered when starting or adjusting the dosage of a NOAC?
A) Serum albumin
B) INR
C) Liver enzymes
D) Partial thromboplastin time
E) Renal function

A

ANSWER: E
Because debilitating knee osteoarthritis is a frequent health concern in older adults, physicians should try to identify and possibly modify factors that increase the risk for this condition. Pooled data from many large studies has been sufficient to clearly identify several major risk factors for the development and progression of osteoarthritis of the knees. Overweight and obesity have consistently been found to approximately double the risk for developing knee osteoarthritis. Other factors that have been identified as risk factors include female sex, advancing age (50–75 years of age), and previous trauma. Smoking, inactivity, moderate physical activity, and socioeconomic status have not been shown to affect one’s risk for developing knee osteoarthritis. However, any of these factors in the extreme may be detrimental to joint health in general.

How well did you know this?
1
Not at all
2
3
4
5
Perfectly
229
Q

A 20-year-old football player presents with pain in the proximal fifth metatarsal. The pain was initially present only after practices, but now it causes push-off pain during practice. There is tenderness to palpation. Plain films show no signs of fracture.
Which one of the following would be most appropriate at this point?
A) Start NSAIDs and allow him to continue practicing as tolerated
B) Place him at non–weight bearing for 2 weeks and repeat the plain films
C) Place him in a hard shoe for 3 weeks and then reexamine
D) Order MRI of the foot
E) Order a bone scan of the foot

A

ANSWER: D
A stress fracture in the proximal fifth metatarsal is particularly prone to nonunion and completion of the fracture. Because complete non–weight bearing or surgical intervention may be necessary with this high-risk fracture, MRI is indicated as the most sensitive test. Bone scans are sensitive but nonspecific. Most stress fractures of the metatarsals occur distally and can be managed with a hard shoe initially, with progressive activity as tolerated. NSAIDs are discouraged because of possible effects on fracture healing.

How well did you know this?
1
Not at all
2
3
4
5
Perfectly
230
Q

An 84-year-old female with severe dementia due to Alzheimer’s disease is a resident of a long-term care facility. She has been hitting the staff while receiving personal care and recently had an altercation with another resident. Behavioral interventions have been unsuccessful in managing her symptoms and you suggest to the patient’s family that she be started on low-dose risperidone (Risperdal). They ask about appropriate use of the drug and the potential for side effects.
Which one of the following would be appropriate advice?
A) Extrapyramidal side effects are more common compared to typical antipsychotics
B) Dementia-related psychosis is an FDA-approved indication
C) No monitoring will be necessary
D) The risk of diabetes mellitus is decreased
E) The risk of mortality is increased

A

ANSWER: E
Both typical and atypical antipsychotics increase the risk of mortality in patients with dementia. The FDA has a black box warning on these medications, including risperidone, about the increased risk of mortality in patients with dementia. Risperidone is not approved by the FDA for dementia-related psychosis. The typical antipsychotics are more commonly associated with extrapyramidal side effects. Diabetes mellitus and agranulocytosis are associated with the atypical antipsychotics, including risperidone. Periodic monitoring of serum glucose levels and CBCs is recommended.

How well did you know this?
1
Not at all
2
3
4
5
Perfectly
231
Q

Which one of the following diabetes mellitus medications is MOST likely to cause weight gain?
A) Empagliflozin (Jardiance)
B) Glimepiride (Amaryl)
C) Liraglutide (Victoza)
D) Metformin (Glucophage)
E) Sitagliptin (Januvia)

A

ANSWER: B
Since many patients with diabetes mellitus are obese, the impact of medications on the patient’s weight is important to consider. Treatment with sulfonylureas, including glimepiride, is associated with weight gain. Empagliflozin, liraglutide, metformin, and sitagliptin are not associated with weight gain. In particular, the SGLT2 inhibitors such as empagliflozin and the GLP1 agonists such as liraglutide are associated with clinically significant weight loss.

How well did you know this?
1
Not at all
2
3
4
5
Perfectly
232
Q

A 52-year-old male presents for evaluation of a long-standing facial rash. He reports that the rash is itchy, with flaking and scaling around his mustache and nasolabial folds.
Which one of the following is most likely to be beneficial?
A) Topical antibacterial agents
B) Topical antifungal agents
C) Topical vitamin D analogues
D) Oral zinc supplementation

A

ANSWER: B
Seborrheic dermatitis is commonly seen in the office setting and affects the scalp, eyebrows, nasolabial folds, and anterior chest. The affected skin appears as erythematous patches with white to yellow greasy scales. The etiology is not exactly known, but it is likely that the yeast Malassezia plays a role. Topical antifungals are effective and recommended as first-line agents. Topical low-potency corticosteroids are also effective alone or when used in combination with topical antifungals, but they should be used sparingly due to their adverse effects. The other agents listed have no role in the management of seborrheic dermatitis (SOR A).

How well did you know this?
1
Not at all
2
3
4
5
Perfectly
233
Q

A 52-year-old male sees you for a routine health maintenance examination. He does not take any medications, does not drink alcohol, and is feeling well. A physical examination is normal with the exception of a BMI of 33.2 kg/m2. Routine laboratory studies reveal mild elevations of ALT (SGPT) and AST (SGOT), which remain elevated on repeat testing 2 months later. Hepatitis B and hepatitis C testing are negative.
In addition to ultrasonography of the liver, which one of the following laboratory studies should be ordered to further evaluate this patient?
A) Serum ferritin
B) Serum phosphorus
C) -Fetoprotein
D) Carcinoembryonic antigen (CEA)
E) Serum protein electrophoresis

A

ANSWER: A
Mild asymptomatic elevations (<5 times the upper limit of normal) of ALT and AST are common in primary care. It is estimated that approximately 10% of the U.S. population has elevated transaminase levels. The most common causes of elevated transaminase levels are nonalcoholic fatty liver disease and alcoholic liver disease. The initial evaluation should include assessment for metabolic syndrome and insulin resistance. Waist circumference, blood pressure, a fasting lipid level, and a fasting glucose level or hemoglobin A1c should be obtained. A CBC with platelets and measurement of serum albumin, iron, total iron-binding capacity, and ferritin levels would also be indicated. Iron studies should be ordered to rule out hereditary hemochromatosis, which is an autosomal recessive disease that causes increased iron absorption in the intestines and release by tissue macrophages.

How well did you know this?
1
Not at all
2
3
4
5
Perfectly
234
Q

A 25-year-old gravida 1 para 0 at 24 weeks gestation comes to your office with right lower extremity swelling and pain. Her pregnancy has been uncomplicated so far and her only medication is a prenatal vitamin. She does not have chest pain, shortness of breath, or fever. She recently started feeling the baby move, and an anatomy scan at 20 weeks gestation was normal.
Lower extremity Doppler ultrasonography confirms a right lower extremity deep vein thrombosis (DVT). Laboratory studies including a CBC, coagulation studies, and renal function are normal.
Which one of the following would be the most appropriate initial treatment of her DVT?
A) Oral apixaban (Eliquis)
B) Oral aspirin
C) Oral warfarin (Coumadin)
D) Subcutaneous enoxaparin (Lovenox)
E) Subcutaneous heparin

A

ANSWER: D
Enoxaparin is the most appropriate pharmacologic therapy for anticoagulation in patients who are pregnant. Aspirin is not used as treatment for deep vein thrombosis. Apixaban, warfarin, and heparin either have not been studied for use in pregnancy or there is data indicating potential fetal harm.

How well did you know this?
1
Not at all
2
3
4
5
Perfectly
235
Q

A 75-year-old white male presents to your office following hospitalization for an episode of heart failure. His edema has resolved but he still becomes symptomatic with minor exertion such as walking less than a block. A recent chest radiograph shows cardiomegaly, and echocardiography reveals an ejection fraction of 25%. He is currently taking furosemide (Lasix), 20 mg daily; carvedilol (Coreg), 25 mg twice daily; and lisinopril (Prinivil, Zestril), 20 mg daily. His vital signs include a pulse rate of 60 beats/min, a blood pressure of 110/70 mm Hg, a respiratory rate of 18/min, and a temperature of 37.0°C (98.6°F). No crackles or hepatojugular reflux are noted on auscultation.
Which one of the following would improve this patient’s symptoms and decrease his mortality risk?
A) Digoxin
B) Hydralazine and isosorbide dinitrate (BiDil)
C) Hydrochlorothiazide
D) Spironolactone (Aldactone)

A

ANSWER: D
For patients with left ventricular systolic dysfunction, clinical trials have demonstrated that ACE inhibitors, -blockers, angiotensin receptor blockers, and aldosterone antagonists decrease hospitalizations and all-cause mortality. In African-American patients, all-cause mortality and hospitalizations have been
reduced by hydralazine and isosorbide dinitrate.

How well did you know this?
1
Not at all
2
3
4
5
Perfectly
236
Q

A 67-year-old female with hypertension and atrial fibrillation has been taking warfarin (Coumadin) for the past 10 years. She has been hemodynamically stable for many years with no complications from her atrial fibrillation. She is scheduled to undergo elective bladder sling surgery for urinary incontinence. She does not have any other significant past medical history.
Which one of the following would be the most appropriate perioperative management of her warfarin?
A) Continue warfarin without interruption
B) Discontinue warfarin the day prior to surgery and provide bridge therapy with low
molecular weight heparin
C) Discontinue warfarin 2 days prior to surgery and restart it 2 days postoperatively
unless there is a bleeding complication
D) Discontinue warfarin 2 days prior to surgery and restart it 5 days postoperatively
unless there is a bleeding complication
E) Discontinue warfarin 5 days prior to surgery and restart it 12–24 hours
postoperatively unless there is a bleeding complication

A

ANSWER: E
Perioperative management of chronic anticoagulation requires an assessment of the patient’s risk for thromboembolism and the risk of bleeding from the surgical procedure. High-risk patients include those with mechanical heart valves, a stroke or TIA within the past 3 months, venous thromboembolism within the past 3 months, or coronary stenting within the previous 12 months. High-risk patients require bridging therapy with low molecular weight heparin, while patients at low risk do not require bridging anticoagulation. For low-risk patients, it is recommended that warfarin be discontinued 5 days prior to surgery and restarted 12–24 hours postoperatively. This patient is at low risk for thromboembolism because her CHA2DS2-VASc score is 3. A patient with atrial fibrillation should receive bridging therapy with a CHA2DS2-VASc score 6. This patient’s surgery is associated with a high risk for bleeding, so it is preferable to stop her warfarin 5 days before the operation.

How well did you know this?
1
Not at all
2
3
4
5
Perfectly
237
Q

A 27-year-old female with a past medical history of polycystic ovary syndrome (PCOS) would like to become pregnant. Which one of the following treatments for PCOS is associated with greater live-birth and ovulation rates?
A) Finasteride (Proscar)
B) Letrozole (Femara)
C) Metformin (Glucophage)
D) Spironolactone (Aldactone)

A

ANSWER: B
In a double-blind randomized trial, letrozole was associated with greater live-birth and ovulation rates compared to clomiphene (SOR A). A Cochrane review indicated that metformin does not increase fertility in patients diagnosed with polycystic ovary syndrome (PCOS). Spironolactone and finasteride are both used to treat PCOS in women who do not desire pregnancy.

How well did you know this?
1
Not at all
2
3
4
5
Perfectly
238
Q

A 58-year-old male with a history of tobacco and alcohol abuse presents with the sudden onset of many well circumscribed brown, oval, rough papules with a “stuck-on” appearance on his trunk and proximal extremities. On examination you also note an unintentional 6-kg (13-lb) weight loss over the last 3 months and conjunctival pallor. A review of systems is positive for more frequent stomachaches, decreased appetite, and mild fatigue.
You order a laboratory workup. Which one of the following would be most appropriate at this point?
A) Reassurance that the skin lesions are benign
B) A skin biopsy
C) Referral to a dermatologist
D) CT of the abdomen and pelvis
E) Upper and lower endoscopy

A

ANSWER: E
This patient’s age, risk factors, red-flag symptoms, and other clinical findings indicate the need for endoscopy. The Leser-Trélat sign may be defined as the abrupt onset of multiple seborrheic keratoses, which is an unusual finding that often indicates an underlying malignancy, most commonly an adenocarcinoma of the stomach. CT is not an initial approach for diagnosing a suspected malignancy of the stomach or colon. Further skin evaluation and lifestyle changes, which are indicated, will not address the need for evaluation of weight loss and other abnormal symptoms and findings.

How well did you know this?
1
Not at all
2
3
4
5
Perfectly
239
Q

A 49-year-old male is concerned about lesions on his penis that he has noticed over the past 6 months. He was circumcised as a child and has had the same female sexual partner for 5 years. He does not have any pain, itching, or dysuria. On examination you note multiple reddish-blue papules on the scrotum and a few similar lesions on the shaft of the penis.
The most likely diagnosis is
A) pearly penile papules
B) lichen nitidus
C) lichen sclerosus
D) angiokeratomas
E) squamous cell carcinoma in situ (Bowen’s disease)

A

ANSWER: D
Penile lesions are usually easily diagnosed from clinical findings. Pearly penile papules are common and benign, and present as small, skin-colored, dome-shaped papules in a circular pattern around the coronal sulcus.
Lichen nitidus is benign but uncommon. It presents as discrete, pinhead-sized hypopigmented papules that are asymptomatic. Papules are often found scattered all over the penis, as well as on the abdomen and upper extremities.
Lichen sclerosus is more common and appears as hypopigmented lesions with the texture of cellophane. The lesions are usually located on the glans or prepuce. Atrophy, erosions, and bullae are common, and patients often present with itching, pain, bleeding, and possibly phimosis or obstructed voiding. Lichen sclerosus is associated with squamous cell cancer in a small percentage of cases.
Carcinoma in situ is a premalignant condition that is more common in uncircumcised males over age 60. Lesions are typically beefy red, raised, irregular plaques and can be found on the glans, meatus, frenulum, coronal sulcus, and prepuce. Lesions can be ulcerated or crusted. Pruritus and pain are common. A biopsy is important for making the diagnosis.
Angiokeratomas are lesions that are usually asymptomatic, circumscribed, red or bluish papules. They may appear solely on the glans of the penis, but are also found on the scrotum, abdomen, thighs, groin, and extremities. They may be misdiagnosed as pearly papules or carcinoma. Treatment is not necessary unless the lesions are bleeding or extensive. It is important to realize that angiokeratomas on the shaft of the penis, the suprapubic region, or the sacral region can be associated with Fabry disease. Patients with this finding should be promptly referred.

How well did you know this?
1
Not at all
2
3
4
5
Perfectly
240
Q

You are evaluating a 64-year-old female in the emergency department for pyelonephritis. Her past medical history is negative and she has previously been in good health. The patient appears acutely ill but is oriented. On examination her weight is 100 kg (220 lb), her temperature is 38.9°C (102.0°F), her pulse rate is 110 beats/min, her respiratory rate is 24/min, her blood pressure is 136/72 mm Hg, and her oxygen saturation is 94% on room air. Initial laboratory findings include a venous lactate level of 4.0 mmol/L (N 0.6–1.7).
You decide to start normal saline intravenously. Which one of the following would be the most appropriate initial rate?
A) 100 mL/hr
B) 150 mL/hr
C) 200 mL/hr
D) 3000 mL over 30 minutes
E) 3000 mL over 3 hours

A

ANSWER: E
The Surviving Sepsis Campaign recommends that patients with elevated serum lactate or hypotension receive isotonic intravenous fluids such as normal saline or lactated Ringer’s solution at an initial rate of 30 mL/kg in the first 3 hours using small boluses of approximately 500 mL. A serum lactate value >36 g/dL (4 mmol/L) is correlated with increased severity of illness and poorer outcomes even if hypotension is not yet present. Patients who are hypotensive or whose serum lactate level is >36 g/dL require intravenous fluids or colloid to expand their circulating volume and effectively restore perfusion pressure. The administration of 30 mL/kg of fluid is recommended as a fluid challenge, which should be started as early as possible in the course of septic shock.

How well did you know this?
1
Not at all
2
3
4
5
Perfectly
241
Q

You see a patient who is being treated for opioid use disorder with buprenorphine. Which one of the following can be used as adjuvant treatment to reduce stress-related opioid cravings and increase abstinence?
A) Clonidine (Catapres)
B) Methadone
C) Naloxone
D) Naltrexone (Vivitrol)
E) Nifedipine (Procardia)

A

ANSWER: A
Clonidine (0.1–0.3 mg every 6–8 hours) is a useful adjunct to buprenorphine in the treatment of opioid use disorder to help increase the rates of abstinence and decrease stress-related opioid cravings (SOR C). Naloxone is an opioid antagonist used to treat overdoses. Nifedipine is a common antihypertensive like clonidine but it has no role in the treatment of opioid use disorder. Methadone and naltrexone are used to treat opioid use disorder but neither of these agents would be used simultaneously with buprenorphine.

How well did you know this?
1
Not at all
2
3
4
5
Perfectly
242
Q

In a 60-year-old patient who has not previously received pneumococcal vaccine, which one of the following would be an indication for both 13-valent pneumococcal conjugate vaccine (PCV13, Prevnar 13) and 23-valent pneumococcal polysaccharide vaccine (PPSV23, Pneumovax 23)?
A) Alcoholism
B) Chronic renal failure
C) Cigarette smoking
D) COPD
E) Diabetes mellitus

A

ANSWER: B
Both 13-valent pneumococcal conjugate vaccine (PCV13) and 23-valent pneumococcal polysaccharide vaccine (PPSV23) are recommended for patients with chronic renal failure. Indications for PPSV23 alone in immunocompetent persons younger than 65 include chronic lung disease, diabetes mellitus, chronic heart disease, smoking, and alcoholism.

How well did you know this?
1
Not at all
2
3
4
5
Perfectly
243
Q

A 36-year-old male presents with a 2-day history of painless right-sided facial droop. There are no associated symptoms and his medical history is otherwise unremarkable. An examination is remarkable for an unfurrowed right brow, mouth droop, a sagging right lower eyelid, and a complete inability to move the muscles of the right face and forehead. No other weakness is elicited and no rash is seen.
Which one of the following would be the most appropriate management at this point?
A) Reassurance only
B) Valacyclovir (Valtrex) alone
C) A tapering dose of prednisone alone
D) Valacyclovir and a tapering dose of prednisone
E) Immediate transfer to the emergency department

A

ANSWER: D
Early recognition and effective treatment of acute Bell’s palsy (idiopathic facial paralysis) has been shown to decrease the risk of chronic partial paralysis and pain. Corticosteroids have been shown in a meta-analysis to decrease chronic symptoms, but a Cochrane meta-analysis of 10 studies concluded that antiviral medication along with corticosteroids is significantly more effective than corticosteroids alone. The medications are most effective if started within 72 hours of symptom onset. The same analysis showed that antiviral medications alone were less effective than corticosteroids alone. This patient’s presentation is not consistent with stroke or another emergency. Because supranuclear input to the facial nerves comes from both cerebral hemispheres, strokes and other central pathologies affecting the facial nerves typically spare the forehead, which is not the case in this patient.

How well did you know this?
1
Not at all
2
3
4
5
Perfectly
244
Q

In a patient presenting with truncal obesity, hypertension, type 2 diabetes mellitus, hirsutism, osteopenia, and skin fragility, which one of the following tests is needed to confirm the diagnosis of Cushing syndrome?
A) A dexamethasone suppression test
B) Inferior petrosal sinus sampling
C) Plasma corticotropin
D) Plasma free cortisol
E) Urinary free cortisol

A

ANSWER: E
In a patient presenting with obesity, hypertension, type 2 diabetes mellitus, and hirsutism, who also has thin skin and osteopenia, an elevated 24-hour collection showing high urinary free cortisol confirms the presence of Cushing syndrome. The dexamethasone suppression test, though still commonly used, no longer has a place in the diagnosis and treatment of patients with Cushing syndrome. Corticotropin-dependent and corticotropin-independent causes of Cushing syndrome can be separated by measuring plasma corticotropin. Plasma free cortisol measurements should be obtained only to determine the success or failure of transsphenoidal microadenomectomy or adrenalectomy. Inferior petrosal sinus sampling is used to confirm the source of corticotropin secretion before surgical intervention.

How well did you know this?
1
Not at all
2
3
4
5
Perfectly
245
Q

A 74-year-old female with a long-standing history of coronary artery disease is hospitalized for pneumonia. The patient improves with treatment and is hemodynamically stable. An EKG performed on the third day of hospitalization is shown below.
[Progressive PR prolongation w drop beat]
Which one of the following would be the most appropriate next step?
A) Cardiac rhythm monitoring with no additional treatment
B) Atropine
C) Transcutaneous pacing
D) Transvenous pacing

A

ANSWER: A
Second degree Mobitz type I (Wenckebach) heart block is characterized by an intermittent blockade of electrical impulses from the atria to the ventricles at the level of the atrioventricular node. This prevents generation of a QRS complex. It is characterized by progressive prolongation of the PR interval until a P wave is not followed by a QRS complex. P waves come at regular intervals so PP intervals are normal. Following the missed QRS complex, the PR interval returns to its baseline duration. A pacemaker is not recommended in patients with second degree Mobitz type I heart block who are asymptomatic. It is recommended in symptomatic patients, however, and is guided by electrophysiologic studies.

How well did you know this?
1
Not at all
2
3
4
5
Perfectly
246
Q

A U.S. hospital or birthing center seeking to be certified as “Baby-Friendly” by the Baby-Friendly Hospital Initiative must satisfy which one of the following criteria in addition to meeting other requirements?
A) Demonstrating proper use of an infant car seat to parents prior to discharge
B) Providing no other food or fluids to breastfeeding infants without a medical
indication
C) Providing a pacifier to each baby prior to discharge
D) Providing easy access to a variety of infant formulas
E) Providing on-site daycare facilities for staff

A

ANSWER: B
The Baby-Friendly Hospital Initiative is a global program established by UNICEF and WHO to promote healthy infant feeding and mother-baby bonding. The primary objective is to educate the public on the benefits of breastfeeding and encourage, promote, and facilitate breastfeeding as outlined in the UNICEF/WHO Ten Steps to Successful Breastfeeding chart. These steps promote breastfeeding to the public and provide guidelines for hospitals and birthing centers for the successful initiation and continuation of breastfeeding.
Baby-friendly facilities must have a written breastfeeding policy that is routinely communicated to all health care staff, and all health care staff must be trained in the skills necessary to implement this policy. All pregnant women should be informed about the benefits and management of breastfeeding. Mothers should be helped to initiate breastfeeding within an hour after birth and shown how to breastfeed and to maintain lactation, even if they are separated from their infants. Breastfeeding infants should not be given food other than breast milk, unless medically indicated. If mothers choose to give formula after appropriate education, they should be instructed in proper preparation and use.
Rooming in should be practiced, allowing mothers and infants to remain together 24 hours a day. Mothers should be encouraged to breastfeed on demand. Breastfeeding infants should not be given pacifiers or artificial nipples. Mothers should be referred to breastfeeding support groups on discharge from the hospital. In addition, the hospital must comply with the International Code of Marketing of Breast Milk Substitutes, which requires that formula companies cannot give free gifts to staff or mothers, that breast milk substitutes are not marketed in the maternity unit, and that breast milk supplements and infant feeding supplies are purchased at fair market price.

How well did you know this?
1
Not at all
2
3
4
5
Perfectly
247
Q

According to the recommendations of the American Heart Association, which one of the following patients requires endocarditis prophylaxis?
A) A 10-year-old female with a previous history of Kawasaki disease without valvular dysfunction
B) A 22-year-old female who underwent surgical repair of a ventricular septal defect 1 year ago
C) A 28-year-old female with mitral valve prolapse without regurgitation
D) A 35-year-old female with a history of infectious endocarditis in her 20s that was
related to intravenous drug use
E) A 42-year-old female with a history of rheumatic fever with chorea who has normal
cardiovascular findings

A

ANSWER: D
The American Heart Association and the American College of Cardiology have decreased the number of indications for antibiotic prophylaxis prior to dental procedures. Currently antibiotics are indicated for prosthetic cardiac valves, previous infective endocarditis, unrepaired cyanotic congenital heart disease or a repaired congenital defect with a residual shunt, and a cardiac transplant with valve regurgitation due to a structurally abnormal valve. Amoxicillin, 2 g, is the antibiotic prophylaxis of choice.

How well did you know this?
1
Not at all
2
3
4
5
Perfectly
248
Q

Which one of the following is most commonly associated with oligohydramnios?
A) Anencephaly
B) Esophageal atresia
C) Hydrops
D) Maternal -thalassemia
E) Posterior urethral valves

A

ANSWER: E
Amniotic fluid volume is regulated in part by fetal swallowing, inspiration, and urination. Some malformations of the urinary tract, including renal agenesis and persistent obstruction from posterior urethral valves, lead to oliguria or anuria, and are associated with marked oligohydramnios.
Anencephaly, esophageal atresia, heart failure, and maternal -thalassemia are associated with polyhydramnios. Anencephaly is probably the most common cause of polyhydramnios, via transudation from the exposed meninges; swallowing difficulties and excessive urination may also be contributing factors. Esophageal atresia is almost always associated with polyhydramnios due to an inability to swallow. Intrauterine heart failure, whether due to dysrhythmias, structural defects, or severe anemia, often leads to fetal hydrops, which is associated with polyhydramnios. -Thalassemia, relatively common in Asians, can also cause fetal hydrops and polyhydramnios.

How well did you know this?
1
Not at all
2
3
4
5
Perfectly
249
Q

You see a 47-year-old female for follow-up of a rash. She is a carpenter and was seen 4 days ago for increasing redness and tenderness of her anterior shin after hitting the area with a board 3 days earlier. She was afebrile during that visit and the area was red but not fluctuant. She chose observation rather than treatment at that time. The patient smokes 10 cigarettes daily. Past medical, surgical, and family histories are otherwise negative. Screening for diabetes mellitus was normal last year.
Today the patient’s anterior shin is still tender. She is afebrile and other vital signs are unremarkable. The extent of the infection was drawn 4 days ago with an indelible marker by your partner. Currently the area of redness extends beyond this border. There is no fluctuance or drainage of the wound. The skin appears mildly indurated.
Which one of the following would be best to provide coverage against Streptococcus pyogenes or methicillin-resistant Staphylococcus aureus (MRSA) in this patient?
A) Amoxicillin/clavulanate (Augmentin) and ciprofloxacin (Cipro)
B) Cephalexin and dicloxacillin
C) Dicloxacillin and fosfomycin (Monurol)
D) Doxycycline and trimethoprim/sulfamethoxazole (Bactrim)
E) Trimethoprim/sulfamethoxazole and cephalexin

A

ANSWER: E
Clindamycin or a combination of trimethoprim/sulfamethoxazole (or doxycycline or minocycline) plus cephalexin (or dicloxacillin or amoxicillin/clavulanate) should provide adequate coverage for Streptococcus and methicillin-resistant Staphylococcus aureus (MRSA) for mild to moderate cellulitis.
Doxycycline plus trimethoprim/sulfamethoxazole would provide inadequate coverage for streptococcal bacteria. Cephalexin plus dicloxacillin would provide inadequate coverage for MRSA. The primary indication for ciprofloxacin is treatment of infections with gram-negative rods. Fosfomycin is indicated only for urinary tract infections. Neither is typically used in the treatment of cellulitis

How well did you know this?
1
Not at all
2
3
4
5
Perfectly
250
Q

The mother of a newborn infant is concerned because her baby’s eyes are sometimes crossed. Assuming the intermittent eye crossing persists, which one of the following is the most appropriate age for ophthalmologic referral?
A) 10–14 days
B) 6 months
C) 12 months
D) 24 months

A

ANSWER: B
In many normally developing infants there may be imperfect coordination of eye movements and alignment during the early days and weeks of life, but proper coordination should be achieved by age 4–6 months. Persistent deviation of an eye in an infant requires evaluation.

How well did you know this?
1
Not at all
2
3
4
5
Perfectly
251
Q

A 47-year-old male presents with a 3-day history of fever, chills, low back pain, and urinary frequency. He does not have any nausea, vomiting, or abdominal pain. There is no significant past medical history.
The patient’s vital signs include a temperature of 38.1°C (100.6°F), a pulse rate of 88 beats/min, and a respiratory rate of 14/min. The examination reveals a mildly tender lower abdomen with no guarding or rebound tenderness; no costovertebral angle tenderness; and an enlarged, homogeneous, exquisitely tender prostate.
Which one of the following is indicated to help guide this patient’s treatment?
A) A serum prostate-specific antigen level
B) A culture of prostate secretions after massage of the prostate
C) A culture of midstream voided urine
D) CT of the abdomen and pelvis with intravenous and oral contrast
E) An ultrasound-guided prostate biopsy

A

ANSWER: C
This patient has clinically diagnosable acute bacterial prostatitis, and no further testing, including imaging, is required to establish the diagnosis. Culture of a midstream voided urine may aid in identifying the pathogen, but prostate massage should be avoided because it may increase the risk of bacteremia. A prostate biopsy is not indicated in the presence of acute infection, and a prostate-specific antigen level is not indicated because it is likely to be elevated in the presence of infection.

How well did you know this?
1
Not at all
2
3
4
5
Perfectly
252
Q

A 57-year-old female is admitted to the hospital with lower lobe pneumonia. She has no history of diabetes mellitus. She has not met sepsis criteria but had a blood glucose level of 172 mg/dL in the emergency department.
Insulin should be started if this patient has a persistent blood glucose level greater than or equal to
A) 120 mg/dL
B) 140 mg/dL
C) 160 mg/dL
D) 180 mg/dL

A

ANSWER: D
Insulin therapy should be initiated in hospitalized patients with persistent hyperglycemia, starting at a threshold of 180 mg/dL. Once insulin therapy is started, a target glucose range of 140–180 mg/dL is recommended for the majority of hospitalized patients, regardless of whether they have a critical illness.

How well did you know this?
1
Not at all
2
3
4
5
Perfectly
253
Q

A 69-year-old male presents for follow-up of hypertension treated with spironolactone (Aldactone) and amlodipine (Norvasc). His past medical history is remarkable only for a kidney stone several years ago. A physical examination is unremarkable. A comprehensive metabolic panel is unremarkable except for a calcium level of 12.0 mg/dL (N 8.0–10.0).
Which one of the following is the most likely cause of his elevated calcium level?
A) Excessive ingestion of calcium supplements
B) His current medication regimen
C) Occult malignancy
D) Primary hyperparathyroidism
E) Vitamin D deficiency

A

ANSWER: D
The most common cause of hypercalcemia is hyperparathyroidism. This is seldom symptomatic and is often discovered through routine blood testing. Hypercalcemia due to cancer can be caused by secretion of the parathyroid hormone–related protein and by osteoclastic bone resorption. Other causes of hypercalcemia include thiazide diuretics, lithium, vitamin D intoxication, hyperthyroidism, milk alkali syndrome from excessive calcium antacid ingestion, adrenal insufficiency, and lymphoma.

How well did you know this?
1
Not at all
2
3
4
5
Perfectly
254
Q

A 7-month-old male is admitted to the hospital for respiratory syncytial virus bronchiolitis. His temperature is 37.9°C (100.2°F), pulse rate 160 beats/min, respiratory rate 70/min, and oxygen saturation 92% on room air. Auscultation of the lungs reveals diffuse wheezing and crackles accompanied by nasal flaring and retractions.
Which one of the following interventions would most likely be beneficial?
A) Bronchodilators
B) Corticosteroids
C) Epinephrine
D) Nasogastric fluids
E) Oxygen supplementation to maintain O2 saturation above 95%

A

ANSWER: D
The mainstay of therapy for acute respiratory syncytial virus bronchiolitis is supportive care, and maintaining hydration is important. Infants with respiratory rates >60/min may have poor feeding secondary to difficulty breathing and oral rehydration may increase the risk of aspiration. In these cases, nasogastric or intravenous fluids should be administered. Oxygen saturation of 90% or more on room air is sufficient for infants with bronchiolitis, and using supplemental oxygen to maintain higher oxygen saturations only prolongs hospitalization because of an assumed need for oxygen. Bronchodilators should not be administered to infants with bronchiolitis, because they have not been shown to have any effect on the need for hospitalization, oxygen saturation, or disease resolution. In addition, there is no evidence to support the use of epinephrine or corticosteroids in the inpatient setting.

How well did you know this?
1
Not at all
2
3
4
5
Perfectly
255
Q

A healthy 43-year-old executive presents with problems falling asleep and staying asleep. Doxepin (Silenor) and extended-release melatonin have not helped.
In addition to behavioral interventions, which one of the following would be the most appropriate pharmacologic therapy for this patient’s insomnia at this time?
A) Diphenhydramine (Benadryl)
B) Doxylamine (Unisom)
C) Eszopiclone (Lunesta)
D) Olanzapine (Zyprexa)
E) Zaleplon (Sonata)

A

ANSWER: C
Although behavioral interventions are the mainstay of treatment for insomnia, they often need to be supplemented by pharmacologic therapy. When both doxepin and extended-release melatonin fail to provide benefit, a member of the Z-drug class should be tried next. Among the Z-drugs only eszopiclone provides an early peak onset and a long half-life, with a 1-hour approximate time to peak and a 6-hour half-life. While zaleplon has an equally short time to peak of 1 hour, it also has a 1 hour half-life. Antihistamines, including diphenhydramine and doxylamine, as well as atypical antipsychotics such as olanzapine, are not indicated unless used primarily to treat another condition.

How well did you know this?
1
Not at all
2
3
4
5
Perfectly
256
Q

A 21-year-old gravida 1 para 0 is diagnosed with overt hyperthyroidism early in the first trimester. The most appropriate management at this time is
A) observation only
B) methimazole (Tapazole)
C) propylthiouracil
D) radioactive iodine
E) thyroidectomy

A

ANSWER: C
Overt hyperthyroidism during pregnancy is associated with adverse effects to the mother and fetus, so treatment is required. Since methimazole is associated with birth defects when used in the first trimester, propylthiouracil is preferred. Methimazole should be considered after the first trimester because the risk of congenital anomalies is less than the risk of liver failure associated with propylthiouracil. Surgery and radioactive iodine should only be used if there is a clear indication, and radioactive iodine would not be appropriate during pregnancy.

How well did you know this?
1
Not at all
2
3
4
5
Perfectly
257
Q

A 45-year-old male with a 30-pack-year smoking history reports a chronic cough with a small amount of phlegm production and dyspnea with strenuous exercise. You order spirometry, which shows a pre- and postbronchodilator FEV1/FVC ratio of 0.6 and an FEV1 of 85% of predicted.
Which one of the following agents would be the best initial pharmacologic management?
A) An inhaled corticosteroid
B) A short-acting anticholinergic
C) A long-acting anticholinergic
D) A long-acting 2-agonist
E) Theophylline

A

ANSWER: B
This patient has COPD and is in a risk category of A (low risk, fewer symptoms) based on the Global Initiative for Chronic Obstructive Lung Disease (GOLD) combined assessment of COPD. As a result, either a short-acting anticholinergic or a short-acting 2-agonist should be selected as the initial pharmacologic management. Long-acting 2-agonists or long-acting anticholinergics are indicated for patients with a GOLD combined assessment category of B or worse. Long-acting inhaled corticosteroids are indicated for patients with a GOLD combined assessment category of C or worse. Due to its narrow therapeutic window, modest benefit, and need for monitoring, theophylline is not recommended as an initial agent and should be considered as an alternative only for patients with severe refractory symptoms.

How well did you know this?
1
Not at all
2
3
4
5
Perfectly
258
Q

A 30-year-old white male presents to the emergency department with a 4-day history of fever to 101°F, a sore throat, rhinorrhea, and cough. An examination reveals rhinorrhea and a boggy nasal mucosa, but is otherwise unremarkable. A chest radiograph shows a questionable infiltrate.
Which one of the following would help determine if antibiotic treatment would be appropriate?
A) A C-reactive protein level
B) A procalcitonin level
C) A WBC count with differential
D) An erythrocyte sedimentation rate
E) CT of the chest

A

ANSWER: B
Using a procalcitonin-guided therapy algorithm reduces antibiotic use by 3.47 days without increasing either morbidity or mortality in adults with acute respiratory infections. If the procalcitonin level is <0.10 mg/dL, a bacterial infection is highly unlikely and it is strongly recommended that antibiotics not be prescribed. If the procalcitonin level is 0.10–0.24 mg/dL a bacterial infection is still unlikely and it is recommended that antibiotics not be used. If the level is 0.25–0.50 mg/dL a bacterial infection is likely and antibiotics are recommended. It is strongly recommended that antibiotics be given if the level is >0.50 mg/dL, because a bacterial infection is very likely.

How well did you know this?
1
Not at all
2
3
4
5
Perfectly
259
Q

Additional workup or referral to an endocrinologist for evaluation of precocious puberty would be indicated in which one of the following patients?
A) A 7-year-old female with some pubic hair
B) An 8-year-old female with breast buds
C) An 8-year-old male with some pubic hair and axillary odor
D) An 8-year-old male with penile enlargement
E) A 10-year-old female who has recently begun having menses

A

ANSWER: D
Penile enlargement in an 8-year-old male is a sign of precocious puberty. Isolated sparse pubic and axillary hair growth and axillary odor is referred to as premature adrenarche, and represents high levels of dehydroepiandrosterone rather than activation of the hypothalamic-pituitary-gonadal axis that leads to puberty. The isolated findings of premature adrenarche are generally considered benign. An 8-year-old with breast buds and a 10-year-old with menarche are within the normal range of expected pubertal development. Penile enlargement typically represents full activation of the hypothalamic-pituitary-gonadal axis and warrants endocrinologic evaluation in boys younger than 9 years of age.

How well did you know this?
1
Not at all
2
3
4
5
Perfectly
260
Q

An 80-year-old male sees you for the first time. He is asymptomatic except for some fatigue. His pulse rate is 50 beats/min. An EKG shows a prolonged PR interval.
Which one of the following medications in his current regimen is the most likely explanation for these findings?
A) Donepezil (Aricept)
B) Escitalopram (Lexapro)
C) Lisinopril (Prinivil, Zestril)
D) Memantine (Namenda)
E) Zolpidem (Ambien)

A

ANSWER: A
The 2015 American Geriatrics Society Beers Criteria for potentially inappropriate medication use in older adults 65 years of age states that donepezil use should be avoided in patients with syncope, due to an increased risk of bradycardia (Moderate Evidence Level; Strong Strength of Recommendation). Donepezil is a cholinesterase inhibitor. Due to their cholinergic effect, these medications have a vagotonic effect on the sinoatrial and atrioventricular nodes. This can cause bradycardia or heart block in patients with or without underlying cardiac conduction abnormalities. Syncope has been reported with these medications.
83
Memantine is an N-methyl-D-aspartate receptor antagonist and is not associated with bradycardia. Escitalopram, lisinopril, and zolpidem are also not associated with bradycardia.

How well did you know this?
1
Not at all
2
3
4
5
Perfectly
261
Q

A 14-year-old female is brought to your office for an annual well child check and sports preparticipation physical examination. She says she does a lot of running during basketball practices and games but has trouble controlling her weight. Most of her family is overweight. She does not have any difficulty participating in sports, and has no symptoms such as chest pain, shortness of breath, or headaches. She has no significant past medical history.
On examination the patient’s height is 154 cm (61 in) and she weighs 63 kg (139 lb). Her BMI is 26.4 kg/m2, which places her in the 90th percentile for her age. Her blood pressure is 130/85 mm Hg, which places her between the 95th and 99th percentile for her age, height, and sex. Her chart reveals that her blood pressure was at this level at the last two visits. The physical examination is otherwise normal.
In addition to counseling and support for weight loss, which one of the following would be most appropriate at this point?
A) Informing the patient and her parents that she is prehypertensive and having her return for a blood pressure check in 3 months
B) Plasma renin and catecholamine levels
C) An imaging study of the renal arteries
D) A fasting basic metabolic panel, a lipid profile, and a urinalysis
E) Antihypertensive drug therapy

A

ANSWER: D
In a pediatric patient, blood pressure should be evaluated using comparisons based on age, sex, and height. Although this adolescent’s blood pressure is prehypertensive for an adult according to JNC 8 guidelines, it is stage 1 hypertension (between 95% and 99%) for her age, sex, and height. All pediatric patients with confirmed hypertension should have further evaluation to check for renal dysfunction as well as other cardiac risk factors. Additionally, renal ultrasonography is recommended to evaluate for renal disease and echocardiography to evaluate for end-organ damage that would affect treatment goals. Additional studies, such as plasma renin and catecholamine levels or renovascular imaging, may be indicated in children with abnormalities on initial evaluation that suggest secondary causes of hypertension.
Pharmacologic therapy is usually recommended for pediatric patients with symptomatic hypertension, secondary hypertension, target organ damage, diabetes mellitus, or persistent hypertension in spite of nonpharmacologic treatment. A low-sodium diet may be helpful for decreasing blood pressure, and given this patient’s obesity, intensive counseling about lifestyle changes is appropriate.

How well did you know this?
1
Not at all
2
3
4
5
Perfectly
262
Q

A 20-year-old college student comes to the urgent care clinic with right knee pain and swelling after injuring her knee in a recreational basketball game. Her feet were planted when another player collided with her, causing her upper torso to rotate. She felt immediate pain in the knee and was unable to complete the game.
Which one of the following is the most accurate and appropriate maneuver to detect an anterior cruciate ligament tear?
A) The anterior drawer test
B) The lever sign test
C) The Lachman test
D) The McMurray test
E) The pivot shift test

A

ANSWER: C
The Lachman test is the most accurate test for an anterior cruciate ligament (ACL) tear (SOR A). Accurate testing can lead to appropriate referral and treatment for ACL tears, and early detection can lead to better outcomes. The Lachman test has higher validity based on a sensitivity of 68% for partial ruptures and 96% for complete ruptures. The other two commonly used tests are the anterior drawer test, which has a sensitivity of 38% and a specificity of 81%, and the pivot shift test, which is more technically difficult than the other two tests and has a sensitivity ranging from 24% to 85%. The pivot shift test is effective if done correctly but should not be used alone to diagnose an ACL tear (SOR A).
85
The lever sign test is a newer test that holds promise for detecting ACL tears and is easily performed in the office. However, sensitivity and specificity reports vary (SOR B). The McMurray test is used to detect meniscal tears.

How well did you know this?
1
Not at all
2
3
4
5
Perfectly
263
Q

A 34-year-old female presents with a 1-month history of increasing foot pain. She does not have any significant past musculoskeletal history, and she started a new exercise program 6 weeks ago. She has pain in the lateral side of her heel that is present both with activity and at rest. On examination you note tenderness below the lateral malleolus extending to the midfoot.
Which one of the following is the most likely diagnosis?
A) Calcaneal apophysitis (Sever’s disease)
B) Calcaneal stress fracture
C) Peroneal tendinopathy
D) Plantar fasciitis
E) Tarsal tunnel syndrome

A

ANSWER: C
This patient most likely has peroneal tendinopathy, which is a degeneration of the peroneal tendon that involves pain or tenderness in the lateral calcaneus below the ankle along the path to the base of the fifth metatarsal. Initial treatment options include activity modification, decreasing pressure to the affected area, anti-inflammatory or analgesic medications, and eccentric exercises. Calcaneal apophysitis, or Sever’s disease, is a common growth-related injury that typically affects adolescents between 8 and 12 years of age. Symptoms often present after a growth spurt or starting a new high-impact sport or activity, and common examination findings include tight heel cords and a positive calcaneal squeeze test. A calcaneal stress fracture, which most commonly occurs immediately inferior and posterior to the posterior facet of the subtalar joint, involves pain that intensifies with activity and often worsens to include pain at rest. It typically follows an increase in weight-bearing activity or a switch to running or walking on a hard surface. Plantar fasciitis is characterized by sharp, shooting pain in the arch and medial aspect of the foot that often is worse upon arising and taking the first few steps of the morning. Examination of the foot reveals tenderness at the site and pain with dorsiflexion of the toes. Tarsal tunnel syndrome involves entrapment of the posterior tibial nerve and causes a burning, tingling, or shooting pain and numbness that radiates into the plantar aspect of the foot, often into the toes. The pain associated with tarsal tunnel syndrome typically worsens with activity and is relieved with rest.

How well did you know this?
1
Not at all
2
3
4
5
Perfectly
264
Q

A 30-year-old female with type 2 diabetes and obesity sees you for follow-up. She has experienced several episodes of symptomatic hypoglycemia, and because of this she stopped all of her medications except metformin (Glucophage). Her hemoglobin A1c has increased to 8.4%.
Which one of the following would be the best additional treatment for this patient?
A) Basal insulin (Lantus)
B) Rapid-acting insulin (Humalog)
C) Exenatide (Byetta)
D) Glipizide (Glucotrol)
E) Repaglinide

A

ANSWER: C
Exenatide is a GLP-1 receptor agonist that is not associated with hypoglycemia and can also assist with weight loss, which would be helpful in this patient with obesity. All of the other listed medications, including both types of insulin, sulfonylureas, and meglitinides, can be associated with hypoglycemia. Since this patient’s hemoglobin A1c is only moderately elevated at 8.4%, exenatide is reasonable, although it can be an expensive option. If her hemoglobin A1c was severely elevated, insulin would be indicated, with close monitoring for hypoglycemia.

How well did you know this?
1
Not at all
2
3
4
5
Perfectly
265
Q

Polypharmacy increases the risk of adverse health outcomes. According to the Choosing Wisely campaign, adding to a threshold of how many medications in a patient’s regimen should prompt a thorough review to determine if any of the medications can be discontinued?
A) 3 B) 5 C) 7
D) 10

A

ANSWER: B
Polypharmacy, which is defined as regular use of five or more medications, increases the risk of adverse medical outcomes. Patients who take five or more medications can find it difficult to understand and adhere to the complicated regimens. Risk factors for polypharmacy include having multiple medical conditions that are managed by multiple specialist or subspecialist physicians, residing in a long-term care facility, having poorly updated medical records, and using automated refill services. Inappropriate prescribing of drugs that are not discontinued after their usual effective or recommended period is known as legacy prescribing and can contribute to inappropriate polypharmacy.
According to the Choosing Wisely campaign, any prescriptions beyond a threshold of five medications should trigger a thorough review of the patient’s complete regimen, including over-the-counter medications and dietary supplements, to determine if any of the medications can be discontinued. Tools such as the Beers criteria list and the Medication Appropriateness Index can be used to identify potentially inappropriate medication use, but no single tool or strategy has been determined to be superior. If discontinuation of particular high-risk medications is not possible because of medical conditions, then dose reductions should be considered.

How well did you know this?
1
Not at all
2
3
4
5
Perfectly
266
Q

A 70-year-old female with a history of coronary artery disease, a femorofemoral bypass 3 years ago, and hypertension sees you for a follow-up visit. She has intermittent right arm pain that is worse with exercise. The pain increases with all arm exercises and improves with rest. The patient’s blood pressure is 140/70 mm Hg in the left arm and 120/64 mm Hg in the right arm.
Which one of the following would be the most appropriate next step?
A) Radiographs of the right shoulder
B) Arterial duplex ultrasonography of the upper extremities
C) MR angiography
D) No imaging, and referral to physical therapy

A

ANSWER: B
This patient has peripheral artery disease (PAD) of the right arm. PAD of the upper extremities is characterized by pain with exertion and can cause gangrene and ulceration. It is more common in patients who have had lower extremity occlusive disease. A blood pressure differential of 15 mm Hg between arms suggests stenosis and warrants further testing. Initial testing in symptomatic patients includes arterial duplex ultrasonography of the upper extremities. CT angiography and MR angiography may be appropriate to clarify the diagnosis or plan intervention. Neither radiography nor physical therapy would be appropriate.

How well did you know this?
1
Not at all
2
3
4
5
Perfectly
267
Q

A 19-year-old female presents with a 4-year history of intermittent facial acne. She tells you that her acne has never completely resolved, and it worsens during her menstrual period. She has tried various over-the-counter facial cleansers although she does not recall what they contained. On examination she has scattered open and closed comedones, and pustules on her forehead and around her mouth.
Which one of the following treatments would you recommend?
A) Topical adapalene (Differin)
B) Topical clindamycin (Cleocin T)
C) Oral doxycycline
D) Oral isotretinoin (Absorica)

A

ANSWER: A
This patient has mild inflammatory acne as indicated by her combination of comedones and pustules. She does not have extensive skin involvement and should benefit from the use of a topical agent. Topical retinoids, including adapalene, tretinoin, tazarotene, and trifarotene, are appropriate for the treatment of mild to moderate acne as single agents, although they may be more effective when combined with a topical antibiotic or benzoyl peroxide. Topical antibiotics can lead to bacterial resistance and should not be used as monotherapy. Oral antibiotics are appropriate for the treatment of moderate to severe acne that has failed to respond to topical treatment. Oral isotretinoin is reserved for the treatment of severe nodular acne.

How well did you know this?
1
Not at all
2
3
4
5
Perfectly
268
Q

An otherwise healthy 21-year-old male sees you for follow-up after a hospitalization for pneumonia. This was his second pneumonia infection of the year. He reports a history of multiple sinus infections and upper respiratory infections over the years that were treated with antibiotics on an outpatient basis. Laboratory studies reveal a normal CBC and a decreased IgA level. A trial of pneumococcal polysaccharide vaccine (PPSV23, Pneumovax 23) reveals no measurable response.
This presentation is most consistent with
A) selective IgA deficiency
B) common variable immunodeficiency
C) severe combined immunodeficiency
D) DiGeorge syndrome
E) Wiskott-Aldrich syndrome

A

ANSWER: B
Common variable immunodeficiency (CVID) is the only immunodeficiency condition listed that can present later in life, while severe combined immunodeficiency, DiGeorge syndrome, and Wiskott-Aldrich syndrome typically present prior to 6 months of age. CVID is a condition of impaired humoral immunity and thus should be considered in a patient this age in the setting of recurrent bacterial infections such as sinusitis or pneumonia. The blunted response to a vaccination challenge implies impaired IgG antibody response, which differentiates CVID from a selective IgA deficiency. Because severe combined immunodeficiency is associated with significant abnormalities of both T-cell and B-cell function, it presents very early in life with multiple severe, opportunistic infections, and failure to thrive. DiGeorge syndrome is associated with multiple other physical abnormalities such as cardiac malformations and dysmorphic facial features. Wiskott-Aldrich syndrome is linked to the X chromosome (primarily affecting males) and associated with eczema and thrombocytopenia.

How well did you know this?
1
Not at all
2
3
4
5
Perfectly
269
Q

Which one of the following vitamins is a well established therapy for the treatment of nausea and vomiting in pregnancy?
A) Vitamin A
B) Vitamin B6
C) Vitamin B12
D) Vitamin C
E) Vitamin E

A

ANSWER: B
About half of pregnant women experience nausea and vomiting during pregnancy, which increases the risk of dehydration, poor weight gain, and impaired fetal growth. Pregnancy-related nausea and vomiting often begins by 4 weeks estimated gestation and typically resolves by the end of 12 weeks estimated gestation. When treating common causes of nausea and vomiting during pregnancy, lifestyle modifications such as frequent small meals and avoidance of high-protein or fatty foods are considered the safest intervention and first-line therapy. If these conservative measures are not effective, other well established low-risk therapies can be added in a stepwise fashion. These options include vitamin B6 (pyridoxine), over-the-counter antihistamines such as doxylamine, and natural ginger (<1500 mg daily). In addition, combination doxylamine/pyridoxine is approved by the FDA for the prevention of nausea and vomiting in pregnancy. Prescription antiemetics such as metoclopramide or trimethobenzamide are reserved for severe or refractory cases. Vitamin A, vitamin B12, vitamin C, and vitamin E are not appropriate for the treatment of pregnancy-related nausea and vomiting.

How well did you know this?
1
Not at all
2
3
4
5
Perfectly
270
Q

A 62-year-old female sees you for a routine health maintenance visit and asks for your advice regarding vitamin D supplementation. She is healthy and active, and jogs 1–2 hours three times weekly.
Which one of the following would be the most appropriate advice regarding vitamin D supplementation in this patient?
A) It is not recommended because she is asymptomatic
B) It will reduce the risk of certain cancers
C) It will reduce the risk of depression
D) It will reduce the risk of diabetes mellitus
E) It will reduce the risk of fractures

A

ANSWER: A
The U.S. Preventive Services Task Force and the American Academy of Family Physicians concluded that evidence is insufficient for vitamin D testing and for vitamin D supplementation in asymptomatic adults. Vitamin D supplementation does not reduce the risk of cancer, depression, diabetes mellitus, or fractures.

How well did you know this?
1
Not at all
2
3
4
5
Perfectly
271
Q

A 56-year-old male who has heart failure with reduced ejection fraction sees you for follow-up. He is stable but over the past year has noted an increase in dyspnea with moderate activity. His blood pressure is well controlled today. His current medications include carvedilol (Coreg), losartan (Cozaar), and escitalopram (Lexapro).
Which one of the following additions to his current medication regimen has the best evidence for reducing his risk of mortality from heart failure?
A) Aspirin
B) Atorvastatin (Lipitor)
C) Furosemide (Lasix)
D) Hydrochlorothiazide
E) Spironolactone (Aldactone)

A

ANSWER: E
This patient has symptomatic New York Heart Association class II heart failure, and an escalation in therapy is warranted. Both -blockers and aldosterone antagonists have been shown to reduce mortality in patients with symptomatic heart failure (SOR A). Management of associated cardiovascular disease such as hyperlipidemia and hypertension is important to prevent disease progression, but of the medications listed (aspirin, atorvastatin, furosemide, hydrochlorothiazide, and spironolactone) spironolactone is the best choice to reduce heart failure–related mortality.

How well did you know this?
1
Not at all
2
3
4
5
Perfectly
272
Q

Chronic kidney disease is defined by abnormal kidney structure or function lasting a minimum of
A) 2 months
B) 3 months
C) 6 months
D) 12 months
E) 24 months

A

ANSWER: B
Chronic kidney disease (CKD) is one of the most common chronic disease states encountered by family physicians, affecting 15% of the total U.S. adult population, and substantially impacting health care costs as well as morbidity and mortality. In the United States, diabetes mellitus and hypertension are the most common causes. CKD is defined by abnormal kidney structure or function lasting greater than 3 months, with associated implications for health. Diagnostic criteria include a persistent glomerular filtration rate <60 mL/min/1.73 m2, albuminuria, urine sediment abnormalities, renal imaging abnormalities, and serum acid-base or electrolyte abnormalities.

How well did you know this?
1
Not at all
2
3
4
5
Perfectly
273
Q

A 55-year-old female with type 2 diabetes sees you because of early satiety, nausea, vomiting, bloating, and postprandial fullness that is sometimes accompanied by upper abdominal pain. Since these symptoms have developed she has also noted increasing difficulty with blood glucose control.
Which one of the following would be the best study for confirming the most likely diagnosis?
A) Gastric emptying scintigraphy with a solid meal
B) Hepatobiliary scintigraphy (HIDA)
C) An upper gastrointestinal series with small-bowel follow-through
D) Abdominal ultrasonography
E) Abdominal CT

A

ANSWER: A
Gastroparesis is a complication of diabetes mellitus, and presents with nausea, vomiting, early satiety, bloating, postprandial fullness, and/or upper abdominal pain. Gastric emptying scintigraphy with a solid meal is the first-line study for confirming the diagnosis. Hepatobiliary scintigraphy (HIDA) is used to evaluate biliary dyskinesia and is not indicated in this patient. An upper gastrointestinal radiographic series, abdominal ultrasonography, and CT of the abdomen can help to rule out obstructive pathology, biliary tract disease, and other gastrointestinal conditions but would not confirm the diagnosis. The patient should also undergo esophagogastroduodenoscopy to exclude obstruction.

How well did you know this?
1
Not at all
2
3
4
5
Perfectly
274
Q
A
How well did you know this?
1
Not at all
2
3
4
5
Perfectly
275
Q

The administrator in your practice recently attended a quality improvement conference. He would like to start a clinic-wide program to focus on lead screening in asymptomatic children 5 years of age and younger.
After reviewing U.S. Preventive Services Task Force guidelines, which one of the following should you tell the practice administrator?
A) There is insufficient evidence to recommend for or against lead screening in children 5 years of age and younger
B) All children 5 years of age and younger should undergo lead screening
C) All children who live in housing built before 1978 should undergo lead screening
D) There are accurate and reliable screening questionnaires to guide lead screening

A

ANSWER: A
The U.S. Preventive Services Task Force (USPSTF) has found adequate evidence that questionnaires and other clinical prediction tools to identify asymptomatic children with elevated blood lead levels are inaccurate. The USPSTF went on to conclude that the current evidence is insufficient to assess the balance of benefits and harms of screening for elevated blood lead levels in asymptomatic children 5 years of age and younger. Although children living in older housing with lead-based paint are at higher risk of elevated blood lead levels than those living in housing built after 1978, the USPSTF does not recommend routine screening in asymptomatic children based on this risk factor.

How well did you know this?
1
Not at all
2
3
4
5
Perfectly
276
Q

A 60-year-old male who has type 2 diabetes comes to your office with an acute onset of fever, chills, and malaise. He says that he is feeling progressively worse. His temperature is 40°C (104°F). An examination reveals redness, tenderness, and swelling of the penis, scrotum, and perineal area.
Which one of the following medications is most likely to cause this condition?
A) Dapagliflozin (Farxiga)
B) Exenatide (Byetta)
C) Insulin glargine (Lantus)
D) Pioglitazone (Actos)
E) Sitagliptin (Januvia)

A

ANSWER: A

Nectoritzing fascitis of the peineum !!!

SGLT2 inhibitors (canagliflozin, dapagliflozin, empagliflozin, and ertugliflozin) are associated with a higher rate of genitourinary infections, including necrotizing fasciitis of the perineum (Fournier’s gangrene). While rare, this is a life-threatening infection associated with this class of medications that is being used more frequently to treat diabetes mellitus. Because of this risk, the FDA issued a Drug Safety Warning in 2018 due to case reports. The drug classes that include exenatide, insulin glargine, pioglitazone, and sitagliptin are not associated with this condition.

How well did you know this?
1
Not at all
2
3
4
5
Perfectly
277
Q

Conditions associated with adhesive shoulder

A

Diabetes adn hypothyroidism

How well did you know this?
1
Not at all
2
3
4
5
Perfectly
278
Q

A 55-year-old male sees you because of a second flare of gout. He has also had an elevated blood pressure at the last few visits to your clinic and is hypertensive again today.
In addition to treating his gout flare, which one of the following would be the most appropriate agent to treat his hypertension in light of his presenting problem?
A) Atenolol (Tenormin)
B) Hydralazine
C) Hydrochlorothiazide
D) Lisinopril (Prinivil, Zestril)
E) Losartan (Cozaar)

A

ANSWER: E
The 2020 American College of Rheumatology guideline for the management of gout generated numerous recommendations, including the management of concurrent medications in patients with gout. In such patients, losartan is the preferred antihypertensive agent when possible (SOR C). Hydrochlorothiazide should typically be changed to another agent, such as losartan, when feasible in patients with gout (SOR C). Both hydrochlorothiazide and losartan are known to have effects on the serum urate concentrations, with hydrochlorothiazide causing an increase and losartan causing a decrease. The American College of Rheumatology guideline does not recommend for or against the use of atenolol, hydralazine, and lisinopril as antihypertensive treatment in patients with gout.

How well did you know this?
1
Not at all
2
3
4
5
Perfectly
279
Q

A 60-year-old male comes to your office with a 1-year history of the gradual onset of mild fatigue and dyspnea. There are no symptom triggers. He has a 20-pack-year history of cigarette smoking but stopped at age 35. An examination is significant only for a BMI of 30 kg/m2. Office spirometry reveals a decreased FVC and a normal FEV1/FVC ratio, and there are no changes after bronchodilator administration.
Which one of the following would you recommend at this point?
A) The 6-minute walk test
B) Bronchoprovocation testing such as a methacholine challenge test
C) Full pulmonary function testing
D) Bronchoscopy
E) A ventilation-perfusion scan

A

ANSWER: C
Family physicians are often required to manage dyspnea and evaluate common office spirometry results. The American Thoracic Society recommends full pulmonary function testing when office spirometry suggests a restrictive pattern, which is the case with this patient’s normal FEV1/FVC ratio and decreased FVC. Full laboratory pulmonary function testing gives further information about gas exchange and lung volumes, which allows a more definitive diagnosis.

How well did you know this?
1
Not at all
2
3
4
5
Perfectly
280
Q

To determine compliance with prescribed medications and detect use of illicit substances, your clinic uses urine drug screening with an immunoassay qualitative point-of-care test to monitor patients who are on long-term opioid therapy. Which one of the following is most likely to result in a false-negative result and require confirmatory testing for detection?
A) Cannabis
B) Cocaine
C) Codeine
D) Morphine
E) Oxycodone (OxyContin)

A

ANSWER: E
Immunoassay drug screenings can be performed at the point of care and are relatively inexpensive. Typical immunoassays can detect nonsynthetic opioids such as morphine and codeine, as well as illicit substances such as amphetamines, cannabinoids, cocaine, and phencyclidine. However, these immunoassays do not reliably detect synthetic or semisynthetic opioids such as oxycodone, oxymorphone, methadone, buprenorphine, and fentanyl, as well as many benzodiazepines. Confirmatory testing is needed in situations with an unexpected negative result in order to distinguish a false negative from a true negative.

How well did you know this?
1
Not at all
2
3
4
5
Perfectly
281
Q

A 30-year-old female who is an established patient calls your office to request a test for COVID-19. The patient spent several hours inside the home of another individual who just received a positive COVID-19 test result. She states that her sense of taste seems diminished, but she has no respiratory symptoms and otherwise feels well.
Which one of the following is the typical incubation period for COVID-19?
A) 1day
B) 5 days
C) 14 days
D) 30 days

A

ANSWER: B
SARS-CoV-2 is a respiratory coronavirus that is responsible for COVID-19. Knowledge of the natural history of the viral infection will inform testing strategies and many other aspects of counseling of patients. The incubation period measures the time from exposure to symptom onset. The typical incubation period for COVID-19 is approximately 4–5 days, though it can range from 1–14 days.

How well did you know this?
1
Not at all
2
3
4
5
Perfectly
282
Q

A 45-year-old female with a 4-year history of type 2 diabetes is taking only metformin (Glucophage) and maintaining a hemoglobin A1c of 6.6%. Her LDL-cholesterol level is 94 mg/dL. She has no complications related to diabetes and her medical history is otherwise unremarkable.
Which one of the following should be added to her current medication regimen?
A) A DPP-4 inhibitor
B) An SGLT2 inhibitor
C) A low-intensity statin
D) A moderate-intensity statin
E) A high-intensity statin

A

ANSWER: D
All patients between 40 and 75 years of age with diabetes mellitus and an LDL-cholesterol level 70 mg/dL should begin taking a moderate-intensity statin. It is not necessary to calculate a 10-year risk for atherosclerotic cardiovascular disease because the results do not alter the recommendation. This patient’s hemoglobin A1c is <7%, which is acceptable, and she does not need additional hypoglycemic medications. She has no diabetes-specific risk-enhancing conditions such as a long duration of illness, chronic kidney disease, retinopathy, neuropathy, or an ankle-brachial index <0.9. Older age and risk-enhancing conditions may require increasing the statin to high-intensity dosages. A DPP-4 inhibitor, an SGLT2 inhibitor, and a low-intensity statin would not be appropriate for this patient at this time.

How well did you know this?
1
Not at all
2
3
4
5
Perfectly
283
Q

A 23-year-old primigravida comes to your office for her initial obstetric visit. She is at 13 weeks gestation based on the dates of her last menstrual period. She is a nonsmoker and does not drink alcohol or use illicit substances. Her vital signs are remarkable for a blood pressure of 142/92 mm Hg and a BMI of 32 kg/m2. She says that she has been diagnosed with hypertension in the past but has not taken any medications for it.
In addition to a prenatal vitamin, which one of the following would you recommend for her?
A) No additional medications
B) Aspirin
C) Ferrous sulfate
D) Folic acid
E) Labetalol (Trandate)

A

ANSWER: B
The U.S. Preventive Services Task Force (USPSTF) recommends prescribing low-dose aspirin after 12 weeks gestation for asymptomatic women at high risk for preeclampsia. Women at high risk include those with a history of preeclampsia, chronic hypertension, multiple pregnancy, type 1 or 2 diabetes, renal disease, autoimmune disease, or any combination of these. Many women become iron deficient in pregnancy but not all will require additional iron supplementation beyond what is available in the prenatal vitamin. The USPSTF found insufficient evidence to recommend for or against routine iron supplementation for pregnant women. Additional folic acid is recommended for women with increased risk for neural tube defects (NTDs), and while obesity increases the risk for NTD it is not an indication alone for a higher dosage of folic acid than the levels found in prenatal vitamins. In pregnant patients with chronic hypertension, treatment with antihypertensive medications is recommended only when the blood pressure is >150/100 mm Hg, because aggressive blood pressure lowering may result in placental hypoperfusion.

How well did you know this?
1
Not at all
2
3
4
5
Perfectly
284
Q

34-year-old female with asthma sees you for routine follow-up. She tells you that she uses her short-acting -agonist (SABA) approximately twice a week.
Which one of the following management strategies would you recommend for prevention of exacerbations?
A) Continued use of a SABA as needed
B) An inhaled corticosteroid (ICS)/long-acting -agonist (LABA) as needed
C) A daily maintenance ICS/LABA
D) A daily maintenance ICS plus a SABA as needed
E) A daily maintenance ICS plus a daily leukotriene receptor antagonist

A

ANSWER: B
For patients with mild asthma, recent evidence has shown that an inhaled corticosteroid (ICS)/long-acting -agonist (LABA), such as budesonide/formoterol, as needed was as effective at preventing exacerbations as a daily maintenance ICS plus a short-acting -agonist (SABA) at one-fifth of the total corticosteroid dose. In addition, it was more effective at preventing exacerbations than continued use of a SABA alone as needed. A daily maintenance ICS inhaler plus either a LABA or a leukotriene receptor antagonist are
management strategies for persistent asthma.

How well did you know this?
1
Not at all
2
3
4
5
Perfectly
285
Q

A 42-year-old female sees you because of intermittent right upper abdominal pain that occurs after eating. The episodes have been gradually worsening and now last up to an hour. She has tried over-the-counter antacids, ibuprofen, and acetaminophen, which have not helped. She tells you that the last episode occurred earlier this week and the pain was so severe that it woke her up and she went to the emergency department (ED). A comprehensive metabolic panel, CBC, and lipase level performed in the ED were all normal. Right upper quadrant abdominal ultrasonography today is negative for gallstones but notable for increased echogenicity of the liver.
Which one of the following would be the most appropriate next step in the evaluation?
A) Plain radiography of the abdomen
B) CT of the abdomen
C) Hepatobiliary scintigraphy (HIDA)
D) Magnetic resonance cholangiopancreatography (MRCP)
E) Endoscopic retrograde cholangiopancreatography (ERCP)

A

ANSWER: C
This patient presents with classic biliary symptoms and normal right upper quadrant ultrasonography, liver enzymes, and pancreatic enzymes. Abdominal ultrasonography was negative for gallstones. The next most appropriate test is hepatobiliary scintigraphy, also known as a hepatobiliary iminodiacetic acid (HIDA) scan. While a normal HIDA scan does not exclude a diagnosis of functional gallbladder disease (also referred to as acalculous cholecystitis, biliary dyskinesia, and biliary dysmotility), an abnormal study identifies patients for whom cholecystectomy is strongly recommended. Plain radiography of the abdomen and CT of the abdomen are helpful to evaluate for other etiologies of abdominal pain but are not the most appropriate next step for a patient with classic biliary symptoms and a normal laboratory workup. Magnetic resonance cholangiopancreatography (MRCP) is reserved for suspected choledocholithiasis. Endoscopic retrograde cholangiopancreatography (ERCP) is an invasive test also used for choledocholithiasis and in conjunction with sphincterotomy and stone extraction.

How well did you know this?
1
Not at all
2
3
4
5
Perfectly
286
Q

A 6-month-old male is brought to your office by his mother for a well child examination. The mother does not have any concerns. Interactions between the mother and child are appropriate and the child appears well.
Which one of the following screenings is recommended at this visit?
A) Autism
B) Iron deficiency
C) Maternal depression
D) Otoacoustic emissions (OAE) testing

A

NSWER: C
The American Academy of Pediatrics (AAP) recommends formal screening for maternal depression with the Edinburgh Postnatal Depression Scale or the Patient Health Questionnaire–2 (PHQ-2) at the 1-, 2-, 4-, and 6-month well child visits. The AAP recommends screening for autism at 18 months, but the U.S. Preventive Services Task Force (USPSTF) finds insufficient evidence to recommend screening unless there are parental concerns. The AAP recommends screening for iron deficiency at 12 months, but the USPSTF finds insufficient evidence for screening at this time. Otoacoustic emissions (OAE) testing is performed during the newborn screening and is not recommended at 6 months of age.

How well did you know this?
1
Not at all
2
3
4
5
Perfectly
287
Q

A 42-year-old female presents to your office with heavy menstrual periods and pelvic pressure. Her symptoms began several years ago and have gradually worsened. Laboratory findings are notable for a mild microcytic anemia. Pelvic ultrasonography identifies a 7-cm submucosal mass. She wants to avoid a hysterectomy but desires a treatment that will provide symptom relief, decrease the volume of the mass, and have a sustained effect.
Which one of the following would be most appropriate for this patient?
A) Expectant management
B) A GnRH agonist
C) A selective estrogen receptor modulator
D) A levonorgestrel-releasing IUD (Mirena)
E) Uterine artery embolization and occlusion

A

ANSWER: E
This patient presents with a symptomatic fibroid. Although she does not express a desire to maintain fertility, she prefers uterine preservation. The Agency for Healthcare Research and Quality Effective Health Care Program review found consistent evidence that uterine artery embolization and occlusion is effective for reducing fibroid size, with lasting effects up to 5 years and moderate evidence for reducing bleeding and improving quality of life. Expectant management is an appropriate option only for patients who have asymptomatic fibroids. GnRH agonists are effective for providing symptom relief and reducing fibroid size, but their use results in a hypoestrogenized state and should not be continued long term for a sustained effect in premenopausal women. Treatment with a selective estrogen receptor modulator such as raloxifene does not affect fibroid size or bleeding patterns. There is limited data regarding the efficacy of a levonorgestrel-releasing IUD for the treatment of uterine fibroids.

How well did you know this?
1
Not at all
2
3
4
5
Perfectly
288
Q

A 58-year-old male with a history of a neurogenic bladder comes to your office as a new patient. He recently elected to have placement of a chronic indwelling urethral catheter rather than performing intermittent catheterization at home, and he asks how to reduce his risk of urinary tract infections (UTIs). His last UTI was approximately 1 year ago and required intravenous antibiotics.
Which one of the following is most effective for preventing UTIs in patients with chronic indwelling urethral catheters?
A) Routine daily hygiene of the meatal surface with soap and water
B) Daily periurethral cleaning with iodine
C) Daily oral antibiotics based on prior urine culture sensitivities
D) Routine instillation of an antimicrobial solution into the drainage bag
E) Regularly scheduled catheter exchanges at fixed intervals

A

ANSWER: A
Although use of chronic indwelling urethral catheters should be avoided whenever possible, there are still some patients that will require one. Prevention of catheter-associated urinary tract infections (CAUTIs) is important. The most important measure to prevent CAUTIs is routine cleaning of the meatal surface with soap and water while bathing or showering. Use of specific periurethral antiseptics or instillation of antiseptics into the drainage bag does not reduce rates of CAUTI. Daily oral antibiotics are not indicated to prevent CAUTIs. Catheters and drainage bags should only be changed when clinically indicated, such as when there is an infection or obstruction.

How well did you know this?
1
Not at all
2
3
4
5
Perfectly
289
Q

A 68-year-old male presents with a burn on his lower leg after trying to light a bonfire with kerosene. Examination of the affected leg reveals the presence of blistering, along with a denuded central area that does not blanch with pressure. The underlying fat and connective tissue are not involved.
Which one of the following is the proper classification of this burn?
A) Superficial burn
B) Superficial partial-thickness burn
C) Deep partial-thickness burn
D) Full-thickness burn

A

ANSWER: C
Decisions regarding the management of burn wounds depend on first identifying the depth of the burn. Superficial burns are red, painful, and blanching, and they do not blister. Superficial partial-thickness burns blister and blanch with pressure. Deep partial-thickness burns blister, but do not blanch with pressure. Full-thickness burns extend through the entire dermis and into the underlying tissues, and they are dry and leathery. Patients with deep partial-thickness or full-thickness burns should be evaluated by a burn specialist.

How well did you know this?
1
Not at all
2
3
4
5
Perfectly
290
Q

Superficial burn

A

Red painful and blanching, do not blister

How well did you know this?
1
Not at all
2
3
4
5
Perfectly
291
Q

Superficial partial thickness burn

A

blister and blanch with pressure

How well did you know this?
1
Not at all
2
3
4
5
Perfectly
292
Q

Deep partial thickness burns

A

blister but DO NOT blanch

How well did you know this?
1
Not at all
2
3
4
5
Perfectly
293
Q

Full thickness burns

A

Extend entire dermis into underlying tissue

How well did you know this?
1
Not at all
2
3
4
5
Perfectly
294
Q

A 71-year-old female with a history of well controlled hypertension, diabetes mellitus, and osteoporosis presents with a 2-day history of fever, chills, and a productive cough. She lives at home with her husband, who has not noted any confusion but says she has been weak and unable to bathe herself.
On examination the patient has a temperature of 38.2°C (100.8°F), a blood pressure of 110/68 mm Hg, unlabored respirations at a rate of 22/min, and an oxygen saturation of 94% on room air. You note that she has good air entry, there are no abnormal breath sounds, and there is no egophony or increased fremitus. The cardiovascular examination is unremarkable.
Laboratory Findings
WBCs elevated
Hemoglobin low
Platelets normal
Creatinine normal
BUN normal
14,000/mm3 (N4500–11,000) 12.5g/dL(N14.0–17.5) 250,000/mm3 (N150,000–350,000) 1.0mg/dL(N0.6–1.2)
14 mg/dL (N 8–23) Posteroanterior and lateral chest radiographs show an infiltrate in the right middle lobe.
Which one of the following would be the most appropriate treatment for this patient?
A) Azithromycin (Zithromax)
B) Amoxicillin plus metronidazole (Flagyl)
C) Amoxicillin/clavulanate (Augmentin) plus azithromycin
D) Azithromycin plus levofloxacin
E) Clindamycin (Cleocin) plus doxycycline

A

ANSWER: C
Community-acquired pneumonia (CAP) is an infection of the lung parenchyma that is not acquired in a hospital, long-term care facility, or other health care setting, and it is a significant cause of morbidity and mortality in adults. This patient has CAP in the presence of a significant comorbidity (diabetes mellitus). After CAP is diagnosed the first decision to make is whether hospitalization is needed. In all patients with CAP, mortality and severity prediction scores should be used to determine inpatient versus outpatient care (SOR A). This patient has a CURB-65 score of 1 (age 65 years), so she can be treated as an outpatient.
For outpatients with comorbidities, amoxicillin/clavulanate is a possible treatment option, but it should be paired with a macrolide. Macrolides such as azithromycin are the treatment of choice for previously healthy outpatients with no history of antibiotic use within the past 3 months. Azithromycin monotherapy, amoxicillin plus metronidazole, azithromycin plus levofloxacin, or clindamycin plus doxycycline would not be appropriate treatment strategies for this patient with a significant comorbidity.

How well did you know this?
1
Not at all
2
3
4
5
Perfectly
295
Q

An 8-year-old female is brought to your office because of left arm pain after she fell down on the sidewalk while roller skating. She has pain, swelling, and a mild deformity of her distal forearm over the radius. Posteroanterior and lateral radiographs confirm an incomplete compression fracture of the distal radius.
In addition to a short arm splint, which one of the following would be appropriate management of this fracture?
A) Ultrasonography in 3 weeks
B) Repeat radiography in 4 weeks
C) Return to activity in 4 weeks if she is pain free
D) Follow-up and reevaluation in 6 weeks
E) Referral to an orthopedist

A

ANSWER: C
This patient has a compression fracture of the distal radius, also known as a buckle fracture. There is no cortical disruption and these are inherently stable fractures. Radiography or ultrasonography may be used as the initial imaging study if a buckle fracture is suspected. Treatment consists of short arm immobilization, which is most easily performed with a removable splint or wrist brace. The Choosing Wisely campaign states that these fractures do not require repeat imaging if there is no longer any tenderness or pain with palpation after 4 weeks of splinting, and the patient can return to full activity as tolerated. These fractures do not require referral to an orthopedist and can be managed in the office.

How well did you know this?
1
Not at all
2
3
4
5
Perfectly
296
Q

An obese 32-year-old male is admitted to the hospital with a new onset of acute pancreatitis. A lipid panel reveals a triglyceride level of 1150 mg/dL and an HDL-cholesterol level of 30 mg/dL. Other laboratory studies are normal. His 10-year risk of atherosclerotic cardiovascular disease is <5%. His family history is positive for recurrent pancreatitis in his father and paternal grandfather.
In addition to lifestyle modifications, which one of the following would be most appropriate for this patient after he is discharged?
A) Atorvastatin (Lipitor)
B) Colesevelam (Welchol)
C) Ezetimibe (Zetia)
D) Fenofibrate (Tricor)
E) Omega-3-acid ethyl esters (Lovaza)

A

ANSWER: D
Fibrates reduce the likelihood and recurrence of pancreatitis due to severe hypertriglyceridemia when triglyceride levels are 500 mg/dL, measured in a fasting or nonfasting state (SOR A). This patient’s risk of atherosclerotic cardiovascular disease is <7.5% and his LDL-cholesterol level is within normal range, so initiating a statin or ezetimibe is not indicated. Colesevelam may be used to reduce LDL-cholesterol and glucose levels but is not considered a first-line treatment. Omega-3-acid ethyl esters will reduce the triglyceride levels but this patient has severe hypertriglyceridemia, so fibrate therapy is recommended to prevent recurrent pancreatitis.

How well did you know this?
1
Not at all
2
3
4
5
Perfectly
297
Q

A 23-year-old gravida 1 para 1 who is a single mother of a 3-day-old infant comes to your office for a newborn follow-up. She reports some sleep disturbance, mild depression without suicidal ideation, and financial concerns. Her past medical history is significant for persistent depressive disorder.
The U.S. Preventive Services Task Force recommends which one of the following to help prevent perinatal depression in patients such as this?
A) Exercise
B) Amitriptyline
C) Sertraline (Zoloft)
D) Referral for cognitive-behavioral therapy

A

ANSWER: D
The U.S. Preventive Services Task Force (USPSTF) recommends counseling interventions to prevent perinatal depression in patients who are at risk. This patient has risk factors for perinatal depression, including young age, single motherhood, and a history of depression. Other risk factors include low socioeconomic status and depressive symptoms. The USPSTF found that the benefits of counseling interventions outweigh the harms. The USPSTF could not find evidence that exercise, amitriptyline, or sertraline were beneficial.

How well did you know this?
1
Not at all
2
3
4
5
Perfectly
298
Q

A 34-year-old male sees you because he was recently informed that a partner he had unprotected sex with last month has been diagnosed with HIV. You would advise this patient to initiate ongoing antiretroviral therapy
A) immediately, because HIV testing is not necessary prior to initiation
B) at the time of diagnosis of HIV infection
C) when his CD4 cell count drops to <200 cells/ L
D) when his CD4 cell count drops to <500 cells/ L
E) when he develops an AIDS-defining illness

A

ANSWER: B
Antiretroviral therapy (ART) should be prescribed at the time of diagnosis of HIV infection unless the patient has expressed a desire to not initiate treatment. ART should not be delayed until the CD4 cell count drops to a predetermined level or until an AIDS-defining illness occurs. It is recommended to initiate prophylaxis for Pneumocystis pneumonia when the CD4 cell count drops below 200 cells/ L.

How well did you know this?
1
Not at all
2
3
4
5
Perfectly
299
Q

An 82-year-old female with atrial fibrillation treated with digoxin is started on verapamil sustained-release capsules (Calan SR) for hypertension and angina. Although she initially tolerates the medication and has a good clinical response, when you see her 1 month later she has lost 3 kg (7 lb) and reports persistent anorexia and nausea over the past 2–3 weeks. A serum chemistry profile, TSH level, and CBC are normal. Her serum digoxin level is 1.4 ng/mL (therapeutic range 0.8–1.5). Her vital signs are stable and a physical examination is notable only for rate-controlled atrial fibrillation.
Which one of the following would be most appropriate at this point?
A) Prescribe a therapeutic trial of an H2-blocker
B) Order an upper gastrointestinal contrast study
C) Withhold digoxin for several days and reinstitute at a lower dosage if necessary
D) Order imaging to look for a central nervous system abnormality

A

ANSWER: C
This patient presents with typical symptoms of digitalis toxicity, which is common in elderly patients and may occur when the serum level is in the suggested therapeutic range. When drug toxicity is suspected the first step in management would be to discontinue or reduce the dosage of the suspected agent. In this case the addition of verapamil will increase the serum level of digoxin, which will reach a new steady-state level in several days. Therefore, the side effects may not occur for several days while the level is increasing. Because this patient has a good response to verapamil, it is not advisable to stop it, as the digitalis toxicity should be reversed by a dosage reduction. A therapeutic trial of an H2-blocker, an upper gastrointestinal contrast study, or imaging to look for a central nervous system abnormality would not be appropriate at this time.

How well did you know this?
1
Not at all
2
3
4
5
Perfectly
300
Q

A 58-year-old male sees you for evaluation of left ankle pain after he slipped on some ice in his driveway last night. He felt immediate pain over the lateral ankle, which started swelling over the next hour. He elevated his foot, applied ice, and took ibuprofen. This morning the ankle remains swollen and also appears bruised. He is able to walk on it with some pain. On examination you note typical findings of an ankle sprain. He asks you for medication to manage his pain.
Which one of the following medications has the best evidence for providing pain relief while also minimizing side effects?
A) Topical diclofenac gel (Pennsaid)
B) Topical menthol gel
C) Oral hydrocodone/acetaminophen (Lortab)
D) Oral ibuprofen
E) Oral tramadol

A

ANSWER: A
In 2020 the American College of Physicians and the American Academy of Family Physicians published a guideline regarding the treatment of acute pain from musculoskeletal injuries (non–low back related). This systematic review found good evidence to support the recommendation that topical NSAIDs be used as first-line therapy to reduce pain and improve physical function. Topical NSAIDs were the only intervention that improved multiple outcomes and were not associated with a statistically significant increase in the risk for adverse events. Oral NSAIDs and acetaminophen were recommended as second-line therapies, as they were found to be effective for pain relief but were associated with an increased risk for adverse events. Topical menthol gel was not found to be effective as monotherapy but may be considered when combined with a topical NSAID. The guideline specifically recommends avoidance of opioids, including tramadol, noting a prevalence of 6% for prolonged opioid use resulting from an initial prescription. Nonpharmacologic approaches with evidence of benefit include specific acupressure and use of a transcutaneous electrical nerve stimulation (TENS) unit.

How well did you know this?
1
Not at all
2
3
4
5
Perfectly
301
Q

A 7-year-old male is brought to the urgent care clinic with a 2-day history of fever and sore throat, with no associated cough. His temperature is 38.3°C (100.9°F) and a rapid antigen test confirms a group A -hemolytic Streptococcus infection. A prescription for penicillin is sent to the pharmacy, but the medication is never picked up due to a lack of transportation. The patient is brought to your office 2 weeks later with a fever, joint pain, shortness of breath, and chest pain. His vital signs are significant for a temperature of 38.8°C (101.8°F) and a heart rate of 118 beats/min.
On examination the patient’s affect is appropriate, he has a 3/6 holosystolic murmur heard best over the apex, and he has tenderness and swelling of his knees bilaterally and of his left ankle. An antistreptolysin O titer is positive, his erythrocyte sedimentation rate is 124 mm/hr (N <10), and a chest radiograph is significant for cardiomegaly.
Which one of the following would be the most appropriate therapy?
A) Hydroxychloroquine (Plaquenil)
B) Methylprednisolone acetate (Depo-Medrol)
C) Naproxen
D) Intravenous immunoglobulins
E) Plasmapheresis

A

ANSWER: C
Using the Jones criteria for diagnosis, this patient has acute rheumatic fever, with two major criteria (carditis and polyarthritis) and two minor criteria (fever and positive erythrocyte sedimentation rate). NSAIDs such as naproxen can provide significant relief and should be administered as soon as acute rheumatic fever is diagnosed (SOR B). Hydroxychloroquine is not FDA approved for the treatment of acute rheumatic fever and would not be appropriate. Treatment with corticosteroids, intravenous immunoglobulins, and plasmapheresis is not considered appropriate for acute rheumatic fever but may be indicated for management of pediatric autoimmune neuropsychiatric disorders associated with streptococcal infections (PANDAS).

How well did you know this?
1
Not at all
2
3
4
5
Perfectly
302
Q

A 51-year-old female presents with concerns about a change in her cognition. She says she has difficulty retrieving words, loses her train of thought, and goes into a room and forgets why she came there. She also has had more frequent hot flashes and sleep disturbances. She still menstruates but has noticed a change from her previous pattern. A physical examination is unremarkable, and recent laboratory tests were all normal, including vitamin B12 and thyroid studies. Cognitive testing is normal.
Which one of the following would be the most appropriate next step?
A) Reassurance only
B) CT of the head
C) MRI of the brain
D) Hormone therapy
E) Referral to a neurologist

A

ANSWER: A
Women experience subjective cognitive difficulties during their menopausal transition. This may include retrieving numbers or words, losing one’s train of thought, forgetting appointments, and forgetting the purpose of behavior such as entering a room. Clinical studies of these women showed intact cognitive test performance. The treatment consists of patient education and reassurance, since studies have shown that 62% of women report subjective cognitive problems during their menopausal transition. Imaging and referral to a neurologist are not indicated, and there are no trials that support the use of hormone therapy.

How well did you know this?
1
Not at all
2
3
4
5
Perfectly
303
Q

An otherwise healthy 46-year-old female presents with a 10-day history of recurring pain in the right cheek and gums. She says the pain feels like electric shocks lasting a few seconds and recurring “hundreds of times a day.” She says that smiling and brushing her teeth can trigger the pain. She does not have a history of recent dental work, trauma, fever, or myalgia. She feels well aside from the facial pain. She has tried over-the-counter analgesics without relief. Her vital signs and an HEENT examination are normal. There is no pain with dental percussion, and the skin and mucous membranes of the nose and mouth are unremarkable.
Which one of the following would be the most appropriate treatment for this patient’s condition?
A) Carbamazepine (Tegretol), 200 mg twice daily
B) Prednisone, 40 mg daily
C) Sumatriptan (Imitrex), 6 mg subcutaneously
D) Valacyclovir (Valtrex), 1000 mg three times daily
E) High-flow oxygen for 20 minutes

A

ANSWER: A
Trigeminal neuralgia is a clinically diagnosed condition that is characterized by brief, sudden, unilateral pain in the distribution of one of the three branches of the trigeminal nerve. The pain, which is often triggered by minimal stimulus, is paroxysmal with episodes of remission that can last for months. First-line treatment for the condition is carbamazepine or oxcarbazepine (SOR C). Other suggested treatments include lamotrigine, baclofen, and surgical treatments, including microvascular decompression. There is a paucity of good evidence for treatments other than carbamazepine.
Valacyclovir would be an appropriate treatment for herpes zoster, which can also cause unilateral electrical shock–like pain in the face. However, patients with herpes zoster typically would have developed the characteristic lesions by 10 days. In addition, the pain is rarely paroxysmal in nature with herpes zoster. Short-term corticosteroids are not recommended for trigeminal neuralgia and there is minimal evidence of their effectiveness for herpes zoster. Sumatriptan is an appropriate treatment for migraines that are also unilateral but are associated with photophobia, phonophobia, nausea, and persistent severe pain.
High-flow oxygen is used to relieve cluster headaches that occur in the orbital, temporal, or supraorbital areas and persist for 15–180 minutes. Cluster headaches are accompanied by tearing and nasal discharge.

How well did you know this?
1
Not at all
2
3
4
5
Perfectly
304
Q

A 68-year-old male with a history of COPD, hypertension, and hyperlipidemia presents with a worsening cough and dyspnea with exertion over the past 3 months. His symptoms were previously well controlled with tiotropium (Spiriva) daily and albuterol (Proventil, Ventolin) as needed, and he has not had any COPD exacerbations in the past year until these symptoms began. He has not had any change in sputum production. Recently he has been using his albuterol inhaler several times a day to help relieve his shortness of breath with exertion.
A physical examination reveals a temperature of 37.0°C (98.6°F), a heart rate of 78 beats/min, a respiratory rate of 16/min, a blood pressure of 144/82 mm Hg, and an oxygen saturation of 95% on room air. A cardiac evaluation reveals a regular rate and rhythm and he has no peripheral edema or cyanosis. His lungs are clear with no wheezes or crackles, and there is a mild prolonged expiratory phase.
According to current GOLD guidelines, which one of the following would be the most appropriate next step in the management of this patient’s symptoms?
A) Add azithromycin (Zithromax)
B) Add inhaled fluticasone (Flovent)
C) Add inhaled salmeterol (Serevent)
D) Add inhaled fluticasone/salmeterol (Advair)
E) Discontinue tiotropium and start inhaled fluticasone

A

ANSWER: C
COPD is currently the third leading cause of death in the United States and is commonly treated by primary care providers. In patients on monotherapy with a long-acting bronchodilator such as a long-acting muscarinic agonist (LAMA) or long-acting -agonist (LABA) who have continued dyspnea, the Global Initiative for Chronic Obstructive Lung Disease (GOLD) guidelines recommend escalating therapy to two bronchodilators. This patient has persistent dyspnea and is being treated with a single agent, a LAMA, so his regimen needs to be escalated to include a LABA such as salmeterol. Once the symptoms are stabilized, treatment can be de-escalated to a single agent. For patients with frequent COPD exacerbations or with a diagnosis of asthma and COPD, the guidelines recommend adding an inhaled corticosteroid (ICS) such as fluticasone to a LABA, LAMA, or both. Triple therapy with a LABA, a LAMA, and an ICS is not indicated at this time as the patient has not yet been treated with a combination of a LAMA and LABA and has not had any recent exacerbations. The addition of azithromycin may be considered in patients who are already on triple therapy with a LABA, a LAMA, and an ICS and still having exacerbations. Monotherapy with an ICS is not indicated in COPD and has been shown to increase the risk of developing pneumonia.

How well did you know this?
1
Not at all
2
3
4
5
Perfectly
305
Q

A 69-year-old male presents with a several-week history of difficulty swallowing that has gradually worsened. At first he noted trouble with passing larger boluses of food through his mid-chest area. Now he states that even ice cream is a problem. He has a past history of GERD that he has treated with omeprazole (Prilosec) intermittently over the past several years. He also takes enteric-coated aspirin, 81 mg daily, for his heart. There is no history of tobacco use. He drinks about six beers a week. On examination you note a 5-kg (11-lb) weight loss over the past 3 months but the remainder of the examination is normal.
Which one of the following would be the most appropriate next step in the evaluation?
A) A trial of omeprazole, 40 mg daily, for 8 weeks
B) Chest CT with and without contrast
C) Barium esophagography
D) High-resolution esophageal manometry
E) Esophagogastroduodenoscopy

A

ANSWER: E
This patient has several risk factors for a significant problem such as esophageal cancer, including age over 50, weight loss, and progressive symptoms. Esophagogastroduodenoscopy (EGD) is needed without delay. This approach would allow biopsy of any lesions seen and therapeutic dilatation if a benign-appearing stricture is noted. Biopsies are also needed to diagnose eosinophilic esophagitis. If the EGD does not identify a problem, further workup should then proceed. CT may identify a source of extrinsic pressure. Barium esophagography may detect mild narrowing or esophageal webs missed on EGD. Esophageal motility disorders may be diagnosed with esophageal manometry. Waiting 8 weeks to see if his symptoms improve with regular use of a proton pump inhibitor would not be appropriate in this patient with symptoms that are worrisome for esophageal cancer.

How well did you know this?
1
Not at all
2
3
4
5
Perfectly
306
Q

An otherwise healthy 57-year-old male presents with mild fatigue, decreased libido, and erectile dysfunction. A subsequent evaluation of serum testosterone reveals hypogonadism.
Which one of the following would you recommend at this time?
A) No further diagnostic testing
B) A prolactin level
C) A serum iron level and total iron binding capacity
D) FSH and LH levels
E) Karyotyping

A

ANSWER: D
In men who are diagnosed with hypogonadism with symptoms of testosterone deficiency and unequivocally and consistently low serum testosterone concentrations, further evaluation with FSH and LH levels is advised as the initial workup to distinguish between primary and secondary hypogonadism. If secondary hypogonadism is indicated by low or inappropriately normal FSH and LH levels, prolactin and serum iron levels and measurement of total iron binding capacity are recommended to determine secondary causes of hypogonadism, with possible further evaluation to include other pituitary hormone levels and MRI of the pituitary. If primary hypogonadism is found, karyotyping may be indicated for Klinefelter’s syndrome.

How well did you know this?
1
Not at all
2
3
4
5
Perfectly
307
Q

A 35-year-old female comes to your office for evaluation of a tremor. During the interview you note jerking movements first in one hand and then the other, but when the patient is distracted the symptom resolves. Aside from the intermittent tremor the neurologic examination is unremarkable. She does not drink caffeinated beverages and takes no medications.
Which one of the following is the most likely diagnosis?
A) Parkinson’s disease
B) Cerebellar tremor
C) Essential tremor
D) Physiologic tremor
E) Psychogenic tremor

A

ANSWER: E
Psychogenic tremor is characterized by an abrupt onset, spontaneous remission, changing characteristics, and extinction with distraction. Cerebellar tremor is an intention tremor with ipsilateral involvement on the side of the lesion. Neurologic testing will reveal past-pointing on finger-to-nose testing. CT or MRI of the head is the diagnostic test of choice. Parkinsonian tremor is noted at rest, is asymmetric, and decreases with voluntary movement. Bradykinesia, rigidity, and postural instability are generally noted. For atypical presentations a single-photon emission CT or positron emission tomography may help with the diagnosis. One of the treatment options is carbidopa/levodopa.
Patients who have essential tremor have symmetric, fine tremors that may involve the hands, wrists, head, voice, or lower extremities. This may improve with ingestion of small amounts of alcohol. There is no specific diagnostic test but the tremor is treated with propranolol or primidone. Enhanced physiologic tremor is a postural tremor of low amplitude exacerbated by medication. There is usually a history of caffeine use or anxiety.

How well did you know this?
1
Not at all
2
3
4
5
Perfectly
308
Q

A nulliparous 34-year-old female comes to your office for evaluation of fatigue, hair loss, and anterior neck pain. These symptoms have been gradually worsening for the past few months. Her past medical history is unremarkable. She has gained 5 kg (11 lb) since her last office visit 18 months ago. Examination of the thyroid gland reveals tenderness but no discrete nodules. Her TSH level is 7.5 U/mL (N 0.4–4.2), her T4 level is low, and her thyroid peroxidase antibodies are elevated.
Which one of the following would be the most appropriate next step?
A) Continue monitoring TSH every 6 months
B) Begin thyroid hormone replacement and repeat the TSH level in 6–8 weeks
C) Begin thyroid hormone replacement and repeat the TSH level along with a T3 level in
6–8 weeks
D) Order ultrasonography of the thyroid
E) Order fine-needle aspiration of the thyroid

A

ANSWER: B
This patient has thyroiditis with biochemical evidence for autoimmune (Hashimoto’s) thyroiditis. The most appropriate plan of care is to begin thyroid hormone replacement and monitor with a repeat TSH level 6–8 weeks later. It is not necessary to include a T3 level when assessing the levothyroxine dose. There is no need to routinely order thyroid ultrasonography when there are no palpable nodules on a thyroid examination. Fine-needle aspiration may be necessary to rule out infectious thyroiditis when a patient presents with severe thyroid pain and systemic symptoms.

How well did you know this?
1
Not at all
2
3
4
5
Perfectly
309
Q

A 70-year-old male presents to your office for follow-up after he was hospitalized for acute coronary syndrome. He has not experienced any pain since discharge and is currently in a supervised cardiac rehabilitation exercise program. His medications include aspirin, lisinopril (Prinivil, Zestril), and metoprolol, but he was unable to tolerate atorvastatin (Lipitor), 40 mg daily, because he developed muscle aches.
Which one of the following would you recommend?
A) Evolocumab (Repatha)
B) Ezetimibe/simvastatin (Vytorin)
C) Fenofibrate (Tricor)
D) Niacin
E) Omega-3 fatty acid supplements

A

ANSWER: B
High-intensity statin therapy is recommended for patients younger than 75 years of age with known coronary artery disease. For those who are intolerant of high-intensity statins, a trial of a moderate-intensity statin is appropriate. There is evidence to support ezetimibe plus a statin in patients with acute coronary syndrome or chronic kidney disease. Omega-3 fatty acids, fibrates, and niacin should not be prescribed for primary or secondary prevention of atherosclerotic cardiovascular disease because they do not affect patient-oriented outcomes. PCSK9 inhibitors such as evolocumab are injectable monoclonal antibodies that lower LDL-cholesterol levels significantly and have produced some promising results, but more studies are needed to determine when this would be cost effective.

How well did you know this?
1
Not at all
2
3
4
5
Perfectly
310
Q

A 30-year-old gravida 1 para 0 develops erythematous patches with slightly elevated scaly borders during her first trimester. There was a 2-cm herald patch 2 weeks before multiple smaller patches appeared. The rash on the back has a “Christmas tree” pattern. She has not had any prenatal laboratory work.
This condition is associated with
A) no additional pregnancy risk
B) a small-for-gestational-age newborn
C) congenital cataracts
D) multiple birth defects
E) spontaneous abortion

A

ANSWER: E
This patient has classic pityriasis rosea. This is generally a benign disease except in pregnancy. The epidemiology and clinical course suggest an infectious etiology. Pregnant women are more susceptible to pityriasis rosea because of decreased immunity. Pityriasis rosea is associated with an increased rate of spontaneous abortion in the first 15 weeks of gestation. It is not associated with an increased risk for a small-for-gestational-age newborn, congenital cataracts, or multiple birth defects.

How well did you know this?
1
Not at all
2
3
4
5
Perfectly
311
Q

In asymptomatic patients with sarcoidosis, which one of the following organ systems should be examined yearly to detect extrapulmonary manifestations of the disease?
A) Cardiac
B) Neurologic C) Ocular
D) Integumentary

A

ANSWER: C
Sarcoidosis has numerous extrapulmonary manifestations. Because inflammation of the eye can result in permanent impairment and is often asymptomatic, patients require yearly eye examinations as well as additional monitoring with disease flares. Although skin involvement is common it is usually readily apparent and rarely has serious sequelae. Cardiac sarcoidosis can potentially lead to progressive heart failure and sudden death, but evaluation is needed only in patients who are symptomatic. Similarly, evaluation for neurologic involvement is needed only in patients who are symptomatic.

How well did you know this?
1
Not at all
2
3
4
5
Perfectly
312
Q

A 57-year-old male presents with left posterior heel pain that started several weeks ago. An examination reveals a nodular appearance at the site of insertion of the Achilles tendon to the calcaneus, and local tenderness of the distal tendon.
Which one of the following would be the safest and most appropriate initial management?
A) Local injection with a corticosteroid
B) Local injection with platelet-rich plasma
C) Physical therapy with eccentric calf-strengthening exercises
D) Immobilization of the ankle in a cast or boot for 4–6 weeks
E) Surgical debridement of the calcification about the distal tendon

A

ANSWER: C
This patient has typical symptoms and findings of Achilles tendinopathy. The best management involves eccentric calf-strengthening exercises. A local injection with corticosteroids or with platelet-rich plasma is ineffective and may increase the risk of a tendon rupture. Immobilization and surgical debridement may be considered if more conservative therapies have failed.

How well did you know this?
1
Not at all
2
3
4
5
Perfectly
313
Q

A 32-year-old female presents with a 4-month history of nasal drainage, congestion, and loss of her sense of smell. She reports having a cold about 4 months ago that never resolved. On examination the nasal turbinates are swollen and you note mucopurulent drainage on the right.
Which one of the following is the most likely cause of her symptoms?
A) Chronic rhinosinusitis
B) Granulomatosis with polyangiitis (Wegener’s granulomatosis)
C) Nasal polyposis
D) Sarcoidosis
E) Seasonal allergic rhinitis

A

ANSWER: A
The American Academy of Otolaryngology defines chronic rhinosinusitis as the presence of two of four cardinal symptoms, which include nasal drainage, nasal obstruction, facial pain or pressure, and hyposmia or anosmia, along with objective signs on examination or radiographic studies. This patient has three cardinal symptoms of chronic rhinosinusitis and objective evidence on the physical examination. No nasal polyps were seen on the examination. Granulomatosis with polyangiitis and sarcoidosis can both present similarly but are uncommon causes of chronic rhinosinusitis. Allergic rhinitis can be associated with chronic rhinosinusitis but would also present with allergic symptoms.

How well did you know this?
1
Not at all
2
3
4
5
Perfectly
314
Q

A 19-year-old female member of a college cross-country team presents with a 1-week history of right knee pain. She does not have any acute injury to the knee. An examination reveals no deformity and she has a normal gait. She has tenderness and subtle swelling localized 1 cm distal to the right medial joint line, and examinations of the knee and hip are otherwise normal.
Which one of the following is the most likely diagnosis?
A) Fibular head stress fracture
B) Iliotibial band syndrome
C) Medial meniscal tear
D) Pes anserine bursitis
E) Tibial apophysitis (Osgood-Schlatter disease)

A

ANSWER: D
This patient has medial knee pain related to repetitive use, most likely caused by pes anserine bursitis. Iliotibial band syndrome is often related to overuse but causes pain in the lateral knee. The fibular head is also lateral to the knee joint. Osgood-Schlatter disease is also often related to overuse but causes pain at the insertion of the patellar ligament on the midline proximal tibia. A medial meniscal tear would localize to the medial joint line rather than distal to the joint line and would more likely be associated with positive findings from other examinations, such as a McMurray test.

How well did you know this?
1
Not at all
2
3
4
5
Perfectly
315
Q

When titrating the dosage of opioids, the CDC recommends that you should also consider prescribing naloxone when the opioid dosage reaches what morphine milligram equivalent (MME) per day threshold?
A) 30 B) 50 C) 80 D) 90 E) 100

A

ANSWER: B
To mitigate the risk of opioid harm, it is essential to understand morphine milligram equivalents (MME). The evidence shows that the risk of an opioid overdose increases at the threshold of 50 MME/day. It is therefore recommended by the CDC that a prescription for naloxone be ordered when an opioid dosage reaches 50 MME/day, which is a high dosage. In general one should avoid prescribing 90 MME/day because of the substantially higher risk of an overdose at this dosage level.

How well did you know this?
1
Not at all
2
3
4
5
Perfectly
316
Q

A 62-year-old female with stage 3 chronic kidney disease and an estimated glomerular filtration rate of 37 mL/min/1.73 m2 is found to have a mildly low ionized calcium level. Which one of the following would you expect to see if her hypocalcemia is secondary to her chronic kidney disease?
A) Elevated parathyroid hormone (PTH) and elevated phosphorus
B) Elevated PTH and low phosphorus
C) Low PTH and elevated phosphorus
D) Low PTH and low phosphorus

A

ANSWER: A
Chronic kidney disease–mineral and bone disorder (CKD-MBD) is found in many patients with CKD and is associated with an increased risk of bone fractures and cardiovascular events due to vascular calcification. In patients with CKD, phosphate is not appropriately excreted and the subsequent hyperphosphatemia leads to secondary hyperparathyroidism and binding of calcium. Decreased production of calcitriol in patients with CKD also leads to hypocalcemic hyperparathyroidism. Patients with CKD stages 3a–5 should have phosphorus, calcium, parathyroid hormone, and 25-hydroxyvitamin D levels checked regularly, and consultation with a nephrologist or endocrinologist should be obtained if CKD-MBD is suspected.

How well did you know this?
1
Not at all
2
3
4
5
Perfectly
317
Q

A 63-year-old female sees you for evaluation of recurrent right foot swelling and redness. She has a history of obesity and type 2 diabetes with retinopathy, nephropathy, and peripheral neuropathy. She presented with similar symptoms 2 weeks ago and was diagnosed with cellulitis and treated with a 10-day course of amoxicillin/clavulanate (Augmentin). Her symptoms seemed to initially improve with this therapy along with elevation of the foot but then worsened. She does not have any pain in the foot, fever, or chills. She does not recall any trauma or other inciting event.
The patient’s vital signs include a temperature of 37.1°C (98.8°F), a pulse rate of 72 beats/min, and a blood pressure of 124/82 mm Hg. Her right foot appears swollen, red, and warm to the touch, and is not tender to palpation. There are no open sores or calluses. Her dorsalis pedis pulse is 2+. Monofilament testing confirms a diagnosis of peripheral neuropathy. A WBC count is normal. Radiographs reveal soft-tissue edema with no other abnormalities.
The most appropriate treatment at this point would be
A) immobilization
B) antibiotics
C) bisphosphonates
D) corticosteroids
E) surgical repair

A

ANSWER: A
This patient has acute Charcot neuroarthropathy, an inflammatory condition that occurs in obese patients with peripheral neuropathy and ultimately leads to foot deformities (the classic rocker-bottom foot) and resultant ulcerations and infections. Its clinical appearance can easily be initially mistaken for cellulitis. However, the absence of tenderness and other signs of infection such as fever, an elevated WBC count, and inflammatory markers is not consistent with cellulitis. Radiography is an appropriate initial imaging modality but the results are often interpreted as normal early in the disease process. MRI is the modality of choice for a definitive diagnosis and may demonstrate periarticular bone marrow edema, adjacent soft-tissue edema, joint effusion, and microtrabecular or stress fractures.
The treatment of acute Charcot neuroarthropathy is immobilization with total contact casting, which increases the total surface area of contact to the entire lower extremity, distributing pressure away from the foot. Immobilization is typically required for at least 3–4 months but in some cases may be needed for up to 12 months. Bisphosphonates were found to be ineffective as adjunctive therapy in acute Charcot neuroarthropathy. Corticosteroids and antibiotics have no role in the treatment of Charcot foot but would be appropriate therapy for cellulitis or gout, which are important alternative diagnoses to consider. The role of surgery is more controversial but may be indicated in the acute phase of Charcot neuroarthropathy in patients with severe dislocation or instability.

How well did you know this?
1
Not at all
2
3
4
5
Perfectly
318
Q

A 38-year-old female with a 6-month history of mild shortness of breath associated with some intermittent wheezing during upper respiratory infections presents for follow-up. You previously prescribed albuterol (Proventil, Ventolin) via metered-dose inhaler, which she says helps her symptoms. You suspect asthma. Pulmonary function testing reveals a normal FEV1/FVC ratio for her age.
Which one of the following would be the most appropriate next step?
A) Consider an alternative diagnosis
B) Assess her bronchodilator response
C) Perform a methacholine challenge
D) Prescribe an inhaled corticosteroid
E) Proceed with treatment for COPD

A

ANSWER: C
Spirometry is central to confirming the diagnosis of asthma, which is characterized by a reversible obstructive pattern of pulmonary function. In this case the patient’s FEV1/FVC ratio is normal, which neither confirms nor rules out asthma. A methacholine challenge is recommended in this scenario to assess for the airway hyperresponsiveness that is the hallmark of asthma. Methacholine is a cholinergic agonist. Bronchoconstriction (defined as a reduction in FEV1 20%) observed at low levels of methacholine administration (<4 mg/mL) is consistent with asthma. If the FEV1/FVC ratio is reduced on initial spirometry, a bronchodilator response should be tested. A fixed or partially reversible obstructive pattern suggests an alternative diagnosis such as COPD, and full reversal after bronchodilator use is consistent with asthma. Inhaled corticosteroids are not appropriate for intermittent asthma.

How well did you know this?
1
Not at all
2
3
4
5
Perfectly
319
Q

A 67-year-old male diagnosed with polymyalgia rheumatica is started on long-term prednisone therapy. Which one of the following is the recommended first-line agent to prevent steroid-induced osteoporosis?
A) Alendronate (Fosamax)
B) High-dose vitamin D
C) Raloxifene (Evista)
D) Teriparatide (Forteo)

A

ANSWER: A
Patients are at risk of developing glucocorticoid-induced osteoporosis if they are on long-term glucocorticoid therapy, defined as >2.5 mg of prednisone for a duration of 3 months or longer. The American College of Rheumatology recommends pharmacologic treatment for these patients, as well as for patients receiving glucocorticoids who have a bone mineral density T-score –2.5 at either the spine or the femoral neck and are either male and 50 years of age or female and postmenopausal. Therapy is also recommended in patients 40 years of age who do not meet these criteria but have a 10-year risk of major osteoporotic fracture of at least 20% or a risk of hip fracture of at least 3% according to the FRAX tool.
Oral bisphosphonates are recommended as first-line agents for preventing glucocorticoid-induced osteoporotic fractures, although intravenous bisphosphonates can be used if patients are unable to use the oral forms. Supplementation of calcium (800–1000 mg) and vitamin D (400–800 IU) is also recommended. Raloxifene and teriparatide are options when bisphosphonate therapy fails or is contraindicated (SOR A).

How well did you know this?
1
Not at all
2
3
4
5
Perfectly
320
Q

At a routine well child check, the mother of an 18-month-old female expresses concern about the child’s development. Which one of the following should prompt consideration of a developmental delay?
A) A vocabulary of less than six words
B) Failure to point to pictures or body parts when named
C) Inability to follow one-step directions
D) Inability to run well
E) Inability to copy a vertical line

A

ANSWER: C
At 18 months of age a child should follow one-step directions. Approximately 90% of 18-month-olds say at least three words, and 50%–90% say six words. The ability to point to body parts or pictures after they are named is expected at 2 years of age. Not walking at 18 months would be a red flag for delay, but running well may not yet be accomplished. At 18 months a child would be expected to scribble spontaneously but not to copy a vertical line.

How well did you know this?
1
Not at all
2
3
4
5
Perfectly
321
Q

A 22-year-old male presents to your office the morning after falling onto his outstretched right hand as he tripped while leaving a bar. He has a deep, dull ache in the right wrist on the radial side. The pain is worsened by gripping and squeezing. On examination there is some wrist fullness and the wrist is tender to palpation over the anatomic snuffbox. Radiographs of the wrist are negative.
Which one of the following would be most appropriate at this time?
A) Rest, ice, compression, elevation, and NSAIDs with no specific follow-up
B) Rest, ice, compression, elevation, and NSAIDs with a follow-up examination in 2 weeks
C) Placement of a thumb spica splint, with a follow-up examination in 2 weeks
D) CT of the wrist to detect an occult fracture
E) Ultrasonography of the wrist to detect a ligament injury

A

ANSWER: C
The history, symptoms, and physical examination findings in this case suggest a scaphoid fracture. The scaphoid bone is the most commonly fractured carpal bone and a fall on an outstretched hand can produce enough force to cause this fracture. This fracture is most common in males 15–30 years of age.
The finding of anatomic snuffbox tenderness is highly sensitive but not specific for a scaphoid fracture. Initial radiographs often do not demonstrate a fracture. When there is a high clinical suspicion for a scaphoid fracture but radiographs are negative, it is reasonable to immobilize in a thumb spica splint and reevaluate in 2 weeks.

How well did you know this?
1
Not at all
2
3
4
5
Perfectly
322
Q

A 28-year-old white female comes to your office at 37 weeks gestation with a 24-hour history of painful vesicles on the vulva. She does not have a past history of similar lesions. You make a presumptive diagnosis of genital herpes.
Of the following, the most sensitive and specific test is
A) exfoliative cytology (Tzanck test)
B) a polymerase chain reaction (PCR) test
C) an enzyme-linked immunosorbent assay (ELISA)
D) HSV serology (IgG/IgM)

A

ANSWER: B
When genital herpes occurs during pregnancy, the best method of diagnosis is either a tissue culture or a polymerase chain reaction (PCR) test, which is more sensitive. Enzyme-linked immunosorbent assays are sensitive, but not as sensitive or specific as PCR.

How well did you know this?
1
Not at all
2
3
4
5
Perfectly
323
Q

A 55-year-old patient with a history of alcoholism is admitted through the emergency department with acute pancreatitis. Which one of the following tests performed at the time of admission can best predict the severity of pancreatitis?
A) Hematocrit
B) C-reactive protein
C) Serum amylase
D) Serum lipase
E) CT of the abdomen

A

ANSWER: A
Knowing the severity of pancreatitis helps predict how aggressive management should be. Hematocrit, BUN, and creatinine levels are the most useful predictors of the severity of pancreatitis, reflecting the degree of intravascular volume depletion. C-reactive protein is often elevated, but it is not as useful as hematocrit for predicting severity. Serum amylase and lipase have no prognostic value. CT evidence of severe pancreatitis lags behind clinical and laboratory evidence, and early CT underestimates the severity of the acute process.

How well did you know this?
1
Not at all
2
3
4
5
Perfectly
324
Q

A 34-year-old male with sickle cell disease has a new onset of mild to moderate thirst and polyuria. He ate a large meal about 2 hours ago.
An examination reveals a BMI of 32 kg/m2. Results of a urinalysis performed by your staff include 3+ glucose and no ketones. His blood glucose level is 288 mg/dL and his hemoglobin A1c is 5.2%.
Which one of the following would be most appropriate at this point to help diagnose and monitor this patient’s glycemic control?
A) A serum fructosamine level
B) A repeat hemoglobin A1c
C) A 2-hour glucose tolerance test
D) Hemoglobin electrophoresis
E) Referral to an endocrinologist

A

ANSWER: A
This patient with sickle cell disease has a new onset of diabetes mellitus. Hemoglobinopathies falsely lower hemoglobin A1c as a result of hemolysis and abnormal glycation. Fructosamine correlates well with hemoglobin A1c levels and is recommended instead of hemoglobin A1c for monitoring glucose control in patients with diabetes and hemoglobinopathies. A 2-hour glucose tolerance test or hemoglobin electrophoresis would not provide useful information. Referral to an endocrinologist is not indicated at this point because the patient has not failed primary care management.

How well did you know this?
1
Not at all
2
3
4
5
Perfectly
325
Q

A 14-year-old male sees you for a well child examination. He had one dose of HPV vaccine at his last well child examination 1 year ago.
Which one of the following is true regarding HPV vaccine for this patient?
A) He does not require additional HPV vaccine
B) He should receive one dose of the vaccine now and no additional HPV vaccine in the
future
C) He should receive the vaccine now and again in 4 months
D) He should receive the vaccine now and again in 6 months
E) He should receive the vaccine now, in 2 months, and in 4 months

A

ANSWER: B
HPV vaccine is currently recommended for males and females at age 11. Catch-up vaccination is recommended until age 21 in males and 26 in females. Children who receive the first dose of the vaccine before the age of 15 and receive two doses are considered adequately vaccinated. If the first dose is given after age 15, a three-dose series is recommended.

How well did you know this?
1
Not at all
2
3
4
5
Perfectly
326
Q

A 34-year-old female presents to your office after she was bitten on the hand by a neighbor’s cat. The patient has no allergies and has been in good health. You decide to treat the patient with a prophylactic antibiotic.
Which one of the following would be the antibiotic of choice?
A) Amoxicillin/clavulanate (Augmentin)
B) Azithromycin (Zithromax)
C) Cephalexin (Keflex)
D) Clindamycin (Cleocin)
E) Metronidazole (Flagyl)

A

ANSWER: A
Prophylactic antibiotics should be given for all closed-fist injuries unless the skin has not been penetrated, and for puncture wounds caused by cat bites. The antibiotic should have both aerobic and anaerobic activity and include Pasteurella coverage for animal bites and Eikenella coverage for human bites. Suggested regimens include amoxicillin/clavulanate. If the patient is allergic to penicillin, clindamycin plus levofloxacin or moxifloxacin, which has anaerobic coverage, can be used. Azithromycin, cephalexin, and metronidazole are not first-line antibiotics following a cat bite.

How well did you know this?
1
Not at all
2
3
4
5
Perfectly
327
Q

A 30-year-old female presents for follow-up after an emergency department visit for an episode of symptomatic supraventricular tachycardia that was diagnosed as Wolff-Parkinson-White syndrome. Which one of the following would be most appropriate for the initial long-term management of this patient?
A) Adenosine (Adenocard)
B) Amiodarone (Cordarone)
C) Diltiazem (Cardizem)
D) Metoprolol
E) Catheter ablation

A

ANSWER: E
Catheter ablation is the most appropriate treatment for a patient with symptomatic Wolff-Parkinson-White syndrome (WPW). Catheter ablation has a very high immediate success rate (96%–98%). The most significant risk associated with the procedure is permanent atrioventricular block, which occurs in approximately 0.4% of procedures. Adenosine and amiodarone are used for the acute management of supraventricular tachycardia, but not for long-term management. Node-blocking medications such as diltiazem and metoprolol should not be used for the long-term treatment of WPW, due to the increased risk of ventricular fibrillation.

How well did you know this?
1
Not at all
2
3
4
5
Perfectly
328
Q

A 7-year-old male received one dose of trivalent inactivated influenza vaccine at another health care facility 5 weeks ago. This was the first time he received influenza vaccine, and it resulted in soreness at the injection site. His mother reports that he has had mild hives after eating peanuts and eggs in the past. Your office has stocked only quadrivalent inactivated influenza vaccine.
Which one of the following would you recommend?
A) No further influenza immunization this year
B) Immunization now with quadrivalent inactivated vaccine
C) Delaying immunization until trivalent inactivated vaccine can be given
D) Delaying immunization until 8 weeks after the first vaccine
E) No immunization because of a potential egg allergy

A

ANSWER: B
The CDC’s Advisory Committee on Immunization Practices recommends that patients with egg allergy receive influenza vaccination. Previously unvaccinated patients ages 6 months to 8 years should receive two doses of either trivalent or quadrivalent vaccine separated by 1 month.

How well did you know this?
1
Not at all
2
3
4
5
Perfectly
329
Q

Three weeks after he had knee surgery, a 64-year-old male presents for follow-up of an emergency department visit for a pulmonary embolism. He has no previous history of pulmonary embolism and is otherwise in good health. He is being treated with apixaban (Eliquis).
The recommended duration of anticoagulation therapy for this patient is
A) 1 month
B) 3 months
C) 6 months
D) 9 months
E) 12 months

A

ANSWER: B
Patients who have a venous thromboembolism (VTE) require anticoagulation therapy for treatment and prevention of recurrence. The risk of recurrence is greatest in the first year after the event and remains elevated indefinitely. The risk for VTE recurrence is dependent on patient factors, such as active cancers and thrombophilia. Current guidelines recommend treatment for at least 3 months. In patients who have a reversible provoking factor such as surgery, anticoagulation beyond 3 months is not recommended.

How well did you know this?
1
Not at all
2
3
4
5
Perfectly
330
Q

A 30-year-old gravida 2 para 1 in her second trimester is evaluated for hypothyroidism. The normal TSH range in pregnancy is
A) lower than in the nonpregnant state
B) higher than in the nonpregnant state
C) the same as in the nonpregnant state
D) not useful for evaluating hypothyroidism after the first trimester

A

ANSWER: A
The TSH reference range is lower during pregnancy because of the cross-reactivity of the -subunit of hCG. Levels of hCG peak during weeks 7–13 of pregnancy, and hCG has mild TSH-like activity, leading to slightly high free T4 levels in early pregnancy. This leads to a feedback decrease in TSH.

How well did you know this?
1
Not at all
2
3
4
5
Perfectly
331
Q

A 69-year-old male presents for an annual health maintenance examination. His medical history is significant for hypertension and worsening back pain over the last 6 months. Laboratory studies reveal a hemoglobin level of 8.6 g/dL (N 14.0–18.0) and a mean corpuscular volume of 88 m3 (N 80–94). The remainder of the CBC is normal. A peripheral smear and a ferritin level are both normal. A comprehensive metabolic panel is normal except for a serum creatinine level of 1.6 mg/dL (N 0.7–1.3).
Which one of the following would be the most appropriate next step in the evaluation of this patient?
A) A vitamin B12 level
B) A haptoglobin level
C) Serum protein electrophoresis
D) Flow cytometry
E) A bone marrow biopsy

A

ANSWER: C
This patient’s laboratory results and back pain suggest multiple myeloma (MM). He has a normocytic anemia and evidence of renal insufficiency, which can indicate MM. The laboratory findings along with worsening back pain indicate a need to order serum protein electrophoresis to look for MM. Flow cytometry is generally used in patients with an elevated WBC count and suspected lymphoma. The remainder of this patient’s CBC is normal, which makes a bone marrow issue less likely. His mean corpuscular volume is also normal, making vitamin B12 deficiency less likely. A haptoglobin level could be ordered, but protein electrophoresis is a better choice because the peripheral smear demonstrated no evidence of a hemolytic problem.

How well did you know this?
1
Not at all
2
3
4
5
Perfectly
332
Q

A 32-year-old female presents with heat intolerance, excessive weight loss, and anxiety. She gave birth 6 months ago and recently stopped breastfeeding. On examination her thyroid gland is slightly diffusely enlarged and nontender. Laboratory studies reveal a decreased TSH level and elevated free T3 and T4 levels. You suspect that she has postpartum thyroiditis.
Which one of the following tests would be most useful to confirm the diagnosis?
A) Radioactive iodine uptake
B) Thyroid peroxidase antibody levels
C) Thyroid ultrasonography
D) Thyrotropin receptor antibody levels

A

ANSWER: A
Postpartum thyroiditis is defined as a transient or persistent thyroid dysfunction that occurs within 1 year of childbirth, miscarriage, or medical abortion. Release of preformed thyroid hormone in the bloodstream initially results in hyperthyroidism. During the hyperthyroid phase, radioactive iodine uptake will be low, which can help to confirm the diagnosis. Pregnancy and breastfeeding are contraindications to radionuclide imaging. Thyroid peroxidase antibody levels are elevated with chronic autoimmune thyroiditis (Hashimoto’s thyroiditis), and patients present with symptoms of hypothyroidism. The Endocrine Society and American Association of Clinical Endocrinologists do not recommend routine thyroid ultrasonography in patients with abnormal thyroid function tests if there is no palpable abnormality of the thyroid gland. Thyrotropin receptor antibody levels are positive in Graves disease.

How well did you know this?
1
Not at all
2
3
4
5
Perfectly
333
Q

A patient comes to your outpatient clinic with a persistent migraine that she has been unable to treat effectively at home. The symptoms began several hours ago and are typical for her. She has already tried her usual treatments of ibuprofen, 800 mg, and rizatriptan (Maxalt), 10 mg, but they have not provided any relief. She took a second dose of rizatriptan 2 hours later without benefit. She is in significant pain, which is causing mild nausea, and she has photophobia and phonophobia.
Which one of the following would be most appropriate at this point?
A) Oral butalbital/acetaminophen/caffeine (Fioricet)
B) Oral ergotamine/caffeine (Cafergot)
C) Subcutaneous sumatriptan (Imitrex)
D) Intramuscular morphine
E) Intramuscular prochlorperazine

A

ANSWER: E
Multiple studies have determined that parenteral antiemetics have benefits for the treatment of acute migraine beyond their effect on nausea. Most outpatient clinics do not have the ability to administer intravenous metoclopramide, which is the preferred treatment. However, most clinics do have the ability to administer intramuscular prochlorperazine or promethazine. Due to concerns about oversedation, misuse, and rebound, treatment with parenteral opiates is discouraged but may be an option if other treatments fail. Oral butalbital/acetaminophen/caffeine and oral ergotamine/caffeine have less evidence of success in the treatment of acute migraine. Sumatriptan is contraindicated within 24 hours of the use of rizatriptan.

How well did you know this?
1
Not at all
2
3
4
5
Perfectly
334
Q

A 48-year-old male with an 8-week history of the gradual onset of nonradiating, worsening left heel pain sees you for follow-up. He started running on his neighborhood streets 3 months ago to train for a 10K race. The pain limits his training significantly. His symptoms improved when he began taking ibuprofen and took 2 weeks off from running a month ago. A radiograph of the left foot 4 weeks ago was normal. There is no other pain and he feels well otherwise.
On examination you note that his left heel is slightly swollen compared to the right and very tender when squeezed on the sides. The anterior aspect and Achilles tendon insertion of the heel are nontender. There is no erythema or warmth and the remainder of the left lower extremity examination is normal. His vital signs are normal.
Which one of the following would you recommend at this point?
A) A C-reactive protein level
B) Nerve conduction velocity testing
C) A repeat radiograph
D) Ultrasonography
E) MRI

A

ANSWER: E
This patient has a calcaneal stress fracture as suggested by the history of increased running on a hard surface, improvement with rest, and a positive calcaneal squeeze on examination. A delay in diagnosis increases the risk of delayed union. MRI is the preferred imaging modality because radiographs often do not detect a calcaneal stress fracture. A C-reactive protein level could be indicated if there were symptoms or signs of infection or autoimmune illness. The clinical picture does not suggest a neurologic condition, so nerve conduction velocity testing is not appropriate. While there are some case reports of the diagnosis of stress fractures using ultrasonography, this is not the preferred imaging method.

How well did you know this?
1
Not at all
2
3
4
5
Perfectly
335
Q

A 48-year-old female presents with the nail findings shown below. Her past medical history is significant for systemic lupus erythematosus.
This finding is known as
A) clubbing
B) koilonychia
C) leukonychia
D) onychomycosis
E) squamous cell carcinoma

A

ANSWER: B
Koilonychia, also known as spoon nail, is a finding that can be associated with multiple systemic conditions such as iron deficiency, hypothyroidism, and systemic lupus erythematosus. Clubbing of the nails involves thickening of the soft tissue proximal to the nail. Leukonychia is a white discoloration of the nail plate and can be a normal variant. Onychomycosis is a fungal infection generally presenting as a thickened, yellow, dystrophic nail. Squamous cell carcinoma presents as an erythematous, proliferating mass that disrupts normal nail morphology.

How well did you know this?
1
Not at all
2
3
4
5
Perfectly
336
Q

A previously healthy 45-year-old female presents with upper abdominal pain and dysphagia. An upper GI series reveals no significant reflux. On esophagogastroduodenoscopy the esophagus has a ringed appearance and a biopsy reveals >15 eosinophils/hpf. Helicobacter pylori testing is negative. She does not currently take any medications.
Which one of the following would be the best initial treatment?
A) Budesonide oral suspension, 1 mg twice daily
B) Fexofenadine, 180 mg daily
C) Pantoprazole (Protonix), 40 mg once daily
D) Prednisone, 40 mg daily for 7 days
E) Ranitidine (Zantac), 150 mg once daily

A

ANSWER: A
The clinical presentation and esophagogastroduodenoscopy findings indicate eosinophilic esophagitis (EoE) in this patient. In the absence of other causes of eosinophilia, the presence of >15 eosinophils/hpf is considered diagnostic. Application of corticosteroids to the esophagus is generally the treatment of choice, either in the form of an oral suspension of budesonide or an inhaled corticosteroid sprayed into the mouth and swallowed. Although EoE can occur in patients with other atopic illnesses, this patient does not have any symptoms of allergies or asthma, so an antihistamine such as fexofenadine is not needed. EoE does not respond to proton pump inhibitor therapy or H2-blocker therapy and systemic corticosteroids are not necessary.

How well did you know this?
1
Not at all
2
3
4
5
Perfectly
337
Q

A 72-year-old female presents for a routine health maintenance visit. Which one of the following medications in her current regimen places her at risk for osteoporosis?
A) Atorvastatin (Lipitor)
B) Hydrochlorothiazide
C) Metformin (Glucophage)
D) Phenytoin (Dilantin)
E) Ranitidine (Zantac)

A

ANSWER: D
Medications reported to be associated with osteoporosis and increased fracture risk include antiepileptic drugs, long-term heparin, cyclosporine, tacrolimus, aromatase inhibitors, glucocorticoids, gonadotropin-releasing hormone agonists, thiazolidinediones, excessive doses of levothyroxine, proton pump inhibitors, SSRIs, parenteral nutrients, medroxyprogesterone contraceptives, methotrexate, and aluminum antacids. Atorvastatin, hydrochlorothiazide, metformin, and ranitidine are not associated with osteoporosis.

How well did you know this?
1
Not at all
2
3
4
5
Perfectly
338
Q

A 46-year-old male who uses injectable heroin daily presents to establish primary care. He reports no symptoms and feels well overall. He does not smoke cigarettes, use alcohol, or use any drugs other than heroin. He works as an accountant for the federal government and has not had any arrests for illegal activity.
In addition to assessing his readiness for treatment of his addiction, you should order testing for
A) HIV only
B) HIV, hepatitis B, and hepatitis C
C) HIV, hepatitis B, hepatitis C, and TB
D) HIV, hepatitis A, hepatitis B, hepatitis C, and TB
E) HIV, hepatitis A, hepatitis B, hepatitis C, TB, gonorrhea, and Chlamydia

A

ANSWER: D
Persons who inject drugs are at increased risk for HIV, hepatitis A, hepatitis B, hepatitis C, and latent tuberculosis. These patients should be screened at the initial visit and treated for any conditions found, according to routine guidelines. If titers are low or absent for hepatitis A or B, the patient should be vaccinated. Although gonorrhea and Chlamydia screening is recommended for females under 25 years of age who use injectable drugs, it is not appropriate in this asymptomatic patient.

How well did you know this?
1
Not at all
2
3
4
5
Perfectly
339
Q

A 37-year-old female presents with a 3-day history of left thumb pain. She first noted diffuse pain in her left thumb after a fall while skiing. She does not recall the mechanism of injury. The pain is greatest at the medial metacarpophalangeal joint, but there is no mass or instability. A radiograph does not show any fractures.
An examination reveals mild laxity in her ulnar collateral ligament with 30° of joint opening when abduction stress is applied to the distal thumb while stabilizing the metacarpal. The right side has only 15° of joint opening.
Which one of the following would be most appropriate at this point?
A) Reassurance and follow-up if symptoms do not improve
B) Anti-inflammatory medication for 7–10 days
C) Immobilization of the left thumb in a thumb spica cast or brace for 6 weeks
D) Immediate referral to an orthopedic surgeon for surgical repair

A

ANSWER: C
Ulnar collateral ligament (UCL) disruption, or “skier’s thumb,” should be suspected in traumatic thumb injuries. It is important to recognize and treat this injury because it can lead to joint instability and a weak pincer grip if untreated. Initial treatment of UCL disruption involves immobilization of the affected thumb in a thumb spica cast or brace for 6 weeks. In the absence of an avulsion fracture, indications for referral to an orthopedic surgeon would include 35°–40° of joint opening or no end point on stress abduction testing. A Stener lesion (entrapment of the UCL outside of the adductor aponeurosis) would usually present with joint instability and a tender mass and would necessitate an orthopedic referral.

How well did you know this?
1
Not at all
2
3
4
5
Perfectly
340
Q

A 65-year-old female with hypertension, osteoporosis, and GERD presents to your office for a well woman visit. She reports no new symptoms or concerns. A review of laboratory work performed prior to her visit reveals lipid levels at goal, normal glucose and sodium levels, a calcium level of 10.6 mg/dL (N 8.6–10.3), an albumin level of 4.1 g/dL (N 3.6–5.1), and a 25-hydroxyvitamin D level of 35 ng/mL (N 20–50). Her calcium level was 10.5 mg/dL on a basic metabolic panel 6 months ago.
The patient’s medications include hydrochlorothiazide, 12.5 mg daily; lisinopril (Prinivil, Zestril), 10 mg daily; alendronate (Fosamax), 70 mg weekly; omeprazole (Prilosec), 20 mg daily as needed; and vitamin D, 2000 IU daily. The patient’s blood pressure is 110/60 mm Hg. An examination is normal.
In addition to ordering follow-up laboratory studies and scheduling a follow-up visit in 1 month, which one of the following would be most appropriate?
A) Discontinue alendronate
B) Discontinue hydrochlorothiazide
C) Discontinue lisinopril
D) Discontinue omeprazole and begin ranitidine (Zantac)
E) Increase vitamin D to 5000 IU daily

A

ANSWER: B
This patient has hypercalcemia with a normal albumin level. Hydrochlorothiazide can cause drug-induced hypercalcemia. Alendronate, lisinopril, and omeprazole do not cause hypercalcemia. A high vitamin D level can cause hypercalcemia, so increasing vitamin D is not appropriate at this point. A laboratory evaluation can help differentiate between PTH- and non–PTH-mediated hypercalcemia.

How well did you know this?
1
Not at all
2
3
4
5
Perfectly
341
Q

A 60-year-old patient is admitted to the hospital for a COPD exacerbation. For this patient, which one of the following would be the most appropriate prednisone dosage?
A) 40 mg daily for 5 days
B) 40 mg daily for 10 days
C) 60 mg daily, tapered over 6 days
D) 60 mg daily for 10 days

A

Item 137
ANSWER: A
For patients with a COPD exacerbation, systemic glucocorticoids can improve FEV1, improve oxygenation, shorten recovery time, and reduce the length of hospitalization (level of evidence A). Prednisone, 40 mg daily for 5 days, is recommended for COPD exacerbations. Studies have shown that oral administration is equally efficacious compared to the intravenous route. The duration of therapy should not be longer than 5–7 days (level of evidence A).

How well did you know this?
1
Not at all
2
3
4
5
Perfectly
342
Q

A 17-year-old female presents to your office with open and closed comedones on her nose, forehead, and chin. No cystic lesions are noted and the acne does not extend to her back or chest.
Of the following, which one would be most effective?
A) Topical adapalene (Differin) and benzoyl peroxide
B) Topical clindamycin (Cleocin T)
C) Topical salicylic acid
D) Topical tazarotene (Tazorac)
E) Oral norethindrone/ethinyl estradiol

A

ANSWER: A
Acne affects 85% of 12- to 25-year-olds in the United States. This patient has mild acne as evidenced by the absence of cystic lesions and localization to the facial T-zone. Topical retinoids are first-line treatment for any level of severity of acne (SOR A). Adapalene is an effective retinoid and is available over-the-counter. Benzoyl peroxide is also very effective in the control of acne because it reduces the concentration of cystic acne with no risk of bacterial resistance. The combination of a topical retinoid and benzoyl peroxide is more effective than either agent alone.
Topical antibiotics such as clindamycin and erythromycin should not be used as monotherapy due to high rates of microbial resistance. There is little evidence that salicylic acid is effective in combating acne despite its widespread use.
Tazarotene is effective in the treatment of acne but is teratogenic (pregnancy category X) and should be avoided in women of reproductive age. Combined oral contraceptives can be effective, but norethindrone and other first-generation progestins are androgenic and can worsen acne.

How well did you know this?
1
Not at all
2
3
4
5
Perfectly
343
Q

A 48-year-old male is brought to the medical tent of a marathon after he became confused and dizzy in the middle of the race. On examination his rectal temperature is 41.1°C (106.0°F).
Which one of the following should be initiated immediately?
A) Passive cooling in an air-conditioned space
B) Gradual cooling with a cold water spray and a fan
C) Ice packs applied to the groin, neck, and axilla
D) Full-body immersion in an ice bath
E) Cold intravenous fluids

A

ANSWER: D
Heatstroke can be nonexertional from prolonged exposure to a high heat index, or it can be exertional, as in this case. A core temperature >40°C (104°F) is consistent with heatstroke. In treating patients with either clinical variant of heatstroke, cold or ice-water immersion is the most effective treatment and should be initiated as soon as possible, without delaying for transfer to the hospital setting (SOR A). Treatment should continue until the core body temperature is <39°C (102°F). If cold water immersion is not possible other forms of cooling such as cold intravenous fluids, ice packs, cold water immersion of the extremities, and evaporative cooling have been shown to have some benefit. Once the body temperature is decreased patients should be transferred to a hospital for evaluation for known complications of heatstroke, including coagulopathy, renal and hepatic dysfunction, hypoglycemia, electrolyte disturbance, and rhabdomyolysis.

How well did you know this?
1
Not at all
2
3
4
5
Perfectly
344
Q

A 25-year-old landscaper presents with a 1-day history of pain and swelling in his left hand. Three days ago he sustained a puncture wound in the palm of the hand when he was stuck by a large thorn. He has generalized achiness and chills in addition to the hand pain. He has no significant past medical history.
On examination the patient’s temperature is 37.9°C (100.2°F). His left third finger is diffusely swollen, erythematous, and held in flexion. There is tenderness along the third tendon in the palm. A radiograph of the hand is negative for a fracture or foreign body.
Which one of the following would be the most appropriate next step?
A) Amoxicillin/clavulanate (Augmentin), 875/125 mg twice daily for 10 days
B) Incision and drainage in the office with a culture of the wound
C) A hand splint and nonurgent referral to an orthopedic surgeon
D) Urgent MRI of the hand
E) Urgent surgical consultation

A

ANSWER: E
Pyogenic flexor tenosynovitis usually develops 2–5 days after a penetrating hand injury. The flexor tendon sheath has a poor vascular supply and the synovial fluid is a prime growth medium for bacteria. Flexor tenosynovitis is a clinical diagnosis characterized by the four “Kanavel” signs: pain with passive extension, tenderness with palpation of the tendon sheath, flexed position of the involved finger, and fusiform swelling of the finger. Treatment includes prompt intravenous antibiotics and surgical debridement and irrigation.
Flexor tenosynovitis requires urgent surgical consultation and treatment. Patients with suspected flexor tenosynovitis should be seen by a surgeon within 72 hours of symptom onset (SOR C). Oral antibiotics and splinting of the hand alone are insufficient treatments for the condition. Incision and drainage would also not be sufficient to clear the infection. Ordering MRI can unnecessarily delay surgical consultation, although the surgeon may obtain one to guide treatment.

How well did you know this?
1
Not at all
2
3
4
5
Perfectly
345
Q

You are administering a mental status examination to a 92-year-old male with suspected dementia. You give the patient a pencil and ask him to show how it is used. He gives you a bewildered look and eventually puts the pencil in his mouth and demonstrates using it as a toothbrush.
This task assesses
A) executive functioning
B) gnosia
C) orientation
D) praxis
E) visuospatial proficiency

A

ANSWER: D
Praxis is the ability to carry out intentional motor acts and is commonly assessed by giving the patient a common object such as a hairbrush or pencil and asking the patient to show how it is used. A patient unable to carry out such motor acts is referred to as having apraxia (SOR C). Several other common components of the cognitive assessment will be impaired in persons with dementia. Executive functioning is the ordering and implementation of cognitive functions necessary to engage in appropriate behavior and is often assessed by asking a patient to draw a clock with the hands set at a certain time.
Gnosia is the ability to name objects and their function and is often assessed by showing a patient a common object such as a pen, watch, or stethoscope and asking whether he or she can identify it and describe how it is used. Orientation is the ability of the patient to recognize his or her place in time and space. Orientation is commonly assessed by asking a patient the date, the current location, his or her name, and his or her place of birth. Visuospatial proficiency is the ability to perceive and manipulate objects and shapes in space. It is often assessed by asking the patient to copy intersecting pentagons or a three-dimensional cube on paper.

How well did you know this?
1
Not at all
2
3
4
5
Perfectly
346
Q

An 8-year-old male was brought to your office 7 months ago because of frequent diarrhea and abdominal discomfort that had been present for 11⁄2–2 years. An examination revealed no abnormalities. A CBC, a serum iron level, and a metabolic panel were normal. Serology was consistent with celiac disease and a duodenal biopsy confirmed the diagnosis. The patient’s symptoms resolved after his parents took him to a nutritionist who recommended a gluten-free diet. Today you see the child for preventive care and he remains asymptomatic.
In addition to normal well child care, which one of the following would you recommend?
A) No further testing
B) Gradual reintroduction of gluten into the diet
C) IgG antigliadin antibody
D) IgA tissue transglutaminase antibody
E) A duodenal mucosal biopsy to ensure healing

A

ANSWER: D
Celiac disease affects approximately 1% of the U.S. population and can affect all ages. Individuals with northern European ancestry are most commonly affected. The condition is caused by autoimmunity induced by gluten-containing foods in susceptible individuals. Untreated celiac disease is associated with anemia, malabsorption, osteoporosis, weight loss, and gastrointestinal lymphomas. In children, growth stunting and delayed puberty are also common. With strict adherence to a gluten-free diet most complications from celiac disease are preventable and, in children, growth and development return to normal. The World Gastroenterology Organisation recommends annual monitoring of children and adolescents with celiac disease by anthropometry, pubertal development, and celiac serology. The preferred serology is IgA antibody to tissue transglutaminase (IgA anti-tTG) due to its balance of good performance and low cost. Failure of IgA anti-tTG titers to decrease in 6 months suggests continued ingestion of gluten. Repeat duodenoscopy with a biopsy would be unnecessary and would subject the child to potential harm.

How well did you know this?
1
Not at all
2
3
4
5
Perfectly
347
Q

A 13-year-old male is admitted to the hospital with diabetic ketoacidosis. Aggressive fluid resuscitation with normal saline was initiated in the emergency department and the following laboratory results were obtained:
Glucose Sodium  Potassium  Bicarbonate Aniongap
400mg/dL
136 mEq/L (N 136–145) 2.8 mEq/L (N 3.5–5.1) 15mEq/L(N22–29) 14mEq/L(N10–20)
In addition to continued fluid resuscitation, which one of the following would be the most appropriate next step in the management of this patient?
A) Administration of sodium bicarbonate
B) Potassium replacement
C) An intravenous insulin drip
D) Subcutaneous insulin using a basal/bolus technique
E) Bedside ketone capillary measurement

A

ANSWER: B
Correcting the potassium level is the best treatment choice for this patient. A low serum potassium level in diabetic ketoacidosis (DKA) indicates a significant potassium deficiency, placing the patient at risk for a cardiac arrhythmia, among other complications. Potassium deficiency is usually the product of urinary losses due to glucose osmotic diuresis and secondary hyperaldosteronism. However, serum potassium can remain normal when there is a whole body deficiency, as a result of movement of potassium out of cells in response to the acidosis, insulin deficiency, and hyperosmolality. This patient’s serum potassium is low, which indicates severe deficiency.
55
Sodium bicarbonate can be used in some cases of DKA but is not the most important next step in this case. If insulin therapy is initiated prior to potassium replacement the insulin can force more potassium into cells, worsening the serum deficiency. If the serum potassium level is <3.3 mEq/L potassium should be replaced prior to insulin therapy. Ketone measurement can also be part of DKA management but is not the most appropriate next step for this patient.

How well did you know this?
1
Not at all
2
3
4
5
Perfectly
348
Q

A 25-year-old male presents to your office with anxiety. During questioning he reveals that he struggles with opioid addiction since he underwent an appendectomy 2 years ago, and is anxious because of random drug screens in his workplace. He averages using about 30 mg of hydrocodone daily and goes through cycles of use and withdrawal. Screening is negative for alcohol and substance use disorders other than opioids, and for depression or other mental health disorders. You consider maintenance treatment for opioid use disorder with buprenorphine.
For this patient, buprenorphine therapy
A) is inferior to methadone maintenance therapy
B) should not be combined with naloxone therapy because of potential side effects
C) should be initiated when he is in mild to moderate withdrawal from opioids
D) can be initiated only after inpatient detoxification
E) should be continued for 1 year and then tapered

A

ANSWER: C
Buprenorphine therapy is an important option for maintenance therapy for patients with opioid use disorder. It can be initiated in the outpatient setting but should be done when the patient is in mild to moderate withdrawal in order to avoid the risk of precipitated withdrawal (SOR C). Buprenorphine therapy is more convenient than methadone maintenance therapy and is equally as effective. Buprenorphine/naloxone combinations are preferred over buprenorphine monotherapy due to lower abuse potential, except when naloxone is contraindicated such as in patients who are pregnant or breastfeeding (SOR C). Because relapse rates are higher in patients who discontinue medication-assisted therapy for opioid use disorder, long-term use is recommended.

How well did you know this?
1
Not at all
2
3
4
5
Perfectly
349
Q

A 45-year-old female presents with pain in her right hand and wrist. She is right hand–dominant and most symptoms are in the right hand and wrist. She has been awakening at night with numbness of the hand that improves after she shakes her wrist. Her work as a software engineer involves lots of typing.
On examination you note a positive Tinel’s sign at the right wrist consistent with carpal tunnel syndrome. She has an important project at work over the next several months and will not be able to take any time off.
Which one of the following treatments has the best evidence for delaying the need for definitive surgical therapy?
A) Night splints
B) Physical therapy
C) Therapeutic ultrasound
D) Oral corticosteroids
E) Corticosteroid injection

A

ANSWER: E
Carpal tunnel syndrome of mild to moderate severity can be treated nonsurgically. Patients with severe symptoms or nerve damage seen on electromyography should be referred for surgical therapy. Nonsurgical management options include splinting, physical therapy, therapeutic ultrasound, and corticosteroids (oral or injection). Oral prednisone, 20 mg daily, for 10–14 days improves symptoms and function compared with placebo, for up to 8 weeks, but oral corticosteroids are less effective than corticosteroid injections. In a 2013 double-blind, placebo-controlled, randomized clinical trial comparing methylprednisolone injection (40 mg and 80 mg) to saline injection, patients in the 80-mg injection group were less likely to have surgery at 12 months.
While corticosteroid injections have the best evidence for delaying the need for surgery, night splints, physical therapy, and therapeutic ultrasound have some evidence of benefit. Splinting was found to be effective in a Cochrane review. It is low cost and safe and especially recommended in pregnancy-related disease. Limited evidence supports the use of physical therapy to treat carpal tunnel syndrome. Nerve glide exercises are simple hand and finger movements that are easy to learn, can be performed at home, and can be combined with other treatments such as splinting. Therapeutic ultrasound also has limited evidence of benefit. It requires an experienced therapist and requires multiple sessions, typically 5 days/week for 2–4 weeks.

How well did you know this?
1
Not at all
2
3
4
5
Perfectly
350
Q

Which one of the following is an indication to consider ultrasonography to screen for developmental dysplasia of the hip in a newborn?
A) A firstborn female infant with a vertex presentation at delivery
B) A firstborn male infant with a vertex presentation at delivery
C) A large-for-gestational-age male infant with a vertex presentation at delivery
D) A male infant who was in a breech position until 28 weeks gestation
E) A female infant with a first degree relative with developmental dysplasia of the hip

A

ANSWER: E
Screening for developmental dysplasia of the hip (DDH) is somewhat controversial because the benefit of treatment remains somewhat unclear. Despite the widespread practice of screening for DDH, ethical newborn practices are difficult to determine. The American Academy of Family Physicians and the U.S. Preventive Services Task Force have found insufficient evidence to recommend routine screening for DDH. The American Academy of Pediatrics, however, recommends routine screening of all newborns with physical examination maneuvers, and targeted screening ultrasonography for infants who were breech in the third trimester, have a family history of DDH, or have a personal history of instability. Given this, decisions should be individualized. Additional risk factors include female sex, firstborn status, oligohydramnios, and a large-for-gestational-age infant.

How well did you know this?
1
Not at all
2
3
4
5
Perfectly
351
Q

Which one of the following is recommended regarding oral fluoride supplementation?
A) Supplementation for all children whose primary water source is well water
B) Starting supplementation at 6 months of age if the primary water supply is fluoride
deficient
C) Starting supplementation at 2 years of age even in children who have received topical
fluoride varnish
D) No supplementation if fluoride varnish is applied once all primary teeth have come in
or by 3 years of age, then yearly thereafter
E) No supplementation for most children

A

ANSWER: B
The U.S. Preventive Services Task Force recommends oral fluoride supplementation for the prevention of dental caries beginning at age 6 months for children whose primary water supply is fluoride deficient (B recommendation). Well water may be fluoridated naturally depending on the aquifer, but the water is highly variable and should be tested before deciding on the need for supplementation. Testing well water is also advisable because excessive fluoride may lead to fluorosis of the bones. Bottled water is variable, making it undependable as an adequate source of fluoride. Topical fluoride, in toothpaste or applications of fluoride varnish, is effective in preventing tooth decay in children and can be used in addition to properly fluoridated water. Twice-yearly application of fluoride varnish to primary teeth should begin when the first tooth comes in and repeated every 6 months thereafter in children (SOR B).

How well did you know this?
1
Not at all
2
3
4
5
Perfectly
352
Q

A 48-year-old female presents to your office for follow-up of painful rectal bleeding with bowel movements. She has increased her fiber consumption and is using a stool softener as you recommended at her last visit 2 weeks ago. She reports that her pain has worsened since yesterday and she is very uncomfortable. An examination reveals a firm and tender right posterior hemorrhoid below the dentate line.
Which one of the following would be the most appropriate next step in providing relief for this thrombosed hemorrhoid?
A) Topical corticosteroids
B) Bioflavonoid supplements
C) Oral antibiotics
D) Rubber band ligation
E) Excision

A

ANSWER: E
This patient has tried first-line treatment for hemorrhoids with increased fiber intake but has returned with symptoms of a thrombosed external hemorrhoid. Office-based surgical excision of the thrombosed external hemorrhoid within 2–3 days of symptom onset may provide significant symptomatic relief (SOR B) and result in a lower risk of recurrence. While conservative treatment with topical therapies such as corticosteroids may be helpful, symptomatic relief is prolonged with excision of the thrombosed hemorrhoid.
Bioflavonoids are used outside the United States for symptomatic treatment of hemorrhoids but evidence is lacking and they are not approved by the FDA for this use. Oral antibiotic therapy has no role in the treatment of thrombosed external hemorrhoids but may be beneficial in treating an abscess, which would present with a gradual onset of pain and a fluctuant rectal mass. Rubber band ligation is an appropriate treatment for grades I–III internal hemorrhoids (SOR A).

How well did you know this?
1
Not at all
2
3
4
5
Perfectly
353
Q

A 15-year-old male presents with a 2-day history of dark-colored urine, lower extremity edema, and fatigue. Approximately 2 weeks ago he said he had a “bad sore throat” that was treated empirically with amoxicillin. On examination his blood pressure is 144/92 mm Hg, his pulse rate is 76 beats/min, and his other vital signs are normal. Other than mild dependent edema there are no additional significant physical examination findings. A urinalysis dipstick shows 3+ hematuria.
Which one of the following findings on microscopic evaluation of the urine sediment would help to confirm the diagnosis in this patient?
A) Gram-positive cocci in chains
B) RBC casts
C) WBC casts
D) Eosinophils
E) Oxalate crystals

A

ANSWER: B
This is a classic presentation for acute poststreptococcal glomerulonephritis (APSGN), with the onset of gross hematuria associated with hypertension and systemic edema. This is most commonly seen in school-age children, usually 1–2 weeks after an episode of pharyngitis or 3–4 weeks after an episode of impetigo, caused by so-called nephritogenic strains of Group A -hemolytic Streptococcus. The hematuria is caused by immune complex–mediated glomerular injury.
66
Bacteriuria may be seen in both upper and lower urinary tract infections, but may also be a spurious finding, especially with the combined presence of epithelial cells. The classic finding on microscopic urinalysis for acute glomerulonephritis is the presence of RBC casts. WBC casts are seen with acute pyelonephritis. The presence of urinary eosinophils indicates acute interstitial nephritis. Calcium oxalate makes up the most common type of kidney stones.
Antibiotics prescribed for antecedent pharyngitis do not prevent APSGN. Treatment is supportive, controlling blood pressure and edema with a thiazide or a loop diuretic. The prognosis for resolution and full recovery of the vast majority of patients with APSGN is excellent, especially in the pediatric age group.

How well did you know this?
1
Not at all
2
3
4
5
Perfectly
354
Q

You have diagnosed a Chlamydia infection in a 24-year-old male. He reports having three sexual partners in the last month. He does not feel comfortable revealing their names or genders at this time.
In almost all U.S. states, which one of the following would be the most appropriate plan for timely treatment of the involved parties with azithromycin (Zithromax) as a single dose?
A) One prescription in the patient’s name
B) One prescription in the patient’s name and instructions to the patient to inform his
partners
C) One prescription in the patient’s name with three refills
D) One prescription in the patient’s name and three prescriptions for expedited partner
therapy
E) One prescription in the patient’s name and three blank prescriptions

A

ANSWER: D
Expedited partner therapy (EPT) is the clinical practice of treating the sex partners of patients diagnosed with Chlamydia or gonorrhea by providing prescriptions or medications to the patient to take to his/her partner without the health care provider first examining the partner. When patients have been diagnosed with gonorrhea or Chlamydia, EPT has been shown to reduce the overall burden of disease in a given population when the partners cannot be linked to care. In this case, as the partners’ identities are unknown, it is impossible for the clinician to examine them or even contact them, so four prescriptions should be written, one with the patient’s name and the other three for EPT. The CDC states that EPT is particularly effective in treating the female partners of infected males. The CDC considers having the partners visit a health care provider to be the optimal course of treatment but this is not often practically feasible due to a lack of resources or social factors.
Although conventional practice is to treat only the patient, this does not provide timely treatment for the patient’s partners. A prescription written to “EPT” can be filled at the pharmacy without the individual’s name or date of birth. Kentucky and South Carolina are the only states that do not allow this practice. A prescription with three refills would be unethical, as presumably the physician would be advising the patient to distribute medications that had been prescribed to only the patient. Blank prescriptions would require the partners to reveal their identities, which may lead to a reluctance to fill the prescriptions.

How well did you know this?
1
Not at all
2
3
4
5
Perfectly
355
Q

A 24-year-old male presents with swelling of the right testicle. The pain started yesterday and has persistently worsened. A physical examination reveals swelling on the right side of his scrotum. His entire testicle is painful, and elevation of the scrotum improves his pain. Ultrasonography reveals hyperemia, swelling, and increased blood flow to the testis and epididymis.
Which one of the following would be the most appropriate management?
A) Doxycycline
B) Levofloxacin (Levaquin)
C) Trimethoprim/sulfamethoxazole (Bactrim)
D) Doxycycline plus ceftriaxone
E) Urgent surgical evaluation

A

ANSWER: D
This patient presents with acute epididymitis. Typical symptoms develop gradually over 1–2 days with posterior scrotal pain and swelling. Additional symptoms may include fever, hematuria, dysuria, and urinary frequency. The pain may radiate to the lower abdomen. Physical examination findings may include tenderness of the epididymis and testis along with swelling of the scrotum. Elevation of the scrotum may decrease the pain (Prehn sign). Typical ultrasound findings include hyperemia, swelling, and increased blood flow to the epididymis.
With testicular torsion the pain is often sudden in onset and severe, with associated nausea and vomiting and no other urologic symptoms. A physical examination often demonstrates a high-riding testis that may lie transversely in the scrotum. The cremasteric reflex may be absent. Ultrasound findings would demonstrate decreased or absent blood flow with testicular torsion.
In sexually active adults <35 years of age, gonorrhea and Chlamydia are the most common causative organisms of acute epididymitis. Ceftriaxone, 250 mg intramuscularly or intravenously once, with oral doxycycline, 100 mg twice daily for 10 days, would be the appropriate treatment for acute epididymitis (SOR C). In men over the age of 35 or those with a history of recent urinary tract surgery or instrumentation, enteric organisms are the most likely cause and monotherapy with oral levofloxacin or ofloxacin for 10 days would be the recommended treatment.

How well did you know this?
1
Not at all
2
3
4
5
Perfectly
356
Q

A 70-year-old female comes to your office with a 10-day history of a subjective fever at home, facial and tooth pain, sinus pressure, and a green nasal discharge. There has been no change in her symptoms. The patient has a history of allergic rhinitis treated with immunotherapy, and a history of developing hives while taking penicillin. On examination her temperature is 38.1°C (100.6°F).
In addition to nasal saline and analgesics, which one of the following would be the most appropriate management?
A) Standard-dose amoxicillin/clavulanate (Augmentin)
B) High-dose amoxicillin/clavulanate
C) Azithromycin (Zithromax)
D) Doxycycline

A

ANSWER: D
This patient has a history of symptoms consistent with acute bacterial rhinosinusitis that have persisted for 10 days, warranting empiric antibiotic therapy (SOR B). Doxycycline is an appropriate alternative to amoxicillin/clavulanate for a patient with a history of a reaction to penicillin. Macrolides and trimethoprim/sulfamethoxazole are not recommended as empiric therapy because of high rates of resistance.

How well did you know this?
1
Not at all
2
3
4
5
Perfectly
357
Q

A 24-year-old patient wants to start the process of transitioning from female to male. He has been working with a psychiatrist who has confirmed the diagnosis of gender dysphoria.
Which one of the following would be the best initial treatment for this patient?
A) Clomiphene
B) Letrozole (Femara) C) Leuprolide (Eligard)
D) Spironolactone (Aldactone) E) Testosterone

A

ANSWER: E
For patients with gender dysphoria or gender incongruence who desire hormone treatment, the treatment goal is to suppress endogenous sex hormone production and maintain sex hormone levels in the normal range for their affirmed gender. For a female-to-male transgender patient this is most easily accomplished with testosterone. When testosterone levels are maintained in the normal genetic male range, gonadotropins and ovarian hormone production is suppressed, which accomplishes both goals for hormonal treatment without the need for additional gonadotropin suppression from medications such as leuprolide.
Clomiphene can increase serum testosterone levels, but only in the presence of a functioning testicle. Letrozole is an estrogen receptor antagonist, but it would not increase serum testosterone levels. Spironolactone has androgen receptor blocking effects and would not accomplish either of the hormone treatment goals.

How well did you know this?
1
Not at all
2
3
4
5
Perfectly
358
Q

Based on American Cancer Society guidelines for cervical cancer screening, when should HPV DNA co-testing first be performed along with Papanicolaou testing?
A) At the onset of sexual activity
B) At age 21
C) At age 25
D) At age 30
E) At age 35

A

ANSWER: D
According to American Cancer Society guidelines for cervical cancer screening, Papanicolaou (Pap) testing should begin at age 21 irrespective of sexual activity and should be continued every 3 years until age 29. The preferred screening strategy beginning at age 30 is Pap testing with HPV co-testing, which should be continued every 5 years until age 65. Cervical screening may be discontinued at that time if the patient’s last two tests have been negative and the patient was tested within the previous 5 years.

How well did you know this?
1
Not at all
2
3
4
5
Perfectly
359
Q

A 24-year old female presents to your office with a 3-month history of difficulty sleeping. She says that she struggles to fall asleep and wakes up multiple times at night at least three times a week. She tries to go to bed at 10:00 p.m. and wakes up at 6:30 a.m. to start her day. She lies awake for an hour in bed before falling asleep and spends up to 2 hours awake in the middle of the night trying to fall back asleep. Lately she has been feeling fatigued and having difficulty concentrating at work. You conduct a full history and physical examination and tell her to return in 2 weeks with a sleep diary. At this follow-up visit you see from her diary that she is sleeping an average of 51⁄2 hours per night.
Which one of the following would be the most appropriate recommendation?
A) Set her alarm for 5:30 a.m.
B) Add a mid-afternoon nap
C) Move her bedtime to 9:00 p.m.
D) Move her bedtime to 12:30 a.m.
E) Stay up for an hour if she wakes up at 3:00 a.m.

A

ANSWER: D
This patient presents with symptoms of chronic insomnia. Cognitive-behavioral therapy for insomnia (CBT-I) and brief behavioral therapy for insomnia (BBT-I) are effective nonpharmacologic treatments for chronic insomnia. Modified CBT-I and BBT-I can be administered by a primary care physician. The basic principles include stimulus control (sleep hygiene) and sleep restriction. Reducing time in bed increases sleep efficiency. In this case, 6 hours of time in bed would improve the patient’s sleep efficiency and a bedtime of 12:30 a.m. would accomplish this goal. Generally, reduced time in bed is accomplished by postponing bedtime rather than getting up earlier. Naps generally do not improve sleep efficiency. While getting out of bed is recommended after being in bed for 30 minutes without falling asleep, or being awake for 30 minutes after being asleep, staying up for a prescribed period of time is not recommended.

How well did you know this?
1
Not at all
2
3
4
5
Perfectly
360
Q

A 32-year-old female who is one of your longtime patients calls you because of a 24-hour history of painful urination with urinary frequency and urgency. She is otherwise healthy and does not have any fever, chills, back pain, or vaginal discharge. She uses an oral contraceptive pill and states that her last menstrual period was normal and occurred last week.
Which one of the following would be most appropriate at this time?
A) Empiric antibiotic treatment
B) A urinalysis
C) A urine culture
D) Plain abdominal radiographs
E) Pelvic ultrasonography

A

ANSWER: A
This patient has symptoms of acute simple cystitis and does not have any symptoms that would suggest a complicated urinary tract infection or vaginal infection. In these cases treatment with oral antibiotic therapy may be prescribed without further evaluation (SOR B). Simple cystitis is a clinical diagnosis and a urinalysis and urine culture are not necessary. The patient does not have any symptoms that warrant evaluation with abdominal radiographs or pelvic ultrasonography.

How well did you know this?
1
Not at all
2
3
4
5
Perfectly
361
Q

A 70-year-old female develops thrombocytopenia during a prolonged hospitalization for endocarditis. Her current medications include scheduled unfractionated heparin injections for venous thromboembolism prophylaxis. You suspect heparin-induced thrombocytopenia (HIT).
Assuming that her thrombocytopenia is caused by HIT, which one of the following is the most likely complication?
A) Anaphylaxis
B) Disseminated intravascular coagulation
C) Hemorrhage
D) Sepsis
E) Thrombosis

A

ANSWER: E
Heparin-induced thrombocytopenia (HIT) is an immune-mediated process that occurs in approximately 1 in 5000 hospitalized patients. Patients are at highest risk 7–10 days after exposure to unfractionated heparin, and the risk is particularly high after cardiac surgery, which is associated with an estimated rate of 1%–3%. In contrast to other causes of thrombocytopenia, HIT places patients at a paradoxically increased risk of thrombotic complications, with clotting events occurring in roughly 50% of confirmed cases of HIT. Lower-extremity deep vein thrombosis and pulmonary embolism are the most common thrombotic complications, followed by arterial thromboses, stroke, and myocardial infarction, in descending order of frequency. Thromboses often occur concurrently with the development of thrombocytopenia or shortly thereafter. The risk of HIT can be determined with the 4T scoring system, which evaluates the acuity of thrombocytopenia, timing of onset, presence of thrombosis, and alternative causes of thrombocytopenia. Patients with an intermediate or high pretest probability should be managed with prompt discontinuation of heparin and initiation of full-dose anticoagulation with a non-heparin anticoagulant, such as argatroban, danaparoid, fondaparinux, or bivalirudin, pending results of further HIT diagnostic evaluation. Anaphylaxis, disseminated intravascular coagulation, hemorrhage, and sepsis are all less common complications of HIT compared to thrombotic events.

How well did you know this?
1
Not at all
2
3
4
5
Perfectly
362
Q

A 57-year-old male with a history of heart failure sees you for follow-up. He describes symptoms of mild dyspnea on exertion with ordinary activities such as shopping or yard work. An echocardiogram shows an ejection fraction of 37%.
According to the New York Heart Association criteria, this patient’s heart failure would be classified as which one of the following?
A) Class I
B) Class II
C) Class III
D) Class IV

A

ANSWER: B
The appropriate classification of heart failure is important for monitoring the disease. The most common currently used system is the New York Heart Association (NYHA) functional classification. In this system, class I is defined as heart disease in a patient with no symptoms and no limitations of physical activity. Patients with class II heart failure have mild symptoms with normal physical activity. Class III heart failure refers to significant limitations of activity, including symptoms with less than normal activities. Patients with class IV heart failure have symptoms at rest and are unable to carry on activity without discomfort.

How well did you know this?
1
Not at all
2
3
4
5
Perfectly
363
Q

A 24-year-old female presents with progressively worsening vulvar pain for 3 days. On examination a 3×3-cm tender, fluctuant mass is noted on the right labia minora. She had a similar episode of this problem last year.
Which one of the following would be the most appropriate management?
A) Expectant management
B) Fine-needle aspiration
C) Incision and drainage
D) Marsupialization
E) Excision under general anesthesia

A

ANSWER: D
The most appropriate management of a recurrent Bartholin gland abscess would be marsupialization, which has a 0% recurrence rate at 6 months. Local anesthesia can be used in the office to effectively treat Bartholin gland abscesses and sedation is not required (SOR A). If the Bartholin gland abscess is >5 cm, referral to a gynecologist is recommended. Expectant management, fine-needle aspiration, or incision and drainage would likely lead to recurrence.

How well did you know this?
1
Not at all
2
3
4
5
Perfectly
364
Q

A 25-year-old male presents with a 4-month history of crampy abdominal pain, diarrhea, and fatigue. His symptoms began gradually but have become more severe and he is now experiencing rectal bleeding. He says that his abdominal pain seems to temporarily improve after eating. He has smoked five cigarettes per day for the past 8 years. He is surprised to learn that he has lost 7 kg (15 lb) when he is weighed today.
His vital signs include a blood pressure of 116/70 mm Hg, a heart rate of 76 beats/min, a respiratory rate of 12/min, and a temperature of 37.7°C (99.9°F). A physical examination reveals abdominal tenderness and mild distention. An anorectal examination is significant for a perianal fistula. A laboratory evaluation is notable for mild anemia. His kidney and liver function are normal.
Which one of the following is the most likely diagnosis?
A) Celiac disease
B) Chronic pancreatitis
C) Crohn’s disease
D) Irritable bowel syndrome
E) Ulcerative colitis

A

ANSWER: C
Crohn’s disease may present insidiously with diarrhea, abdominal pain, rectal bleeding, fever, weight loss, and fatigue. Red-flag symptoms include perianal lesions, a first degree relative with inflammatory bowel disease, weight loss of 5% of the patient’s usual weight, abdominal pain for more than 3 months, nocturnal diarrhea, fever, the absence of abdominal pain for 30–45 minutes after eating, and the absence of rectal urgency. This patient exhibits symptoms consistent with Crohn’s disease. While anemia is also common in celiac disease, rectal bleeding is not. Chronic pancreatitis does not generally present with improved pain after eating. Irritable bowel syndrome is not associated with fever, rectal bleeding, anemia, or perianal fistulas. Ulcerative colitis is not associated with perianal lesions.

How well did you know this?
1
Not at all
2
3
4
5
Perfectly
365
Q

A 34-year-old female at 32 weeks gestation presents with a right-sided, pounding headache that began 8 hours ago and is similar to headaches she has had in the past. She is sensitive to light and sound, and has vomited several times since the onset of pain. She has taken acetaminophen without relief. She takes prenatal vitamins but no other routine medications. On examination her blood pressure is normal.
Which one of the following would be the most appropriate treatment for this patient?
A) Dihydroergotamine
B) Metoclopramide (Reglan)
C) Naproxen
D) Oxycodone (OxyContin)
E) Sumatriptan (Imitrex)

A

ANSWER: B
Metoclopramide and acetaminophen are the only two medications considered safe for abortive migraine treatment during pregnancy (SOR B). The dopamine antagonist antiemetics are considered second-line abortive treatments in the general population. Dihydroergotamine should not be used during pregnancy due to its oxytocic properties and the potential risk of intrauterine growth restriction with its use. NSAIDs are not considered safe during pregnancy, particularly in the first and third trimesters. Opioids are only moderately useful for migraine treatment and should be avoided during pregnancy due to their abuse potential. Triptans are generally considered safe during the first trimester but not in the second and third trimesters. Their use has been associated with uterine atony, increased risk of bleeding during delivery, and increased risk of preterm birth.

How well did you know this?
1
Not at all
2
3
4
5
Perfectly
366
Q

Which one of the following U-100 insulin products has the longest duration of action?
A) Degludec (Tresiba)
B) Glargine (Lantus)
C) Isophane NPH (Humulin N)
D) Lispro (Humalog)
E) Regular (Humulin R)

A

ANSWER: A
Among the available U-100 insulin products, the one with the longest duration of action is ultralong-acting degludec, which lasts 42 hours. The duration of action of rapid-acting lispro is 3–6.5 hours, short-acting regular is 5–8 hours, intermediate-acting isophane is 12–16 hours, and long-acting glargine is 11–24 hours.

How well did you know this?
1
Not at all
2
3
4
5
Perfectly
367
Q

When performing a geriatric assessment, which one of the following is an instrumental activity of daily living?
A) Bathing
B) Dressing
C) Transferring between the bed and a chair
D) Using the telephone
E) Using the toilet

A

ANSWER: D
The foundation of geriatric assessment is assessing the individual’s ability to perform tasks required for living. Activities of daily living are self-care activities that are performed daily, such as eating, bathing, dressing, transferring between the bed and a chair, and toileting, including bladder and bowel function. Instrumental activities of daily living include activities necessary to live independently, such as using a telephone, doing housework, preparing meals, taking medications properly, and managing finances.

How well did you know this?
1
Not at all
2
3
4
5
Perfectly
368
Q

A 9-year-old male with a history of moderate persistent asthma is brought to the emergency department with an acute exacerbation. His symptoms began with a runny nose and nasal congestion 2 days ago. His parents state that he has not had any fevers or chills and he was eating and drinking well until a few hours ago when his breathing started to appear more labored. After multiple treatments with inhaled albuterol (Proventil, Ventolin) and oral prednisolone he remains tachypneic and wheezy.
Which one of the following intravenous medications should be added to the patient’s current treatment to reduce the likelihood of hospital admission?
A) Ketorolac
B) Magnesium sulfate
C) Methylprednisolone
D) Omalizumab (Xolair)
E) Theophylline

A

ANSWER: B
Children who present to the emergency department with an asthma exacerbation and fail to improve adequately with inhaled short-acting bronchodilators and corticosteroids may benefit from treatment with intravenous (IV) magnesium sulfate. A 2016 Cochrane review of three randomized, controlled trials found that this reduced hospital admissions by 68%. Ketorolac is not known to have any benefit in the treatment of asthma. Oral administration of corticosteroids is as effective as IV administration, so there is no reason to give IV methylprednisolone. Omalizumab may be used to prevent exacerbations in patients with severe asthma who do not achieve adequate control with high-dose inhaled corticosteroids, but it has no role in the management of acute exacerbations. IV theophylline is not recommended for asthma exacerbations given its safety profile and poor efficacy compared to short-acting bronchodilators.

How well did you know this?
1
Not at all
2
3
4
5
Perfectly
369
Q

A 62-year-old female sees you for a routine health maintenance examination. She has a history of breast cancer diagnosed 6 years ago that was treated with lumpectomy, radiation, and endocrine therapy. She is feeling well today and has no symptoms of concern. There is no family history of breast, ovarian, colon, or prostate cancers.
In addition to mammography, which one of the following annual tests would improve this patient’s chance of survival?
A) No tests
B) CT of the chest
C) MRI of the breast
D) Breast ultrasonography
E) A bone scan

A

ANSWER: A
Breast cancer, the most common noncutaneous malignancy among women, has a 5-year survival rate of almost 90%, so medical care of such patients is increasingly common. To help provide guidance to primary care physicians, the American Cancer Society and the American Society of Clinical Oncology published their joint Breast Cancer Survivorship Care Guideline in 2016. This guideline includes a recommendation for annual mammography for women with prior treatment for breast cancer to screen for local recurrence or a new primary breast cancer. MRI is not recommended in the absence of specific high-risk criteria such as a BRCA mutation. Similarly, other imaging modalities such as ultrasonography are not recommended in the absence of symptoms. Imaging is not indicated to screen for metastatic disease. Though breast cancer most commonly metastasizes to the lung, bone, and liver, there is no evidence that screening CT or a bone scan provides mortality or quality-of-life benefits.

How well did you know this?
1
Not at all
2
3
4
5
Perfectly
370
Q

A 50-year-old male with no significant past medical history presents with a 5-day history of fever to 101°F, chills, and mild diffuse joint and muscle pains. He also reports a mild headache but has not had any sore throat, rhinorrhea, cough, shortness of breath, nausea, vomiting, or change in bowel habits. He noticed a round red rash (shown below) a few days ago on his leg that has grown in size since then. It is minimally pruritic but not painful. He has no other rashes. He recently traveled to Vermont for his family’s annual fall hiking trip but does not recall any insect bites. He does not take any medications and has no drug allergies. He has a blood pressure of 120/74 mm Hg, a pulse rate of 84 beats/min, and a temperature of 37.8°C (100.0°F). Cardiac, pulmonary, musculoskeletal, and abdominal examinations are normal.
Which one of the following is the most likely diagnosis?
A) Adenovirus
B) Ehrlichiosis
C) Influenza
D) Lyme disease
E) Rocky Mountain spotted fever

A

ANSWER: D
This patient has a classic presentation for Lyme disease. Lyme disease is transmitted by a tick bite, but not all patients remember being bitten by a tick. The classic erythema migrans lesion appears a couple of weeks after the tick bite. The first-line treatment for Lyme disease is either amoxicillin or doxycycline. Macrolides can be used if patients have true allergies to -lactams and doxycycline, but they are less effective. This patient is not exhibiting the respiratory symptoms typically associated with adenovirus. Ehrlichiosis and Rocky Mountain spotted fever typically present with headaches and fevers but not with an erythema migrans rash. Although influenza should be considered in the differential diagnosis, it would not present with an erythema migrans rash.

How well did you know this?
1
Not at all
2
3
4
5
Perfectly
371
Q

Which one of the following oral iron preparations is most effective for the treatment of iron deficiency anemia in a patient with non–dialysis-dependent chronic kidney disease?
A) Ferric citrate (Auryxia)
B) Ferrous fumarate
C) Ferrous gluconate
D) Ferrous sulfate

A

ANSWER: A
Oral iron supplements, including ferrous fumarate, ferrous gluconate, and ferrous sulfate, are generally ineffective when used by hemodialysis patients and are only modestly effective when used by patients with non–dialysis-dependent chronic kidney disease. The one exception is ferric citrate, which is highly efficacious in all patients with chronic kidney disease. Intravenous iron preparations such as iron dextran are similarly effective in both groups and are considered the gold standard for the treatment of iron deficiency in patients on chronic hemodialysis.

How well did you know this?
1
Not at all
2
3
4
5
Perfectly
372
Q

One of your patients will turn 65 in 2 weeks and your practice manager routinely encourages scheduling a Welcome to Medicare preventive visit soon after patients’ 65th birthdays. This patient continues to work full time and is currently insured through his employer-sponsored health insurance.
In order to bill for a Welcome to Medicare visit after this patient turns 65, which one of the following is true?
A) The patient must be enrolled in Medicare Part A
B) The patient must be enrolled in Medicare Part B
C) The patient must be enrolled in Medicare Part D
D) The patient must be over age 65 and the specific type of Medicare enrollment is not relevant

A

ANSWER: B
The Welcome to Medicare preventive visit, also known as an Initial Preventive Physical Examination (IPPE), is a one-time service that can be provided within the first year of a patient’s enrollment in Medicare Part B. Medicare Part B covers provider visits and outpatient services such as laboratory testing. Beneficiaries are automatically enrolled in Part A when they apply to Medicare, which provides coverage for hospital-based and hospice care. Because Part A does not typically carry a monthly premium, some working older adults who continue to have insurance through their employer may opt to obtain Part A only, and wait on Part B coverage, which does have a monthly premium. Medicare Part D is prescription drug coverage.

How well did you know this?
1
Not at all
2
3
4
5
Perfectly
373
Q

Routine follow-up blood tests for colorectal cancer survivors should include
A) carcinoembryonic antigen (CEA) levels only
B) liver function tests only
C) CBCs and CEA levels only
D) CBCs and liver function tests only
E) CBCs, CEA levels, and liver function tests

A

ANSWER: A
The Choosing Wisely campaign recommends checking only carcinoembryonic antigen (CEA) levels following curative treatment for colorectal cancer (SOR C). No routine laboratory studies such as a CBC or liver function tests should be ordered for follow-up.

How well did you know this?
1
Not at all
2
3
4
5
Perfectly
374
Q

You are advising a group of medical students who are planning a tobacco cessation program for expectant mothers. The medical students want to build an advertising program that touts the pregnancy benefits of tobacco cessation.
You tell the medical students that evidence suggests that tobacco cessation in pregnant women
A) decreases the risk for cesarean delivery
B) decreases the risk for preeclampsia
C) decreases the need for epidural anesthesia
D) increases infant birth weight
E) increases the risk for preterm delivery

A

ANSWER: D
Smoking during pregnancy increases the risk for fetal growth restriction. The U.S. Preventive Services Task Force identified evidence that tobacco cessation increases infant birth weight and decreases the risk for preterm delivery. There is no evidence that tobacco cessation in pregnant women decreases the risk for cesarean delivery, the risk for preeclampsia, or the need for epidural anesthesia.

How well did you know this?
1
Not at all
2
3
4
5
Perfectly
375
Q

A 13-year-old female is brought to your office by her adoptive mother. They do not know the patient’s biological family history. They are concerned because, unlike all of her friends, she has not yet started to menstruate. Breast development began 2 years ago. On examination her breasts show a secondary mound from the areola and nipple above the contour of her breast. She has dark, coarse hair covering the mons pubis consistent with a stage 4 sexual maturity rating.
If her sexual development continues to be normal, at what age should you recommend evaluation for primary amenorrhea?
A) 13 B) 14 C) 15 D) 16 E) 17

A

ANSWER: C
Delayed puberty in girls is defined as the absence of breast development by age 13. Typically, menarche starts 2.5 years after the onset of breast development, with an average age of 12.5 years (normal range 9–15 years). In girls with otherwise normal sexual development, the absence of menarche by 15 years of age should prompt an evaluation for primary amenorrhea.

How well did you know this?
1
Not at all
2
3
4
5
Perfectly
376
Q

A 57-year-old female with a history of diabetes mellitus, hypertension, and depression sees you for a routine follow-up visit. Her vital signs include a heart rate of 88 beats/min, a blood pressure of 162/84 mm Hg, and a BMI of 32 kg/m2.
The recommended antihypertensive regimen for reducing cardiovascular events in this patient is an ACE inhibitor plus
A) an -blocker
B) an angiotensin receptor blocker
C) a calcium channel blocker
D) a loop diuretic

A

ANSWER: C
The ACCOMPLISH trial demonstrated that an ACE inhibitor (ACEI) in combination with a calcium channel blocker (CCB) reduced both fatal and nonfatal cardiovascular events in patients with diabetes mellitus and hypertension. The benefit of an ACEI and a CCB for reducing cardiovascular events was greater than that of an ACEI and a thiazide diuretic. Evidence has shown that combination therapy for most patients should include a CCB, an ACEI or angiotensin receptor blocker (ARB), or a thiazide diuretic (SOR A). The American College of Cardiology/American Heart Association guidelines recommend against centrally acting medications such as -blockers for first-line therapy. Combining ACEIs and ARBs is not recommended, as the risk of side effects such as hyperkalemia outweighs the benefits. Loop diuretics are not considered first-line antihypertensive agents.

How well did you know this?
1
Not at all
2
3
4
5
Perfectly
377
Q

An elderly homeless male is brought to the emergency department. He is clearly hypothermic due to cold exposure and has superficial frostbite of his extremities. He is still conscious and shivering.
In addition to rewarming him, which one of the following should you administer?
A) Acetazolamide (Diamox Sequels)
B) Amitriptyline
C) Ceftriaxone
D) Ibuprofen
E) tPA

A

ANSWER: D
Frostbite is a freezing injury that occurs when initial cooling causes vasoconstriction and localized ischemia. Continued cold exposure leads to ice crystal formation, which causes cellular lysis, electrolyte abnormalities, and microvascular occlusion. Rewarming creates an inflammatory response. Ibuprofen is the most appropriate agent for the treatment of frostbite until the wounds heal or surgery is performed (SOR C). Acetazolamide can cause frostbite at high altitudes. Amitriptyline is used to treat the pain of immersion foot (also called trench foot), which is a nonfreezing injury that happens when the foot is exposed to prolonged wet conditions above 0°C (32°F). Antibiotics are indicated if open or dirty wounds are present (SOR B). tPA has a role in treating patients with frostbite, but it is used only to decrease the risk of amputation when rewarming patients with grade 3, grade 4, or deep frostbite (SOR B).

How well did you know this?
1
Not at all
2
3
4
5
Perfectly
378
Q

A 72-year-old female presents with bothersome palpitations. She is otherwise healthy and is not taking any medications. A physical examination is normal, including thyroid and eye examinations. Laboratory studies reveal a serum TSH level of 0.2 U/mL (N 0.4–4.0) and normal T3 and free T4 levels. An EKG reveals frequent premature atrial contractions but is otherwise normal. Ultrasonography of the thyroid does not reveal any nodules, thyroid scintigraphy shows diffuse uptake, and an anti–thyrotropin-receptor (thyroid-stimulating immunoglobulin) antibody level is significantly elevated.
Which one of the following is the most likely diagnosis?
A) Central hypothyroidism
B) Graves disease
C) Iodine deficiency
D) Solitary toxic thyroid nodule
E) Toxic multinodular goiter

A

ANSWER: B
This patient has subclinical hyperthyroidism caused by Graves disease. A positive anti–thyrotropin-receptor (thyroid-stimulating immunoglobulin) antibody result is virtually diagnostic of Graves disease. Central hypothyroidism is associated with a low TSH level and low T3 and T4 levels. Iodine deficiency is associated with goiter and hypothyroidism. Nodular thyroid disease is unlikely given the imaging results. Treatment of this patient’s mild Graves disease is probably indicated, given her age and cardiac symptoms.

How well did you know this?
1
Not at all
2
3
4
5
Perfectly
379
Q

An EKG performed on a patient with a history of heart failure and an ejection fraction of 30% reveals sinus rhythm with a heart rate of 55 beats/min and new left bundle branch block with a QRS interval of 160 msec. The patient is taking a -blocker but has a history of acetylcholinesterase inhibitor–induced angioedema.
Which one of the following would be most appropriate at this point?
A) Amiodarone
B) Ivabradine (Corlanor)
C) Sacubitril/valsartan (Entresto)
D) Sinoatrial node ablation
E) Cardiac resynchronization therapy

A

ANSWER: E
Cardiac resynchronization therapy is strongly recommended for patients with symptomatic heart failure, an ejection fraction <35%, and a left bundle branch block with a QRS interval >150 msec. Amiodarone is an antiarrhythmic and would not be indicated for this patient. Ivabradine, a sinoatrial node modulator, is used in patients with symptomatic heart failure as an add-on therapy to decrease the heart rate. It is indicated in patients with a heart rate >70 beats/min despite -blockade. Sacubitril/valsartan is contraindicated in patients with a history of angioedema. Sinoatrial node ablation is indicated for some patients with sinus node dysfunction.

How well did you know this?
1
Not at all
2
3
4
5
Perfectly
380
Q

A 66-year-old female with multiple medical problems has routine laboratory work performed during a regularly scheduled clinic visit. All of the laboratory values are normal except for a serum calcium level of 11.0 mg/dL (N 8.5–10.2).
Which one of the following medications in her current regimen is most likely to cause an elevated calcium level?
A) Alendronate (Fosamax)
B) Lithium
C) Omeprazole (Prilosec)
D) Sertraline (Zoloft)
E) Spironolactone (Aldactone)

A

ANSWER: B
Hypercalcemia is a commonly encountered laboratory abnormality. It is important for family physicians to be aware of common medications that can cause elevated calcium levels. Of the options listed, lithium is the only medication that can cause high calcium levels. In addition, thiazide diuretics, excluding aldosterone receptor antagonists such as spironolactone, often cause elevated calcium levels. Hypercalcemia is not a side effect of alendronate, omeprazole, sertraline, or spironolactone.

How well did you know this?
1
Not at all
2
3
4
5
Perfectly
381
Q

A 58-year-old athletic trainer presents with acute low back pain. He thinks the pain began after he started a new weightlifting regimen 3 weeks ago. The pain does not radiate, and prolonged standing exacerbates the pain. He does not have any bowel or bladder incontinence, fever, or saddle anesthesia. He asks which type of therapy is most likely to shorten the course of the pain.
Which one of the following would be the best recommendation?
A) Ice
B) Targeted exercises
C) Oral analgesics
D) Spinal manipulation
E) Evidence does not support the superiority of any modality

A

ANSWER: E
Recommendations for the treatment of back pain often include ice, targeted exercises, oral analgesics, and spinal manipulation. Although there is some evidence that certain modalities are better than placebo, there is very little evidence to show that one modality is superior to another in relieving back pain or shortening the course of the pain. There is some evidence that spinal manipulation is superior to sham manipulation.

How well did you know this?
1
Not at all
2
3
4
5
Perfectly
382
Q

A 23-year-old male presents for evaluation of wrist pain after he fell while skateboarding. On examination he has left wrist pain with snuffbox tenderness. A radiograph of his wrist shows a fracture through the midportion of the scaphoid with 2 mm of displacement.
Which one of the following would be the most appropriate treatment of this injury?
A) A thumb spica cast
B) A short arm cast
C) A long arm cast
D) Closed reduction followed by a thumb spica cast
E) Surgical fixation

A

ANSWER: E
Scaphoid fractures have a high risk for nonunion because the blood supply arises distally from branches of the radial artery. The proximal pole of the scaphoid is entirely dependent on this distal blood supply. To improve healing and decrease the risk of nonunion and avascular necrosis, displaced fractures should be treated with surgical fixation. Nondisplaced fractures of the distal third of the scaphoid may be treated with a short arm thumb spica cast for 4–6 weeks. Middle and proximal fractures should be treated with a long arm thumb spica cast for 6 weeks, followed by a short arm thumb spica cast.

How well did you know this?
1
Not at all
2
3
4
5
Perfectly
383
Q

A 21-year-old female comes to your office because of concerns about excess hair growth. She has dark hairs on her upper lip and chin and around her nipples that have been consistently present since puberty, and she is dissatisfied with the cosmetic results of various hair removal methods. She has regular menses and does not wish to become pregnant at this time. An examination is consistent with some terminal hairs in the distribution she describes and is otherwise unremarkable.
Which one of the following would be most appropriate at this point?
A) An early morning total testosterone level
B) A full hormonal workup including thyroid function testing, prolactin,
17-hydroxyprogesterone, and dehydroepiandrosterone sulfate (DHEAS) levels
C) Transvaginal ultrasonography
D) A 6-month trial of oral contraceptive pills

A

ANSWER: D
This patient most likely has benign idiopathic hirsutism. It is estimated that approximately 50% of women with mild hirsutism have idiopathic hirsutism. In the absence of other worrisome findings on the history or examination, such as a rapid onset, virilization, or a high degree of hirsutism, the most appropriate next step is a trial of pharmacologic therapy, using oral contraceptive pills as the first-line agent if the patient does not desire pregnancy. A minimum 6-month trial is needed because of the length of the hair growth cycle. An early morning total testosterone level, a full hormonal workup, and transvaginal ultrasonography would be appropriate if there were other signs and symptoms of hyperandrogenism on the history or examination.

How well did you know this?
1
Not at all
2
3
4
5
Perfectly
384
Q

You are working in an urgent care clinic when a 68-year-old male with chronic hypertension sees you for refills of his medications. He has been out of his medication for the past month and could not get an appointment with his primary care physician for refills. His blood pressure is persistently 190/115 mm Hg, even after he has rested in a quiet room for 30 minutes. His blood pressure previously had been well controlled. He has a moderate headache but otherwise feels well. An examination, including a funduscopic examination, is normal.
Which one of the following management options would be most appropriate at this time?
A) Refill his usual medications and arrange for follow-up in 1 week
B) Administer oral labetalol (Trandate) every 30 minutes until his blood pressure is
<180/110 mm Hg
C) Administer oral nifedipine (Procardia) every 30 minutes until his blood pressure is
<180/110 mm Hg
D) Administer sublingual nifedipine every 30 minutes until his blood pressure is <180/110
mm Hg
E) Refer for immediate hospitalization for intravenous antihypertensive treatment

A

ANSWER: B
This patient has a hypertensive urgency, defined as symptomatic acute severe hypertension without evidence of acute end-organ injury. Hypertensive urgencies may be managed in the ambulatory setting. Emergent intravenous treatment at the hospital is not indicated. This patient should be treated with an oral agent with a fairly rapid onset of action, such as clonidine, labetalol, captopril, or prazosin. Topical nitroglycerin is also an option. Nifedipine may cause unpredictable blood pressure reduction and should be avoided. The patient may be discharged to resume his usual medications after his symptoms have improved and his blood pressure is below 160–180/110 mm Hg, with follow-up within a week.

How well did you know this?
1
Not at all
2
3
4
5
Perfectly
385
Q

The U.S. Preventive Services Task Force recommends which one of the following for breast cancer screening?
A) Annual screening mammography for women ages 40–75
B) Annual screening mammography for women ages 50–75
C) Biennial screening mammography for women ages 35–75
D) Biennial screening mammography for women ages 50–75
E) Biennial screening mammography for women ages 45–50 and annual screening
mammography for women ages 51–75

A

ANSWER: D
Current U.S. Preventive Services Task Force guidelines for breast cancer screening recommend biennial screening mammography for women ages 50–75 (B recommendation). Biennial screening mammography can be considered for women age 40–49 after discussing the risks and benefits with the patient (C recommendation).

How well did you know this?
1
Not at all
2
3
4
5
Perfectly
386
Q

At a routine visit, a 65-year-old male former smoker reports shortness of breath and a cough that has been worsening slowly over the last 6 months. On examination you hear bibasilar inspiratory crackles. An EKG, chest radiograph, and echocardiogram are normal. CT of the chest shows multiple bilateral patchy areas of consolidation.
The most appropriate next step is to
A) take a detailed history of medication use and lifetime environmental exposures
B) start an antibiotic
C) start furosemide (Lasix)
D) start an inhaled short-acting -agonist as needed
E) refer for pulmonary rehabilitation

A

ANSWER: A
Idiopathic pulmonary fibrosis occurs most often in male former smokers over the age of 60. For patients with newly diagnosed interstitial lung disease (ILD) with suspected idiopathic pulmonary fibrosis, the American Thoracic Society recommends taking a detailed history of medication use and environmental exposures over the patient’s lifetime. In an observational study of 1084 patients, 47% were identified as having hypersensitivity pneumonitis on a detailed assessment of new-onset ILD with an unknown cause. Laboratory testing for connective tissue disease is also recommended. Antibiotics would be appropriate to treat a bacterial infection. Furosemide is used to treat heart failure. An inhaled short-acting -agonist and pulmonary rehabilitation would not be appropriate at this time.

How well did you know this?
1
Not at all
2
3
4
5
Perfectly
387
Q

In a patient with documented Helicobacter pylori infection, which one of the following studies should be performed to document clearance of the infection?
A) An H. pylori IgG level
B) An H. pylori IgM level
C) A stool antigen test 1 week after the completion of antibiotic therapy
D) A urea breath test 1 month after the completion of antibiotic therapy
E) A gastric biopsy

A

ANSWER: D
After treatment for a Helicobacter pylori infection it is essential to document clearance of the infection. This is typically done with a stool antigen test or a urea breath test performed 1 month after the completion of antibiotic therapy. If the patient is taking a proton pump inhibitor it should be discontinued prior to the test. H. pylori IgG or IgM levels and gastric biopsies are not appropriate for documenting clearance of H. pylori infection.

How well did you know this?
1
Not at all
2
3
4
5
Perfectly
388
Q

A 26-year-old female with a history of bipolar disorder sees you for follow-up of chronic joint pain. The review of systems is positive for intermittent rashes on the dorsal hands and face and left-sided pleuritic chest pain. An examination reveals tender swelling of the hand and wrist joints bilaterally. Examination of the lungs reveals diminished breath sounds at the left base with point-of-care ultrasonography findings consistent with a small pleural effusion. Initial laboratory tests are significant for a platelet count of 96,000/mm3 (N 130,000–450,000), a positive antinuclear antibody test with a 1:80 titer (N <1:40), and negative Lyme disease and HIV tests.
Which one of the following would help confirm your suspected diagnosis?
A) Elevated anticyclic citrullinated peptide antibodies
B) Elevated anti–smooth muscle antibodies
C) Elevated anti-centromere antibodies
D) Low complement levels
E) Positive HLA-B27

A

ANSWER: D
The clinical findings on examination, including symmetric polyarthritis, thrombocytopenia, positive antinuclear antibodies (ANAs), and pleural effusion, meet the American College of Rheumatology criteria for a diagnosis of systemic lupus erythematosus (SLE). The rash and the patient’s neuropsychiatric history may also factor into the diagnosis, but they are not described specifically in this case. A positive ANA is sensitive but not specific for SLE. Although additional laboratory testing may not be needed to confirm SLE in this case due to classic clinical findings, low complement levels help confirm SLE and may be helpful because the ANA is only mildly elevated.
49
Elevated anticyclic citrullinated peptide antibodies help confirm a diagnosis of rheumatoid arthritis. Anti–smooth muscle antibodies are used to confirm autoimmune hepatitis, which can also cause an elevated ANA. Anti-centromere antibodies, a subset of ANAs, are more closely associated with systemic sclerosis. A positive HLA-B27 test is associated with the seronegative spondyloarthropathies, such as psoriatic arthritis.

How well did you know this?
1
Not at all
2
3
4
5
Perfectly
389
Q

A 36-year-old male sees you for follow-up of progressive fatigue and lightheadedness that has worsened over the past 3 months. He has lost 5 kg (11 lb) during this time. On examination he has a BMI of 21 kg/m2, a blood pressure of 88/48 mm Hg, and a pulse rate of 66 beats/min. A skin examination is notable for patches of nonpigmented skin on the hands. Initial laboratory testing is significant for a sodium level of 132 mEq/L (N 135–145) and a potassium level of 5.3 mEq/L (N 3.5–5.0).
Which one of the following tests would confirm the most likely diagnosis?
A) 17-hydroxyprogesterone
B) ACTH stimulation
C) Dexamethasone suppression
D) Late night salivary cortisol
E) Plasma renin and aldosterone

A

ANSWER: B
This clinical case is consistent with Addison’s disease, or adrenal insufficiency. This case is most likely the result of autoimmune disease, given the concurrent vitiligo, but it may also be idiopathic or secondary to cancers such as lymphoma or infections such as tuberculosis. Adrenal insufficiency is suggested by a low morning cortisol level, but the test of choice to confirm this diagnosis is the ACTH stimulation test.
17-Hydroxyprogesterone deficiency causes congenital adrenal hyperplasia, which typically presents in childhood. This test is part of newborn screening in the United States. Acquired 17-hydroxyprogesterone deficiency can present in adulthood as adrenal insufficiency but a low 17-hydroxyprogesterone level does not confirm adrenal insufficiency. Late night salivary cortisol is an initial test for corticosteroid excess (Cushing syndrome) and the dexamethasone suppression test is used to confirm that disorder. Renin and aldosterone levels can be helpful to characterize mineralocorticoid deficiency but they are not diagnostic.

How well did you know this?
1
Not at all
2
3
4
5
Perfectly
390
Q

A patient born in 1954 requests screening for hepatitis C. His anti-HCV antibody screen is positive.
Which one of the following would be most appropriate at this point?
A) A confirmatory second anti-HCV test in 4–6 weeks
B) A qualitative HCV RNA test
C) Quantitative HCV genotyping
D) Treatment with a dual antiviral regimen pending additional testing
E) Referral for a percutaneous liver biopsy

A

ANSWER: B
The CDC recommends that all adults 18 years of age or older receive a one-time screening for hepatitis C virus (HCV). Persons with risk factors for HCV exposure should be screened periodically, based on risk level. If the anti-HCV antibody screen is positive, then a qualitative HCV RNA test is the next step (SOR C). Prior to initiating treatment, a quantitative HCV RNA and genotype testing is necessary (SOR A). In addition, assessing the degree of fibrosis will provide information regarding the urgency of treatment. Percutaneous liver biopsy is generally the preferred evaluation after obtaining quantitative and genotype results, all of which can guide treatment decisions.

How well did you know this?
1
Not at all
2
3
4
5
Perfectly
391
Q

You are caring for an 84-year-old female with Alzheimer’s dementia. She is disoriented to time but usually oriented to person and place. She is homebound but is frequently visited by family. The patient’s daughter feels conflicted but wants her mother to go to a skilled care facility. The patient is adamant that she does not want to leave her home. During a recent home visit you noted that she smelled of urine and had a severe intertriginous rash suspicious for Candida. Her bedside Mini-Mental State Examination score was 24/30.
Which one of the following ethical principles should be the primary consideration when determining whether this patient should continue to live at home?
A) Autonomy B) Competence C) Harm
D) Justice E) Safety

A

ANSWER: A
Four widely recognized principles of medical ethics include respect for autonomy, beneficence, nonmaleficence, and justice. Respect for patient autonomy is central to medical ethics and to the doctor-patient relationship. Physicians should involve patients in health care decisions commensurate with the patient’s capacity to understand and make decisions. Even when a condition affects a patient’s decision-making capacity, the patient may be able to participate in some aspects of the decision-making process. Competence (or incompetence) is a legal term, which can be used to refer to a patient being granted autonomy or not. Harm and safety are not ethical principles, but are related to the principle of nonmaleficence, or “do no harm.” The principle of justice addresses the question of who ought to receive the benefits of an intervention and/or bear its burdens. An injustice occurs when a person who is entitled to a benefit is denied without good reason or when some burden is imposed unduly.

How well did you know this?
1
Not at all
2
3
4
5
Perfectly
392
Q

A 38-year-old male presents with a 2-month history of increased postprandial nausea and nonbloody, painless, loose stools. He feels well otherwise. Ten months ago he underwent bariatric surgery, which involved creating a small stomach pouch. There were no complications from the surgery and he does not take any medications. A review of his diet reveals that he has three small meals and two snacks daily. He has one serving of vegetables or bread with each meal. He has 16 oz of coffee with breakfast, 12 oz of soda with lunch, 12 oz of beer with dinner, and a cup of water or milk with snacks. His snack is usually cheese or peanut butter and a cracker. An examination is unremarkable.
You are concerned with his diet habits and your recommendations include
A) no fluid for 15 minutes before or after meals and snacks
B) limiting carbonated beverages to 8 oz per meal or snack
C) increasing daily servings of fibrous vegetables and whole grain breads
D) eating one meal and three snacks daily
E) eating a diet lower in fat

A

ANSWER: A
Hundreds of thousands of Americans have undergone bariatric surgery, and family physicians are often asked to provide long-term postoperative management. Many bariatric surgery procedures create a small stomach pouch. Dietary compliance is essential to minimize feeding intolerance symptoms such as postprandial nausea, emesis, and diarrhea. Post bariatric surgery diet recommendations typically include the following:
Ref:
*
* *
*
Avoid fluid 15 minutes before and after meals. Fluids with meals will move food more quickly through the pouch and decrease the feeling of fullness.
Avoid carbonated beverages entirely.
Eat three small protein-rich meals and one or two snacks daily. Lower fat diets are not typically recommended.
Whole grains and fibrous vegetables often exacerbate symptoms so there is no need to increase these foods.

How well did you know this?
1
Not at all
2
3
4
5
Perfectly
393
Q

A healthy 52-year-old male who runs for 40 minutes 4–5 times per week comes to your office because he has recently noted pain in the inner aspect of his right knee along the joint line. He also reports mild swelling and stiffness in his knee along with a “clicking” sensation when he flexes and extends it. In addition, he says that at unpredictable times he feels like his knee “might give way.” During the examination he reports pain along the medial joint line with palpation, and the McMurray test is positive. A subsequent MRI demonstrates a degenerative tear in the lateral portion of the medial meniscus with no significant degenerative arthritis. You begin to discuss a management plan with him, considering various surgical and nonsurgical options.
Given this patient’s age and clinical findings, which one of the following has been shown to produce better long-term outcomes than conservative management?
A) Intra-articular corticosteroids
B) Hylan GF 20 (Synvisc)
C) Arthroscopic meniscectomy
D) Arthroscopic meniscus repair
E) No additional treatment strategies

A

ANSWER: E
A systematic review found that arthroscopic procedures for degenerative meniscal tears in middle-aged adults with little or no arthritis do not significantly improve long-term pain or function compared to conservative management consisting of physical therapy and a standardized exercise program. No studies have compared conservative management to intra-articular injections of corticosteroids or hylan GF 20 for managing degenerative meniscal tears, either alone or when combined with conservative management.

How well did you know this?
1
Not at all
2
3
4
5
Perfectly
394
Q

n a patient on chronic warfarin (Coumadin) therapy who has a stable INR in the therapeutic range, which one of the following antibiotics would be most likely to elevate the INR?
A) Cephalexin (Keflex)
B) Clindamycin (Cleocin)
C) Penicillin G
D) Rifampin (Rifadin)
E) Trimethoprim/sulfamethoxazole (Bactrim)

A

ANSWER: E
Trimethoprim/sulfamethoxazole is one of the antimicrobials most likely to increase the INR of a patient taking warfarin. If trimethoprim/sulfamethoxazole is used in a patient on warfarin, reducing the warfarin dosage by 25%–40% is recommended, with close monitoring of the INR. The patient’s INR should be checked within 3–5 days of starting or stopping any antimicrobial.
55
First generation cephalosporins such as cephalexin, fourth generation cephalosporins, clindamycin, and penicillin G have a lower likelihood of affecting the INR. Rifampin decreases the INR and the warfarin dosage should be increased if rifampin is started. Other antimicrobials that significantly affect the INR include metronidazole and fluconazole. Azithromycin, ciprofloxacin, clarithromycin, and levofloxacin may impact the INR with a variable patient-specific effect.

How well did you know this?
1
Not at all
2
3
4
5
Perfectly
395
Q

A 6-month-old male is brought to your office by his parents for a well child check. He is their first child. On examination you note that he has two erupted teeth. The family lives in a town with fluoridated water and the parents would like to know how to care for their child’s teeth.
Which one of the following is the most effective preventive measure for this child’s newly erupted teeth?
A) No care is needed at this point
B) A parent wiping or brushing the teeth with water twice daily
C) A parent brushing the teeth with a smear of low-fluoride toothpaste twice daily
D) A fluoride supplement daily
E) Topical sealants

A

ANSWER: C
Early and consistent dental care in infants has been shown to reduce the rate of early childhood caries. The American Academy of Pediatric Dentistry recommends the use of low-fluoride toothpaste for tooth cleaning, starting with newly erupted teeth. According to the CDC, dental caries is one of the most prevalent chronic conditions among children in the United States. Dental caries can start soon after eruption of the first teeth. Brushing an infant’s teeth twice daily with a “smear” of fluoridated toothpaste is recommended. Fluoridated toothpaste has been found to be safe and effective for infants’ teeth. The use of fluoride in toothpaste is more effective for the prevention of caries than wiping or brushing the teeth with water. A daily fluoride supplement is not appropriate for this patient because the family’s local water source already contains fluoride. Topical sealants are intended for molar teeth, not incisors.

How well did you know this?
1
Not at all
2
3
4
5
Perfectly
396
Q

A 36-year-old male presents with a 6-week history of a mildly pruritic rash in his groin. An examination reveals small red-brown macules and larger patches with a sharp border. A Wood’s lamp examination reveals coral-red fluorescence.
Which one of the following would be the most appropriate treatment for this condition?
A) 0.1% triamcinolone cream
B) 2.5% hydrocortisone cream
C) Nystatin cream
D) Erythromycin gel
E) Fluconazole (Diflucan) orally

A

ANSWER: D
Erythrasma is a superficial infection caused by Corynebacterium minutissimum. It presents as small, red-brown macules that may coalesce into larger patches with sharp borders. It fluoresces coral red on Wood’s lamp examination. Cutaneous erythrasma is treated with erythromycin (topical, twice daily until the rash resolves, or oral, 250 mg four times daily for 2 weeks). Topical clindamycin, Whitfield ointment, and antibiotic soaps may also be beneficial. Triamcinolone cream, hydrocortisone cream, nystatin cream, and oral antifungals are not effective treatments for this bacterial infection.

How well did you know this?
1
Not at all
2
3
4
5
Perfectly
397
Q

A 34-year-old female comes to your office for follow-up after an emergency department visit because of anxiety. She notes persistent anxiety, poor focus, and palpitations. She also reports that she is not hungry and has lost several pounds. She reports “odd things happening” such as sudden weakness in her legs, falling, and getting lost.
When taking her history you note that the patient is hyperverbal and displays tangential speech. She has a temperature of 37.4°C (99.3°F), a heart rate of 134 beats/min, a respiratory rate of 20/min, and a blood pressure of 117/69 mm Hg. A physical examination reveals an anxious-appearing female who is tremulous at rest. A HEENT examination shows exophthalmos but no thyromegaly. A cardiac examination is unremarkable aside from tachycardia. A pulmonary examination reveals faint bibasilar crackles. An EKG shows sinus tachycardia. Laboratory results are as follows:
CBC  withinnormallimits Basicmetabolicpanel withinnormallimits TSH <0.08 U/mL(N0.35–3.00) FreeT4  4.51ng/dL(N0.89–1.80) FreeT3  >19.0pg/dL(N2.3–4.2)
Which one of the following would be most appropriate at this point?
A) Start methimazole (Tapazole)
B) Check for thyroid receptor antibody
C) Obtain a radioactive iodine uptake scan
D) Refer her to endocrinology
E) Admit her to the hospital

A

ANSWER: E
Hyperthyroidism is a common condition with a generally favorable prognosis. However, it is important to remember that life-threatening complications such as thyrotoxicosis, also known as thyroid storm, can occur. Symptoms of thyroid storm include fever, central nervous system dysfunction, gastrointestinal or liver dysfunction, and cardiovascular complications such as tachycardia and heart failure. The diagnosis is made using the Burch-Wartofsky Point Scale, which produces a total score based on the presence or absence of various diagnostic criteria. In this case, the patient has a score of 45, which is highly suggestive of thyroid storm. This acute, life-threatening condition typically requires care in an intensive-care unit. It would therefore be inappropriate to start treatment with an agent such as methimazole prior to hospitalization. While a thyroid receptor antibody test may be useful in identifying the cause of the condition it should not delay hospitalization. A radioactive iodine uptake test is also useful for identifying the underlying cause of hyperthyroidism but should be avoided until the thyroid storm has resolved. This patient requires hospitalization, so a referral to endocrinology would not be most appropriate at this time.

How well did you know this?
1
Not at all
2
3
4
5
Perfectly
398
Q

A 58-year-old male with uncontrolled type 2 diabetes sees you for follow-up after a recent hospitalization for urosepsis treatment with intravenous antibiotic therapy. His hospital course was complicated by Clostridioides (Clostridium) difficile colitis, and he completed oral vancomycin (Vancocin) therapy 2 weeks ago. He was nearly back to his baseline but has had recurrent watery diarrhea for the past 3 days. You confirm a recurrent infection.
Which one of the following would be the most appropriate treatment?
A) Probiotics
B) Fidaxomicin (Dificid), 200 mg twice daily for 10 days
C) Metronidazole (Flagyl), 500 mg three times daily for 10 days
D) Vancomycin, 125 mg four times daily for 10 days
E) Fecal microbiota transplantation

A

ANSWER: B
This patient presents with his first recurrence of Clostridioides (Clostridium) difficile infection, which was previously treated with vancomycin. Initial episodes can be treated with vancomycin (strong recommendation, high quality of evidence), fidaxomicin (strong recommendation, high quality of evidence), or metronidazole if the other two treatments are unavailable (weak recommendation, high quality of evidence). However, fidaxomicin is recommended for recurrent infection if vancomycin was prescribed for the initial episode (weak recommendation, moderate quality of evidence). If available, a prolonged tapered course of vancomycin could be used if a 10-day course was prescribed initially (weak recommendation, low quality of evidence). Vancomycin is only recommended for a first recurrent episode if metronidazole was used initially (weak recommendation, low quality of evidence). Metronidazole is not recommended for recurrent episodes. Probiotic administration is not recommended due to insufficient data. Fecal microbiota transplantation is only recommended for a second or subsequent recurrent infection (strong recommendation, moderate quality of evidence).

How well did you know this?
1
Not at all
2
3
4
5
Perfectly
399
Q

A 48-year-old female presents with concerns about hair loss. She has noticed gradual thinning of the hair on the top of her head for the last year. Her scalp is now visible through the hair. She is not taking any oral medications. A levonorgestrel IUD (Mirena) was placed 4 years ago. She has not had any recent illnesses or stressors and a review of systems is negative. Her mother had similar hair loss starting in her fifties. On examination you note thin hair on the top of the scalp, an intact frontal hair line, and growth of thin, wispy hairs on her crown.
Which one of the following would be most appropriate for this patient?
A) Removal of the levonorgestrel IUD
B) Spironolactone (Aldactone), 50 mg daily for 6 months
C) Finasteride (Proscar), 5 mg daily indefinitely
D) Minoxidil (Rogaine) 2% solution for 6 months
E) Minoxidil 5% foam indefinitely

A

ANSWER: E
Hair thinning on the crown of the head with the presence of small, wispy hairs among the regular hair is characteristic of female pattern hair loss (FPHL). A family history of similar issues is often present but not necessary for the diagnosis.
Topical minoxidil is the mainstay of treatment for FPHL (SOR A). It is available in a 2% solution or 5% foam for women (the 5% solution is indicated only for men). Treatment for FPHL as well as the male equivalent, androgenic alopecia, must be continued long term. With treatment there is often an initial period of increased hair loss. Regrowth is noticeable around 6 months. Discontinuation of treatment results in loss of regrown hair.
There is no clear association between hormone status and FPHL. Removal of this patient’s levonorgestrel IUD is unlikely to affect hair loss. Spironolactone has also been used for FPHL but evidence is lacking regarding its effectiveness. Finasteride is approved by the FDA only for males with hair loss. There is a high risk of teratogenicity with its use. It has been used in women but evidence of efficacy is minimal.

How well did you know this?
1
Not at all
2
3
4
5
Perfectly
400
Q

A 38-year-old male comes to your office for follow-up of his diabetes mellitus. He takes metformin (Glucophage) and dulaglutide (Trulicity) and his hemoglobin A1c is 6.5%. He has an LDL-cholesterol level of 120 mg/dL, an HDL-cholesterol level of 55 mg/dL, and a triglyceride level of 190 mg/dL. He asks your advice about statin therapy for cholesterol management.
You advise him to start
A) a low-intensity statin now
B) a moderate-intensity statin now
C) statin therapy at 40 years of age
D) statin therapy when his 10-year atherosclerotic cardiovascular disease (ASCVD) risk is >5%
E) statin therapy when his 10-year ASCVD risk is >7.5%

A

ANSWER: C
There is high-quality evidence from randomized, controlled trials that a moderate-intensity statin should be initiated for all patients age 40–75 years with diabetes mellitus regardless of their calculated 10-year atherosclerotic cardiovascular disease (ASCVD) risk. There is not strong evidence supporting the use of statins before age 40 in patients with diabetes unless their LDL-cholesterol level is very high. The ASCVD risk score is valid for patients over 40 years of age and cannot be calculated before then.

How well did you know this?
1
Not at all
2
3
4
5
Perfectly
401
Q

A 46-year-old male who injects heroin daily presents with a 6-month history of progressive dyspnea on exertion, a productive cough, and fatigue. He does not have any fever, chills, malaise, or hemoptysis. He has not had any sick contacts and has never smoked. On physical examination he shows no signs of distress, has a normal oxygen saturation on room air, and has normal breath sounds. A chest radiograph reveals bilateral perihilar shadowing.
A subsequent lung biopsy will most likely show
A) adenocarcinoma
B) branching hyphae
C) foreign body granulomas
D) caseating granulomas
E) noncaseating granulomas

A

ANSWER: C
Although persons who inject drugs are at high risk for a variety of pulmonary infectious diseases, this patient’s presentation, including the relatively slow development of symptoms, is most consistent with pulmonary foreign body granulomas. These result from the injection of crushed pills, talc, or other foreign substances, which are then deposited in the vasculature of the lungs. Adenocarcinoma is not as likely given the patient’s age and nonsmoking history. Branching hyphae would be seen in aspergillosis but this patient does not have fevers or malaise. Caseating granulomas are seen in tuberculosis, which is less likely given the absence of fever and hemoptysis. Noncaseating granulomas, seen in sarcoidosis, would not be more likely in this patient than in the general population.

How well did you know this?
1
Not at all
2
3
4
5
Perfectly
402
Q

A 68-year-old female with a history of diabetes mellitus, hypertension, and heart failure with preserved ejection fraction presents with a long-standing diabetic foot ulcer. The patient reports no signs of any gastrointestinal bleeding, no blood in her urine, no bleeding gums, and no vaginal bleeding. Her medications include metformin (Glucophage), insulin glargine (Lantus), lisinopril (Prinivil, Zestril), atorvastatin (Lipitor), and furosemide (Lasix).
A CBC reveals a WBC count of 7600/mm3 (N 4500–11,000), a hemoglobin level of 9.7 g/dL (N 14.0–17.5), a mean corpuscular volume of 89 m3 (N 80–100), and a platelet count of 412,000/mm3 (N 150,000–400,000).
To further assess the patient’s anemia you obtain the following laboratory results:
Ferritin  Serumiron  Transferrin  Reticulocytecount  VitaminB12 Haptoglobin 
293 ng/mL (N 22–275) 43 g/dL(N50–175) 190 mg/dL (N 177–264) 3.2%(N0.5–1.5) 564pg/mL(N230–1050) 198mg/dL(N63–273)
Which one of the following is the most likely cause of her anemia?
A) Anemia of chronic disease
B) Bone marrow suppression
C) Hemolysis
D) Iron deficiency
E) Vitamin B12 deficiency

A

ANSWER: A
This patient has findings most consistent with anemia of chronic disease, also known as anemia of inflammation. This condition is thought to be primarily a disorder of iron distribution in response to systemic inflammation, which also biases hematopoiesis toward myeloid cell production rather than erythropoiesis and shortens the erythrocyte lifespan. Anemia of chronic disease is a normocytic and normochromic anemia. Iron studies typically show evidence of iron restriction without systemic iron deficiency. A common challenge in diagnosis is when true iron deficiency coexists with anemia of chronic disease.
This patient’s normal WBC and platelet counts make bone marrow suppression less likely. The normal haptoglobin level and low reticulocyte count are not consistent with hemolysis. She has a normocytic rather than microcytic anemia and her ferritin level is elevated. These two factors make iron deficiency less likely despite her low serum iron level. The low normal transferrin level is also consistent with anemia of chronic disease rather than iron deficiency. Her normal vitamin B12 level makes a deficiency unlikely. Her history of a chronic foot ulcer and elevated inflammatory markers (ferritin and platelets) are consistent with anemia of chronic disease.

How well did you know this?
1
Not at all
2
3
4
5
Perfectly
403
Q

A 60-year-old male presents with left lower quadrant abdominal pain. His medical and surgical histories are remarkable only for a history of hypertension controlled with hydrochlorothiazide and lisinopril (Prinivil, Zestril), and no polyps seen on screening colonoscopy 5 years ago. He is afebrile, and a physical examination is notable only for mild abdominal tenderness in the left lower quadrant without peritoneal signs. A urinalysis is normal. You diagnose mild diverticulitis.
Which one of the following management options would be indicated at this time?
A) Rest and clear liquids
B) Avoidance of seeds, nuts, and popcorn
C) Abdominal CT
D) Referral for colonoscopy
E) Hospital admission for intravenous fluids and intravenous antibiotics

A

ANSWER: A
In patients with mild diverticulitis, outpatient management with rest and oral fluids is preferred. Avoidance of seeds, nuts, and popcorn does not reduce recurrence rates. CT of the abdomen may be indicated if the diagnosis is uncertain or if complications are suspected. Colonoscopy is contraindicated acutely and is only necessary for follow-up when age-appropriate cancer screening is indicated, or in cases of complicated disease. Antibiotics may not be necessary in all cases, and hospital admission is unnecessary for mild cases.

How well did you know this?
1
Not at all
2
3
4
5
Perfectly
404
Q

A 60-year-old retired dock worker presents to your office with chronic low back pain due to multiple herniated lumbar discs, with radicular pain down both legs. He rates his pain as a 3 out of 10. He currently takes oxycodone (OxyContin), 10 mg every 12 hours; pregabalin (Lyrica), 150 mg every 12 hours; acetaminophen, 1000 mg every 8 hours; meloxicam (Mobic), 15 mg daily; and cyclobenzaprine, 10 mg every 8 hours. These medications have been prescribed by another physician for the past 5 years. He tells you that his pain is tolerable but his sex drive and energy level have steadily decreased since starting these medications.
Which one of the following medications in this patient’s regimen would be most likely to decrease his libido?
A) Acetaminophen B) Cyclobenzaprine C) Meloxicam
D) Oxycodone E) Pregabalin

A

ANSWER: D
Opioid use may cause numerous adverse reactions, including drowsiness, pruritus, nausea, and constipation. In addition to these well known side effects, chronic opioid use can lead to hypogonadism through inhibition of gonadotropin-releasing hormone and an increase in prolactin. A recently published study found that long-term opioid users were nearly twice as likely to be diagnosed with hypogonadism as short-term opioid users. Given the large number of opioid users in the United States, prescribers should be aware of this adverse effect and screen for hypogonadism when appropriate. None of the other medications in this patient’s regimen, including acetaminophen, cyclobenzaprine, meloxicam, and pregabalin, are the likely cause of his decreased libido.

How well did you know this?
1
Not at all
2
3
4
5
Perfectly
405
Q

A 45-year-old female sees you for follow-up after an emergency department visit in which CT of the abdomen and pelvis was performed to detect kidney stones. Kidney stones were not seen and her flank pain was determined to be musculoskeletal in origin. She is feeling better now. However, the CT showed a simple-appearing 5.2-cm ovarian cyst with assessment limited by artifact. She does not have any symptoms, pelvic pain, bloating, fevers, night sweats, or unintentional weight loss. There is no family history of ovarian or breast cancer. She reports regular menstrual cycles.
Which one of the following would be most appropriate at this point?
A) Reassurance only
B) A CA-125 level
C) Ultrasonography of the pelvis
D) MRI of the pelvis
E) Referral to a gynecologic oncologist

A

ANSWER: C
Ovarian incidentalomas are very common, and appropriate management depends upon the size and appearance of the incidentaloma as well as the menopausal status of the patient. The Society of Radiologists in Ultrasound states that simple cysts 5 cm in premenopausal women and simple cysts 3 cm in postmenopausal women are considered normal and do not require follow-up. The American College of Radiology recommends that immediate ultrasonography be performed in the evaluation of simple-appearing cysts that are incompletely characterized by CT and are >5 cm in premenopausal women or >3 cm in postmenopausal women. This patient’s cyst appears benign, but further evaluation is recommended due to the large size. CA-125 levels have low sensitivity and specificity in premenopausal women and would not be indicated in this case. The initial imaging of choice is pelvic ultrasonography, so MRI of the pelvis is not necessary. Referral to a gynecologic oncologist is not indicated because there is no current evidence to suggest malignancy

How well did you know this?
1
Not at all
2
3
4
5
Perfectly
406
Q

A 56-year-old male comes to your office because of right shoulder pain for the past week. It started when he lifted a heavy piece of furniture while helping a friend move. He felt immediate sharp pain in his shoulder, which has since radiated down the biceps toward the right radial forearm. On examination there is no deformity of the shoulder or arm. He has increased pain with palpation in the anterior shoulder near the bicipital groove of the humerus.
You suspect biceps tendinitis. Anterior shoulder pain with which one of the following examination maneuvers of the right arm would be most consistent with this diagnosis?
A) Active or passive cross adduction of the arm at the shoulder
B) Shooting pain to the thumb with axial compression of the head with the neck flexed
toward the right shoulder
C) Resisted extension of the elbow with the shoulder in a neutral position
D) Resisted internal rotation of the shoulder with the elbow flexed to 90°
E) Resisted supination of the hand with the elbow flexed to 90°

A

ANSWER: E
This patient’s symptoms are consistent with bicipital tendinitis, which causes pain with abduction and external rotation of the arm, and tenderness of the bicipital groove with palpation. Resisted supination of the hand with the elbow flexed to 90° is the Yergason test, and anterior shoulder pain with this maneuver is consistent with bicipital tendinitis. Anterior shoulder pain with cross adduction of the arm is more consistent with acromioclavicular arthritis. Axial compression of the slightly flexed neck is the Spurling test for cervical radiculopathy. Extension of the elbow would activate the triceps, and internal rotation of the shoulder with the elbow flexed would result in less activation of the biceps than resisted supination.
Ref: Churgay CA: Diagnosis and treatment of biceps tendinitis and tendinosis. Am Fam Physician 2009;80(5):470-476. 2) Jameson JL, Fauci AS, Kasper DL, et al (eds): Harrison’s Principles of Internal Medicine, ed 20. McGraw-Hill, 2018, p 2647.

How well did you know this?
1
Not at all
2
3
4
5
Perfectly
407
Q

A 35-year-old male first presented to your office 4 months ago with a persistent chronic cough. He is a nonsmoker with no significant past medical history. Over the past few months he has been evaluated for GERD, asthma, eosinophilic bronchitis, and upper airway cough syndrome without symptomatic relief or diagnosis.
Which one of the following is recommended for chronic refractory cough in this otherwise healthy male?
A) Cyclobenzaprine
B) Duloxetine (Cymbalta)
C) Gabapentin (Neurontin)
D) Lorazepam (Ativan)
E) Propranolol

A

ANSWER: C
In randomized, controlled trials, gabapentin has demonstrated benefit for treating a refractory chronic cough after 4 weeks of treatment (SOR C). Chronic cough may be due to a hypersensitivity of the cough reflex, either centrally or peripherally. Cyclobenzaprine, duloxetine, lorazepam, and propranolol have not proven to be beneficial in reducing or eliminating chronic cough.

How well did you know this?
1
Not at all
2
3
4
5
Perfectly
408
Q

A 36-year-old female presents to your office with a 24-hour history of redness in her right eye. It is associated with mild pain but no drainage. On examination her visual acuity is 20/20 bilaterally, her pupillary reflex is normal, extraocular movements are intact, and there is no discharge noted. There is a focal area of hyperemia of the episcleral blood vessels noted along the medial aspect of the eye. Fluorescein staining is normal.
This patient’s presentation is most consistent with which one of the following?
A) Bacterial conjunctivitis
B) Viral conjunctivitis
C) Episcleritis
D) Iritis
E) Keratitis

A

ANSWER: C
An acute red eye is a common presentation in primary care and it is critical to differentiate serious causes from benign causes. Episcleritis is a self-limited condition that can be idiopathic and presents with mild discomfort and focal hyperemia. Conjunctivitis is typically associated with a discharge that is clear in viral cases and mucopurulent in bacterial cases. Iritis is associated with significant pain, a poorly reactive pupil, diminished vision, and photophobia. This patient does not have changes in visual acuity, photophobia, or severe pain as seen in keratitis, which would also cause an abnormal fluorescein stain showing corneal ulceration.

How well did you know this?
1
Not at all
2
3
4
5
Perfectly
409
Q

A fully immunized 7-month-old male is brought urgently to your office after his parents noted a possible seizure. The mother says that the infant began to “shake all over” for about 3–4 minutes and then promptly fell asleep for about 20 minutes. When he awoke he was alert but fussy and crying. He has been ill for the last few days with a cough, congestion, decreased oral intake, and fevers up to 101°F.
On examination he has an oral temperature of 38.3°C (100.9°F), a heart rate of 170 beats/min, a respiratory rate of 50/min, and an oxygen saturation of 97% on room air. The infant is fussy but consolable. His mucous membranes are moist, his tympanic membranes are clear, and he has a normal oropharynx. He has clear rhinorrhea. Examination of the heart is normal, and examination of the lungs reveals rhonchi and wheezes. He is moving all of his extremities normally.
Which one of the following would be the most appropriate initial step in the evaluation of this child?
A) A basic metabolic panel
B) Radiography of the chest
C) MRI of the brain
D) Electroencephalography
E) A lumbar puncture

A

ANSWER: B
Children who have a simple febrile seizure and appear neurologically intact do not require routine testing except to determine the source of their fever (SOR C). This child has signs of possible pneumonia so a chest radiograph would be warranted to look for the source of infection that triggered the fever.
Routine laboratory testing is not indicated in the workup of simple febrile seizures. There is a low risk that these children will have low sodium or glucose levels, and this would not predict seizure recurrence. Routine neuroimaging such as MRI is not recommended for febrile seizures. Electroencephalography is not useful for predicting the recurrence of simple febrile seizures and would not be indicated in the workup of these seizures. A lumbar puncture is indicated only in cases where the child has neurologic findings suggestive of meningitis, but that is not the case for this child.

410
Q

Which one of the following developmental milestones would be expected in a typical 12-month-old child?
A) Standing independently
B) Identifying at least two body parts
C) Using three words other than names
D) Scribbling spontaneously
E) Building a three-cube tower

A

ANSWER: A
Standing independently is an expected developmental milestone for a 12-month-old child. Identifying at least two body parts, using three words other than names, scribbling spontaneously, and building a three-cube tower are expected milestones for older children.

411
Q

Which one of the following patients should undergo screening for vitamin D deficiency?
A) A 27-year-old male with recurrent major depressive disorder
B) A 58-year-old male with a glomerular filtration rate of 28 mL/min/1.73 m2
C) A 69-year-old female with chronic osteoarthritis pain
D) A 75-year-old female with recurrent falls
E) An 88-year-old male with severe fatigue

A

ANSWER: B
The measurement of vitamin D levels is recommended only for patients with decreased kidney function, various skeletal diseases, or hypercalcemia (SOR C). Vitamin D deficiency is common in patients with chronic kidney disease, and it is associated with cardiovascular morbidity and mortality in those patients. However, good-quality evidence is lacking regarding the effect of vitamin D supplementation in these patients. There is no recommendation for screening the general population for vitamin D deficiency (SOR B), nor should routine vitamin D supplementation be recommended for community-dwelling adults (SOR A). Vitamin D supplementation has not been found to improve depression, osteoarthritis, chronic pain, or fatigue (SOR A). Meta-analyses have found no relationship between vitamin D deficiency and falls, fractures, or mortality.

412
Q

You diagnose polymyalgia rheumatica in a 63-year-old female and begin treatment with oral prednisone, 20 mg daily. You anticipate several months of treatment with prednisone and plan to taper the dose as tolerated but are concerned about her bone health.
Which one of the following would be most appropriate for helping to prevent osteoporotic fractures in this patient?
A) A FRAX score based on DXA results now
B) A FRAX score based on DXA results in 6 months
C) A calcium intake of 1200 mg daily
D) Alendronate, 35 mg weekly

A

ANSWER: A
Glucocorticoid use is the most common cause of secondary osteoporosis. Glucocorticoids increase bone resorption early, so addressing the issue at the start of treatment is vital in preventing fractures. The use of glucocorticoids is associated with an increased risk of fracture in the first 6 months.
A bone density test should be performed shortly after starting corticosteroid treatment (SOR C). The fracture risk can then be calculated using the FRAX assessment tool. If the risk of major osteoporotic fracture is sufficiently elevated, then treatment is recommended. One caveat is that the FRAX score should be adjusted upward if the prednisone dosage is >7.5 mg daily.
Calcium alone has not been shown to reduce the risk of fracture in osteoporosis. Calcium and vitamin D together have been shown to prevent decreases in bone mineral density with low-dose prednisone use but the effect with high doses is unknown. If a patient has an increased fracture risk, oral bisphosphonates should be started. It is not recommended that they be used empirically to reduce the risk of fracture.

413
Q

A 42-year-old transgender male comes to your office for a routine health maintenance examination. The patient’s current medications include testosterone enanthate (Xyosted), 100 mg subcutaneously every 7 days, for gender affirmation, and medroxyprogesterone acetate (Depo-subQ Provera 104), 104 mg subcutaneously every 12 weeks, for contraception. The patient has no current chronic health conditions and no current sexual partners, but he has previously been sexually active with men and women.
Which one of the following health conditions is more likely to occur in this patient compared to a female cisgender patient?
A) Anemia
B) Cervical cancer
C) Dyslipidemia
D) Kidney disease
E) Venous thromboembolism

A

Transgender describes persons whose experienced or expressed gender differs from their sex assigned at birth. In the United States approximately 150,000 youth and 1.4 million adults identify as transgender, though many believe these numbers underestimate the actual prevalence. Transgender men who take testosterone may experience increased muscle mass and decreased fat mass, male pattern baldness, increased sexual desire, clitoromegaly, decreased fertility, deepening of the voice, cessation of menses, acne, and a significant increase in body hair, particularly on the face, chest, and abdomen. Risks of testosterone therapy include more atherogenic lipid profiles, an increase in blood pressure, and erythrocytosis (rather than anemia). Severe liver dysfunction is unusual at therapeutic dosages but is a concern at dosages above the recommended therapeutic range. Testosterone therapy has not been associated with cervical cancer, kidney disease, or venous thromboembolism (VTE). Estrogen-based therapies for male-to-female transgender patients do carry an increased risk for VTE. It is not clear whether increased blood pressure and dyslipidemia in these patients translates into an increase in cardiovascular events. Even so, when identified, treatment such as antihypertensive drugs and statins to address these risk factors is recommended as for any other patient.

414
Q

A 33-year-old female presents with palpitations and excessive sweating. A physical examination is normal. Laboratory findings include a TSH level of 0.02 U/mL (N 0.40–4.00) and a free T4 level of 3.9 ng/dL (N 0.7–1.9). Radionuclide scanning reveals no uptake.
Which one of the following would explain these findings?
A) Thyroid hormone resistance
B) Graves disease
C) A toxic nodular goiter
D) Excess thyroid hormone intake
E) A thyrotropin-secreting pituitary tumor

A

ANSWER: D
Excess thyroid hormone intake would cause a low TSH level with a high free T4 level. Other possibilities include an hCG-secreting tumor and the thyrotoxic phase of subacute thyroiditis. An elevated TSH level would be seen with thyroid-hormone resistance or a thyrotropin-secreting pituitary tumor. Graves disease causes a homogeneous increased thyroid uptake on radionuclide scanning, whereas a hot nodule would be expected with toxic nodular goiter.

415
Q

A 72-year-old female presents with pain, swelling, and decreased range of motion in her right great toe for several months. There is no history of injury or overuse. On examination the metatarsophalangeal joint is swollen and mildly tender, but not red. Dorsiflexion and plantar flexion are approximately 30°. A radiograph shows joint space narrowing and a small bone spur.
Recommended management at this time would be
A) stretching and strengthening exercises
B) a rigid shoe insert
C) ibuprofen
D) a corticosteroid injection
E) surgical referral

A

ANSWER: B
Hallux rigidus affects as many as 50% of women and 40% of men by the age of 70. It is usually due to osteoarthritis of the metatarsophalangeal (MTP) joint and presents as decreased range of motion, swelling, and pain. With progression of the condition, flare-ups become more frequent and more severe, and it can be mistaken for gout. Initial treatment is restriction of motion across the MTP joint. A stiffening shoe insert does relieve pain and most patients see improvement without surgery. Custom orthotics, rigid inserts, or hard-soled shoes are options that are more effective than NSAIDs. Corticosteroid injections, preferably administered with ultrasound guidance, and surgery are reserved for those who fail to respond to more conservative measures. Stretching and strengthening exercises are recommended for plantar fasciitis more so than for hallux rigidus.

416
Q

A 12-year-old male with type 1 diabetes is brought to your office for routine follow-up. Laboratory work performed prior to the appointment shows an LDL-cholesterol level of 120 mg/dL.
In addition to counseling the patient on a heart-healthy diet and daily physical activity, which one of the following would you recommend?
A) No additional measures
B) Fish oil supplements
C) Atorvastatin (Lipitor)
D) Ezetimibe (Zetia)
E) Gemfibrozil (Lopid)

A

ANSWER: C
Pediatric type 1 diabetes is recognized as a high-risk condition for the future development of cardiovascular disease. Current guidelines recommend initiating a statin, in addition to education regarding a healthy diet and physical activity, for pediatric patients in this high-risk category with an LDL-cholesterol level >100 mg/dL. Statins such as atorvastatin are recommended for first-line treatment according to multiple studies that demonstrate their efficacy and benefits in reduction of cardiovascular morbidity and mortality, along with long-term studies demonstrating their safety. Fish oil supplements, ezetimibe, and gemfibrozil would not be appropriate recommendations for this patient at this time.

417
Q

You are reviewing and updating your routine health care examination electronic health record templates to include formal recommendations from the U.S. Preventive Services Task Force. You also consider age-specific causes of mortality in order to create corresponding preventive strategies.
Which one of the following is the leading cause of mortality among people 45–64 years of age?
A) Accidents
B) Diabetes mellitus
C) Heart disease
D) Malignancy
E) Suicide

A

ANSWER: D
The leading cause of mortality among people aged 45–64 years is malignancy. The U.S. Preventive Services Task Force generally recommends a focus on cancer screening in this age group. Accidents are the third most common cause of mortality in people 45–64 years of age, but they are the leading cause of mortality among people 15–44 years of age, and preventive recommendations reflect interventions to prevent accidents. Diabetes mellitus is the sixth most common cause of mortality in people 45–54 years of age, and the fifth most common cause in people 55–64 years of age. Heart disease is the second most common cause of mortality in people 45–64 years of age, but it is the leading cause of mortality in people 65 years of age and older. Suicide is the fourth most common cause of mortality in adults 45–54 years of age, and the eighth most common cause in adults 55–64 years of age.

418
Q

A 64-year-old female presents to the emergency department with a 10-day history of increasing shortness of breath and mild tachycardia. On examination she has an oxygen saturation of 75% on room air.
Which one of the following additional findings would suggest a diagnosis of acute respiratory distress syndrome (ARDS)?
A) Improved oxygen saturation with supplemental oxygen
B) Improvement of her symptoms with diuretic therapy
C) Bilateral airspace opacities seen on a chest radiograph
D) A flattened diaphragm seen on a chest radiograph
E) A right lower lobe infiltrate seen on a chest radiograph

A

ANSWER: C
Acute respiratory distress syndrome (ARDS) will often present similarly to pneumonia and heart failure with dyspnea, hypoxemia, and tachypnea. ARDS typically does not respond to supplemental oxygen or diuretic therapy. Patients decompensate quickly and usually require mechanical ventilation. Chest radiographic findings include bilateral airspace opacities but not a localized infiltrate as with pneumonia, venous congestion or cardiac enlargement as with heart failure, or a flattened diaphragm (associated with COPD).

419
Q

A 61-year-old female comes to your office for a routine health maintenance visit. She brings in laboratory results from a local blood donation facility. She donated blood for the first time in several years but was informed afterward that she is ineligible and should follow up with her primary care physician. She feels well and is asymptomatic. She does not recall receiving the hepatitis B immunization series. The letter she received included the following hepatitis B test results:
Anti-HBcscreeningtest positive HBsAgscreeningtest negative HBVnucleicacidtest(NAT) negative
You order additional hepatitis B testing, with the following notable results: Anti-HBs 56mIU/mL(N<10)
Anti-HBcIgM nonreactive
Which one of the following is the most likely explanation of these laboratory results?
A) These results indicate a false positive
B) She was never infected with hepatitis B
C) She has an acute hepatitis B infection
D) She has a chronic hepatitis B infection
E) She has recovered from a past hepatitis B infection

A

ANSWER: E
This patient’s laboratory studies are consistent with a past natural hepatitis B virus (HBV) infection and she is now immune. If she had never been infected her anti-HBc and anti-HBs would both be negative. If she had an acute infection the HBsAg, anti-HBc IgM, and HBV nucleic acid test (NAT) would have all been positive along with the total anti-HBc. If she had a chronic infection the HBsAg and HBV NAT would be positive in addition to the total anti-HBc. Furthermore, the anti-HBs would be negative in both acute and chronic infection since its presence is associated with recovery from infection. If her anti-HBc screening test were a false positive the anti-HBs would be negative.

420
Q

A 44-year-old male with diabetes mellitus, hypertension, obesity, and chronic pain is on chronic opiate therapy. He comes to your office because of a lack of sex drive, decreasing strength, low overall energy levels, and hot flashes. After ruling out other causes you confirm that he has a low total testosterone level on two separate early morning laboratory tests. He would like to start testosterone therapy.
Which one of the following would be the most appropriate next step?
A) Order a PSA level and perform a digital rectal examination
B) Order LH and FSH levels
C) Order chromosomal studies
D) Discuss risks and benefits of testosterone replacement therapy and start low-dose replacement
E) Inform him that testosterone replacement therapy would not be beneficial for him because of its high risk

A

ANSWER: B
After confirming low testosterone with two morning laboratory tests, the next step is to attempt to determine the cause of the low testosterone. Checking LH and FSH levels is recommended to evaluate for primary hypogonadism. If primary hypogonadism is present, chromosomal studies should be considered. Before initiating testosterone therapy, checking the patient’s PSA level and performing a digital rectal examination are recommended, but in this case the initial workup is not yet complete. It is crucial to discuss the risks and benefits of treatment, and as with all medications, it is recommended to start with the lowest dose needed. However, starting treatment in this case is premature. Evidence for testosterone replacement therapy is not as robust as desired and it does carry risks, but as long as there are no contraindications it can be initiated after a discussion of the risks and benefits.

421
Q

A patient’s office spirometry results demonstrate an obstructive pattern. This would be seen with which one of the following?
A) Asbestosis exposure
B) Cystic fibrosis
C) Idiopathic pulmonary fibrosis
D) Nitrofurantoin exposure
E) Sarcoidosis

A

ANSWER: B
Office spirometry can be very helpful in the development of a differential diagnosis. The differential can be narrowed with the use of office spirometry, as many conditions create either an obstructive or restrictive pattern. Of the options listed, only cystic fibrosis can cause an obstructive pattern. Other causes of an obstructive pattern include asthma, COPD, 1-antitrypsin deficiency, and bronchiectasis, among others. Common diseases or conditions causing restrictive patterns include adverse reactions to nitrofurantoin, methotrexate, and amiodarone. Chest wall conditions such as kyphosis, scoliosis, and morbid obesity can also cause restrictive patterns. Interstitial lung disease, including idiopathic pulmonary fibrosis, sarcoidosis, and asbestosis, also causes a restrictive pattern (SOR A).

422
Q

The mother of a 6-month-old infant is concerned that her child’s feet are “deformed.” On examination the heel bisector line is between the third and fourth digits on the right foot and on the third digit on the left foot. You attempt to flex the feet, and both appear to be rigid.
Which one of the following would you recommend as a corrective intervention?
A) Night splints
B) Adjustable orthotic shoes
C) Braces
D) Physical therapy
E) Surgical correction

A

ANSWER: B
Adjustable orthotic shoes in infants who are not yet walking can be effective for the treatment of metatarsus adductus (SOR B). These orthotics can be adjusted to apply an abduction force on the forefoot while maintaining the heel in a neutral position. Night splints, braces, and physical therapy are not indicated or proven to correct this deformity. Surgery has high complication rates and is rarely indicated to treat metatarsus adductus.

423
Q

You admit a 68-year-old female with an acute stroke to the hospital. She has no other acute cardiovascular conditions. CT rules out a hemorrhagic event. You have determined that the patient is not a candidate for reperfusion therapy with alteplase or thrombectomy.
You advise the nursing staff that you will be initiating antihypertensive therapy if the patient’s blood pressure rises above a threshold of
A) 120/80 mm Hg
B) 140/90 mm Hg
C) 160/100 mm Hg
D) 180/110 mm Hg
E) 220/120 mm Hg

A

ANSWER: E
Because patients with an acute ischemic stroke may require the increased perfusion pressure to limit ischemia, antihypertensive therapy should not be given during the first 48–72 hours as long as they are not candidates for, or recipients of, reperfusion therapy with alteplase or thrombectomy; do not have a comorbid condition requiring acute blood pressure lowering; and do not have a blood pressure >220/120 mm Hg.

424
Q

Which one of the following is an indication for a radionuclide thyroid uptake scan for a patient with a single thyroid nodule confirmed by ultrasonography?
A) The presence of antithyroid antibodies
B) A low TSH level
C) An elevated TSH level
D) A normal TSH level and a tender nodule
E) The presence of the thyroid nodule regardless of other findings

A

ANSWER: B
In all patients found to have a thyroid nodule, the first steps in evaluation should be measuring the TSH level and performing thyroid ultrasonography. If the TSH level is low, then a radionuclide scan is indicated. If the scan indicates hyperfunctioning of the nodule, then fine-needle aspiration is not necessary and radioactive iodine ablation is generally the treatment of choice. With normal to high TSH levels, the need for a biopsy and for follow-up surveillance depends on the findings on ultrasonography. Measuring antithyroid antibodies in a patient with a thyroid nodule is not part of the routine workup.

425
Q

A 15-year-old female is brought to your office by her parents for evaluation because they are concerned about her restrictive eating patterns and weight loss. The patient is unconcerned about these issues and says that she feels well and does not need any evaluation. Her parents tell you that for the past 6 months she has had an increasingly restricted diet to the point that she now drinks only water and eats only vegetables and roasted chicken or turkey. They report that she looks much thinner now than she did 6 months ago, but they are uncertain how much weight she has lost. She says that she does not feel depressed or anxious and she is doing well in school.
On examination she has a height of 163 cm (64 in) and a weight of 43 kg (95 lb), with a BMI of 16 kg/m2. She has a pulse rate of 52 beats/min and a blood pressure of 102/68 mm Hg while seated and 84/58 mm Hg while standing. Evaluation of her teeth shows significant erosion of the enamel.
When considering the psychotherapy aspect of care for this patient, which one of the following is preferred for treatment of her condition?
A) Cognitive-behavioral therapy
B) Dialectical behavioral therapy
C) Family therapy
D) Interpersonal therapy
E) Psychodynamic therapy

A

ANSWER: C
This patient has anorexia nervosa, likely a combination of the restrictive subtype and the binge-eating and purging subtype, given the dental findings on examination. This condition is difficult to treat and carries significant risk of mortality, with an estimated aggregate mortality of 5.6% per decade. Coexisting psychiatric conditions are common, with major depression, anxiety disorders, obsessive-compulsive disorder, and trauma-related disorders predominating. Medical complications include disorders of the esophagus and stomach related to repeated vomiting; cardiovascular conditions associated with bradycardia, orthostatic hypotension, and arrhythmias; renal disease due to chronic dehydration and electrolyte abnormalities; and osteoporosis and bone marrow abnormalities. Treatment may be provided in inpatient or outpatient settings, depending on the severity of disease. Psychotherapy is the foundation of treatment and parental involvement is key for children and adolescents. Parents or guardians typically have a high level of distress around their child’s condition and family therapy helps provide consistent support for treatment goals set by the care team. Other types of one-on-one therapy may be appropriate to augment family therapy and for adolescents with specific comorbidities. Psychotropic drugs have not been consistently and clearly shown to add benefit to psychotherapy, although they are often prescribed.

426
Q

A premenopausal 48-year-old female comes to your office because of a 3-month history of increased frequency and volume of vaginal bleeding. She typically menstruates every 24 days but now has bleeding every 10–14 days and notes an increased volume of blood as well. She feels well otherwise. Her cervical cancer screening is current. A bimanual examination reveals blood at the cervical os. Otherwise the cervix appears normal, the uterus is normal to palpation, and there are no adnexal masses.
Which one of the following should be the next step in the evaluation?
A) Reassurance that irregular menses are common in the perimenopausal period
B) HPV testing
C) CT of the pelvis with and without contrast
D) An endometrial biopsy

A

ANSWER: D
This patient has abnormal uterine bleeding characterized by an increased frequency and volume of vaginal bleeding. Due to the increased risk of endometrial cancer, current guidelines recommend that all women >45 years of age presenting with abnormal uterine bleeding undergo endometrial sampling. Irregular menses can occur during the perimenopausal period but this patient’s increased frequency and volume of vaginal bleeding combined with her age warrant further evaluation. In a patient with an up-to-date Papanicolaou smear and normal-appearing cervix, HPV testing would have no role in the evaluation. Transvaginal ultrasonography is recommended if a bimanual examination is abnormal or if symptoms persist despite treatment. CT is rarely indicated if imaging is necessary, because transvaginal ultrasonography is preferred.

427
Q

A 55-year-old female presents with swelling and some redness in the area of her right ankle that had a gradual onset over the past week. She has not had any injury, fever, or other signs of systemic illness and has no pain. Her past medical history is significant for type 2 diabetes with polyneuropathy that is moderately well controlled, hypertension, hyperlipidemia, and a BMI of 35 kg/m2.
On examination her right ankle and foot are slightly larger than the left, exhibit faint erythema, and feel slightly warmer than the left. No pain is noted with palpation, and her ankle ligaments appear to be intact. Pedal pulses are 2+ bilaterally and she has no calf pain or swelling.
Which one of the following would be the most appropriate next step?
A) Reassure her that the lack of pain indicates the absence of a serious disease process
B) Prescribe antibiotics for presumed cellulitis
C) Recommend compression stockings, leg elevation, and monitoring
D) Provide an ankle stabilizing brace
E) Obtain bilateral weight-bearing foot radiographs

A

ANSWER: E
Acute Charcot neuropathy is a commonly missed diagnosis, and the diagnosis is delayed in up to 25% of cases. The diagnosis should be considered in patients over age 40 with neuropathy and obesity who present with unilateral foot swelling. There may be associated erythema and warmth, and pain may be absent. In a patient with suspected acute Charcot neuropathy, bilateral weight-bearing radiographs are recommended to detect fractures of the midfoot. Acute Charcot neuropathy is frequently painless, and its consequences can be severe, so it would be inappropriate to counsel a patient that lack of pain means the absence of serious disease. Charcot neuropathy is commonly misdiagnosed as cellulitis. In this patient’s presentation, cellulitis is not a clear diagnosis, and Charcot neuropathy needs to be considered before initiating treatment for cellulitis. Compression stockings and leg elevation are appropriate for peripheral edema when other causes of edema have been evaluated and addressed, but in this case the swelling is lower on the leg than what compression stockings would usually treat, and further evaluation is required prior to treatment. There is no evidence for ankle sprain or instability in this patient, so an ankle brace would not be appropriate.

428
Q

The American Thoracic Society/Infectious Diseases Society of America guidelines recommend which one of the following for the diagnosis and initial management of non-severe community-acquired pneumonia in adults?
A) Use of a validated clinical prediction rule to determine the need for hospitalization
B) Urine antigen testing for Legionella
C) Blood and sputum cultures to guide therapy
D) Procalcitonin to determine the need for antibacterial therapy
E) Coverage for methicillin-resistant Staphylococcus aureus (MRSA)

A

ANSWER: A
The American Thoracic Society (ATS) and the Infectious Diseases Society of America (IDSA) guideline recommends use of a validated clinical prediction rule, preferably the Pneumonia Severity Index (PSI), to determine the need for hospitalization in adults diagnosed with community-acquired pneumonia (CAP) (strong recommendation). The yield of blood cultures is around 2% (outpatients) to 9% (inpatients) in adults with non-severe CAP. A sputum culture and a Gram stain of respiratory secretions are recommended in patients classified as having severe CAP, or in those with strong risk factors for methicillin-resistant Staphylococcus aureus (MRSA) or Pseudomonas aeruginosa. Randomized trials have failed to show a benefit for urinary antigen testing for Streptococcus pneumoniae and Legionella. ATS/IDSA guidelines recommend empiric antibiotic therapy for adults with clinically suspected and radiographically confirmed CAP regardless of the initial serum procalcitonin level (strong recommendation). Coverage for MRSA is not recommended in patients without risk factors for MRSA pneumonia.

429
Q

You diagnose nonvalvular atrial fibrillation in a 54-year-old male. His CHA2DS2-VASc score is 2.
Which one of the following should you recommend as first-line therapy for stroke prevention?
A) No antithrombotic therapy
B) Aspirin
C) A direct oral anticoagulant
D) Low molecular weight heparin
E) A vitamin K antagonist

A

ANSWER: C
Direct oral anticoagulants such as apixaban, betrixaban, dabigatran, edoxaban, and rivaroxaban are first-line agents for prevention of stroke in patients with nonvalvular atrial fibrillation with a CHA2DS2-VASc score 2 in men or 3 in women. For patients with atrial fibrillation without valvular heart disease, forgoing antithrombotic therapy is only appropriate in patients with a CHA2DS2-VASc score of 0 in men and 1 in women. Aspirin should not be considered a substitute for anticoagulation but may be suggested for patients with an unprovoked deep vein thrombosis or pulmonary embolism who do not wish to receive lifelong anticoagulation. Low molecular weight heparin is recommended as the anticoagulant of choice in patients with cancer and venous thromboembolism, although direct anticoagulants may be appropriate in some situations. If a patient has moderate to severe mitral stenosis or a mechanical valve, then vitamin K antagonists are the preferred agent.

430
Q

Anticoagulant for VE

A

LMWH (enoxaparin)

431
Q

When to use vit K antagonist

A

Moderate to severe mitral stenosis or mechanical valve

432
Q

Tx for Afib

A

Apixaban (eliquis)
Betrixaban
Dabigatran
Edoxaban
Ribaroxaban (xarelto)

433
Q

A 1-month-old infant is brought to your office by her parents for routine follow-up. The infant was born at 35 weeks and 5 days gestation by normal vaginal delivery after induction of labor for maternal preeclampsia with severe features. The infant was discharged with the mother on the third day of life. There were no additional complications noted. Today the parents report exclusive breastfeeding and tell you the infant spits up after longer feedings. Voiding and stooling are as expected and the infant’s development is normal. The weight at birth was at the 20th percentile, and today’s weight is at the 25th percentile.
Which one of the following would be the most appropriate intervention at this time?
A) Caloric supplementation in addition to breastfeeding
B) Iron supplementation
C) Vitamin E supplementation
D) Referral for early intervention

A

ANSWER: B
In 2018, 1 in 10 infants born in the United States were preterm, with significant racial and ethnic differences noted. Breastfed infants born before 37 weeks gestation should receive iron supplementation at 2 mg/kg/day after 1 month of life. This infant does have some physiologic reflux but since this infant appears asymptomatic, the parents should be counseled on behavioral techniques to reduce spitting up, as there is no clear long-term benefit to antireflux medication. This infant’s growth and development are normal so there is no indication for caloric fortification of breast milk, which is more appropriate for small-for-gestational-age infants or those born below the 10th percentile. There is no specific recommendation for micronutrient supplementation other than iron and vitamin D, so there is no indication to initiate vitamin E supplementation. This child should be screened for developmental delay at each office visit, but there is currently no evidence of delay so referral to early intervention is not indicated.

434
Q

A 34-year-old male began a sexual relationship with a woman 3 months ago and the relationship ended on friendly terms last week. He received a call yesterday from the woman, who said she had developed a rash that resulted in a diagnosis of syphilis and that he should be evaluated and treated if appropriate. He has no symptoms and a serologic test for syphilis is negative. He has no known drug allergies.
Which one of the following would be most appropriate at this time?
A) Daily self-inspection of the penis to identify a chancre
B) Observation with a repeat serologic test for syphilis in 6 weeks
C) Azithromycin (Zithromax), 2 g orally as a single dose
D) Penicillin G benzathine (Bicillin L-A), 2.4 million units intramuscularly as a single dose
E) Penicillin G benzathine, 2.4 million units intramuscularly once weekly for 3 weeks

A

ANSWER: D
The evidence suggests that this patient did not have syphilis prior to this lone contact and a diagnosis of syphilis cannot be confirmed by examination or testing at this point. He should be treated presumptively for early syphilis, even though the serologic test result is negative, because he had sexual contact within the past 90 days with a person who was diagnosed with secondary syphilis. The same is true for individuals exposed to sex partners diagnosed with primary or early latent syphilis during the same time period. When the contact occurred more than 90 days before confirmation of a negative serologic test result, no treatment is necessary.
The recommended treatment for individuals such as this patient and for those with primary or secondary syphilis is a single dose of penicillin G benzathine, 2.4 million units. For patients with a penicillin allergy, oral treatment with doxycycline, 100 mg twice daily; tetracycline, 500 mg four times daily; or azithromycin, 2 g as a single dose, has been effective as an alternate treatment option but should only be used when penicillin is contraindicated and should be followed by close monitoring of serologic tests.

435
Q

You see a 55-year-old female for preoperative clearance prior to a cholecystectomy. When you examine the patient she asks you to also look at the lesion on her foot (shown below).
Which one of the following is the most likely etiology of the lesion?
A) Arterial B) Venous C) Infectious
D) Neuropathic E) Pressure

A

ANSWER: D
The likely etiology of this patient’s lesion is neuropathy, most likely due to poorly controlled diabetes mellitus. Peripheral neuropathy can predispose patients to abnormal gait patterns and/or unrecognized trauma. These deep ulcers usually present over a bony prominence and are surrounded by a callus (SOR A).
64
Ulcers of arterial origin are due to tissue ischemia and are most typically deep but on the anterior leg, distal dorsal foot, and toes, and have a dry, fibrous base with poor granulation tissue. Tendons can be exposed. Venous ulcers are due to venous hypertension and chronic venous insufficiency. These ulcers are shallow and exudative with good granulation tissue in the base. Common locations are over bony prominences such as the medial malleolus. Infectious lesions would typically have erythema and extensive exudation. Pressure ulcers occur on areas of high pressure in patients with limited mobility, especially on the sacrum, heels, and hips.

436
Q

Which one of the following is the most common cause of koilonychia (spoon-shaped nails)?
A) Chronic pulmonary disease
B) Hyperthyroidism
C) Iron deficiency anemia
D) Onychomycosis
E) Psoriasis

A

ANSWER: C
Iron deficiency anemia is the most common cause of koilonychia, which is also known as spoon nail because it appears as a central depression in the nail that curves outward away from the nailbed, giving the nail the appearance of a spoon. If iron deficiency anemia is the cause of koilonychia, the nail will return to a normal appearance when the anemia is corrected. Chronic pulmonary disease is associated with clubbing of the nails. Hyperthyroidism can result in onycholysis and brown discoloration of the nail plate. Onychomycosis causes onycholysis, hyperkeratosis, and yellow streaks. Psoriasis typically causes pitted nails, although patients can also have some hyperkeratosis and onycholysis.

437
Q

A 22-year-old gravida 1 para 0 was recently diagnosed with gestational diabetes. Her fasting blood glucose levels have consistently been 120–130 mg/dL since she began following nutrition and exercise recommendations.
Based on guidelines from the American Diabetes Association and the American College of Obstetricians and Gynecologists, which one of the following would be preferred at this point?
A) No change in management
B) Loosening the calorie restrictions
C) Adding glyburide
D) Adding insulin
E) Adding metformin (Glucophage)

A

ANSWER: D
Pharmacologic treatment should be initiated in patients with gestational diabetes mellitus (GDM) when nutrition and exercise therapy are not adequate to meet goals. Accepted goals are fasting blood glucose levels <95 mg/dL, 1-hour postprandial glucose levels <140 mg/dL, and 2-hour glucose levels <120 mg/dL. Although oral antidiabetic medications are being used more frequently in GDM, insulin is the preferred treatment recommended by the American Diabetes Association and the American College of Obstetricians and Gynecologists. Oral medication may be initiated in patients who refuse insulin or are unable to comply with insulin management. This recommendation is made predominantly because metformin has not shown superiority and there is a lack of long-term outcome studies in the offspring exposed to metformin. Glyburide has not shown outcomes equivalent to those of metformin or insulin.

438
Q

An otherwise healthy 72-year-old male presents with a 4-week history of catching and triggering of his right middle finger. When he awakens in the morning the finger is locked in flexion at the proximal interphalangeal joint and he has to manually extend the finger. He enjoys playing golf and painting, both of which are compromised by the triggering of his finger. He has not had any pain or numbness.
Which one of the following would you tell him regarding his treatment options for this condition?
A) Less than 15% of cases spontaneously resolve by 1 year
B) Splinting of the affected finger can result in complete resolution
C) NSAID injections have been found to be more effective than corticosteroid injections
D) Surgical release is the most cost-effective treatment

A

ANSWER: B
This patient presents with trigger finger, which has a lifetime prevalence of 2%–3% in the adult population, with higher prevalence rates in patients with diabetes mellitus. There are several options for conservative treatment that are appropriate prior to consideration of surgical release. Splinting, which is a first-line treatment, has been shown to be effective. Single-joint orthoses at either the metacarpophalangeal or the proximal interphalangeal joint can be effective (SOR B). The duration of splinting can range from 6 weeks to 3 months.

A retrospective case series analysis of trigger finger managed by observation only found that trigger finger resolved spontaneously in 52% of patients, with the majority resolving within 1 year. Corticosteroid injections are generally effective but efficacy depends on the severity of the condition and on the number of fingers involved. They are more effective than NSAID injections (SOR B). Surgical release is considered the most effective treatment but not the most cost-effective. A series of three corticosteroid injections could result in savings of up to $72,000 in one study.

439
Q

A 62-year-old male with hypertension and metabolic syndrome sees you for follow-up. A fasting triglyceride level is 300 mg/dL. You address lifestyle and other potential causes of his elevated triglycerides, including his current medications.
If included in his current regimen, which one of the following hypertension medications would be most likely to contribute to his hypertriglyceridemia?
A) Amlodipine (Norvasc)
B) Diltiazem (Cardizem)
C) Lisinopril (Prinivil, Zestril)
D) Metoprolol

A

ANSWER: D
Several medications can be secondary causes of hypertriglyceridemia, including -blockers, with the exception of carvedilol. Others include oral estrogens, glucocorticoids, bile acid sequestrants, protease inhibitors, retinoic acid, anabolic steroids, sirolimus, raloxifene, tamoxifen, and thiazides. Calcium channel blockers, ACE inhibitors, and angiotensin receptor blockers are not associated with hypertriglyceridemia.

440
Q

Croup Tx

A

Mild: dexamethasone

Moderate-severe: nebulized epinephrine

441
Q

A sexually active 45-year-old female who has been using oral contraceptives for years without experiencing any problems asks how much longer she should continue contraception. She is happy with her current low-dose estrogen combined oral contraceptive and would like to continue this if possible. She asks if the health risks are high enough to warrant a change to another option. She does not smoke and is in excellent health.
Which one of the following would be most appropriate for this patient?
A) Continue her current oral contraceptive
B) Discontinue contraception
C) Obtain FSH and LH levels today and discontinue contraception only if the results
confirm infertility
D) Replace the oral contraceptive with a barrier method

A

ANSWER: A
The defined age at which a woman loses natural fertility is not known. The median age for menopause in the United States is approximately 51 years of age, but it can normally occur anytime between 40 and 60 years of age. The American College of Obstetricians and Gynecologists and the North American Menopause Society both currently recommend that women continue contraceptive use until menopause or age 50–55 years. For women on hormonal contraception no current laboratory test can confirm the menopausal state. Natural pregnancy is uncommon for women over 44 years of age, but the risks associated with pregnancy beyond that age may exceed the risks associated with use of combined oral contraceptives (COCs) in women who do not have certain chronic conditions. For this group, increased risks for developing breast cancer in women over 40 years of age and stroke for women over 45 years of age who continue to use COCs has been shown to be nonsignificant in recent studies (level of evidence 2). It is not clear that the increased risk for myocardial infarction or thromboembolism associated with the use of COCs is any higher above baseline for women over the age of 45 years than for younger women.

442
Q

A 42-year-old female presents with a several-month history of fatigue, arthralgias in her knees and hips, myalgias, hair loss, and a recent episode of gross hematuria diagnosed at an urgent care center as a urinary tract infection. She has no urinary tract symptoms at this time. A friend of hers who had similar symptoms for months was recently diagnosed with systemic lupus erythematosus (SLE), and the patient asks whether she might have SLE.
Which one of the following would be most helpful in reassuring her that the likelihood of her having SLE is low?
A) Absence of the typical malar rash
B) Absence of RBC casts on microscopic urinalysis
C) A negative serum antinuclear antibody
D) Normal levels of complement C3, C4, and CH50
E) Joint pain limited to large joints

A

C) A negative serum antinuclear antibody

The diagnosis of systemic lupus erythematosus (SLE) can be difficult and is often not established for months or even years, due to the significant overlap of symptoms with many other conditions. The American College of Rheumatology has established 11 diagnostic criteria, at least 4 of which must be met over time, to establish a diagnosis of SLE. The vast majority (>95%) of patients with SLE have a positive antinuclear antibody (ANA) test, thus it is sensitive as an initial test in a patient for whom there is clinical suspicion for SLE. However, testing for other immunologic subgroup ANA markers should be performed in a patient with a positive ANA. If one or more of those are positive, then the likelihood of SLE is higher. The majority of patients with a positive ANA do not have SLE but a negative ANA is very unlikely in a patient who has SLE.
The typical malar rash of SLE is one of the 11 clinical criteria but is only present in approximately 30% of patients with SLE. Up to 80% of patients may have some form of cutaneous involvement over the course of the disease but hair loss is not specifically a feature of SLE. Other potentially helpful but nonspecific findings in SLE include proteinuria and RBC cellular casts, both of which are indicators of nephritis, but their absence does not rule it out. The subgroup markers (anti-dsDNA, anti-SmDNA, complement C3, C4, CH50) should only be obtained in patients suspected of having SLE who have a positive ANA. Myalgias or arthralgias and synovitis in two or more joints (not limited to large or small joints) is another one of the clinical diagnostic criteria.

443
Q

A 44-year-old female sees you for evaluation of frequent, disabling headaches. She describes the headaches as throbbing, usually left-sided, and associated with nausea. Her headaches worsen with sound and movement, and they last for 48 hours nearly every week. She has not had any aura, weakness, numbness, or speech disturbance.
Based on current evidence, which one of the following medications would have the greatest potential for reducing the number of headache days for this patient?
A) Fluoxetine (Prozac)
B) Gabapentin (Neurontin)
C) Lisinopril (Prinivil, Zestril)
D) Propranolol
E) Verapamil (Calan)

A

ANSWER: D
According to the International Classification of Headache Disorders, this patient meets criteria for the diagnosis of migraine without aura. Many medications have been studied for the prevention of migraine. Divalproex, topiramate, metoprolol, propranolol, and timolol have been shown to be effective for migraine prevention by consistent, good-quality evidence. One of these medications should be offered as first-line treatment. Studies of fluoxetine have demonstrated inconsistent results. Gabapentin has been evaluated in six randomized, controlled trials with mixed results. Three studies have been conducted with ACE inhibitors and angiotensin receptor blockers for migraine prevention, with just one study showing some benefit. Verapamil has previously been considered effective, but on reevaluation of previous studies, the supporting data for verapamil is insufficient to prove efficacy.

444
Q

A 65-year-old female is diagnosed with osteoporosis based on a screening DXA scan. After a shared decision-making discussion you decide to initiate treatment with alendronate (Fosamax).
Which one of the following would you recommend for the duration of pharmacologic treatment?
A) 1 year
B) 3 years
C) 5 years
D) 10 years
E) Lifelong treatment

A

ANSWER: C
Alendronate decreases the risk of osteoporosis-related hip and vertebral fractures in postmenopausal women. The risk for atypical subtrochanteric fractures increases significantly with duration of treatment. The American College of Physicians evidence-based guideline recommends a maximum treatment duration of 5 years with alendronate. Continuation of treatment beyond 5 years should be reassessed at that point and determined based upon an individualized discussion of risks and benefits.

445
Q

A 59-year-old male presents with difficulty breathing during exercise. He says that his symptoms have gradually worsened over the past year and he has had to discontinue his morning walks. He reports mild lower extremity edema and weight gain. He has a blood pressure of 115/79 mm Hg, a heart rate of 88 beats/min, and an oxygen saturation of 92% on room air. A physical examination is notable for mild jugular venous distention and 1+ bilateral lower extremity edema. Examination of the heart reveals a normal rate and rhythm with an S3 heart sound. The lungs are clear to auscultation. You order a CBC, a comprehensive metabolic panel, an EKG, a chest radiograph, and echocardiography.
While awaiting the results you consider the differential diagnosis. Which one of the following conditions is the most common cause of pulmonary hypertension?
A) Chronic thromboembolism
B) COPD
C) Idiopathic pulmonary arterial hypertension
D) Left heart disease
E) Sleep-disordered breathing

A

ANSWER: D
This patient has signs and symptoms of pulmonary hypertension. Diagnostic tests, particularly echocardiography, can confirm this diagnosis. It is important to determine the etiology since addressing the underlying condition is the preferred treatment for most cases of non-severe pulmonary hypertension. Left heart disease, including both preserved and reduced systolic function, is the most common cause of pulmonary hypertension, while chronic thromboembolism, COPD, and sleep-disordered breathing are other possible but less common causes. Idiopathic pulmonary arterial hypertension is a rare cause.

446
Q

You are covering a weekend shift in the local intensive-care unit. When providing care for a very ill adult patient with hypoproliferative thrombocytopenia who is not currently bleeding, prophylactic platelet transfusion should be considered if the platelet count is below a threshold of
A) 10,000/ L B) 20,000/ L C) 25,000/ L D) 50,000/ L E) 100,000/ L

A

ANSWER: A
The threshold for transfusing platelets to prevent spontaneous bleeding in the setting of hypoproliferative thrombocytopeniainmostadultsis<10,000/ L(SORA).Aplateletcount<20,000/ Listhethreshold for use of elective central venous catheter placement. For elective diagnostic lumbar puncture, major elective non-neuraxial surgery, and interventional procedures, the threshold is a platelet count <50,000/ L. For neuraxial surgery a threshold <100,000/ L is recommended.

447
Q

A 15-year-old female is brought to your office for a routine wellness check. Her only concerns are that she has never menstruated and she is not growing as fast as her peers. She is very active and plays volleyball on a travel team. An examination reveals that her height is now at the 25th percentile, although it was at the 90th percentile when she was 8 years old. She has breast buds that do not extend beyond the areola and her pubic hair is fine and sparse. Laboratory findings include a negative pregnancy test and a normal CBC, metabolic panel, TSH level, and prolactin level. She has an estradiol level of 12 pg/mL (N 25–75), an LH level of 40 mIU/mL (N 5–20), and an FSH level of 50 mIU/mL (N 3–20).
The most likely diagnosis is
A) congenital adrenal hyperplasia
B) constitutional delay of puberty
C) pituitary adenoma
D) polycystic ovary syndrome
E) primary ovarian insufficiency

A

ANSWER: E
Primary amenorrhea is the lifelong absence of menses. If menarche has not occurred by age 15, or no menses have occurred 3 years after the development of breast buds, an evaluation is recommended. The patient’s history should include a review of eating and exercise habits, sexual activity, changes in body weight, perfectionistic tendencies, substance abuse, chronic illness, and timing of breast and pubic hair development. A family history of late growth spurts or late menses may indicate constitutional delay, which manifests as short stature that continues on the same percentile until puberty, when there is a delayed growth spurt to achieve normal height.
A physical examination should note trends in height, weight, and BMI. An evaluation should be performed to look for signs of virilization, which would indicate androgen excess found in congenital adrenal hyperplasia, polycystic ovary syndrome, Cushing syndrome, or adrenal tumors.
Laboratory testing is usually initiated with a pregnancy test and prolactin, LH, FSH, and TSH levels. Primary ovarian insufficiency is associated with low estradiol levels and high levels of LH and FSH. Generally, the LH/FSH ratio is <1. Patients with congenital adrenal hyperplasia will have low estrogen, LH, and FSH levels. Virilization is generally noted in congenital adrenal hyperplasia, and a 17-hydroxyprogesterone level should be obtained to assess for this condition. Functional hypothalamic amenorrhea will also cause low levels of LH, FSH, and TSH. While polycystic ovary syndrome is associated with low estrogen, LH, and FSH levels, prolactin may be elevated. A pituitary adenoma will cause the prolactin level to be elevated.

448
Q

A 42-year-old male presents with a 3-month history of epigastric pain, bloating, and occasional vomiting after eating. He has not had any weight loss, blood in the stools, or difficulty swallowing. He does not report any significant acid reflux symptoms.
Which one of the following would be the best management strategy for this patient?
A) Implement lifestyle changes and follow up in 3 months
B) Test for Helicobacter pylori and treat if positive
C) Initiate a 2-month trial of proton pump inhibitor therapy
D) Order esophagogastroduodenoscopy
E) Order a barium swallow

A

ANSWER: B
This patient presents with dyspepsia but does not have any alarm symptoms such as weight loss, blood in the stools, or difficulty swallowing. An important cause of dyspepsia is gastric infection with Helicobacter pylori. In patients younger than 55 years of age with no alarm symptoms, a test-and-treat strategy is effective and safe, with esophagogastroduodenoscopy reserved for patients not meeting these criteria (SOR A). Lifestyle interventions and proton pump inhibitor therapy are more effective for GERD. A barium swallow would not be appropriate for this patient at this time.

449
Q

A 15-year-old basketball player presents with a 3-week history of bilateral knee pain that is greater in the left knee. The pain increases with jumping, walking down stairs, and kneeling. He has not had any recent injury. He tells you that he has grown more than 5 inches in the past year. A physical examination is notable for tight quadriceps and hamstrings, and tenderness to palpation over the tibial tuberosity bilaterally.
Which one of the following is the most likely diagnosis?
A) Anterior cruciate ligament tear
B) Medial meniscus tear
C) Osgood-Schlatter disease
D) Patellar sleeve fracture
E) Patellofemoral pain syndrome

A

ANSWER: C
Apophysitis is a traction injury at the bony site of the tendon attachment. It most often occurs in children or adolescents who are rapidly growing. Rapid growth results in bone lengthening, which occurs more rapidly than lengthening of the associated muscle and tendons. Osgood-Schlatter disease is one type of apophysitis affecting the patellar tendon attachment at the proximal tibia. Anterior cruciate ligament tears will present with joint laxity and meniscal tears with joint line tenderness. Anterior cruciate ligament and medial meniscus tears are usually associated with trauma, especially in younger patients. A patellar sleeve fracture results from a similar type of apophysitis (Larsen-Johansson disease), which affects the patellar tendon attachment at the lower pole of the patella. It most often occurs in athletic children between the ages of 10 and 12. Patellofemoral pain syndrome may present with a similar history, but the pain is generally felt under the patella and localized tenderness over the tibial tuberosity is not present.

450
Q

A 36-year-old female sees you for a routine health maintenance visit. She reports worsening hair growth on her chin and abdomen over the last few years. The excessive hair growth first appeared in her late teens and she has been dissatisfied with the cosmetic results of various hair removal methods. She is generally healthy aside from a BMI of 31 kg/m2. She does not take any medications, is a nonsmoker, and has had a bilateral tubal ligation. Her menses are regular.
A complete physical examination is consistent with some terminal hairs in the distribution she described and is otherwise unremarkable. Laboratory results are normal, including fasting lipids, a comprehensive metabolic panel, a hemoglobin A1c, and a TSH level.
Which one of the following would be the recommended first-line treatment for this patient’s condition?
A) Drospirenone/ethinyl estradiol (Yasmin)
B) Flutamide
C) Leuprolide (Eligard)
D) Metformin (Glucophage)
E) Spironolactone (Aldactone)

A

ANSWER: A
Hirsutism affects 5%–15% of women and can adversely affect quality of life. It is caused by increased androgen production. Most cases are caused by benign conditions. Polycystic ovary syndrome (PCOS) accounts for 70% of cases with another 25% attributable to idiopathic hyperandrogenism and idiopathic hirsutism. First-line therapy for hirsutism in women who do not desire pregnancy and for whom cosmetic treatments are not effective is combined oral contraceptives (SOR B), which decrease androgen production in the ovaries by decreasing LH levels.
Flutamide is an antiandrogen treatment that has been found to be effective but should be avoided due to potential hepatotoxicity. Leuprolide can be used in patients who do not respond to combined oral contraceptives and antiandrogen treatments. However, there are serious side effects with its use, including bone loss and hypoestrogenism. Metformin is not effective for the treatment of hirsutism (SOR B). While this patient likely has PCOS, the anti-insulin medications will not affect excess hair growth. Antiandrogen treatments such as spironolactone and finasteride are second-line therapies that can be added to the combined oral contraceptives if there is no improvement after the first 6 months (SOR A).

451
Q

Primary prevention

A

targets individuals who may be at risk to devleop a medical condition and intervenes to prevent onset of condition
- Vaccines
- antismoking
- education safe sex

452
Q

Secondary prevention

A

targets individuals who have developed an asymptomatic disease and institutes treatmetn to prevent complications

Pap
Screening HTN
Screening DM and Hyperlipidemia

453
Q

Tertiary prevention

A

targets individuals with a known disease with a goal of limiting or preventing future complications

screening DM patients for microalbumineuria

454
Q

A 25-year-old gravida 1 para 1 presents for insertion of a levonorgestrel-releasing intrauterine
device (Mirena). She is on the last day of her menses, which began 5 days ago. A urine
pregnancy test in the office is negative. You insert the device without complications and she asks
how long she needs to use backup contraception.

Which one of the following would be the most appropriate advice? (check one)
Backup contraception is not necessary
She should use backup contraception for the next 48 hours
She should use backup contraception for the next 7 days
She should use backup contraception for the next 14 days
She should use backup contraception for the next month

A

Back up is not needed

The Centers for Disease Control and Prevention (CDC) provides specific recommendations for backup
contraception after IUD insertion. According to the CDC guidelines, this patient does not need to use
backup contraception if her IUD is inserted today because it was inserted within 7 days after menstrual
bleeding started. If the levonorgestrel IUD is inserted more than 7 days after menstrual bleeding starts,
the patient needs to abstain from sexual intercourse or use additional contraceptive protection for the next
7 days.

455
Q

A 72-year-old female taking hydrochlorothiazide for hypertension develops trigeminal neuralgia and you start her on carbamazepine (Tegretol). She is at risk for which one of the following metabolic consequences? (check one)
Calcium pyrophosphate deposition
Hypercalcemia
Hyponatremia
Hyperuricemia

A

Hyponatremia

Elderly patients, especially those taking hydrochlorothiazide, are at risk for developing hyponatremia while taking carbamazepine. Carbamazepine is one of the medications that can cause the syndrome of inappropriate antidiuretic hormone secretion, as it interferes with the ability to dilute the urine. It does not lead to the other derangements listed (SOR A).

456
Q

A 27-year-old male has been treating his plaque psoriasis with high-potency topical corticosteroids for several years. He comes to your office to discuss other options since the lesions on his trunk and extremities are becoming resistant to this therapy.

Which one of the following treatment strategies would be most appropriate? (check one)
Switch to topical tazarotene (Avage, Tazorac)
Add topical calcipotriene (Dovonex, Sorilux)
Begin oral acitretin (Soriatane)
Begin an oral corticosteroid
Begin etanercept (Enbrel) injections

A

Oral corticosteroids are not indicated in the treatment of plaque psoriasis. All of the other options are indicated only if topical treatments fail. Of the options listed, the combination of a topical corticosteroid and topical calcipotriene is considered the most appropriate for this patient. Another option would be to add topical tazarotene to the topical corticosteroid. However, when tazarotene is used as monotherapy it often fails to clear plaques and increases the incidence of skin irritation.

457
Q

A patient is admitted to the hospital with severe acute pancreatitis, based on diagnostic criteria for severity. After appropriate intravenous hydration, which one of the following is associated with shorter hospital stays and lower mortality? (check one)
Parenteral nutrition
Nothing by mouth until the pain has resolved
Clear liquids by mouth after 48 hours
Bolus nasogastric enteral nutrition
Continuous nasogastric enteral nutrition

A

The American College of Gastroenterology recommends that patients with severe acute pancreatitis receive enteral nutrition. Enteral feedings help prevent infectious complications, such as infected necrosis, by maintaining the gut mucosal barrier and preventing translocation of bacteria that may seed pancreatic necrosis. Currently, continuous enteral feeding is preferred over bolus feeding. A meta-analysis has shown that continuous nasogastric enteral feeding started in the first 48 hours decreases mortality and the length of hospital stay.

Total parenteral nutrition is not recommended because of infectious and line-related complications. It should be avoided unless the enteral route cannot be used.

458
Q

Pneumonia vaccine recommendations

A

All patients 65 and older should be given either PCV20 once or PCV 15 followed by PPSV23 at least one year later

If immunocompromised can do 8 week later

459
Q

HTn plus low potassium

A

Primary hyepraldosteronism

Check aldosterone/renin ratio

460
Q

Rash associated with coccidioidomycosis

A

Erythema nodosum

461
Q

Erythema ab igne

A

cutaneous rash caused by prolonged heat exposure

462
Q

Erythema infectiosum

A

associated with parvovirus B19

Slapped cheek appearance

463
Q

Erythema migrans

A

expanding, erythematous, annular rash with or without central clearing and is often associated with tick exposure (Lyme disease).

464
Q

Erythema multiforme

A

Erythema multiforme consists of raised, annular, target-like lesions with central erythema and is usually associated with herpes simplex virus type 1.

465
Q

A 50-year-old male presents with chronic abdominal pain. A workup leads you to suspect peptic ulcer disease, and you refer him for endoscopy, which shows a small duodenal ulcer. The endoscopist also notes some small esophageal varices without red wale signs.
Further evaluation confirms that the patient has compensated cirrhosis in the setting of alcohol use disorder. He readily accepts this diagnosis and enters an Alcoholics Anonymous program. His ulcer symptoms resolve with antibiotic therapy for Helicobacter pylori. He says he has abstained from alcohol for 6 weeks, and he would like to further reduce his risks from cirrhosis.
The most appropriate next step in the management of his esophageal varices would be
A) octreotide (Sandostatin)
B) omeprazole (Prilosec)
C) propranolol
D) endoscopic variceal ligation
E) repeat endoscopy in 1–2 years

A

ANSWER: E
Primary prevention of variceal hemorrhage is an important consideration in the management of patients with cirrhosis. Although this patient’s varices were diagnosed incidentally, patients with cirrhosis and clinically significant portal hypertension should be screened for varices every 2–3 years with esophagogastroduodenoscopy (EGD). EGD can be deferred in patients with platelet counts <150,000/mm3 and transient elastography with liver stiffness <20 kPa. Once esophageal varices are identified, the criteria for initiating prophylaxis to prevent variceal hemorrhage is based on the risk of bleeding. Findings associated with a high risk of bleeding include small varices in patients with decompensated cirrhosis, small varices with red wale signs (thinning of the variceal wall), and medium to large varices. Patients with small varices not meeting these criteria have a low risk of hemorrhage and do not require prophylaxis. They should be rescreened with EGD every 1–2 years.

For patients requiring treatment due to high-risk features, options for primary prophylaxis of hemorrhage include nonselective -blockers such as propranolol or endoscopic variceal ligation. Treatment decisions are based on patient preference, other potential contraindications, and local resources. The need for repeat endoscopy in these cases will depend on the clinical circumstances. If nonselective -blockers are used, they should be continued indefinitely. Octreotide is only given intravenously for acute hemorrhage. There is no evidence that omeprazole slows the progression of esophageal varices.

Wale sign: thinning of the variceal wall

466
Q

An otherwise healthy 29-year-old gravida 2 para 1 at 28 weeks gestation presents to your office with a laceration sustained while doing yard work. After thoroughly cleaning the wound, you decide not to suture it because of the risk of infection. The patient received Tdap during her previous pregnancy 6 years ago and you confirm in her medical records that she completed her primary immunizations as a child.
Which one of the following would be most appropriate regarding tetanus prophylaxis?
A) No tetanus prophylaxis
B) Tetanus immune globulin now
C) Td now
D) Tdap now
E) Tdap at 38 weeks gestation

A

ANSWER: D
This patient needs a tetanus toxoid–containing vaccine for the management of her wound. Since pregnant people should receive a dose of Tdap between 27 and 36 weeks gestation to protect against pertussis, Tdap is the best choice for this patient. Tetanus immune globulin would be appropriate if this patient had not previously completed the primary series or were showing signs of clinical tetanus. Td would be an appropriate option for tetanus prophylaxis in nonpregnant patients who have previously received Tdap. Because this patient requires some form of tetanus prophylaxis at this time, waiting until 38 weeks to administer Tdap is not appropriate.

466
Q

Which one of the following cardiovascular medications may lead to hyperthyroidism?
A) Amiodarone
B) Digoxin
C) Flecainide
D) Metoprolol
E) Valsartan (Diovan)

A

ANSWER: A
Amiodarone-induced thyrotoxicosis (AIT) is a less common cause of hyperthyroidism and can be particularly difficult to accurately diagnose and treat. AIT type 1 is a form of iodine-induced thyrotoxicosis caused by the high iodine content in amiodarone. AIT type 2 is a form of amiodarone-induced thyroiditis. Digoxin, flecainide, metoprolol, and valsartan do not cause hyperthyroidism.

467
Q

Which one of the following statements regarding hormone therapy for transgender patients is true?
A) Hormone therapy to facilitate development of secondary sex characteristics is generally reversible
B) Patients who receive hormone therapy generally report improved quality of life, higher self-esteem, and decreased anxiety
C) Masculinizing hormone therapy is associated with reduced muscle mass and fat redistribution
D) Patients receiving feminizing hormone therapy are at increased risk for erythrocytosis

A

ANSWER: B
Hormone therapy is not required for all transgender patients, but those who receive treatment generally report improved quality of life, higher self-esteem, and decreased anxiety. Feminizing and masculinizing hormone therapies, including the use of estrogen and/or androgen therapies such as testosterone, are partially irreversible. Thus, it is important to make a reasonable, educated decision and use informed consent prior to treatment. Patients who receive masculinizing therapy are at increased risk for erythrocytosis and those who receive feminizing hormone therapy often experience reduced muscle mass and fat redistribution.

468
Q

A 37-year-old female presents to your clinic with a long-standing history of abnormal menstrual cycles, often occurring irregularly more than 40 days apart. She has ongoing struggles with weight gain, acne, and facial hair growth. She states that she is not currently sexually active. Her last Papanicolaou smear 2 years ago was normal. Her vital signs and a physical examination are unremarkable other than a BMI of 36 kg/m2. An office urine pregnancy test is negative. Laboratory evaluation reveals a hemoglobin A1c of 6.2%, and normal TSH, prolactin, and 17-hydroxyprogesterone levels.
Which one of the following is required to confirm the most likely diagnosis?
A) No additional evaluation
B) A serum C-peptide test
C) A dexamethasone suppression test
D) Ultrasonography of the pelvis
E) CT of the abdomen and pelvis

A

ANSWER: A
This patient has signs and symptoms consistent with polycystic ovary syndrome (PCOS). The Rotterdam 2003 criteria are the most widely used diagnostic criteria for PCOS, endorsed by multiple national and international professional societies. These criteria require the presence of two out of the following three features: oligomenorrhea, hyperandrogenism, and the presence of polycystic ovaries on ultrasonography. When the first two of these criteria are clearly met, ultrasonography to establish the presence of polycystic ovaries is not required. Therefore, a diagnosis is already warranted for this patient and additional evaluation is not needed. When patients require imaging, pelvic ultrasonography is the preferred modality rather than CT. While this patient has evidence of insulin resistance, as is common for patients with PCOS, a C-peptide test is not indicated. Dexamethasone suppression testing is not indicated because this patient does not have any other clinical signs and symptoms that would be consistent with Cushing syndrome.

469
Q

Once hemolysis is excluded, the most common cause of unconjugated hyperbilirubinemia is
A) alcoholic liver disease
B) biliary tract disease
C) fatty liver disease
D) Gilbert syndrome
E) Wilson disease

A

ANSWER: D
Unconjugated hyperbilirubinemia can be defined as an elevated indirect bilirubin level. While unconjugated hyperbilirubinemia is most commonly seen in hemolysis, another common cause is Gilbert syndrome, which stems from a genetic defect that affects how the liver processes bilirubin. Alcoholic liver disease, biliary tract disease, fatty liver disease, and Wilson disease do not lead to unconjugated hyperbilirubinemia.

470
Q

A 35-year-old female presents to your office with a feeling of vague fullness in her neck for the last month. She has noticed a gradual onset of fatigue, constipation, and cold intolerance over that time. A few weeks ago the patient took a selfie and was surprised by how puffy her face appeared in the photo.
On examination her thyroid is diffusely enlarged and nontender and feels pebbly on palpation. An HEENT examination, including an eye examination, is otherwise normal.
Which one of the following is the most likely diagnosis?
A) Chronic autoimmune (Hashimoto) thyroiditis
B) Graves disease
C) Lymphadenitis
D) Lymphoma
E) Thyroid cancer

A

ANSWER: A
This patient’s clinical picture is most consistent with chronic autoimmune thyroiditis, traditionally known as Hashimoto thyroiditis. This diagnosis is suggested by her neck fullness and symptoms of hypothyroidism. Additionally, a nontender goiter that feels like pebbles on examination is classically reported with chronic autoimmune thyroiditis.
Graves disease typically presents with symptoms of hyperthyroidism and, in many patients, orbitopathy (eye bulging). A patient with lymphadenitis typically shows symptoms of a causative infection. Lymphadenitis tends to rapidly enlarge the lymph nodes, which are also typically painful and tender. Lymphoma more commonly presents with fevers, night sweats, unintentional weight loss, itchy skin, and dyspnea.

This patient lacks a discrete thyroid nodule, which makes thyroid cancer less likely. Thyroid nodules are more frequently painful, while the neck fullness in chronic autoimmune thyroiditis is usually painless and nontender.

471
Q

A 12-year-old female presents with a sore throat and tonsillar exudate, and a rapid antigen test is positive for streptococcal pharyngitis. She returns to your office after completing a 10-day course of penicillin this morning. She says that although she saw some initial improvement, she now has a sore throat again, accompanied by a runny nose and cough. Her mother asks if another antibiotic would be appropriate. A physical examination reveals nonexudative pharyngitis, but a rapid antigen test for group A Streptococcus is again positive.
Which one of the following would be the most appropriate treatment at this point?
A) No further antibiotic therapy
B) Oral azithromycin (Zithromax) for 5 days
C) Oral ciprofloxacin (Cipro) for 10 days
D) A single dose of intramuscular benzathine penicillin
E) A single dose of intramuscular ceftriaxone

A

ANSWER: A
Most bacteriologic treatment failures for group A Streptococcus (GAS) represent a GAS carrier state. This patient had clinical improvement followed by a second illness with typical features of a viral infection. Oral azithromycin, oral ciprofloxacin, intramuscular benzathine penicillin, and intramuscular ceftriaxone are not appropriate for the treatment of viral infections in a patient who is a pharyngeal GAS carrier.

472
Q

The American Academy of Pediatrics recommends obtaining a blood pressure reading at well child checks beginning at what age?
A) 18 months
B) 2 years
C) 3 years
D) 4 years
E) 5 years

A

ANSWER: C
The American Academy of Pediatrics recommends annual blood pressure screening at well child checks beginning at 3 years of age. This recommendation does not differ for children who have a strong family history of hypertension.

473
Q

A 45-year-old male presents for follow-up of migraine headaches, which had previously responded well to occasional as-needed use of ibuprofen but have recently worsened in severity and frequency. He is interested in trying an abortive therapy.
Triptan use would be CONTRAINDICATED with a history of which one of the following in this patient?
A) Coronary artery stent placement
B) Depression with psychotic features
C) Diabetes mellitus with a hemoglobin A1c 7.5%
D) Hypertension requiring two medications to achieve control
E) Stage 4 chronic kidney disease

A

ANSWER: A
Because of their vasoconstricting properties, triptan medications are contraindicated in patients with established coronary artery disease, cerebrovascular disease, or peripheral vascular disease; patients with uncontrolled or multiple cardiovascular risk factors; and patients with certain high-risk migraine syndromes, including basilar and hemiplegic migraines. Triptan use for the treatment of migraine headaches would not be contraindicated with a history of depression with psychotic features, poorly controlled diabetes mellitus, hypertension requiring two medications, or stage 4 chronic kidney disease.

474
Q

A 27-year-old male presents to establish care after relocating to the community. He was diagnosed with β-thalassemia major at birth and has been transfusion-dependent since early childhood. His microcytic anemia is stable with blood transfusions every 4 weeks, but his most recent DEXA scan indicates an advancement from osteopenia to osteoporosis.
In addition to bisphosphonates, calcium, and vitamin D, which one of the following medications may improve his bone density?
A) Hydroxyurea (Hydrea)
B) Vitamin C
C) Zinc
D) Deferoxamine (Desferal)
E) Luspatercept (Reblozyl)

A

ANSWER: C
In addition to bisphosphonates, calcium, and vitamin D, zinc supplementation is recommended to improve bone density in patients with thalassemia and osteoporosis (SOR C). Though hydroxyurea is an indicated therapy to minimize the frequency of blood transfusions needed in transfusion-dependent thalassemia, it does not improve bone density (SOR C). Vitamin C supplementation does not improve bone health in patients with thalassemia and osteoporosis. Deferoxamine infusions are indicated when ferritin levels are >1000 ng/mL in patients with transfusion-dependent thalassemia to reduce iron overload (SOR C). Luspatercept reduced transfusion burden by 33% in a phase 3, randomized study but is not indicated to improve bone density.

475
Q

While reviewing laboratory studies for a patient who was recently started on antihypertensive medication, you note new hyperkalemia. Which one of the following medications is most likely to cause this finding?
A) Amlodipine (Norvasc)
B) Chlorthalidone
C) Hydrochlorothiazide
D) Metoprolol
E) Olmesartan (Benicar)

A

ANSWER: E
Hyperkalemia is a known side effect of ACE inhibitors and angiotensin receptor blockers such as olmesartan. The risk of hyperkalemia is increased with chronic kidney disease, diabetes mellitus, moderately severe to severe heart failure, NSAID use, and older adults. Chlorthalidone and hydrochlorothiazide can cause hypokalemia, while amlodipine and metoprolol have no significant effect on potassium levels.

476
Q

HTN medication and hyperkalemia

A

ACE inhibitors/ ARBS

477
Q

HTN meds and hypokalemia

A

Hhydrchlorothiazide and chlorthalidone

478
Q

An obese 40-year-old female with diabetes mellitus sees you for evaluation of painful, deep-seated nodules in both axillae. On examination you note nodules in the axillae with purulent drainage and associated scarring.
This condition is associated with which one of the following?
A) Amyotrophic lateral sclerosis
B) Crohn’s disease
C) Dermatitis herpetiformis
D) Systemic lupus erythematosus
E) Trauma

A

ANSWER: B
The patient has hidradenitis suppurativa, a chronic folliculitis affecting intertriginous areas in the axillae and the groin that may also occur around the anus and nipples. Treatment depends on severity and ranges from topical to systemic antibiotics. Hidradenitis suppurativa is associated with obesity, diabetes mellitus, Crohn’s disease, arthritis and spondyloarthropathy, metabolic syndrome, polycystic ovary syndrome, pyoderma gangrenosum, and trisomy 21. There are three stages: stage I is single or multiple abscesses without sinus tracts or scarring, stage II is abscess recurrence with sinus tracts and scarring and widely separated lesions, and stage III is diffuse abscesses with interconnecting sinus tracts. Amyotrophic lateral sclerosis has no typical skin manifestation. Dermatitis herpetiformis is associated with celiac disease and has clusters of pruritic lesions. Systemic lupus erythematosus has cutaneous manifestations of a malar rash and may involve subcutaneous lesions without scarring. Hidradenitis suppurativa is not associated with trauma.

479
Q

Hidradenitis suppurative associated iwth

A

Crohns disease

480
Q

A 43-year-old female with lifelong asthma asks if she would be a candidate for treatment with a biologic agent such as omalizumab (Xolair). A CBC reveals mild eosinophilia, indicating type 2 inflammatory asthma.
In which one of the following patient scenarios should biologic treatment for asthma be considered?
A) Any patient with poorly controlled, severe asthma
B) A patient with severe non–type 2 asthma that is poorly controlled despite adherence to
optimal therapy with long-term controller medication
C) A patient with type 2 inflammatory asthma that is poorly controlled despite therapy with
as-needed inhaled albuterol (Proventil, Ventolin) and low-dose inhaled corticosteroids
D) A patient with severe type 2 inflammatory asthma that is poorly controlled despite
adherence to optimal therapy with long-term controller medication

A

ANSWER: D
Biologic therapy for asthma targets type 2 inflammation pathways. According to the 2019 Global Initiative for Asthma (GINA) guidelines, diagnosis and management of severe asthma includes determination of the asthma phenotype to assess for type 2 inflammation. Type 2 asthma includes allergic and eosinophilic asthma. Non–type 2 asthma is driven by neutrophils and is associated with smoking and obesity. Type 2 inflammation is diagnosed by elevated eosinophils in the blood or sputum, elevated fractional exhaled nitric oxide, or a need for oral corticosteroid maintenance therapy. Biologic therapy may be considered in patients with severe type 2 inflammatory asthma who continue to have significant symptoms despite adherence to optimal therapy, including high-dose inhaled corticosteroids and a long-acting -agonist.

481
Q

A 70-year-old female presents to your office to discuss osteoporosis that was noted on a recent bone density test. Initial laboratory studies reveal an abnormal TSH level of 0.27 μU/mL (N 0.36–3.74). Additional studies reveal the following:
RepeatTSH 0.04μU/mL
FreeT3  3.4pg/mL(N1.7–5.2) FreeT4  1.4ng/dL(N0.7–1.6)
A radioactive iodine uptake scan is notable for multiple areas of increased and suppressed uptake.
Which one of the following is the most likely explanation for these findings?
A) Exogenous thyroid hormone use
B) Graves disease
C) Painless thyroiditis
D) Recent excess iodine intake
E) Toxic multinodular goiter

A

ANSWER: E
This patient has a low serum TSH level in the presence of normal free T4 and total or free T3 levels, which is consistent with subclinical hyperthyroidism. The etiology of overt and subclinical hyperthyroidism should be determined by clinical symptoms, biochemical markers, and, if indicated, diagnostic studies such as a radioactive iodine uptake (RAIU) scan. A scan that shows multiple areas of increased and suppressed uptake is consistent with toxic multinodular goiter. There is no RAIU with exogenous ingestion of thyroid hormone, painless thyroiditis, and recent excess iodine intake. Graves disease causes diffuse RAIU.
It is important to determine the etiology of subclinical hyperthyroidism in order to treat it appropriately. In patients who have TSH levels that are persistently <0.1 U/mL, the American Thyroid Association has a strong recommendation with moderate-quality evidence for treating patients 65 years of age and older; persons with cardiac risk factors, heart disease, or osteoporosis; postmenopausal women not on estrogens or bisphosphonates; and those with hyperthyroid symptoms.

482
Q

An ill-appearing 50-year-old male presents with malaise, nausea, anorexia, and lethargy. He has a recent diagnosis of a high-grade lymphoma and is undergoing aggressive chemotherapy. His last chemotherapy session was 2 days ago. An examination is nonspecific. Initial laboratory studies reveal a creatinine level of 2.1 mg/dL (N 0.6–1.2). His baseline creatinine level is 1.0 mg/dL.
Which one of the following laboratory findings would be expected in this patient?
A) Hypercalcemia
B) Hyperuricemia
C) Hypokalemia
D) Hypophosphatemia
E) Low LDH

A

ANSWER: B
Tumor lysis syndrome is considered the most common oncologic emergency. It is caused by the rapid release of intracellular material from lysis of the malignant cells. The breakdown of nucleic acids releases large amounts of uric acid and leads to acute kidney failure, which limits clearance of potassium, phosphorus, and uric acid. This leads to hyperuricemia, secondary hypocalcemia, hyperkalemia, and hyperphosphatemia. It can result in acute renal failure, arrhythmia, seizure, and sudden death. While tumor lysis syndrome has been reported with many cancer types, it is more common with acute leukemia and high-grade lymphomas. Patients with this condition generally present within 7 days of cancer treatment, including chemotherapy, radiation, or biologic therapies. It can also occur spontaneously. An LDH elevation related to a high cell turnover rate prior to cancer treatment may indicate an increased risk of tumor lysis syndrome. Hypercalcemia, hypokalemia, hypophosphatemia, and low LDH would not be expected laboratory findings in patients with tumor lysis syndrome.

483
Q

Tumor lysis syndrome

A

Limits clearance of potassium, phosphorus and uric acid

Hyperuricemia
Secondary hypocalcemia
Hyperkalemia
Hyperphosphatemia

484
Q

Cardiac stress testing would be most appropriate for which one of the following patients?
A) A 57-year-old female who is scheduled for a knee replacement and has dyspnea when walking up a few stairs
B) A 60-year-old male with diabetes mellitus who was admitted to the hospital for chest pain and acute stroke and has a normal EKG and troponin levels
C) A 66-year-old male with diabetes and hypertension without cardiac symptoms who would like to stratify his risk for heart disease
D) A 68-year-old female with coronary artery disease who is scheduled for a knee replacement and does not have cardiac symptoms when walking up a flight of stairs
E) A 79-year-old male who is scheduled for a transcatheter aortic valve replacement for severe aortic stenosis and has dyspnea when walking up a few stairs

A

ANSWER: A
In the setting of acute symptoms, cardiac stress testing is indicated when there is an intermediate probability of acute coronary syndrome. Cardiac stress testing is also indicated in a preoperative assessment when surgery is at least a moderate risk and the patient cannot reach 4 METs of exertion (climbing a single flight of stairs) without cardiac symptoms. Cardiac stress testing is contraindicated after a recent stroke or TIA and in patients with severe symptomatic aortic stenosis. It is not indicated in asymptomatic patients with no history of revascularization.

485
Q

A 45-year-old male sees you for a routine visit. His medical history includes hypertension treated with hydrochlorothiazide, amlodipine (Norvasc), and losartan (Cozaar). He also has type 2 diabetes treated with metformin and empagliflozin (Jardiance). Laboratory findings are significant for an LDL-cholesterol level of 167 mg/dL and you prescribe simvastatin (Zocor), 80 mg daily. At a follow-up visit 3 months later he tells you that he stopped taking the simvastatin after a week due to muscle pain and weakness.
Which one of the following medications in this patient’s current regimen most likely contributed to his risk for developing statin-induced myopathy?
A) Amlodipine
B) Empagliflozin
C) Hydrochlorothiazide
D) Losartan E) Metformin

A

ANSWER: A
Most statins are metabolized in the liver by cytochrome P450 3A4 (CYP3A4) enzymes. In patients on statin therapy, concurrent use of other medications that are also metabolized by this system, including amiodarone, calcium channel blockers such as amlodipine, certain anti-HIV medications, and certain antifungal medications, can increase the risk of complications such as statin-induced myopathy. In this patient, only simvastatin and amlodipine are metabolized by CYP3A4. Losartan is metabolized by cytochrome P450 enzymes other than 3A4 (2C9), and this patient’s other medications are metabolized by different mechanisms (empagliflozin) or not significantly metabolized (hydrochlorothiazide and metformin).

486
Q

A 77-year-old female presents to your office as a new patient. She recently moved from New York to Florida to live with her daughter, after her partner died 2 years ago. Her past medical history is significant for hypertension, hypothyroidism, osteoarthritis, and cachexia. Her current medications, which were prescribed by her previous primary care physician, include the following:
Acetaminophen Hydrochlorothiazide Levothyroxine (Synthroid) Losartan (Cozaar) Megestrol (Megace)
A multivitamin
Her vital signs are unremarkable except for a 7% weight loss in the past year. A recent TSH level was within the normal range.
Which one of the following medications should be discontinued?
A) Acetaminophen
B) Hydrochlorothiazide C) Levothyroxine
D) Losartan E) Megestrol

A

ANSWER: E
Although it is considered an appetite stimulant, megestrol is not recommended in older adults due to potential harmful side effects and a lack of evidence supporting improved outcomes for the treatment of cachexia in the geriatric population, according to the American Geriatrics Society’s Choosing Wisely recommendation. Acetaminophen is not considered a contributor to weight loss. Though loop diuretics, spironolactone, ACE inhibitors, calcium channel blockers, and propranolol may contribute to weight loss due to their adverse effects, hydrochlorothiazide and angiotensin receptor blockers such as losartan are less likely to contribute to weight loss and the patient’s hypertension is currently controlled on this regimen. The patient’s hypothyroidism is currently stable, so changes to the levothyroxine dosage are unnecessary.

487
Q

A recently divorced 47-year-old male comes to your office appearing disheveled, with the smell of alcohol on his breath. His Patient Health Questionnaire–9 (PHQ-9) score today is 20, and his last PHQ-9 score was 7. He has a history of depression and is currently taking citalopram (Celexa). The patient is tearful during the encounter and admits to thinking the world would be better without him in it. He does not have a weapon with him but keeps a gun in an unlocked drawer in his nightstand.
Which one of the following would be most appropriate at this point?
A) Avoiding direct inquiry about suicide
B) Calling 911
C) Crisis planning
D) Creating a suicide prevention contract
E) Withholding psychogenic medications

A

ANSWER: C
Crisis planning is recommended for patients presenting with suicidal ideation (SOR B). By identifying social support, local resources, and counseling services, suicidal ideation and days spent in the hospital can be reduced. Direct inquiry about suicide is recommended to better evaluate and treat suicidal patients with more favorable outcomes (SOR B). Though calling 911 may be appropriate for transportation for inpatient therapy if involuntary treatment is recommended, further assessment is needed in this case. Suicide prevention contracts do not effectively prevent suicide (SOR B). Psychogenic medications should not be withheld when treating a patient with suicidal ideation. Evidence has shown that the combination of pharmacotherapy and psychotherapy is most effective (SOR C).

488
Q

Side effect of thiazolidinediones

A

Pioglitazone
Rosiglitazone

Weight gain
Salt retention
Edema

489
Q

Montelukast (Singulair) has an FDA boxed warning related to an increased risk of
A) delirium
B) myocardial infarction
C) suicidality
D) venous thromboembolism

A

ANSWER: C
In March 2020, the FDA upgraded its warning label for montelukast to a boxed warning (black box warning) based on the trends for all neuropsychiatric adverse events, including suicidality, associated with montelukast use reported in the FDA Adverse Event Reporting System database from the date of FDA approval in February 1998 through May 2019 (SOR B). The boxed warning does not indicate an increased risk of delirium, myocardial infarction, or venous thromboembolism.

490
Q

A 28-year-old female presents with a 2.5-cm pruritic, erythematous, oval macule on her left thigh. She was seen at an urgent care facility 2 days ago for a urinary tract infection (UTI) and was treated with sulfamethoxazole/trimethoprim (Bactrim). Her UTI symptoms have improved. She reports that she was called earlier this morning and told that her infection was caused by Escherichia coli. The patient reports a similar lesion in the same area about a year ago at the time of her last UTI.
You explain this is most likely secondary to
A) an immunologic reaction to E. coli
B) erythema multiforme
C) nummular eczema
D) the Shiga toxin sometimes produced by E. coli
E) the sulfamethoxazole/trimethoprim used to treat the infection

A

ANSWER: E
This is a typical history for a fixed drug eruption (FDE), which is an immunologic reaction that recurs upon re-exposure to the offending drug. It is most likely related to T-lymphocytes at the dermal-epidermal junction. Sulfonamides and anticonvulsants are the most frequently cited medications, but tetracycline and penicillins have also been reported to cause FDE. FDE is not caused by bacteria. Erythema multiforme does not present as an isolated, recurrent macule and generally has central clearing. Nummular eczema is a coin-shaped, very pruritic patch but does not fit this clinical scenario. Shiga toxin–producing Escherichia coli are rarely found in extra-intestinal sites.

491
Q

A 72-year-old female presents with progressive hand pain and stiffness. She is a seamstress and is concerned because sewing has been more difficult over the past 6 months. She recalls that her mother’s hands were misshapen, but her mother never received a diagnosis. You examine her hands, which are shown below.
Which one of the following would be the most appropriate pharmacotherapy?
A) Colchicine (Colcrys)
B) Diclofenac (Zorvolex)
C) Hydroxychloroquine (Plaquenil)
D) Infliximab (Remicade) injections
E) Methotrexate (Trexall)

A

ANSWER: B
This patient presents with erosive osteoarthritis that involves the distal interphalangeal (DIP) and proximal interphalangeal (PIP) joints with sparing of the metacarpophalangeal (MCP) joints. The primary goals for treating osteoarthritis are to control symptoms such as pain and stiffness and optimize function in order to preserve quality of life. Topical or oral NSAIDs are the most appropriate pharmacotherapy for osteoarthritis of the hand. Colchicine and methotrexate have not been studied for the treatment of osteoarthritis and their use for this condition is not recommended. Colchicine is indicated for the treatment of gout, which is usually pauciarticular and asymmetrical, and methotrexate is effective for rheumatoid arthritis. Conventional synthetic and biologic disease-modifying medications such as hydroxychloroquine and infliximab have not been shown to be effective in the treatment of osteoarthritis. These medications are appropriate for the treatment of systemic lupus erythematosus and rheumatoid arthritis, which have examination findings that involve the MCP and PIP joints but spare the DIP joints.

492
Q

Which one of the following medications for the treatment of type 2 diabetes has been associated with ketoacidosis?
A) Dapagliflozin (Farxiga)
B) Liraglutide (Victoza)
C) Metformin
D) Pioglitazone (Actos)
E) Sitagliptin (Januvia)

A

ANSWER: A
SGLT2 inhibitors such as dapagliflozin have increasingly been shown to be associated with diabetic ketoacidosis under certain circumstances. Liraglutide, metformin, pioglitazone, and sitagliptin are not associated with diabetic ketoacidosis.

493
Q

A 48-year-old male with schizophrenia presents for a new patient visit after recently relocating to your area. He has been stable on clozapine (Clozaril) for the past 15 years and asks you to refill his prescription. He has been told the earliest available appointment with a local psychiatric provider is in 3 months.
Under the Clozapine Risk Evaluation and Mitigation Strategy (REMS) program, which one of the following is required to prescribe clozapine to this patient?
A) A signed patient consent form
B) Serum clozapine levels
C) Creatinine levels
D) Neutrophil counts
E) Specialty training in psychiatry

A

ANSWER: D
Clozapine is a highly effective antipsychotic medication, but its use is limited due to its association with severe drug-induced neutropenia, also referred to as agranulocytosis. Patients must be enrolled in the national Clozapine Risk Evaluation and Mitigation Strategy (REMS) program to receive treatment, and all prescribers and pharmacies must be certified by this program in order to dispense clozapine. The patient’s absolute neutrophil count must be submitted at least every 30 days, or more frequently as determined by stability in treatment. A signed patient consent form should be obtained but is not a part of the Clozapine REMS monitoring system. Monitoring serum clozapine levels and creatinine levels may be appropriate but is not part of the Clozapine REMS program. Family physicians can prescribe clozapine if registered and certified in the Clozapine REMS program, which includes passing a brief knowledge assessment, but specialty training in psychiatry is not required.

494
Q

A 32-year-old female comes to your office because of chronic diarrhea, abdominal cramping, and bloating. She has had these symptoms for many years but has never discussed them in depth with a physician. A thorough history and physical examination are most consistent with irritable bowel syndrome (IBS). You order IgA tissue transglutaminase (tTG) antibody and fecal calprotectin testing to rule out other conditions and both are negative. She has expressed an interest in nonpharmacologic measures as initial management of her IBS.
Which one of the following should you recommend initially, given that it has the best evidence of benefit for her condition?
A) A gluten-free diet
B) A low-FODMAP diet
C) Soluble fiber
D) Prebiotics
E) Probiotics

A

ANSWER: C
This patient has diarrhea-predominant irritable bowel syndrome (IBS-D) and may benefit from validation of her symptoms and a clear diagnosis that has several substantiated treatment options. A 2021 clinical guideline from the American College of Gastroenterology (ACG) is based on a systematic review performed by a committee of experts in this field. Based on this review, soluble fiber (but not insoluble fiber) has good evidence for the alleviation of global IBS symptoms and is recommended strongly as a first-line intervention. In contrast, a gluten-free diet has not been shown to be beneficial for IBS. A diet low in fermentable oligosaccharides, disaccharides, monosaccharides, and polyols (FODMAP) has low-quality evidence supporting its benefit but merits a trial in patients who do not have an adequate response to fiber supplementation. The ACG has not commented on prebiotics as a treatment for IBS. A 2018 systematic review concluded that while there are small individual studies suggesting benefit for prebiotics (and probiotics) there is inadequate long-term consistent evidence to support their routine use. Probiotics have mixed and low-quality evidence for benefit in IBS. Because of the inconsistent data the ACG recommends against their use.

495
Q

A 64-year-old male is hospitalized with anorexia, intractable abdominal pain, and dehydration due to locally advanced pancreatic cancer. He is started on intravenous fluids and morphine, along with a prophylactic dose of subcutaneous heparin. Shortly after admission he develops right-sided chest pain and shortness of breath. His vital signs are normal, except for a respiratory rate of 24/min. An abdominal examination reveals tenderness in the epigastric area. An examination of the heart and lungs is normal. There is no calf tenderness or leg edema. An EKG shows new right bundle branch block.
Which one of the following tests should you order next?
A) A D-dimer level
B) A troponin level
C) Doppler ultrasonography of the lower extremities
D) A ventilation-perfusion (V/Q) scan
E) Computed tomography pulmonary angiography (CTPA)

A

ANSWER: E
This patient’s pretest probability for pulmonary embolism is high given his multiple risk factors, signs, and symptoms. The presence of a new onset of right bundle branch block in a patient presenting with a sudden onset of shortness of breath and chest pain, especially in the setting of active cancer, should raise suspicion of pulmonary embolism. Other EKG abnormalities include tachycardia or bradycardia, an S1Q3T3 pattern, atrial fibrillation, and T-wave inversions in the anterior leads. Patients with cancers of the pancreas and stomach have the highest risk of developing venous thromboembolism (VTE) and should receive pharmacologic VTE prophylaxis during hospitalizations.

In the absence of renal failure, a computed tomography pulmonary angiogram (CTPA) is the most appropriate diagnostic study and would be preferred over a ventilation-perfusion (V/Q) scan. A D-dimer level has a high negative predictive value in the diagnosis of pulmonary embolism; however, it has low specificity, and therefore a high rate of false positives, in patients with active cancer. An elevated troponin level can occur in the setting of pulmonary embolism and is nondiagnostic. Their principal value is in the diagnosis of acute myocardial infarction (SOR A). Doppler ultrasonography of the lower extremities helps identify and locate peripheral deep vein thrombosis and helps support but not confirm the diagnosis of pulmonary embolism. A V/Q scan is a reasonable option when CTPA is contraindicated.

496
Q

Which one of the following disorders is caused by an underlying mechanism of osteochondrosis rather than apophysitis?
A) Legg-Calvé-Perthes disease
B) Osgood-Schlatter disease
C) Sever’s disease
D) Sinding-Larsen-Johansson syndrome

A

ANSWER: A
Osteochondrosis refers to degenerative changes in the epiphyseal ossification areas of growing bones. Legg-Calvé-Perthes disease is a type of osteochondrosis that affects the femoral head. Patients with Legg-Calvé-Perthes disease should be referred to an orthopedist and instructed to avoid all weight-bearing activities until reossification occurs. Osteochondrosis should be differentiated from apophysitis because the etiologies and management strategies differ. Apophysitis is a traction injury to the cartilage and bony attachments of tendons in growing children. Osgood-Schlatter disease, Sever’s disease, and Sinding-Larsen-Johansson syndrome are apophysitis disorders that affect the anterior tibial tubercle, posterior heel, and inferior patellar pole, respectively. Treatment of apophysitis involves stretching, activity modification, icing, and limited use of NSAIDs.

497
Q

A 40-year-old female presents with several pruritic, thickened, scaly areas on her lower back, knees, and elbows. She says that when she tries to remove the scales they often bleed.
Which one of the following would be the most appropriate pharmacologic therapy for this patient?
A) Clobetasol propionate 0.05% lotion (Clobex)
B) Selenium sulfide 2.5% lotion
C) Permethrin cream (Nix)
D) Terbinafine cream 1%
E) Loratadine (Claritin), 10 mg daily

A

ANSWER: A
This patient has psoriasis that is characterized by plaques on her extensor extremities and limited bleeding with removal of the scales (Auspitz sign). First-line treatment for localized plaques is topical corticosteroid therapy, such as clobetasol propionate lotion. Antifungals such as selenium sulfide lotion and terbinafine cream are used to treat dermatophytosis infections including tinea pedis and tinea versicolor. Permethrin cream is indicated for treatment of scabies and lice. Loratadine, an oral antihistamine, is used to treat urticaria.

498
Q

A 17-year-old cross country runner sees you to discuss the results of pulmonary function tests to evaluate his episodic shortness of breath and chest tightness. He had previously been diagnosed with exercise-induced asthma and prescribed albuterol (Proventil, Ventolin), which provided minimal relief. You tell him that the pulmonary function tests revealed normal expiratory findings including normal FEV1 and FVC and a flattened inspiratory flow loop.
The test most likely to confirm a diagnosis for this patient’s shortness of breath is
A) a sleep study
B) chest radiography
C) chest CT
D) esophagogastroduodenoscopy
E) nasolaryngoscopy

A

ANSWER: E
This patient has vocal cord dysfunction, sometimes called paradoxical vocal fold motion, a condition in which the vocal cords close during inspiration when they should be open. It is not entirely understood why this occurs but it is associated with other conditions including asthma, GERD, and anxiety disorders. It typically causes sudden, severe shortness of breath and often has a trigger such as exercise, gastroesophageal reflux, inhalation of an irritant, or stress. Symptoms may include chest or throat tightness, inspiratory stridor, and wheezing predominantly over the upper airway. In less severe situations the voice may be impacted, and patients sometimes also describe a chronic cough that occurs separately from more acute symptoms. Vocal cord dysfunction is confirmed by direct visualization of the vocal cords during inspiration via nasolaryngoscopy. Pulmonary function tests are often performed as part of the assessment for shortness of breath and, if performed while the patient is experiencing symptoms, will show a flattened inspiratory flow loop. Treatment is primarily focused on therapeutic breathing maneuvers and vocal cord relaxation techniques. A speech therapist may assist in instructing patients in these techniques. Associated conditions should also be treated to help prevent vocal cord dysfunction. A sleep study, chest radiography, chest CT, and esophagogastroduodenoscopy would not confirm a diagnosis of vocal cord dysfunction.

499
Q

Heat Stroke

A

Temp > 104 F assoc with neurologic conditions ( Headache, confusion, altered mental status, irritability, seizure)

500
Q

Heat Exhaustion

A

May have neurologic symptoms but are at a lower heat than heat stroke

101- 104

501
Q

Heat injury

A

Like heatstoke has temp > 104 but no neurologic symptoms.

Instead kidney, muscle or liver injury may be present

502
Q

Incidental finding of elevated Alk Phos

A

Concern for pagets disease of bone.

Get xray of skull, pelvis and tibia

503
Q

An 11-year-old female is brought to your office by her parent who is concerned that the child’s spine might be curved. The most appropriate evaluation for scoliosis at this point is
A) comparing the length from the pelvic brim to the pelvic floor on the left and the right
B) scoliometer measurement with the patient bent over to 90°
C) scoliometer measurement with the patient upright and arms to her side
D) determination of the Cobb angle with the patient bent over to 90°
E) determination of the Cobb angle with the patient upright and arms to her side

A

ANSWER: B
The forward bend test, combined with a scoliometer measurement, is the most appropriate initial test when evaluating for scoliosis. A scoliometer should be used with the patient’s spine parallel to the floor (bent over to approximately 90°), with the arms hanging down, palms together, and feet pointing forward. If 5°–7° of trunk rotation is assessed by the scoliometer or by a scoliometer app on a smartphone, radiography can be performed to assess the Cobb angle. This radiography should be performed with the patient upright. A Cobb angle >20° may signify scoliosis, which may benefit from bracing, depending on skeletal maturity. Comparing the length from the pelvic brim to the pelvic floor on the left and the right is not indicated in the evaluation for scoliosis.
The U.S. Preventive Services Task Force changed its recommendation for scoliosis screening from grade D to grade I in 2018. Bracing has been found to reduce by over 50% the chance that mild to moderate curvatures will progress to curvatures of greater than 50°.

504
Q

A 55-year-old female with a BMI of 50 kg/m2 and recently diagnosed severe obstructive sleep apnea (OSA) presents for follow-up after a sleep study. She was unable to tolerate positive pressure therapy.
Her OSA could be most effectively addressed by which one of the following interventions?
A) Use of a nasal dilator device
B) A positional sleep alarm to avoid the supine position
C) Clonidine, 0.1 mg orally before bedtime
D) Uvulopalatopharyngoplasty
E) Bariatric surgery

A

ANSWER: E
Obstructive sleep apnea (OSA) is a common disorder that, if left untreated, can be associated with other serious health conditions such as atrial fibrillation, depression, heart failure, and stroke. Positive pressure therapy is effective and considered the first-line treatment for OSA, although some patients are unable to tolerate this therapy. In obese patients with OSA, bariatric surgery has been shown to reliably result in improvement in >75% of patients and result in remission in 40% of patients after 2 years. Nasal dilator devices and pharmacologic interventions such as clonidine have not been shown to improve symptoms or to be effective for treatment. Positional therapy is not recommended as a long-term solution for severe OSA due to poor long-term compliance. Currently there is insufficient evidence to support oral procedures such as uvulopalatopharyngoplasty as primary interventions for OSA.

505
Q

AAA size needs fixed

A

Men 5.5 cm
Women 5.0 cm

506
Q

A 51-year-old patient asks about recommended lung cancer screenings. The U.S. Preventive Services Task Force recommends annual lung cancer screening with low-dose CT for individuals starting at age
A) 45 with a 15-pack-year smoking history
B) 50 with a 20-pack-year smoking history
C) 55 with a 30-pack-year smoking history
D) 60 with a 35-pack-year smoking history
E) 65 with a 40-pack-year smoking history

A

ANSWER: B
Lung cancer is the second most common cancer in both women and men, after breast cancer for women and prostate cancer for men. It is the leading cause of cancer deaths in the United States, making it important for primary care providers to screen for this disease process. The primary risk factor for lung cancer is tobacco smoking, which accounts for 90% of all lung cancer cases. Lung cancer has a relatively poor prognosis, but early-stage lung cancer is more amenable to treatment and has a better prognosis. Low-dose CT has a reasonable specificity and high sensitivity for lung cancer in patients at high risk. The eligibility criteria were recently updated by the U.S. Preventive Services Task Force due to evidence of mortality benefit, with a recommendation for screening to begin at age 50 for patients with a 20-pack-year smoking history who are current smokers or have quit within the past 15 years.

507
Q

Central vision loss with peripheral spearing

A

Macular degeneration

508
Q

A 62-year-old male presents with a 2-day history of a painful abscess in his perianal area. He has a history of well controlled type 2 diabetes and hypertension and he currently takes metformin and lisinopril (Zestril). He also has a history of recurrent skin abscesses, which have responded well to oral sulfamethoxazole/trimethoprim (Bactrim). He has occasional chills but has not had a fever. On examination his vital signs are normal and you note the presence of a 2.5×2.5-cm perianal abscess. A point-of-care glucose level is 172 mg/dL and the results of a CBC are pending.
Which one of the following would be the most appropriate next step?
A) Continue current management and follow up in 48 hours
B) Perform incision and drainage, obtain a culture, and start sulfamethoxazole/trimethoprim
only
C) Perform incision and drainage, obtain a culture, and start oral linezolid (Zyvox)
D) Perform incision and drainage, obtain a culture, and start sulfamethoxazole/trimethoprim
plus amoxicillin/clavulanate (Augmentin)

A

ANSWER: D
Incision and drainage of an abscess along with MRSA antibiotic coverage is recommended for all abscesses greater than 2×2 cm. Incision and drainage of an abscess is almost always indicated and is a cornerstone of treatment. A wound culture with antibiotic sensitivity must be obtained in all cases to guide therapy. MRSA is the most common causative pathogen, so an antibiotic that provides coverage against MRSA, such as sulfamethoxazole/trimethoprim, doxycycline, or clindamycin, should be used as an empiric first-line agent for treatment of a skin abscess, pending culture results. Additional anaerobic coverage is recommended when an abscess is located in the perirectal area or when an abscess occurs in an area where tissue ischemia is likely.

509
Q

A 67-year-old male presents for follow-up of ongoing chest pain that he experiences when walking up hills. His medical history is significant for hypertension and coronary artery disease. Four months ago he had a positive exercise stress test and underwent coronary angiography, which showed diffuse atherosclerotic disease but no lesions suitable for percutaneous intervention. His current medications include aspirin, 81 mg; atorvastatin (Lipitor), 80 mg; and metoprolol succinate (Toprol-XL), 100 mg. His vital signs include a blood pressure of 120/66 mm Hg and a pulse rate of 68 beats/min. Recent laboratory studies are significant for an LDL-cholesterol level of 58 mg/dL, a triglyceride level of 120 mg/dL, and a troponin level of 0.05 ng/mL (N <0.04).
The addition of which one of the following agents would decrease this patient’s all-cause mortality risk?
A) Clopidogrel (Plavix), 75 mg daily
B) Colchicine (Colcrys), 0.6 mg daily
C) Icosapent ethyl (Vascepa), 2 g twice daily
D) Isosorbide mononitrate, 30 mg daily
E) Rivaroxaban (Xarelto), 2.5 mg twice daily

A

ANSWER: E
This patient presents with stable angina and documented coronary atherosclerosis. His slight troponin elevation is a marker of elevated risk. The addition of low-dose rivaroxaban to aspirin has been shown to decrease cardiac and all-cause mortality in patients with coronary artery disease (CAD) and may be offered to this patient (SOR A). Dual antiplatelet therapy with clopidogrel and aspirin is recommended for 1 year after stenting but is not recommended in patients with stable angina who do not have stents. Colchicine has been associated with decreased cardiac events in patients with CAD but may increase all-cause mortality. Icosapent ethyl has been shown to decrease cardiac events but not mortality in patients with hypertriglyceridemia >150 mg/dL. Isosorbide mononitrate may be indicated to improve angina symptoms but does not improve mortality risk.

510
Q

A 67-year-old male presents to your office with a 1-month history of fever with edema and erythema of his right foot. His medical history is significant for peripheral artery disease and poorly controlled type 2 diabetes with diabetic neuropathy. Significant vital signs include a temperature of 38.6°C (101.5°F), a blood pressure of 155/90 mm Hg, and a pulse rate of 85 beats/min. A physical examination is most notable for a draining ulcer on the ball of his right foot. The edema and erythema are limited to his right foot and he has no calf tenderness. Dorsalis pedis and posterior tibial artery pulses are present but diminished at the right ankle. Sensation to monofilament testing of his right foot is diffusely diminished, which is consistent with his baseline.
The best initial imaging test of the foot ulcer would be
A) radiography
B) ultrasonography
C) CT
D) MRI
E) technetium 99m bone scintigraphy

A

ANSWER: A
This patient has osteomyelitis until proven otherwise. A radiograph of his right foot is the best initial test to look for evidence of this diagnosis. In most treatment settings, radiography is much easier to obtain than ultrasonography, CT, MRI, or technetium 99m bone scintigraphy. In addition, it is generally considerably less expensive than the other options listed. A radiograph also allows the physician to rule out other bony pathologies. MRI is useful if the radiograph is inconclusive, and is more helpful than radiography for determining bony versus soft-tissue infection. CT may be used in cases where MRI is contraindicated. Ultrasonography is not useful for evaluating bony lesions. Bone scintigraphy has low sensitivity, particularly in the setting of recent trauma or surgery.

511
Q

A 23-year-old female with a history of systemic lupus erythematosus presents to discuss contraceptive management. You note that she is no longer taking hydroxychloroquine (Plaquenil) and has not seen her rheumatologist in over a year. She reports generalized fatigue and intermittent joint pain. She is agreeable to obtaining laboratory studies at today’s visit.
In addition to checking anti–double-stranded DNA, complement levels, an erythrocyte sedimentation rate, a C-reactive protein level, a CBC, and a comprehensive metabolic panel, which one of the following laboratory tests would be most helpful in monitoring her disease activity?
A) An anti-cyclic citrullinated peptide level
B) An antinuclear antibody titer
C) A creatine kinase level
D) A fasting lipid panel
E) A urinalysis with microscopic examination

A

ANSWER: E
The majority of patients with systemic lupus erythematosus (SLE) follow a relapsing-remitting course, and management requires monitoring clinical symptoms and laboratory studies. Prolonged remission and persistently active disease are both infrequent. In addition to monitoring anti–double-stranded DNA, complement levels, erythrocyte sedimentation rates, C-reactive protein levels, WBC counts, platelet counts, serum creatinine levels, and glomerular filtration rates, monitoring urine studies for proteinuria is important to assess disease activity over time. Lupus nephritis is a significant cause of morbidity and mortality associated with SLE. Anti-cyclic citrullinated peptide and creatine kinase levels are associated with rheumatoid arthritis and polymyositis, respectively. These tests may assist in ruling out other autoimmune diseases but are not helpful once a diagnosis of SLE is made. While an antinuclear antibody (ANA) titer is helpful in making a diagnosis of SLE, it does not assist with assessing disease activity over time and should not be checked once it is known to be positive. Monitoring lipid levels and atherosclerotic cardiovascular risk is important in managing SLE, but lipid levels do not correlate with flares of SLE.

512
Q

A 45-year-old male presents for follow-up of a recent positive HIV test. He has not had any symptoms. An initial laboratory evaluation is significant for the following:
HIVviralload 124,000copies/mL CD4lymphocytecount 289cells/μL HepatitisCantibody negative Anti-HBs positive Anti-HBc positive HBsAg negative
Renal function is normal. He has an upcoming appointment with the comprehensive HIV clinic to initiate antiretroviral therapy.
Which one of the following would be appropriate to recommend today?
A) Prophylactic emtricitabine/tenofovir (Truvada)
B) Prophylactic sulfamethoxazole/trimethoprim (Bactrim)
C) Hepatitis B vaccine
D) Herpes zoster vaccine (Shingrix)
E) Meningococcal B (MenB) vaccine

A

ANSWER: D
The CDC’s Advisory Committee on Immunization Practices updated its recommendations in 2022 to include a two-dose series of recombinant zoster vaccine for all adults age 19 and older with HIV. Vaccination against meningococcal bacteria A, C, W, and Y (MenACWY) is also recommended, and meningococcal B (MenB) vaccination is only recommended based on the presence of other risk factors, including asplenia, complement deficiency, treatment with complement inhibitors, or risk due to outbreaks. Prophylactic emtricitabine/tenofovir is approved for pre- and postexposure prophylaxis of HIV, but would not be used alone in the care of patients with established HIV. Pneumocystis jirovecii prophylaxis, most commonly with sulfamethoxazole/trimethoprim, is recommended in patients with CD4 lymphocyte counts <200cells/ L.HepatitisBvaccineisrecommendedbutwouldnotbenecessaryforpatientssuchasthis one with natural immunity or confirmed immunity from vaccination.

513
Q

A 67-year-old male with a history of diabetes mellitus, hypertension, and heart failure with reduced ejection fraction has developed stage 5 chronic kidney disease. Which one of the following would be the best option for treatment of his diabetes?
A) Glimepiride (Amaryl)
B) Insulin glargine (Lantus)
C) Metformin
D) Pioglitazone (Actos)

A

ANSWER: B
Patients with end-stage renal disease and diabetes mellitus need careful monitoring of glucose because insulin requirements are difficult to predict and there is an increased risk of hypoglycemia in this setting. The optimal hemoglobin A1c has not been established but maintaining a value between 6% and 9% does decrease mortality. With close monitoring, insulin is preferred for most individuals. Sulfonylureas such as glimepiride and glyburide are associated with a high risk of hypoglycemia and should be avoided in these patients. Metformin should be avoided in those with a glomerular filtration rate <30 mL/min/1.73 m2. Pioglitazone should also be avoided in chronic kidney disease due to the risk of fluid retention and precipitating heart failure.

514
Q

A 4-month-old female is brought to your clinic for a routine well child visit. She has not been seen by a physician since 2 weeks of age due to parental concerns about seeking care during the COVID-19 pandemic. Her growth and development appear to be normal.
Which one of the following vaccines is CONTRAINDICATED today?
A) Haemophilus influenzae type b
B) Hepatitis B
C) Inactivated poliovirus
D) 13-valent pneumococcal conjugate (PCV13, Prevnar 13)
E) Rotavirus

A

ANSWER: E
Vaccine delay and vaccine hesitancy are on the rise in the United States, so family physicians should be familiar with the nuances of vaccine catch-up schedules as well as contraindications. Rotavirus vaccine has age restrictions and should not be initiated after 14 weeks and 6 days of age. In addition, the rotavirus series must be complete by 8 months of age. These age restrictions are intended to ensure the vaccine is administered when it will be of maximal benefit to children given the slightly increased risk of intussusception after vaccine administration. Hepatitis B vaccine should be administered at routine intervals. Haemophilus influenzae type b, inactivated poliovirus, and pneumococcal conjugate can all be administered to this patient today. However, these vaccines have complex follow-up intervals based on the age at prior doses and age at catch-up. Therefore, the clinician should consult the CDC catch-up vaccine schedule to verify dosing intervals.

515
Q

A 26-year-old recreational baseball player presents with recurrent right shoulder pain that tends to gradually worsen during play and is relieved by rest. His other daily activities have not been affected and he has no nighttime pain.
Examination of the right shoulder reveals a normal appearance, no tenderness to palpation, normal abduction strength, and a positive painful arc at 90°. The drop-arm rotator cuff test is negative, the Hawkins impingement sign is mildly positive, the empty-can supraspinatus test is moderately positive, and the Gerber liftoff test is negative. Radiographs of the right shoulder are normal.
Which one of the following would be appropriate at this time to provide long-term pain relief?
A) Complete shoulder rest with temporary use of a shoulder sling
B) Recommending that he permanently stop playing baseball
C) A subacromial corticosteroid injection
D) Physical therapy
E) Referral for arthroscopic surgery

A

ANSWER: D
This patient has shoulder impingement syndrome (with a positive Hawkins impingement sign) and evidence of supraspinatus tendinopathy (with a positive empty-can rotator cuff test). However, the negative drop-arm rotator cuff test is evidence against a complete rotator cuff tear with a negative drop-arm rotator cuff test, and the absence of night pain supports this. Physical therapy, along with pain control using NSAIDs, acetaminophen, or short-term opiate medication, would be most appropriate as initial therapy. Complete shoulder rest is inappropriate since his daily activities are not aggravating the problem, and cessation of play is not necessary since other treatment options are available. A subacromial corticosteroid injection, while commonly done and likely to provide short-term pain relief, is unlikely to provide long-term improvement in pain and function. Surgery is a potential option if other treatments fail and a significant tear is proven, but is not preferable as an initial treatment.

516
Q

A 62-year-old male presents with bright red hemoptysis. While not severe, it is recurrent and has persisted for several weeks. He is otherwise asymptomatic.
A thorough history and physical examination does not provide any additional clues. Gastrointestinal and ear, nose, and throat etiologies are considered and are not thought to be the cause. Posteroanterior and lateral chest radiographs are normal.
Which one of the following would be the most appropriate next step in diagnosis?
A) Antineutrophil cytoplasmic antibody (ANCA) testing
B) A sputum smear for acid-fast bacillus
C) Sputum cytology
D) CT of the chest
E) Bronchoscopy

A

ANSWER: D
A chest radiograph is appropriate in the initial evaluation of hemoptysis (SOR C). If the chest radiograph does not indicate a cause, then CT or CT angiography with intravenous contrast should be performed (SOR C). CT has become the preferred modality over bronchoscopy because it is more effective in determining the etiology. If CT does not identify the cause, bronchoscopy would be the next step. In addition, other tests including a sputum Gram stain, acid-fast bacillus smear, or sputum cytology can be useful depending upon the clinical situation.
If there are concerns about the possibility of immunologic, rheumatologic, or vasculitic disease, testing for immunologic antibodies such as antineutrophil cytoplasmic antibody (ANCA) can be ordered.

517
Q

A 39-year-old female with a BMI of 42 kg/m2 and a history of hypertension, diabetes mellitus, hyperlipidemia, obstructive sleep apnea, and hypothyroidism has been struggling to lose weight. Her medical conditions are controlled. She asks you if weight loss surgery would be better for her than continued dietary efforts.
Which one of the following is true regarding weight loss surgery?
A) Post-surgical dietary recommendations include consuming carbohydrates first at each meal
B) Post-surgical dietary recommendations include drinking fluid with each meal
C) Diabetes remission occurs in the majority of patients 2 years after a Roux-en-Y
procedure
D) Surgical treatment for obesity is equivalent to nonsurgical interventions in overall length
of survival
E) Surgical treatment for obesity leads to a lifelong need for quarterly laboratory studies
to check for nutritional deficiencies

A

ANSWER: C
Surgical treatment for obesity results in remission of diabetes mellitus in 60%–80% of patients at 2 years and sustained remission in 30% at 15 years after a Roux-en-Y procedure. Postsurgical dietary recommendations include consuming protein first at each meal, rather than carbohydrates, to prevent malnutrition, and separating consumption of liquids from solids by 15–30 minutes to avoid food passing through the stomach too quickly, which can lead to a decreased sensation of satiety. Patients who are treated surgically for obesity rather than with nonsurgical interventions have a longer overall length of survival. All-cause mortality is decreased by 30%–50% at 7–15 years after surgery. Evaluation for nutritional deficiencies should be performed quarterly for the first year after surgery; after that, annual checks are recommended.

518
Q

A 60-year-old male with symptomatic low testosterone is started on a testosterone patch. You should order specific periodic monitoring of his PSA level and
A) no other laboratory studies
B) ALT and AST levels
C) BUN and creatinine levels
D) C-reactive protein level
E) hematocrit

A

ANSWER: E
When testosterone therapy is started, baseline and periodic measurements of PSA and hematocrit should be performed. If the hematocrit increases to >54% the testosterone dosage should be stopped or decreased to avoid hemoconcentration. Liver and renal function tests should be monitored routinely with many drugs but not specifically with testosterone. The C-reactive protein level is not monitored during testosterone therapy.

519
Q

In adolescents and adults, what proportion of cases of uncomplicated, acute bronchitis are caused by atypical organisms such as Mycoplasma pneumoniae and Chlamydia pneumoniae?
A) 1% B) 15% C) 30%
D) 65% E) 90%

A

ANSWER: A
Acute bronchitis is caused by a viral infection in 90%–99% of cases. Atypical organisms such as Mycoplasma pneumoniae and Chlamydia pneumoniae are rare causes and have been found in less than 1% of cases of acute bronchitis.

520
Q

A 30-year-old male presents to your office after sustaining a scratch to the eye while playing with his 2-year-old nephew. A penlight examination reveals sensitivity to light and mild conjunctival irritation with no foreign body. Pupillary response, extraocular movements, and visual acuity are all normal. Fluorescein staining reveals a 3-mm corneal abrasion.
Which one of the following would be the most appropriate management?
A) Patching the affected eye
B) Patching the unaffected eye
C) Prednisolone ophthalmic drops
D) Tetracaine ophthalmic drops
E) Oral naproxen

A

ANSWER: E
Corneal abrasions are a common cause of acute eye pain and are often evaluated in primary care settings. Small (4 mm), uncomplicated abrasions typically heal within 1–2 days and usually respond to oral analgesics such as acetaminophen or NSAIDs. A 2013 review reported effective pain relief and earlier return to work with use of topical NSAIDs, although a 2017 Cochrane review subsequently found that evidence may be lacking to support their use, especially considering the higher cost when compared to oral options.
Patching has been proven ineffective for pain relief and can delay healing, although ophthalmologists sometimes use patching to treat large abrasions or to provide a protective barrier for patients who may have difficulty avoiding rubbing their eyes, such as children or people with cognitive impairment. Patching of the unaffected eye is done to treat amblyopia but is not appropriate for managing corneal abrasions. Topical corticosteroids such as prednisolone are not appropriate for treatment of corneal abrasions due to increased susceptibility to infection and the risk of delayed healing and should only be used under the guidance of an ophthalmologist. While point-of-care use of topical anesthetics such as tetracaine may be considered, repeated administration is not recommended as continued use may cause damage to the corneal epithelium, delay healing, and mask symptoms. While topical antibiotics are often prescribed in the setting of corneal abrasion to prevent bacterial superinfection, evidence to support this practice in general is lacking. However, contact lens wearers are at increased risk of infection due to Pseudomonas aeruginosa and should be prescribed an antibiotic with antipseudomonal activity.

521
Q

A 17-year-old male with a history of type 2 diabetes sees you because of fatigue and a 15-lb weight loss in the past month. The patient reports excessive and frequent urination, thirst, and nausea. His only medication has been metformin, but he states that he stopped taking it 6 months ago. His current weight in your office is 93 kg (205 lb), which confirms the reported weight loss. His blood pressure is 130/78 mm Hg, his pulse rate is 90 beats/min, and his temperature is 37.0°C (98.6°F). A physical examination is otherwise unremarkable. A capillary blood glucose level is 348 mg/dL, a hemoglobin A1c is 11.5%, serum ketones are negative, and a urinalysis shows 3+ glucosuria with concentrated urine but is otherwise normal.
Which one of the following would be the most appropriate treatment?
A) Resuming oral metformin
B) Starting oral empagliflozin (Jardiance)
C) Starting subcutaneous insulin
D) Starting subcutaneous liraglutide (Victoza)
E) Hospitalization for continuous intravenous insulin

A

ANSWER: C
This patient presents with symptomatic hyperglycemia in a catabolic state. In such cases insulin therapy is the most reliable way to control hyperglycemia and reverse catabolism. Oral metformin would not be adequate to control this degree of hyperglycemia and might not be tolerated well, given that the current symptoms include nausea and weight loss. Similarly, both empagliflozin, which increases glucosuria and volume contraction, and liraglutide, which decreases gastric emptying and is likely to exacerbate nausea, are likely to be poorly tolerated in this situation. While rapid and effective treatment is essential to prevent further complications, hospitalization is not necessary since the patient has no evidence of diabetic ketoacidosis.

522
Q

A 62-year-old male presents with a 3-day history of left lower quadrant pain and a low-grade fever. Findings on CT are consistent with acute diverticulitis. The patient has a history of intolerance to metronidazole (Flagyl).
If antibiotics are given, the preferred agent for this patient would be
A) amoxicillin/clavulanate (Augmentin)
B) azithromycin (Zithromax)
C) cephalexin (Keflex)
D) ciprofloxacin (Cipro)
E) doxycycline

A

ANSWER: A
The traditional approach to outpatient management of acute diverticulitis consists of clinical diagnosis (with or without imaging), antibiotics, and bowel rest. Two cohort studies found no difference in the effectiveness of outpatient treatment of diverticulitis with amoxicillin/clavulanate or with metronidazole plus a fluoroquinolone. Azithromycin is more appropriate for Campylobacter or Escherichia coli infections that cause lower gastrointestinal bleeding. Cephalexin is not an appropriate treatment, and ciprofloxacin monotherapy will not provide adequate coverage. Doxycycline is a treatment for watery diarrhea caused by Vibrio cholerae and Yersinia infections.

523
Q

A 48-year-old female sees you because she recently felt flutters in her chest while watching television. These were not associated with exertion. She has no significant past medical history and she does not take any medications or use illicit substances.
On examination you hear a regular rhythm with occasional premature beats. An EKG reveals multiple unifocal PVCs. You order 48-hour Holter monitoring, which shows a 15% PVC burden that is unifocal with no episodes of ventricular tachycardia.
Which one of the following would be most appropriate at this time?
A) No further evaluation and reassurance that her palpitations are benign
B) Initiation of a β-blocker
C) Initiation of flecainide
D) Echocardiography
E) Left heart catheterization

A

ANSWER: D
Patients found to have a PVC burden >10% are at risk for PVC-induced dilated cardiomyopathy (PVC-CM). In fact, a PVC burden of 16% has a sensitivity of almost 80% for PVC-CM. Echocardiography should be performed in patients with a PVC burden >10%. Treatment with anti-arrhythmic drugs or radiofrequency ablation reverses cardiomyopathy and its associated increase in morbidity, mortality, and health care spending. Further evaluation for ischemic heart disease may be performed if the patient has risk factors for ischemia. Symptomatic palpitations may be treated with
-blockers or calcium channel blockers, even in patients with lower PVC burdens and no cardiomyopathy. Left heart catheterization would not be appropriate.

524
Q

A 20-year-old female comes to your office for routine follow-up after recently finishing neck irradiation treatment for Hodgkin’s lymphoma. Her past medical history is otherwise significant for allergic rhinitis and GERD. She feels generally well after treatment. A physical examination is unremarkable.
Which one of the following should be performed to monitor for complications from radiation?
A) No routine follow-up surveillance
B) Parathyroid hormone levels
C) Swallow studies
D) Carotid artery ultrasonography
E) Neck CT

A

ANSWER: D
Patients who have been treated with neck irradiation for lymphoma require follow-up surveillance with carotid artery ultrasonography every 10 years. There is evidence that asymptomatic carotid artery disease is more common in patients who have been treated with radiation for Hodgkin’s lymphoma compared to the general population.
The risk of thyroid cancer is increased by neck irradiation. Hypothyroidism is also a common complication from neck irradiation, and a TSH level should be measured annually for up to 5 years. For patients who have completed treatment for lymphoma, additional surveillance laboratory studies include a comprehensive metabolic panel, fasting glucose level, and CBC. Patients with a history of chest or axillary irradiation should receive annual mammography screening starting 8–10 years after treatment or at age 40, whichever comes first. Breast MRI may be appropriate for patients who received chest irradiation treatment between ages 10 and 30. Parathyroid hormone levels, swallow studies, and neck CT would not be appropriate for this patient as surveillance to monitor for complications from radiation.

525
Q

You are evaluating a couple for infertility. The semen analysis demonstrates oligozoospermia on two separate samples. The history and physical examination of the male partner are otherwise unremarkable.
Which one of the following would be the most appropriate next step in the evaluation of his oligozoospermia?
A) A CBC with differential and a basic metabolic panel
B) FSH and early morning total testosterone levels
C) Antisperm antibody testing
D) Scrotal ultrasonography
E) Referral for a testicular biopsy

A

ANSWER: B
A semen analysis is the first step in the evaluation of male infertility. In males with oligozoospermia (especially if the sperm count is <10 million/mL), the American Urological Association recommends an endocrine evaluation with an FSH level and early morning total testosterone levels. The results of that testing can dictate next steps. A CBC and a basic metabolic panel have no role in the evaluation of male infertility. Antisperm antibody testing is rarely recommended and should only be considered in consultation with a fertility specialist. Scrotal ultrasonography is not recommended in individuals with a normal physical examination and should only be performed in individuals with palpable varicoceles on physical examination. A testicular biopsy is not usually required to help differentiate between obstructive and nonobstructive azoospermia.

526
Q

You see a previously healthy 8-year-old female for a well child check. She was born at full term and adopted at birth. She has a history of methamphetamine exposure in utero. She is up to date on vaccinations and is doing well academically. She says she has friends at school and her mother confirms that her teachers report that she interacts well with the other students. Her mother notes, however, that the patient has persistent difficulties with anger and irritability. This behavior has been present since preschool, and while her mother thinks there has been some improvement, she is concerned that it has not resolved.
At home, the child has frequent outbursts, often speaks hatefully when upset, refuses to follow instructions, and throws herself on the floor and kicks in frustration at times. Her sisters sometimes worry about upsetting her because they know she will react dramatically, although she has not been physically aggressive. Her mother notes that the patient often blames her sisters or others when she misbehaves.
Which one of the following would be the most appropriate next step?
A) Reassurance that the behavior should continue to improve with age
B) Education on positive reinforcement of desired behaviors
C) Obtaining further history to evaluate for additional mental health conditions
D) A trial of risperidone (Risperdal)
E) Referral for parent management therapy

A

ANSWER: C
This child displays characteristics of oppositional defiant disorder (ODD). The DSM-5 criteria for a diagnosis of ODD include frequently losing one’s temper, being easily annoyed, antagonism toward authority figures, deliberately annoying others, placing blame on others, and being spiteful or vindictive. These symptoms must occur for at least 6 months, cause distress or negative impacts, and not occur exclusively with substance use or in the course of a psychotic, depressive, or bipolar disorder. Treatment of common comorbid mental health conditions can be associated with improvement in ODD, so it is important to evaluate for attention-deficit/hyperactivity disorder, depression, and anxiety disorders, as well as ODD.
Given the persistence of symptoms and maternal concern in this patient, reassurance alone would not be appropriate. Patients with ODD have a high risk of developing other mental health conditions later, and early therapy is recommended. While positive reinforcement is an important parenting strategy for children with ODD, it would not be expected to be effective in isolation. Medication is rarely indicated for ODD, and not as monotherapy. Parent management therapy is an important part of ODD treatment, but therapy should generally include both child therapy and parent training.

527
Q

A 2-year-old male is brought to the urgent care clinic by his parents in February with a low-grade fever that started the night before. His mother awoke early in the morning when she heard his barking cough. He recently started attending preschool and the mother does not know of any sick contacts.
On initial examination the patient is in mild respiratory distress and appears nontoxic. He does not have any rhinorrhea or congestion. He has a temperature of 38.2°C (100.8°F), a respiratory rate of 40/min, a heart rate of 145 beats/min, and an oxygen saturation of 96% on room air. No rashes or petechiae are present.
The most appropriate next step in management would be
A) humidified air
B) albuterol via nebulizer
C) oral dexamethasone
D) a viral culture
E) a chest radiograph

A

ANSWER: C
This patient has a classic presentation of croup, which peaks in the fall and winter months. There may not be any particular history of sick contacts and it does not present with a prodrome, in contrast to respiratory syncytial virus. The diagnosis of croup is purely clinical and does not require laboratory studies, viral cultures, or imaging (SOR C). The treatment of croup includes corticosteroids such as dexamethasone in mild cases (SOR A) and the addition of epinephrine in moderate to severe cases (SOR A). The inhalation of humidified air does not improve outcomes (SOR B) nor does nebulized albuterol.

528
Q

You see a 26-year-old male who was diagnosed with maturity-onset diabetes of the young at age 22. He has a BMI of 24 kg/m2 and his hemoglobin A1c is now 8.5%.
Which one of the following would be most appropriate for this patient?
A) A ketogenic diet
B) Glipizide (Glucotrol)
C) Metformin
D) Short-acting sliding scale insulin with meals
E) Basal insulin at bedtime

A

ANSWER: B
Maturity-onset diabetes of the young (MODY) is a form of diabetes mellitus in nonobese young adults (under age 30) who have preserved pancreatic -cell function. Nearly 80% of patients with MODY are misdiagnosed as having type 1 or type 2 diabetes. These patients exhibit no signs of insulin resistance (metabolic syndrome, acanthosis nigricans, skin tags, androgenic alopecia), are not obese, have positive C-peptide levels, and have a strong family history of diabetes. MODY does not respond to metformin, but because -cell function is preserved, the hyperglycemia does respond to sulfonylureas. While exercise and a balanced diet of appropriate portions and low carbohydrates are also necessary in patients with MODY, a ketogenic diet is not specifically indicated. Insulin is required only during pregnancy.

529
Q

A 40-year-old female presents to your office with jaundice and dark urine. Testing for acute hepatitis A is positive. Her 12-year-old twin boys, who are in good health, have not been vaccinated against hepatitis A or B and are currently asymptomatic.
Which one of the following would be the most appropriate postexposure prophylaxis for these children?
A) No prophylaxis
B) Hepatitis A vaccine only
C) Immune globulin only
D) Hepatitis A vaccine and immune globulin

A

ANSWER: B
All unvaccinated household contacts and sexual contacts should receive postexposure prophylaxis following significant exposure to hepatitis A within the previous 2 weeks. Healthy individuals 12 months to 40 years of age should receive the hepatitis A vaccine as prophylaxis. Infants younger than 12 months of age should receive immune globulin as postexposure prophylaxis. Individuals >40 years of age, as well as immunocompromised patients, should receive both hepatitis A vaccine and immune globulin.

530
Q

The results of a meta-analysis of lung cancer screening using low-dose CT were as follows: the pooled lung cancer–specific mortality rate in the control group was 2.12%, the estimated lung cancer–specific mortality rate in the screened population was 1.72%, and the absolute risk reduction for lung cancer mortality was 0.4% (2.12% minus 1.72%).
What is the number needed to screen to prevent one death due to lung cancer?
A) 5 B) 23 C) 47
D) 200 E) 250

A

ANSWER: E
The absolute risk reduction (ARR) in this meta-analysis was 0.4%. The number needed to screen is the reciprocal of the ARR. The number needed to screen would equal 1 divided by the ARR (1/ARR), or 1/0.004, which equals 250. Based upon this meta-analysis, 250 individuals would need to be screened to prevent one lung cancer death.

531
Q

You see a 50-year-old male in your office with a 2-week history of cough that began as mild and intermittent, but now occurs as paroxysms that frequently cause him to vomit. He feels very tired after these coughing fits. He also reports rhinorrhea throughout the course of his symptoms. He has been feeling overheated but does not report documented fevers. He notes that he does not typically receive routine vaccinations. His vital signs include a temperature of 37.4°C (99.3°F), a pulse rate of 85 beats/min, a respiratory rate of 20/min, and an oxygen saturation of 93% on room air. He has no medication allergies. A test for COVID-19 is negative.
Of the following treatments, which one would be the most beneficial for this patient’s most likely condition?
A) Continued supportive care only
B) Tdap vaccination
C) Azithromycin (Zithromax)
D) Doxycycline
E) Oseltamivir (Tamiflu)

A

ANSWER: C
The duration and characterization of this patient’s cough are most suggestive of pertussis. Of the options listed, azithromycin is the most appropriate for management of pertussis. Azithromycin is most effective for treatment and minimizing spread of the disease within 21 days of symptom onset. Sulfamethoxazole/trimethoprim and other macrolides, such as erythromycin and clarithromycin, are also acceptable options.

Continued supportive care only does not provide the advantages of cure and minimization of community spread that are accomplished by initiating azithromycin. Symptomatic treatment with over-the-counter medication is appropriate but such supportive care does not replace the therapeutic advantages of azithromycin.
This patient’s objection to routine vaccination should be explored as priorities allow. He should be vaccinated against pertussis with Tdap as soon as feasible, but the vaccination would not provide immediate treatment of his current episode of pertussis.
Doxycycline has shown benefit in other bacterial infections but does not provide effective treatment of pertussis. Based upon the duration of symptoms, quality of his cough, and lack of documented fevers, this patient is not likely to have influenza, so oseltamivir would not be appropriate.

532
Q

A 61-year-old male is found to have a 2-cm right adrenal incidentaloma on CT. He has no history of hypertension, electrolyte abnormalities, headaches, flushing, or sweating.
Which one of the following studies should be performed in patients found to have an adrenal incidentaloma?
A) An ACTH stimulation test
B) A dexamethasone suppression test
C) Paired serum aldosterone and plasma renin activity
D) Serum or urine metanephrines
E) A PET scan

A

ANSWER: B
Adrenal incidentalomas usually do not produce overt hormone excess, but mild autonomous cortisol secretion (MACS) is present in up to 30%–50% of cases. This mild secretion of cortisol may predispose patients to metabolic syndrome, osteoporosis, and cardiovascular events. MACS can be ruled out with an overnight 1-mg dexamethasone suppression test. The remainder of the evaluation can be based on CT findings and clinical symptoms.

An ACTH stimulation test is used to evaluate for adrenal insufficiency, which is not caused by an adenoma. If no hypertension or hypokalemia are present, the serum aldosterone and plasma renin activity paired values may not be necessary. Testing for metanephrines is indicated only if pheochromocytoma is suspected. If no clinical symptoms of pheochromocytoma (such as hypertension, sweating, or headaches) are present and the lesions are <10 Hounsfield units on CT, pheochromocytoma and malignancy are very unlikely. A PET scan would not be indicated in this scenario.

533
Q

A 24-month-old female is brought to your office by her mother because the child will not stand on her right leg. Yesterday the patient was playing at the park and her mother did not notice any injury occur. There has been no recent illness or fever. The child was born at full term, has had no medical problems, and is up to date on vaccinations.
The patient’s vital signs are normal. A physical examination reveals a healthy-appearing child in no apparent distress. She grimaces and pulls away with palpation of the right leg over the lower tibia and she will not bear weight. She has full passive range of motion of her hip, knee, and ankle joints bilaterally without apparent pain. Anteroposterior and lateral radiographs of her right tibia and fibula show no abnormalities.
Which one of the following would be the most appropriate next step in management?
A) Reassurance only
B) A CBC and C-reactive protein level
C) Immobilization with a cam boot, and repeat radiographs in 7 days
D) Bone scintigraphy
E) Referral to an orthopedic surgeon

A

ANSWER: C
A nondisplaced spiral fracture of the distal tibial shaft (toddler’s fracture) should be suspected in children from 9 months to 3 years of age who present with pain in the distal third of the tibia after minor or even unnoticed injury. Toddler’s fractures can have subtle radiographic findings and may not be visible on initial radiographs, so repeat radiography to look for healing is appropriate. Standard treatment is immobilization of the affected leg. While the fracture may heal without immobilization, reassurance alone is not recommended given the unclear diagnosis. If repeat radiography is negative and symptoms have resolved, reassurance may then be appropriate. For children with possible septic arthritis, laboratory studies should be considered, but in this case there are no signs of infection. Bone scintigraphy is more sensitive than radiography and can be considered if follow-up radiography is negative and symptoms persist. Toddler’s fractures routinely heal without complication, so referral to an orthopedic surgeon at this time would be premature.

534
Q

A 57-year-old male who uses tobacco presents with cough and dyspnea. His symptoms were previously controlled with an albuterol (Proventil, Ventolin) inhaler once or twice a month. After a 3-week trial of a tiotropium (Spiriva) inhaler his symptoms are better, but he is still having frequent episodes of coughing and dyspnea. He has been smoking 1–2 packs of cigarettes a day since age 13 and is not interested in quitting.
On examination he is afebrile, his vital signs are stable, and his oxygen saturation is 95% on room air. His lung sounds are diminished, and the remainder of the examination is unremarkable. His in-office peak flow is 300 L/min. You suspect he has moderate COPD and recommend pulmonary function tests but he declines.
In addition to continuing tiotropium, which one of the following medications would you recommend adding to his current regimen?
A) An oral antibiotic
B) An oral corticosteroid
C) An inhaled corticosteroid
D) An inhaled long-acting β-agonist
E) A nebulized short-acting β-agonist

A

ANSWER: D
Guidelines from the Global Initiative for Chronic Obstructive Lung Disease (GOLD), the National Initiative for Health and Care Excellence, and the American College of Chest Physicians all recommend that in addition to smoking cessation, COPD should be treated initially with either a long-acting -agonist (LABA) or a long-acting muscarinic antagonist (LAMA). If symptoms persist with either of those inhaled medications then combination therapy should be initiated. An inhaled corticosteroid (ICS) can be added to the LABA/LAMA regimen for triple therapy if symptoms continue. Long-term use of an ICS as monotherapy is not recommended due to a slight increase in the incidence of pneumonia.

Inhaled corticosteriod: Flovent® (fluticasone) Pulmicort® (budesonide)

LAMA: tiotropium, aclidinium, umeclidinium, and glycopyrrolate

LABA: Serevent (salmeterol), Foradil (formoterol), and Striverdi (olodaterol).

535
Q

A 60-year-old male with moderate COPD presents to your office with shortness of breath and a cough with increased sputum volume. After appropriate evaluation, you diagnose an acute COPD exacerbation.
According to Global Initiative for Chronic Obstructive Lung Disease (GOLD) guidelines, which one of the following additional factors would provide the strongest indication for treatment with antibiotics?
A) A decline in oxygen saturation from baseline
B) Diffuse wheezing on lung auscultation
C) A fever
D) Increased sputum purulence
E) Leukocytosis

A

ANSWER: D
COPD exacerbations, when caused by an infectious agent, may be bacterial or viral. The Global Initiative for Chronic Obstructive Lung Disease (GOLD) guidelines support the use of antibiotics in patients with an acute COPD exacerbation with the three cardinal symptoms of increased dyspnea, increased sputum volume, and increased sputum purulence; in patients with increased sputum purulence and one of the other cardinal symptoms; and in patients who require invasive or noninvasive mechanical ventilation. Hypoxemia and fever, although often seen in the setting of COPD exacerbations, do not provide as strong an indication for treatment with antibiotics. Diffuse wheezing is a hallmark examination finding that is present in most COPD exacerbations, regardless of the underlying cause. Leukocytosis is a relatively nonspecific marker for acute inflammation and may be seen with either viral or bacterial etiologies. Serum biomarkers such as C-reactive protein and procalcitonin have yielded controversial and conflicting evidence in guiding antibiotic therapy.

536
Q

A 40-year-old female sees you to discuss a thyroid nodule that was noted on a report from carotid ultrasonography performed by an independent organization. The patient is asymptomatic. Her family history is negative for malignancy.
On physical examination you confirm the presence of a thyroid nodule. The remainder of the examination, including the lymphatic system, is negative. A TSH level is normal.
Which one of the following would be the most appropriate next step?
A) Observation only
B) Thyroid ultrasonography
C) A radionucleotide thyroid uptake scan
D) Fine-needle aspiration only
E) Fine-needle aspiration with molecular testing

A

ANSWER: B
The first step in the evaluation of a palpable thyroid nodule is to obtain a TSH level and perform thyroid ultrasonography (SOR C). If the TSH level is low, a radionucleotide thyroid uptake scan is the appropriate next step to assess for a hyperfunctioning nodule. If the TSH level is normal or high, next steps are determined by the size and characteristics of the thyroid nodule on ultrasonography. Fine-needle aspiration (FNA) may be indicated depending on the size and nodule characteristics. Molecular testing of FNA specimens is useful in order to guide management of thyroid nodules with indeterminate cytology. Before molecular testing is performed, patients should be counseled about the potential benefits and limitations of the test (SOR C). Observation only would not be appropriate in this scenario.

537
Q

You see a 30-year-old male for a routine health maintenance visit. The physical examination is normal, but he mentions that he has an overwhelming urge to keep checking and rechecking doors to make sure they are locked. He senses that something dangerous will happen if he does not do this. These thoughts and behaviors have become very distressing to him, and have started to interfere with his work and home life.
Which one of the following would be most appropriate at this time?
A) Alprazolam (Xanax)
B) Risperidone (Risperdal)
C) Sertraline (Zoloft)
D) Referral for psychodynamic psychotherapy

A

ANSWER: C
Obsessive-compulsive disorder (OCD) affects 3% of the population and is characterized by intrusive thoughts (obsessions) and repetitive behaviors (compulsions). Obsessions are often religious, sexual, or violent in nature. They may include pathological doubting, preoccupation with contamination, concerns with symmetry, and a sense that something unpleasant or dangerous will happen if a particular ritual is not performed precisely. Typical compulsions include counting, checking, repeating, cleaning, and arranging behaviors. For the diagnosis to be made, these symptoms must be severe enough to cause marked distress or to impair functioning.
Most patients develop symptoms prior to age 35, and a large number of them keep these symptoms a secret. There is often a delay of 5–10 years before the illness comes to medical attention. With optimum treatment, 90% have moderate to marked improvement.

Treatment of OCD requires the integration of various approaches to maximize the outcome. Most patients experience substantial improvement using a combination of pharmacotherapy, particularly SSRIs, and cognitive behavioral therapy. Benzodiazepines such as alprazolam are capable of relieving generalized anxiety, but do not affect obsessions or compulsions. Antipsychotics such as risperidone may be added to an SSRI as second-line pharmacotherapy. Traditional psychodynamic psychotherapy is not effective for OCD.

538
Q

A 50-year-old male presents with a 3-month history of persistent burning and numbness in his anterolateral left thigh. He has not had any injury, back pain, radiation of pain, or weakness. He has not noticed any exacerbating or remitting factors. He has type 2 diabetes and a BMI of 37 kg/m2, and his job is sedentary.
An examination reveals normal deep tendon reflexes in the patella and ankle, and the straight leg raising test is negative bilaterally. His strength is preserved throughout his lower extremities. His pinprick sensation is slightly reduced along the anterolateral thigh. Burning discomfort is reproduced with tapping over the lateral aspect of the inguinal ligament.
Which one of the following is the most likely diagnosis?
A) Cauda equina syndrome
B) Diabetic neuropathy
C) Femoral neuropathy
D) Left S1 radiculopathy
E) Meralgia paresthetica

A

ANSWER: E
Meralgia paresthetica is a common cause of anterolateral hip pain and dysesthesia. It is caused by compression of the lateral femoral cutaneous nerve as it courses under the inguinal ligament into the subcutaneous tissue of the thigh. Tapping over this area during the examination can reproduce symptoms. Obesity is a common cause due to compression of the nerve from overlying pannus. Diabetes mellitus is associated with a sevenfold higher incidence over the general population. Cauda equina syndrome presents with saddle anesthesia and generally marked neurologic disability. Diabetic neuropathy is a peripheral neuropathy initially affecting distal structures such as the toes and feet. Femoral neuropathy would affect sensation in the anteromedial thigh and medial lower leg with weakness in the quadriceps muscle group. The anterolateral thigh would represent the L3-L4 dermatome rather than S1, and the normal straight leg raising test and absence of back pain are evidence against an S1 issue.

539
Q

A 46-year-old female presents with a 2-week history of polyuria. She has not had any discomfort with urination or visible change in her urine. Her past medical history includes hysterectomy for fibroids. Further history reveals no concerning environmental exposures other than a 5-pack-year history of smoking in her twenties. A physical examination, including a pelvic examination, is unremarkable. A urine dipstick reveals only 1+ RBCs. A microscopic urinalysis is negative with the exception of 7 RBCs/hpf, and a urine culture is negative.
Which one of the following would you recommend at this point?
A) Repeat urinalysis in 6 months
B) Renal ultrasonography only
C) Urine cytology and renal ultrasonography
D) Urine cytology and CT urography
E) Cystoscopy and CT urography

A

ANSWER: A
This patient presents with microscopic hematuria without a clear cause. Past guidelines from the American Urologic Association recommended cystoscopy and CT urography for all patients over the age of 35 with microscopic hematuria. However, current guidelines recommend risk stratification and emphasize the use of imaging that has less radiation exposure. Because she is female and younger than 50 years of age, this patient is at low risk of urologic malignancy. Her smoking history and RBC count of <10 RBCs/hpf are also considered low risk. The guidelines recommend the option of repeating urinalysis in 6 months before proceeding with imaging or procedures, given her low risk (SOR C). Renal ultrasonography may be used in intermediate-risk patients, while CT urography is reserved for high-risk patients. Urine cytology is not a recommended test in this setting.

540
Q

Three days after a camping trip in New Hampshire a patient develops influenza-like symptoms of a fever, mild myalgias, and malaise followed by an expanding, erythematous, annular rash with central clearing on his thigh. Which one of the following is the most likely diagnosis for the rash?
A) Erythema migrans
B) Erythema multiforme
C) Nummular eczema
D) Pityriasis rosea
E) Tinea corporis

A

ANSWER: A
Annular lesions can be a presentation of several different conditions. This patient’s history of possible tick exposure and current prodromal constitutional symptoms suggest acute Lyme disease. Erythema migrans is the characteristic rash of acute Lyme disease. Erythema multiforme can be spontaneous, related to a viral or Mycoplasma infection, or associated with a medication reaction. Prodromal symptoms are uncommon in limited erythema multiforme and the clinical context of this case suggests a different etiology. Nummular eczema is an intensely pruritic, annular lesion that is not associated with constitutional symptoms. Pityriasis rosea is thought to be viral in etiology and is usually otherwise asymptomatic. Tinea corporis is a fungal infection and is not associated with systemic symptoms.

541
Q

A 13-year-old male presents with a 3-week history of pain in the anterior right knee that is worse when descending stairs and jumping. He is active in sports but has no recent history of injury. On examination you note tenderness and swelling over the tibial tuberosity, but no redness or warmth.
Which one of the following would be most appropriate before initiating treatment?
A) No imaging
B) Plain film radiographs
C) Ultrasonography
D) MRI
E) A bone scan

A

ANSWER: A
This patient has apophysitis at the insertion of the patellar tendon at the tibial tubercle. The clinical diagnosis of Osgood-Schlatter disease is based on history and examination, and imaging is not needed initially. If this patient’s symptoms persist despite treatment, or if there are atypical features or a history of trauma, imaging would be appropriate. Plain film radiography or ultrasonography would be a good first step, with ultrasonography offering the advantage of no radiation exposure. MRI and a bone scan would not be indicated.

542
Q

A patient’s office spirometry results reveal a normal FEV1/FVC ratio and a decreased FVC. Which one of the following is the most likely explanation for these findings?
A) A normal pattern
B) A mixed pattern
C) A restrictive pattern
D) A reversible obstructive pattern
E) An irreversible obstructive pattern

A

ANSWER: C
Forced vital capacity (FVC) is the total amount of air that can be expelled from full lungs. A decreased FVC on spirometry indicates a restrictive pattern. FEV1 is the volume of air (in liters) that is exhaled in the first second during forced exhalation after maximal inspiration. A normal FEV1/FVC ratio and normal FVC would indicate a normal pattern. A decreased FEV1/FVC ratio with a decreased FVC is consistent with a mixed pattern. A reduced FEV1/FVC ratio indicates an obstructive pattern. A bronchodilator is then utilized to determine whether the obstructive pattern is reversible or irreversible.

543
Q

A 56-year-old male was recently diagnosed with hypertension and started on lisinopril (Zestril). At a follow-up visit his blood pressure remains elevated and his serum creatinine level has increased from 0.9 mg/dL to 1.8 mg/dL (N 0.7–1.3). He has no other known medical issues and has a normal BMI.
Which one of the following should be ordered to confirm the most likely cause of his hypertension?
A) Renin and aldosterone levels
B) A TSH level
C) 24-hour urinary free cortisol
D) 24-hour urinary fractionated metanephrines and normetanephrines
E) CT angiography of the abdomen and pelvis

A

ANSWER: E
Resistant hypertension occurs in 5%–10% of adults with hypertension. In this patient, renal artery stenosis is suggested by the increase in creatinine of more than 50% after starting an ACE inhibitor. CT angiography, renal artery duplex ultrasonography, and MR angiography are appropriate diagnostic tests for renal artery stenosis. Other causes of resistant hypertension include hyperaldosteronism (diagnosed with renin and aldosterone levels), thyroid disorders (diagnosed with TSH levels), Cushing syndrome (diagnosed with 24-hour urinary free cortisol), and pheochromocytoma (diagnosed with 24-hour urinary fractionated metanephrines and normetanephrines).

544
Q

A 72-year-old male presents for follow-up of a recent emergency department (ED) visit for chest pain that was diagnosed as costochondritis. He has a history of hypertension and chronic atrial fibrillation. A CBC in the ED was significant for a hemoglobin level of 11.1 g/dL (N 13.5–17.5) and a mean corpuscular volume of 104 μm3 (N 80–100). Follow-up laboratory studies showed the following:
Peripheralsmear normalotherthanmacrocytosis Reticulocyteindex  1.7%(N0.5–2.5) VitaminB12 512pg/mL(N190–950)
Folate  12pg/mL(N2.7–17)
Which one of the following is the most likely cause of this patient’s anemia?
A) Anemia of chronic disease
B) Chronic alcohol use
C) Gastrointestinal bleeding
D) Hemolysis
E) Myelodysplastic syndrome

A

ANSWER: B
Chronic alcohol use is one of the most common causes of macrocytic anemia. This patient’s normal peripheral smear and reticulocyte index <2% in the setting of anemia suggest decreased red blood cell production, which may be related to alcohol use or nutritional deficiency. Vitamin B12 and folate levels in the normal range rule out deficiencies in these vitamins, though patients with low-normal vitamin B12 and folate levels may warrant testing of homocysteine and methylmalonic acid levels. Anemia of chronic disease is typically characterized by a normocytic or microcytic anemia and a low reticulocyte count. Blood loss and hemolytic anemia are typically associated with a reticulocyte index >2%. Myelodysplastic syndrome is not suggested by the peripheral smear and is less common than anemia related to alcohol use disorder.

545
Q

A 20-year-old pregnant female presents to your clinic to discuss the possibility of delivering her baby at her home. A few of her friends have had their babies at home and she asks for your advice about her delivery.
Which one of the following presents the greatest risk with an out-of-hospital birth?
A) Nulliparity
B) Maternal age <25
C) Prior vaginal delivery
D) Single gestation
E) Cephalic presentation

A

ANSWER: A
Of the options listed, any primigravida should be encouraged to consider a hospital delivery rather than an out-of-hospital delivery. Pregnant patients with a previous cesarean delivery, multiple gestation (twins or higher), or fetal malpresentation (breech or other) should also be strongly encouraged to deliver in a hospital setting (SOR B). Similarly, pregnant patients at 41 weeks gestation should be encouraged to pursue a hospital delivery (SOR B). The age of the mother is not relevant in this decision unless there are other related increased risks and comorbidities that need to be considered.
Patients who do plan a community birth should ensure that their maternity and neonatal health professionals are licensed and meet international Confederation of Midwives Global Standards for Midwifery Education, are practicing within an integrated and regulated health system, and have access to timely and safe transport to a hospital if necessary (SOR C). Unassisted childbirth should also be strongly discouraged (SOR C).

546
Q

A healthy 80-year-old female sees you for a routine visit. She is active and follows a healthy diet. She is enthusiastic about vitamin supplements and asks you regularly about their benefits. Her laboratory chemistry profile demonstrates a persistent calcium level elevation at 10.9 mg/dL (N 8.5–10.2).
You review her prescription medications and do not find any associated with hypercalcemia. You also review her calcium and vitamin D intake. Because you know about her tendency to take supplements, you consider other vitamins that may contribute to the hypercalcemia.
Excessive intake of which one of the following would be the most likely explanation for these findings?
A) Vitamin A
B) Vitamin B1
C) Vitamin C
D) Vitamin E
E) Vitamin K

A

ANSWER: A
Vitamin A intoxication can cause hypercalcemia. This includes analogs of vitamin A such as those used to treat acne. The excessive intake of vitamin A is associated with multisystem effects that can include bone resorption and hypercalcemia. Sources of preformed vitamin A include supplements as well as animal sources such as liver, fish liver oil, dairy, and eggs. Vitamin A toxicity should be considered in unexplained cases of parathyroid hormone–independent hypercalcemia. Vitamins B1, C, E, and K are not associated with hypercalcemia.

547
Q

A 41-year-old male who is a construction worker sees you because he experienced palpitations followed by an episode of syncope after drinking four beers at a hotel bar while on vacation. He says that he was hospitalized briefly, and an electric shock was administered in the emergency department after medications were given. On discharge he was told that he had a brief episode of atrial fibrillation but that an echocardiogram was normal. He was advised to follow up promptly with his personal physician. An EKG performed in your office is shown below.
Which one of the following would you recommend for this patient?
A) Aspirin
B) Oral flecainide to be taken if a similar episode occurs in the future
C) Oral metoprolol therapy
D) Warfarin therapy
E) Catheter ablation therapy

A

ANSWER: E
This EKG shows a short P-R interval with a slurred upstroke in the QRS complex in the precordial leads. These findings are classic for Wolff-Parkinson-White (WPW) syndrome. In symptomatic patients with sustained ventricular tachycardias, especially with the more dangerous paroxysmal atrial fibrillation, catheter ablation is the preferred therapy. Up to 95% success has been reported.
Aspirin alone would be ineffective for anticoagulation in atrial fibrillation, and would not be indicated for this young patient with no other risk factors for stroke. “Pill-in-the-pocket” therapy with agents such as flecainide or propafenone can be used for paroxysmal atrial fibrillation in structurally normal hearts. However, they are not the treatment of choice with WPW syndrome.-Blockers, adenosine, digitalis, diltiazem, and verapamil are contraindicated because they can facilitate conduction on the accessory pathway. This can lead to more rapid atrial fibrillation or even degeneration into ventricular fibrillation. A young patient with a structurally normal heart does not require anticoagulation with warfarin.

548
Q

A 65-year-old female develops gram-negative septicemia from a urinary tract infection. Despite the use of fluid resuscitation she remains hypotensive, with a mean arterial pressure of 50 mm Hg.

Which one of the following would be the most appropriate treatment for this patient? (check one)
Vasopressin
Phenylephrine (Neo-Synephrine)
Epinephrine
Norepinephrine (Levophed)
Low-dose dopamine

A

Norepinephrine

In a patient with sepsis, vasopressors are indicated when fluid resuscitation does not restore organ perfusion and blood pressure. Norepinephrine and dopamine are the preferred pressor agents; however, norepinephrine appears to be more effective and has a lower mortality rate. Norepinephrine is the preferred drug for shock due to sepsis. Its relative safety suggests that it be used as an initial vasopressor. It is a potent vasoconstrictor and inotropic stimulant and is useful for shock. As a first-line therapy norepinephrine is associated with fewer adverse events, including arrhythmia, compared to dopamine. Phenylephrine, epinephrine, or vasopressin should not be used as first-line therapy. Vasopressin is employed after high-dose conventional vasopressors have failed. The use of low-dose dopamine is no longer recommended based on a clinical trial showing no benefit in critically ill patients at risk for renal failure. If an agent is needed to increase cardiac output, dobutamine is the agent of choice.

549
Q

A 60-year-old male presents with a 6-week history of worsening bilateral shoulder, upper arm, and neck pain. He has morning stiffness that lasts at least an hour. The review of systems is otherwise negative. There is no localized tenderness or motor weakness on physical examination. His erythrocyte sedimentation rate is 55 mm/hr.

Which one of the following is the best treatment option for this patient at this time? (check one)
Aspirin, 1000 mg 3 times daily
Indomethacin, 25–50 mg 3 times daily
Methotrexate, 7.5 mg once a week
Prednisone, 10–20 mg once daily
Prednisone, 20 mg 3 times daily

A

Prednisone, 10–20 mg once daily

The patient described has polymyalgia rheumatica (PMR). The hallmark of this condition is the rapid and often dramatic response, typically within a few days, to low-dose corticosteroids. In fact, the lack of response to low-dose prednisone in such a case should prompt the physician to consider another diagnosis.

A related condition, giant cell arteritis, is associated with transient or even permanent vision loss, typically unilateral but sometimes bilateral. This condition usually presents with headache and tenderness of the affected artery, most commonly the temporal artery. Prompt recognition and the initiation of high-dose corticosteroids are keys to preventing blindness.

The other options listed are not pertinent to the management of PMR. While prompt response to low-dose corticosteroids confirms the diagnosis, they are usually continued for 1–2 years, with gradual tapering beginning several months after initiation of treatment (SOR C).

550
Q

A 2-year-old male has a 3-day history of a runny nose and cough, and a 2-day history of fever reaching 40.0°C (104.0°F). He woke up with a rash this morning. His appetite is good and his activity level is normal. On examination the child is afebrile with normal vital signs, and has a fine, maculopapular, erythematous rash on the trunk and extremities. The remainder of the examination is normal.

Which one of the following is the most likely cause of this patient’s rash? (check one)
Atopic dermatitis
Erythema infectiosum
Molluscum contagiosum
Pityriasis rosea
Roseola infantum

A

Roseola infantum

This patient has the classic presentation for roseola infantum, which is caused by human herpesvirus 6. The typical history includes a high fever in a child with either mild upper respiratory symptoms or no other symptoms. After the fever subsides, a rash will appear. The rash is self-limited and no treatment is required.

Pityriasis rosea typically presents with a single herald patch that is oval-shaped and scaly with central clearing, followed by a symmetric rash on the trunk in a typical distribution along the Langer lines. The rash may last up to 12 weeks and no treatment is required.

Erythema infectiosum is caused by parvovirus B19 and is also known as fifth disease. The child will typically have mild symptoms then an erythematous facial rash that has a “slapped cheek” appearance. This is sometimes followed by pink patches and macules in a reticular pattern. Once the rash appears the child is no longer contagious.

Molluscum contagiosum is caused by a poxvirus and is characterized by scattered flesh-colored papules with umbilicated centers. Atopic dermatitis typically presents as scaly, erythematous plaques, commonly on the flexor surfaces of the extremities.

551
Q

A 45-year-old male comes to the urgent care center with left foot pain that began at a ballroom dancing competition. He states that he put all of his weight on the lateral portion of his foot while on tiptoe, and felt the pain immediately. He now cannot bear weight on the foot. On examination it appears slightly edematous with a small amount of ecchymosis. A radiograph reveals a proximal fifth metatarsal metadiaphyseal fracture. The patient is eager to return to competitive dancing as soon as possible.

Which one of the following would be the best treatment for this patient? (check one)
Full weight bearing with the use of a compression dressing as needed for pain and swelling
A posterior splint with no weight bearing for 4 weeks
A walking cast for 6 weeks
Surgical referral

A

Surgical referral

This patient has a Jones fracture. The treatment plan for this type of fracture needs to account for the activity level of the patient. It has been shown that active patients have shorter healing times and return to activity sooner with surgical management. A competitive dancer would be best managed with surgery. If the nonsurgical option is chosen the patient is given an initial posterior splint and followed up in 3–5 days, then placed in a short non–weight-bearing cast for 6 weeks, at which time a repeat radiograph is taken. If the radiograph shows healing, the patient can return to gradual weight bearing. If the radiograph does not show proper healing, then the period of non–weight bearing is extended.

552
Q

A 55-year-old male presents with shortness of breath and abdominal pain. He has a history of diabetes mellitus, coronary artery disease, and hepatitis C. He usually is very active and continues to work at a steel plant. He smokes 1 pack of cigarettes each day and drinks alcohol daily. A chest radiograph shows a moderate right-sided pleural effusion. Thoracentesis reveals a pleural fluid protein to serum protein ratio of 0.3, and a pleural LDH to serum LDH ratio of 0.4. The pleural glucose level is 140 mg/dL and his serum glucose level is 150 mg/dL.

Which one of the following is the most likely cause of this pleural effusion? (check one)
Asbestosis
Cirrhosis with ascites
Malignancy
Pneumonia
Viral illness

A

Evaluation of the pleural fluid is important to assist in determining the cause of the effusion. Protein, glucose, LDH, and cell counts should be measured in the pleural fluid. This can help determine if the effusion is exudative or transudative. A pleural protein to serum protein ratio >0.5 or a pleural fluid LDH to serum LDH ratio >0.6 suggests an exudative effusion. Lower ratios suggest a transudative process. With transudates, the pleural fluid pH is typically between 7.40 and 7.55, with fewer than 1000 WBCs, and the glucose level is similar to the serum glucose level. Cirrhosis with ascites is a cause of transudative effusion. Pleural effusions associated with malignancy, pneumonia, viral illness, and asbestosis tend to be exudative.

553
Q

Transudative effusion

A

Cirrhosis

554
Q

Exudative effusion

A

Pleural effusions associated with malignancy, pneumonia, viral illness, and asbestosis tend to be exudative.

A pleural protein to serum protein ratio >0.5 or a pleural fluid LDH to serum LDH ratio >0.6 suggests an exudative effusion.

555
Q

While reviewing laboratory studies for a patient who was recently started on antihypertensive medication, you note new hyperkalemia. Which one of the following medications is most likely to cause this finding? (check one)
Amlodipine (Norvasc)
Chlorthalidone
Hydrochlorothiazide
Metoprolol
Olmesartan (Benicar)

A

Hyperkalemia is a known side effect of ACE inhibitors and angiotensin receptor blockers such as olmesartan. The risk of hyperkalemia is increased with chronic kidney disease, diabetes mellitus, moderately severe to severe heart failure, NSAID use, and older adults. Chlorthalidone and hydrochlorothiazide can cause hypokalemia, while amlodipine and metoprolol have no significant effect on potassium levels.

556
Q

HTN
Hypokalemia

A

Primary hyperaldosteronism

serum aldosterone/ renin ratio

557
Q

Erythema infectiosum

A

parvovirus B19 infection

Slap cheek

557
Q

Checking cortisol levels

A

Cushing disease

557
Q

5-HIAA

A

Neuroendocrine tumor

Flush and diarrhea

558
Q

Erythema migrans

A

Tick exposure

annular rash with or without central clearing

559
Q

Erythema multiforme

A

consists of raised, annular, target-like lesions with central erythema and is usually associated with herpes simplex virus type 1.

560
Q

Radial nerve affects what

A

paresthesia and pain in the posterior forearm and dorsal hand as well as weakness in wrist and finger extensors, which can result in wrist and finger drop.

561
Q

Octeotride for varices

A

IV Only with acute hemorrhage

562
Q

Esophageal varices tx for high risk features

A

Propranolol

563
Q

Which one of the following cardiovascular medications may lead to hyperthyroidism? (check one)
Amiodarone
Digoxin
Flecainide
Metoprolol
Valsartan (Diovan)

A

Amiodarone

564
Q

Unconjugated bilirubin disease

A

Hemolysis
Gilbert

565
Q

A 35-year-old female presents to your office with a feeling of vague fullness in her neck for the last month. She has noticed a gradual onset of fatigue, constipation, and cold intolerance over that time. A few weeks ago the patient took a selfie and was surprised by how puffy her face appeared in the photo.

On examination her thyroid is diffusely enlarged and nontender and feels pebbly on palpation. An HEENT examination, including an eye examination, is otherwise normal.

Which one of the following is the most likely diagnosis? (check one)
Chronic autoimmune (Hashimoto) thyroiditis
Graves disease
Lymphadenitis
Lymphoma
Thyroid cancer

A

This patient’s clinical picture is most consistent with chronic autoimmune thyroiditis, traditionally known as Hashimoto thyroiditis. This diagnosis is suggested by her neck fullness and symptoms of hypothyroidism. Additionally, a nontender goiter that feels like pebbles on examination is classically reported with chronic autoimmune thyroiditis.

Graves disease typically presents with symptoms of hyperthyroidism and, in many patients, orbitopathy (eye bulging). A patient with lymphadenitis typically shows symptoms of a causative infection. Lymphadenitis tends to rapidly enlarge the lymph nodes, which are also typically painful and tender. Lymphoma more commonly presents with fevers, night sweats, unintentional weight loss, itchy skin, and dyspnea.

This patient lacks a discrete thyroid nodule, which makes thyroid cancer less likely. Thyroid nodules are more frequently painful, while the neck fullness in chronic autoimmune thyroiditis is usually painless and nontender.

566
Q

BP check start at what age

A

3 years old

567
Q

Triptans do what?

Avoid in

A

Vasoconstriction

Avoid in coronary artery disease, cerebrovascular disease, or peripheral vascular disease;

568
Q

First line tx in adolescent with htn

A

HCTZ
ACE inhibitor

569
Q

A 70-year-old female presents to your office to discuss osteoporosis that was noted on a recent bone density test. Initial laboratory studies reveal an abnormal TSH level of 0.27 μU/mL (N 0.36–3.74). Additional studies reveal the following:

Repeat TSH …………. 0.04 μU/mL
Free T3 ……………….. 3.4 pg/mL (N 1.7–5.2)
Free T4 ……………….. 1.4 ng/dL (N 0.7–1.6)

A radioactive iodine uptake scan is notable for multiple areas of increased and suppressed uptake.

Which one of the following is the most likely explanation for these findings? (check one)
Exogenous thyroid hormone use
Graves disease
Painless thyroiditis
Recent excess iodine intake
Toxic multinodular goiter

A

Toxic multinodular goiter

This patient has a low serum TSH level in the presence of normal free T4 and total or free T3 levels, which is consistent with subclinical hyperthyroidism. The etiology of overt and subclinical hyperthyroidism should be determined by clinical symptoms, biochemical markers, and, if indicated, diagnostic studies such as a radioactive iodine uptake (RAIU) scan. A scan that shows multiple areas of increased and suppressed uptake is consistent with toxic multinodular goiter. There is no RAIU with exogenous ingestion of thyroid hormone, painless thyroiditis, and recent excess iodine intake. Graves disease causes diffuse RAIU.

It is important to determine the etiology of subclinical hyperthyroidism in order to treat it appropriately. In patients who have TSH levels that are persistently <0.1 μU/mL, the American Thyroid Association has a strong recommendation with moderate-quality evidence for treating patients 65 years of age and older; persons with cardiac risk factors, heart disease, or osteoporosis; postmenopausal women not on estrogens or bisphosphonates; and those with hyperthyroid symptoms.

570
Q

Tumor lysis syndrome

A

Hyperuricemia
secondary hypocalcemia, hyperkalemia, and hyperphosphatemia

571
Q

A healthy 35-year-old female presents to your office to discuss an upcoming trip to Bangladesh. She currently feels well and has no health problems. She is a nurse and will be traveling with a church group to work in a clinic for 1 month. This area is known to have a high prevalence of tuberculosis (TB). She is worried about contracting TB while she is there and asks for recommendations regarding TB screening. She had a negative TB skin test about 1 year ago at work. A TB skin test today is negative.

Assuming she remains asymptomatic, which one of the following would you recommend? (check one)
Prophylactic treatment with isoniazid starting 1 month prior to departure and continuing throughout her trip
Prophylactic treatment with rifampin (Rifadin) starting 1 month prior to departure and continuing throughout her trip
A repeat TB skin test 2 months after she returns
A chest radiograph 2 months after she returns
An interferon-gamma release assay (IGRA) 6 months after she returns

A

A repeat TB skin test 2 months after she returns

Individuals who travel internationally to areas with a high prevalence of tuberculosis (TB) are at risk for
contracting the disease if they have prolonged exposure to individuals with TB, such as working in a health
care setting. The CDC recommends either a TB skin test or an interferon-gamma release assay prior to
leaving the United States. If the test is negative, the individual should repeat the testing 8–10 weeks after
returning. A chest radiograph in asymptomatic individuals or prophylactic treatment at any point is not
recommended. Isoniazid and rifampin are options for treatment of latent TB.

572
Q
A
572
Q

A nulliparous 34-year-old female comes to your office for evaluation of fatigue, hair loss, and anterior neck pain. These symptoms have been gradually worsening for the past few months. Her past medical history is unremarkable. She has gained 5 kg (11 lb) since her last office visit 18 months ago. Examination of the thyroid gland reveals tenderness but no discrete nodules. Her TSH level is 7.5 μU/mL (N 0.4–4.2), her T4 level is low, and her thyroid peroxidase antibodies are elevated.

Which one of the following would be the most appropriate next step? (check one)
Continue monitoring TSH every 6 months
Begin thyroid hormone replacement and repeat the TSH level in 6–8 weeks
Begin thyroid hormone replacement and repeat the TSH level along with a T3 level in 6–8 weeks
Order ultrasonography of the thyroid
Order fine-needle aspiration of the thyroid

A

Begin thyroid hormone replacement and repeat the TSH level in 6–8 weeks

This patient has thyroiditis with biochemical evidence for autoimmune (Hashimoto’s) thyroiditis. The most
appropriate plan of care is to begin thyroid hormone replacement and monitor with a repeat TSH level 6–8
weeks later. It is not necessary to include a T3 level when assessing the levothyroxine dose. There is no
need to routinely order thyroid ultrasonography when there are no palpable nodules on a thyroid
examination. Fine-needle aspiration may be necessary to rule out infectious thyroiditis when a patient
presents with severe thyroid pain and systemic symptoms.

573
Q

A 72-year-old man with NSTE-ACS (NSTEMI) presents to your rural emergency room with onset of pain two hours ago. What is an appropriate immediate intervention?

a.
Intravenous fibrinolysis within 2 hours

b.
Oral enteric-coated aspirin 81 mg

c.
Oral chewable aspirin (162-325 mg)

d.
Intravenous metoprolol

A

c.
Oral chewable aspirin (162-325 mg)

574
Q

A 68-year-old woman has history of hypertension, type 2 diabetes, and MI which prompted placement of three drug-eluting stents last year. She takes pravastatin 40 mg daily but has not been able to tolerate higher-intensity statin therapy, despite multiple attempts. Which one of the following options would be most appropriate to further reduce her LDL cholesterol and ASCVD risk?

a.
Extended-release niacin, 2000 mg daily

b.
Evolocumab, 140 mg SQ every two weeks

c.
Ezetimibe, 10 mg daily

d.
Fenofibrate, 160 mg daily

A

c.
Ezetimibe, 10 mg daily

575
Q

Affect of spirolactone on potassium

A

Increased potassium

576
Q

BP medication causing gout

A

Thiazide diuretic

HCTZ
Chlorthalidone

577
Q

Thiazide diuretics effects
-Calcium
- Uric acid
-Lithium
- Potassium
- Magnesium

A

Hypercalcemia
Increased uric acid
Increased lithium in body

Hypokalemia
Hypomagnesium

578
Q

Dihydropyridine CCB

A

Amlodipine
Felodipine
Nicardipine
Nifedipine

579
Q

Nondihydropyridine CCB

A

Verapamil
Diltiazem

580
Q

A 45-year-old male sees you for a routine visit. His medical history includes hypertension treated with hydrochlorothiazide, amlodipine (Norvasc), and losartan (Cozaar). He also has type 2 diabetes treated with metformin and empagliflozin (Jardiance). Laboratory findings are significant for an LDL-cholesterol level of 167 mg/dL and you prescribe simvastatin (Zocor), 80 mg daily. At a follow-up visit 3 months later he tells you that he stopped taking the simvastatin after a week due to muscle pain and weakness.

Which one of the following medications in this patient’s current regimen most likely contributed to his risk for developing statin-induced myopathy?

(check one)
Amlodipine
Empagliflozin
Hydrochlorothiazide
Losartan
Metformin

A

Amlodipine

Most statins are metabolized in the liver by cytochrome P450 3A4 (CYP3A4) enzymes. In patients on statin therapy, concurrent use of other medications that are also metabolized by this system, including amiodarone, calcium channel blockers such as amlodipine, certain anti-HIV medications, and certain antifungal medications, can increase the risk of complications such as statin-induced myopathy. In this patient, only simvastatin and amlodipine are metabolized by CYP3A4. Losartan is metabolized by cytochrome P450 enzymes other than 3A4 (2C9), and this patient’s other medications are metabolized by different mechanisms (empagliflozin) or not significantly metabolized (hydrochlorothiazide and metformin).

581
Q

Megestrol

A

Appetite stimulant

Dont use in elderly

582
Q

An 80-year-old male sees you for the first time. He is asymptomatic except for some fatigue. His pulse rate is 50 beats/min. An EKG shows a prolonged PR interval.

Which one of the following medications in his current regimen is the most likely explanation for these findings? (check one)
Donepezil (Aricept)
Escitalopram (Lexapro)
Lisinopril (Prinivil, Zestril)
Memantine (Namenda)
Zolpidem (Ambien)

A

Donepezil (Aricept)

The 2015 American Geriatrics Society Beers Criteria for potentially inappropriate medication use in older adults 65 years of age states that donepezil use should be avoided in patients with syncope, due to an increased risk of bradycardia (Moderate Evidence Level; Strong Strength of Recommendation). Donepezil is a cholinesterase inhibitor. Due to their cholinergic effect, these medications have a vagotonic effect on the sinoatrial and atrioventricular nodes. This can cause bradycardia or heart block in patients with or without underlying cardiac conduction abnormalities. Syncope has been reported with these medications.

Memantine is an N-methyl-D-aspartate receptor antagonist and is not associated with bradycardia. Escitalopram, lisinopril, and zolpidem are also not associated with bradycardia.

583
Q

Which one of the following is a significant risk factor for esophageal adenocarcinoma? (check one)
Aspirin therapy
Ibuprofen therapy
Helicobacter pylori infection
Obesity
Crohn’s disease

A

Obesity

Esophageal adenocarcinoma has become the predominant type of esophageal cancer in North America and
Europe, and gastroesophageal reflux and obesity are the main risk factors. Helicobacter pylori infection,
aspirin therapy, NSAID use, and Crohn’s disease are not significant risk factors.

584
Q

A 43-year-old female complains of easy bruising. She is otherwise asymptomatic. A CBC
reveals a platelet count of 23,000/mm3 (N 150,000–450,000). A peripheral smear reveals giant
platelets. A workup is negative for autoimmune causes, including Graves disease, HIV,
Epstein-Barr virus, cytomegalovirus, varicella zoster, hepatitis C, and Helicobacter pylori. She
is on no prescription or over-the-counter medications and denies alcohol or drug use.

Which one of the following would be the most appropriate initial management? (check one)
Platelet transfusion
Corticosteroids
Thrombopoietin-receptor agonists
A bone marrow biopsy
Splenectomy

A

Corticosteroids

Immune (idiopathic) thrombocytopenic purpura is an acquired immune-mediated disorder defined as
isolated thrombocytopenia not found to have another cause. Treatment is usually restricted to severe
thrombocytopenic cases (platelet count <50,000/mm3) unless there is evidence of acute bleeding.
Corticosteroids are considered the first-line therapy (SOR C). Intravenous immunoglobulin and rituximab
have also been used as first-line agents. Second-line therapies include thrombopoietin-receptor agonists
and splenectomy. Further evaluation, including a bone marrow biopsy, to rule out myelodysplastic
syndrome and lymphoproliferative disorders is indicated in patients over the age of 60 (SOR C). Platelet
transfusion is not indicated in the absence of hemorrhage or a need for surgery.

585
Q

Relative Risk

A

Divide

ERR/CER

[a/ (a+c)] / [b/(b+d)]

586
Q

Absolute Risk

A

Subtract

CER- EER

[b/(b+d)] -[a/ (a+c)]

587
Q

NNT

A

1/ ARR

1/ [b/(b+d)] -[a/ (a+c)]

588
Q

NNH

A

1/ ARR

[a/ (a+c)] - [b/(b+d)]

589
Q

Sensitivity

A

a/ (a+c)

590
Q

Specificity

A

b/ (b+d)

591
Q

PPV

A

a/ a+b

592
Q

NPV

A

d/ (c+d)

593
Q

AAA Screening

A

a. Recommends 1 time screening in men aged 65-75 years who have ever smoked for AAA with ultrasound (B)

594
Q

DM Screening

A

35-70 overweight

595
Q

Aspirin for primary prevention

A

a. Age 40-59 w/ 10% CVD risk: decision to initiate low dose aspirin for primary prevention of CVD in adults should be individual one. (C)
b. 60 years and old: Recommends against initiating low dose aspirin for primary prevention

596
Q

Breast cancer screening

A

a. Recommends biennial screening mammography for women aged 50-74 years (B)

597
Q

Screening for depression

A

a. Recommends screening for MDD in age 12 and up (B)

598
Q

Screening for Hep C

A

a. Recommends screening for hepatitis C virus infection in adults aged 18 to 79 years (B)

599
Q

Screening HIV

A

a. Screen for HIV infection age 15-65. Younger or older who are at risk should also be screened. (A)
b. Recommends screening for HIV in all pregnant patients including those who present in labor with unknown HIV status

600
Q

Aspirin for the Prevention of Preeclampsia
- At what week

A

a. Recommends the use of low-dose aspirin (81 mg/day) as preventive medication after 12 weeks of gestation in persons who are at high risk for preeclampsia (B)

601
Q

Screening for lung cancer

A

a. Recommends annual screening for lung cancer with LDCT in adults 50-80 who have a 20 pack year smoking history and currently smoke or have quit within the past 15 years. Screening should be discontinued who person has not smoked for 15 years or develops health problem that substantially limits life expectancy or the ability or willingness to have curative lung surgery.

602
Q

Screening for prostate cancer

A

a. Decision for Age 55-69 men should be individualized to have PSA screened.
b. Men 70 and older, recommend against screening

603
Q

Statin Use for the Primary Prevention of CVD

A

a. Recommends that clinicians prescribe statin for the primary prevention of CVD for adults aged 40 to 75 who have 1 or more CVD risk factors (dyslipidemia, diabetes, HTN, or smoking) and an estimated 10 year risk of a CV event of 10% of greater.

604
Q

Gestational Diabetes Screening

A

a. Recommends screening at 24 weeks or after (B)

605
Q

Leading causes of death

A
  1. Heart disease
  2. Cancer
  3. COVID
  4. Unintentional injury
  5. Stroke
606
Q

Antidepressants for smoking cessation

A

Bupropion
Nortriptyline

(SSRI, TCA ineffective)

607
Q

Most useful smoking cessation in those with COPD

A

Bupropion

608
Q

Caution with smoking cessation with bupropion in who

A

Those with unstable angina and acute coronary syndrome

609
Q

Vaccines to not give in pregnancy

A

MMR
Varicella
HPV
Influenza LAV (different from IV?)

610
Q

Fish Scaling rash

A

Ichthyosis (hereditary)

611
Q

Violet plaques

A

Lichen Planus

612
Q

Ichthyosis

A

Fish scaling (Hereditary)

613
Q

Lichen Planus

A

Violet Plaques

614
Q
A

Lichen Plaques

615
Q

Target lesions

A

Erythema Multiforme

616
Q

Furuncle vs carbuncle

A

Furuncle (boil)
- deep infection of the hair follicle in which purulent material extends through the dermis into the subcutaneous tissue

Carbuncle
-Coalescence of several furuncle into a single inflammatory mass

617
Q

Hot tub follicuitis tx

A

Avoid exposure to contaminated water
Will resolved

Severe cases: ciprofloxacin

618
Q

Closed comedones

A

white head

619
Q

Tx Acne

A

Comedonal acne
- ketatinolytic agent (topic retinoid)

Mild inflammatory acne
- ketatinolytic agent or benzyl peroxide

Moderate inflammatory acne
- retinoid + benzyl peroxide + antibiotic (doxycycline)

Severe: Isotretinoin (highly teratogenic)

620
Q

Coral pink fluorescence on woods lamp
-Tx

A

Erythrasma
Corynebacterium minutissimum

Topical clindamycin
Mod: erythromycin

621
Q

Tinea capitis tx

A

Near oral treatment
Griseofluvin
Terbinafine
Itraconazole
Fluconazole

622
Q

Shockable rhythms

A

VT/ VF

623
Q

Number one causes of trauma deaths overall

A

Firearms

624
Q

Number one causes of trauma deaths in children

A

MVA

625
Q

Fall on warfarin

A

Start 4 factor prothrombin complex concentrate + IV vitamin K

626
Q

Spleenectomy what vaccines after (4)

A

Pneumococcal vaccine
Hib
Meningococcal
Influenza

627
Q

Tx cat bite

A

Augmentin

Any bite: cat human dog

628
Q

Salt water injury organism and tx

A

Vibrio fulnificus

3rd gen cephalosporin
Doxycycline

629
Q

Fish tank exposure organism & tx

A

Mycobacterium marinum

Minocycline
Clarithromycin

630
Q

Lyme disease organism & tx

Two manifestations of disease

A

Borrelia bugdorferi

Doxycycline

Erythema migranes and bells palsy

631
Q

Rocky mountain spotted fever

A

Rickettsia Rickettsii

Flu like
Rash

632
Q

Serotonin syndrome signs & Tx

A

Agitation anxiety
Hyperthermia
Hyperreflexia/ clonus/ tremor
Sweaty

633
Q

What could falsely elevated A1c

A

Iron def anemia
Splenectomy
Renal failure
Aplastic anemia

634
Q

Falsely lower a1c

A

liver disease
hemolytic anemia
blood loss
hemoglobin variants (sickle cell)

635
Q

DM diagnosis

Fasting
2 hr
Random

A

Fasting >= 126
2 hr >= 200
Random >= 200

636
Q

Diabetes medication that affects B12

A

Metformin

637
Q

Risk w/ SGLT2

A

Predispose to DKA
Increase fractures
Risk of foot amputations
Increased UTI and genital infections

638
Q

GLP-1 risks

A

N/V
Pancreatitis

639
Q

-gliptins

elderly?

A

DPP-4 inhibits

Use linagliptin in elderly not renally excreted

640
Q

DM causing fluid overload

A

Thiazolidinediones (Pioglitazone)

641
Q

Safest DM with decreased renal function

A

Glipizide

642
Q

Dm TX in children (3)

A

Metformin
Liraglutide
Insulin

643
Q

Medications that can cause hypothyroidism (2)

A

Lithium
Amiodarone

644
Q

Enlarged thyroid that is non tendered check what

A

Radioiodine uptake study

645
Q

Graves disease

A

Hyperthyroidism

TSH receptor stimulating antibodies

646
Q

Hastimotos

A

Hypothyroidism
Thyroiditis

647
Q

Thyroid radioiodine uptake scan shows diffused uptake

TX

A

Graves disease
- TSH receptor stimulating antibodies

Tx: Methimazole better than PTU
Surgery

648
Q

Palpate firm neck mass on thyroid what test to get

A

TSH

649
Q

Thyroid nodule work up

A
  1. TSH

If low TSH
Get scan (Hot/cold)

Hot –> Surgery
Cold –> FNA

If normal/ high TSH
- FNA

650
Q

Painless non tender goiter

A

Hashimotos
Type of thyroiditis

651
Q

Graves in Pregnancy

A

PTU 1st trimester

Then methimazole

652
Q

Levothyroxine in pregnancy

A

Increase dose by 25-50%

653
Q

Causes of hypercalcemia(6)

A

Primary hyperparathyroidism
Sarcoidosis
Hodgkin’s lymphoma
SCC
Thiazide diuretic
Lithium

654
Q

AM cortisol test

A

Cushings syndrome

655
Q

ACTH stimulation test

A

Addison disease

656
Q

Cushings syndrome test

A

AM cortisol
Dexamethasone suppression

657
Q

Hyperpigmentation
Hyperkalemia
Hypotension

-Name
-Test

A

Addision disease

ACTH suppression test

658
Q

A 38-year-old female presents for follow-up of a second hospitalization in the past 3 months for acute hepatitis, thrombocytopenia, and alcohol withdrawal symptoms treated with benzodiazepines. She says that prior to her hospitalization a week ago she had been drinking a half pint of vodka daily. She reports that her drinking has gradually increased over the past 10 years but increased significantly 6 months ago after she lost her job at a bar and grill. She knows her alcohol consumption is causing damage to her liver and tells you that her aunt died of alcoholic cirrhosis this year. Despite this knowledge she does not want to stop drinking at this time. She has looked into several alcohol cessation programs in the area but does not think that they are a good fit for her.

She currently lives with her boyfriend who also uses alcohol and cocaine regularly. She is not currently speaking to her mother because they “don’t see eye to eye.” She tells you that she has not consumed alcohol since her discharge from the hospital 2 days ago. She reports that her abdominal pain, nausea, and vomiting have resolved and she is feeling well.

The most likely diagnosis is (check one)

alcohol intoxication
alcohol withdrawal
alcohol use disorder in early remission
severe alcohol use disorder

A

severe alcohol use disorder

This patient presents with 6 out of 11 symptoms of alcohol use disorder within a 12-month period,
including a strong desire or urge to use alcohol, recurrent alcohol use that has contributed to the inability
to fulfill work obligations, continued alcohol use despite interpersonal problems with her family, continued
alcohol use despite knowledge that it is causing physical damage to her liver, development of a tolerance
to the effects of alcohol over time, and withdrawal symptoms that require treatment with benzodiazepines.
Mild alcohol use disorder is defined by the presence of 2–3 of the 11 symptoms documented in the DSM-5,
whereas 3–5 symptoms indicate moderate alcohol use disorder and 6 or more symptoms indicate severe
alcohol use disorder. This patient has severe alcohol use disorder that is currently active. Early remission
is defined as the absence of symptoms for at least 3 months but less than 12 months. She is not currently
intoxicated, and she does not currently have withdrawal symptoms related to her alcohol use over a week
ago.

659
Q

A 35-year-old male presents with a 2-week history of lower extremity edema. He is in good health and does not take any medications. You note weight gain, and mild dyspnea with exertion. An examination is unremarkable except for 2+ to 3+ pitting edema of the lower extremities to his knees bilaterally. A CBC and metabolic panel are unremarkable except for a low albumin level. A urinalysis reveals 3+ protein on the dipstick with no microscopic findings.

Which one of the following would be the most appropriate next step? (check one)
Urine microscopy to check for eosinophils
A spot urine protein/creatinine ratio
Renal ultrasonography
Echocardiography
Referral for a renal biopsy

A

A spot urine protein/creatinine ratio

Individuals with nephrotic syndrome often present with edema and fatigue with no evidence of severe liver
disease or heart failure. Hallmarks of this problem include heavy proteinuria, hypoalbuminemia, and
peripheral edema, often with hyperlipidemia as well. While most of these cases are idiopathic, secondary
causes such as diabetes mellitus, systemic lupus erythematosus, and medication reactions should be
considered.
To confirm proteinuria in the nephrotic range a spot urine protein/creatinine ratio is now suggested instead
of a 24-hour collection of urine. Checking urine for eosinophils has been recommended in the past for
evaluation for acute interstitial nephritis but subsequent studies have shown a lack of specificity and
sensitivity. Renal ultrasonography would be indicated if the glomerular filtration rate were reduced.
Echocardiography would be appropriate if heart failure were suspected. While a renal biopsy is often
recommended, it is most useful in patients with suspected underlying systemic lupus erythematosus or
similar disorders when a biopsy can guide management decisions and prognosis.

660
Q

Type 2 DM medication causes ketoacidosis

A

SGLT2 inhbitors

Dapagliflozin

661
Q

Influenza vaccine seprated by what time frame for first dose

A

1 month

662
Q

VTE require anticoagulation for how long

A

3 months

663
Q

A 32-year-old female presents with heat intolerance, excessive weight loss, and anxiety. She gave birth 6 months ago and recently stopped breastfeeding. On examination her thyroid gland is slightly diffusely enlarged and nontender. Laboratory studies reveal a decreased TSH level and elevated free T3 and T4 levels. You suspect that she has postpartum thyroiditis.

Which one of the following tests would be most useful to confirm the diagnosis? (check one)
Radioactive iodine uptake
Thyroid peroxidase antibody levels
Thyroid ultrasonography
Thyrotropin receptor antibody levels

A

Postpartum thyroiditis is defined as a transient or persistent thyroid dysfunction that occurs within 1 year
of childbirth, miscarriage, or medical abortion. Release of preformed thyroid hormone in the bloodstream
initially results in hyperthyroidism. During the hyperthyroid phase, radioactive iodine uptake will be low,
which can help to confirm the diagnosis. Pregnancy and breastfeeding are contraindications to radionuclide
imaging. Thyroid peroxidase antibody levels are elevated with chronic autoimmune thyroiditis
(Hashimoto’s thyroiditis), and patients present with symptoms of hypothyroidism. The Endocrine Society
and American Association of Clinical Endocrinologists do not recommend routine thyroid ultrasonography
in patients with abnormal thyroid function tests if there is no palpable abnormality of the thyroid gland.
Thyrotropin receptor antibody levels are positive in Graves disease.

664
Q

A 72-year-old female presents for a routine health maintenance visit. Which one of the following medications in her current regimen places her at risk for osteoporosis? (check one)
Atorvastatin (Lipitor)
Hydrochlorothiazide
Metformin (Glucophage)
Phenytoin (Dilantin)
Ranitidine (Zantac)

A

Medications reported to be associated with osteoporosis and increased fracture risk include antiepileptic
drugs, long-term heparin, cyclosporine, tacrolimus, aromatase inhibitors, glucocorticoids,
gonadotropin-releasing hormone agonists, thiazolidinediones, excessive doses of levothyroxine, proton
pump inhibitors, SSRIs, parenteral nutrients, medroxyprogesterone contraceptives, methotrexate, and
aluminum antacids. Atorvastatin, hydrochlorothiazide, metformin, and ranitidine are not associated with
osteoporosis.

665
Q

Aortic dissection BP

A

SBP rapidly lower to 100-110s or lower

666
Q

Systolic murmur

A

Pulmary stenosis
Aortic stenosis

Mitral regurg
Tricuspid regurg

667
Q

Tx A fib with COPD

A

Diltazem

(Beta blocker as well)

668
Q

Anticoagulation Calculation

A

CHA2DS2-VASc

Congestive HF
HTN
Age > 75 (+2)
Diabetes
Stroke or TIA (+2)

Vascular disease
Age 65-74
Sex (female)

0= ASA
1= ASA or anticoag
>2= anticoag

669
Q

Delta waves

  • Disease
  • Affected area
  • Tx (3)
A

Wolf parkinson white
Bundle of kent

If SVT: vagal adenosine

If Afib: Ibutilide and procainamidee

Fix: Catheter ablation

670
Q

Torsades de pointes
- due to
- tx

A

Prolonged QT
Tx IV magnesium

671
Q

QT prolonging drugs (7)

A

Macrolides (erythromycin, azithromycin)
Azyl antifungal (ketoconazole)
Fluroquinoles (ciprofloxacin, levofloxacin)
Zofran
Tricycle antidepressants (amitriptyline, nortriptyline)
Amiodarone
Procainamide

672
Q

DOACs
-Are what
- 2 examples

A

Factor Xa
- Apixaban (eliquis)
- Rivaroxaban (xarelto)

673
Q

Tx VT

A

if pulseless: defib

stable: lidocaine & amiodarone

674
Q

AAA vs aortic dissection

A

AAA
- smoking

aortic dissection
- HTNA

675
Q

Increase in prevalence of the disease in a population effect

A

Increase in PPV
Decrease NPV

676
Q

Vaccine at birth

A

Hepatitis B
RSV (Birth- 6 month anytime)

677
Q

Vaccine 2 month

A

RSV (Birth- 6 month anytime)
Hep B (2nd dose)

Rotavirus
DTAP
Hib (haemophilus infuenzae)
Pneumococcal conjugate
Inactived polio

678
Q

4 months vaccines

A

Rotavirus
DTAP
Hib (haemophilus infuenzae)
Pneumococcal conjugate
Inactived polio

679
Q

6 month vaccine

A

3rd dose Hep B
DTAP (3rd dose)
Pneumococcal (3rd dose)
Inactivated polio (3rd dose)
COVID
Flu

680
Q

12 month vaccine

A

Pneumo
MMR
Varicella
Hep A (2 dose series) (6 month apart, have until 18 yrs old)

681
Q

Hep A vaccine

A

2 dose series
Give 6 months apart
Can give from 12 mo to 18 yr

682
Q

Hib

A

4 dose

If given after 15 month no more catch up dose

2 mn, 4 mn, 6 mn, 12 month

683
Q

DTAP

A

Dose 5 not necessary if dose 4 given at 4 years or older

2 mn, 4 mn, 6 mn, 15/18 mn, 4-6 yr

684
Q

Compartment syndrome pressure

A

> 30 mmHg

685
Q

Fall on warfarin

A

4 factor prothrombin complex concentrate _ Vit K 10 mg IV

686
Q

Tetanus for a dirty wound

A

> 5 year since last dose

687
Q

Frost Bite Grade & TX

A

Grade 1
- No blisters
- Ibuprofen

Grade 2
- Blisters
- Ibuprofen

Grade 3
- Hemorrhagic blisters & black eschar
- Ibuprofen + TPA

Grade 4
- Muscle/ bone involvement
- Ibuprofen + TPA

688
Q

Heat exhaustion vs Heat stroke

A

Heat exhaustion
- Temp 104
- No neuro

Heat stroke
- Temp 104
- Neuro

689
Q

Bend and straighten pink and ring finger

A

Ulnar nerve

690
Q

Ulnar nerve

A

Ring and pinky finger

Spread fingers apart
Bend and straighten ring and pinky finger

691
Q

Extension of the wrist

A

Radial nerve

692
Q

Back of hand, thumb, pointer and 1/2 middle finger

A

Radial nerve

693
Q

Tx Acetaminophen toxicity

A

N- acetylcysteline

694
Q

Anticholinergic signs

A

Hyperthermia
Mydriasis
Thirst
Flushing
Delirm

DRY

Hot as Hades
Blind as a bat
Dry as a bone
Red as a beat
Mad as a hatter

Difference from serotonin is dry not sweating

695
Q

Tx serotonin syndrome

A

Cyprohepatadine

696
Q

Hypothyroid causing meds

A

Lithium
Amiodarone

697
Q

TX Bloody diarrhea if needed

A

Campylobacter
- Azithroymcin

Shigella
- TMP-SMX (bactrim) , quinolone

Salmonella
- Levofloxacin

698
Q

GI virus most common in children

  • most common in adults
A

Children: Rota

Adulta: Noro

699
Q

Ulcerative colitis description (4)

Tx

A

Starts in rectum
No SKIPS
Only mucosa
Rectal bleeding

Tx Sulfasalazine, 5- ASA

700
Q

Crohn’s disase (6)

Tx

A

Full wall thickness
Any part of GI tract (terminal ileum)
SKIPS around
Cobblestonning
Ulcerations
Strictures

Tx Budesonide
5-ASA
Steroids

701
Q

Drug induced parkinsonism(3)

A

Metoclopramide (For nausea/vomiting)

Prochlorperazine (antipsychotic, antiemetic)

Sodium Valproate (anticonvulsant)

702
Q

Osteoporosis drugs that are only vertrabral fx (2)

A

Ibandronate
RaloxifeneR

703
Q

Rouleaux formation

A

Stacked coins
AML

704
Q

Newborn fever

A

> 100.4

Group B strep
Ecoli
Listeria

705
Q

When to get renal ultrasound

A

Febrile UTI < 2 y.o
Recurrent UTI

706
Q

Thumb print sign
- disease
- organism
- tx (2)

A

Epiglottitis

Haemophilus

Airway Management
Cephalosporin +/- clindamycin

707
Q

Herpangina

A

Hand foot and mouth
Coxsackie virus

708
Q
A
709
Q
A
710
Q

Coxsackie virus

A

Hand foot and mouth
aka Herpangina

711
Q

Barking Cough
- disease
- tx

A

Croup aka laryngotracheobronchitis

Parainfluenza virus

Nebulized epinephrine and dexamethasone

712
Q

Tx pertussis close contacts

A

Azithromycin

713
Q

Red cheeks

A

Fifth disease
Erythema infectiosum

Parvo B 19

714
Q

Henoch Schonien Purpura
-Who
- Due to
- Signs (3)
- Additional sign (2)
- Tx

A

Children < 10
Follows URI

Purpura, Colicky abdominal pain, arthritis

Can get hematuria and proteinuria

Tx: Prednisone

715
Q

Metronidazole resistant trichomonas

A

Tinidazole

Dont use in pregnancy

716
Q

Gonorrhea tx

A

Ceftriaxone 500 mg IM if < 150 kg (300 lb)

1 g if > 300 lb

717
Q

Tx chlamydria

A

Doxycycline 100 mg BID 7 days

Azithromycin 1 g 1x dose ( use in pregnancy)

718
Q

RBC casts

A

Glomerulonephritis

719
Q

WBC casts

A

Pyelonephritis

720
Q

AKI criteria

A

Increase in serum creatinine of >= 0.3 within 48 hours

Increase in serum creat of >= 1.5 x baseline within 7 days

721
Q

Most common cause of AKI

A

Acute tubular necrosis

722
Q

Medications that cause Acute Interstitial Nephritis(8)

A

PPIs (omeprazole)
Allopurinol
NSAIDS
Ciprofloxacin
Penicilllin
cephalosporin
Sulfonamides
Thiazides/ furosemide

723
Q

Types of Nephritic Glomerular disease ( 4)

A

Post infxn glomerulonephritis
IgA nephropathy
Membranoproliferative glomerulonephritis
Henoch-Schonlein purpura

724
Q

Types of Nephrotic Glomerular disease ( 5)

A

Diabetic nephropathy
Minimal change disease
Focal segmental glomerulosclerosis
Preeclampsia
Primary amyloidosis

725
Q

Stop before contrast (4)

A

Metformin
Diuretics
NSAIDS
ACE/ ARB

726
Q

Nephrotoxic meds (10)

A

NSAIDS
LIthium
Antibiotics
- Aminoglycosides
- Vancomycin
- Amphotericin
- IV accylovir
ACE/ARB
Diuretic
PPI
Methotrexate

727
Q

Medication to not use if creatinine clearance <60

A

Nitrofurantoin

728
Q

FENa

A

100 x (UNa x plasma cr)/ (plasma Na x Ucr)

< 1% prerenal
1-2 intrinsic renal
2-3 ATN

729
Q

Instead of FENa what to use if on diuretic

A

Fractional excretion of urea

730
Q

AKI causing what abnormalitiy (4)

A

Metabolic acidosis

Hyperphosphatemia
Hyperkalemia
Hypercalcemia/ hypocalcemia

731
Q

Tx Hyperkalemia

A

Calcium gluconate
Insulin
Sodium bicarbonate

732
Q

When to discontinue ACE/ARB in renal

A

Creatinine increased by more than 30%

733
Q

Top causes of CKD to ESRD and transplantation

A

1) DM
2 ) HTN

734
Q

CKD definitition

A

GFR < 60 for 3 months

735
Q

CKD stages

A

Stage 1: > 90
Stage 2: 60-89
Stage 3a: 45-59
Stage 3b: 30-44
Stage 4: 15-29
Stage 5: < 15 ESRD

736
Q

Nephrology referral at what stage

A

Stage 4

< 30

737
Q

CKD BP goals

A

140/90

738
Q

CKD with GFR < 30

A

Loop diuretic

739
Q

Labs in CKD
- Ca
- PO4
- PTH

A

Low Ca
HIgh PO4
HIght PTH

740
Q

When to use ACE/ARB in CKD ( 4)

A

1) CKD GFR < 60
2) CKD & Albuminuria > 30

3) HTN

4) NOn-diabetic CKD if albuminuria > 300

741
Q

Blood pressure and Ischemic CVA

A

Tx if BP > 220/120

742
Q

Blood pressure and hemorrhagic CVA

A

Tx if BP 150-220 lower to < 140

743
Q

TIA or minor CVA tx

A

Duel therapy Asparin + clopidogrel 3 months

744
Q

CVA tx

A

Monotherapy clopidogrel or ASA not both

745
Q

When to get endoartectomy

A

TIA or non-disabiling CVA with ipsilateral 70-99% stenosis

746
Q

Tx Focal seizures

A

< 16 Oxcarbazepine
> 60 Lamotrigine

747
Q

Tx absence seizures

A

Ethosuxamide

748
Q

Toxicities of AEDs (4)

A

Suicidality 2x risk

Neurotoxicities
-ataxia
- dizziness
- somnuolence
- fatigue
- HA

Rash

Liver enzymes

749
Q

DRESS syndrome

A

Macrulopapular rash
Fever
Arthralgias
Lymphadenopathy

Causing by:
Carbamazepine, lamotrigine, valproate, allopurinol, sulfonamides

750
Q

Contraceptives and AEDs

A

Decreased effectiveness of hormones

Doesnt affect Depo or IUDs

751
Q

AEDS to not breast feed on

A

Lamotrigine

752
Q

Folic acid on AEDS

A

4 mg per day

753
Q

Prophylaxis cluster headache

A

Verapamil

754
Q

Prophylaxis tension headaches

A

TCA
- Amitriptyline
- Imipramine

755
Q

Tx Chronic daily headache

A

TOpamax
depakote

756
Q

Asymptomatic bacteria
- def
- tx

A

> 100,000 cfu/mL

Cephalexin

757
Q

Start therapy for herpes in pregnancy when

A

36 weeks

758
Q

Test for gestational diabetes when

A

24 weeks

759
Q

Test Group B strep when

A

36-38 weeks

760
Q

Rhogam when

A

28 weeks

761
Q

Preventing preeclampsia in pregnancy

A

starting aspirin after 12 weeks

762
Q

Tx Intrahepatic cholestasis of pregnancy

A

Ursodeoxycholic acid

763
Q

Prevent postpartum hemorrhage

A

Oxytocin at shoulder delivery

764
Q

Medication contraindicated in management of postpartum hemorrhage with HTN

A. Oxytocin (pitocin)
B. Carboprost tromethamine (Hemabate)
C. Misoprostil (cytotec)
D. Methylergonovine

A

D. Methylergonovine

765
Q

Diabetes medication contraindicated in patents with medullary thyroid cancer

A

GLP-1 agonist

Dulaglutide (Trulicity®).
Exenatide (Byetta®).
Exenatide extended-release (Bydureon®).
Liraglutide (Victoza®).
Lixisenatide (Adlyxin®).
Semaglutide injection (Ozempic®).
Semaglutide tablets (Rybelsus®)

766
Q

Wrist pronation and flexion

A

Median nerve

767
Q

Wrist extension

A

Radial nerve

768
Q

Tx hirsutism

A

OCPs
Spironolactone
Finasteride

769
Q

Ovulation in PCOS

A

Letrozole

770
Q

Heavy menses at start of menstruation

A

Von Willebrand disease

771
Q

Most common cause of post menopausal bleeding

A

Endometrial atrophy

772
Q

Stops paps at what age

A

65

773
Q

Hour limit on emergency contraception

A

120 hours

774
Q

3x risk of VTE with what birth control

A

PATCH

775
Q

Tx of mastitis

A

Clindamycin

MRSA: clindamycin/ bactrim

OTher wise: clindamycin, cephalexin, dicloxacilllin

776
Q

Meds that affect contraception ( 4)

A

Rifampin
Anticonvulsants (except valproic acid)
Antifungals
HIV meds

777
Q

Positive Neers or Hawkins test

A

Shoulder impingement

778
Q

Adhesive capsulitis associated with

A

DM
Parkinson’s

779
Q

Most sensitive test for carpal tunnel syndrome

A

Flick sign

aka phalens test

780
Q

Carpal tunnel syndrome associated iwth (5)

A

Thyroid disease
Diabetes
Pregnancy
ALcholism
RA

781
Q

Heel pain with tender medial calcanela tubercle

A

Plantar fasciitis

782
Q

Trigger finger more common in (5)

A

40-60
Women
DM
RA
Repeative hand activities

783
Q

6 month milestone

A

sit w/o support

784
Q

9 month milestone

A

pull to stand

785
Q

12 mn milestone

A

walk independently

786
Q

2 yr milestone

A

stairs, kick ball

787
Q

3 yr milestone

A

stand 1 foot

788
Q

Calf pseudohypertrophy

A

Duchenne muscular dystrophy

789
Q

Non-ossifying fibroma

A

Resolved with skeletal maturity

790
Q

CLub foot surgery

A

9-12 months if needed

791
Q

Developmental dysplasia of the hip risk factors(4)

  • side
A

Left side most common

Girls
First born
Breech
Oligohydramnios

792
Q

Syndactyly

A

webbed fingers or toes

793
Q

Most common bone cancer

A

Osteosarcoma

794
Q

Patella inward
Feet inward

A

Femoral anteversion

80% improved by age 9-10
Surgery at 9-10 if severe

795
Q

Most common congenital foot deformity

A

Metatarsus adductus

796
Q

Patella forward
Feet inward

A

Interanl tibial torsoin

PT orthotics splints are nto effective **

Resolved by 8

797
Q

Osgood sclatters

A

Pain at shin

798
Q

Scoliosis

A

Observe is cobb angle < 30
perform serial radiographs q 6 months

Brace cobb angle 30-40
Surgery cobb angle >40

799
Q

Inability to full extend after elbow injury

A

Fracture

800
Q

Fall on outstretch hand and elbow pain

A

Supracondylar fracture

Long cast 3 week

801
Q

Tx nurse maid elbow

A

Hyperpronation

802
Q

Most commonly fractured bone

A

Clavicle fx

803
Q

Fall on outstretch hand nerve damage

A

Median nerve

804
Q

Physical exam after hip fracture

A

Shortened leg and externally rotated

805
Q

Cant flex DIP

A

Jersey Finger

Avulsion to teh flexor digitorum profundus

Surgery

806
Q

Jammed finger
Cant extend at PIP

A

Cetnral slip injury to PIP joint

Boutonniere deformity

Immobilize in extension

807
Q

Forced flexion of extended DIP. Cant extend tip of finger

A

Mallet finger

Avulsion to extensor digitorum tendon

Splint in extension

808
Q

Crutch palsy affects

A

Radial nerve

809
Q

Ottawa Ankle rules

A

Malleolar pain (posterior margin)

Navicular pain

Proximal 5th metatarsal pain

Cant bear 4 steps

810
Q

Common fracture in 9 months- 3 yr

A

Non displaced spiral fracture

Long leg cast 4 weeks

811
Q

Management hypertrophic cardiomyopathy

A

Beta blockers

812
Q

Commotio Cordis

A

Blunt chest wall trauma causing arrthymia

Need defibritllation

813
Q

Risk with Marfan

A

Ascending aortic dilation
Aortic dissection

814
Q

Mono return to play

A

Asymptomatic
3 weeks after postiive test

815
Q

Evalution of side line concussion

A

SCAT (5)
SPorts concussion assessment tool

816
Q

Concussion

A

No same day return to play

817
Q

Most common injury body region

A

Face/ Head

818
Q

Most common injured knee structure

A

MCL

819
Q

Test for ACL tear

A

Lachmann

820
Q

Test for PCL test

A

posterior drawer

821
Q

Test for MCL

A

Valgus

822
Q

Test for meniscal tear

A

Thessaly maneuver

823
Q

Handlebar neuropathy

A

Entrapment of ulnar nerve at Guyans canal

Paresthesias in palmar 5th and ulnar aspect of 4th digit

824
Q

Throwers neuropathy

A

Throwers and pitchers
Ulnar nerve at cubital tunnel

Paresthesias/pain in teh ulner forearm and dorsal aspect of 5th and ulnar aspect of 4th digit of throwing arm

825
Q

Saddle neuropathy

A

Cyclists

Pudendal nerve

826
Q

1 month development

A

Looks @ face
Responds to voice
Lifts head

827
Q

2 month development

A

vocalizes

828
Q

4 month development

A

Laughs
Follows past midline
No persistent clenching hands

829
Q

6 month development

A

Rolls over
Hand to hand transfer

830
Q

9 month development

A

sit w/o support
dada mama

831
Q

12 month development

A

Stands
Babbles

832
Q

18 month development

A

Walks backward
3-6 words

833
Q

2 yr development

A

speaks 2 word together

834
Q

3 yr development

A

Throws ball
Tricycle

835
Q

4 yr development

A

dresses self
Speech understandable

836
Q

Autism screen at what age

A

18 and 24 month

837
Q

Screen for obesity in children starting at

A

6 year

838
Q

ADHD associated with

A

Oppositional defiant disorder (ODD)

839
Q

Pneumonia most common pathogen

  • overall
  • children
A

Strep pneum

5-18 yr: mycoplasma pneumoniae
- Tx macrolide

840
Q

Reading tuberculus skin test

A

Read induration not erythema

841
Q

Ohio farm with lung nodules

A

Histoplasmosis

842
Q

Southwest organism

A

Cocccidiomycosis

843
Q

Tx Outpatient pneumonia health

Tx Outpatient pneumonia w/ comorbidities

Inpatient

Severe

A

Outpatient
-Amoxicillin
-Doxycycline

Outpatient w/ comorbidities
- Augmentin plus macrolide or doxy or
Fluoroquinolone

Inpatient
- Fluoroquinolone or
Beta lactam + macrolide
(Ampicillin/sulbactam, ceftriaxone plus clarithromycin, azithromycin)

Severe
- Beta lactam plus macrolide
- Beta lactam plus fluroquinolone

844
Q

TB Treatment

A

Isonazid and Rifapentine for 12 weeks or
Isonazid 6 months (9 months if HIV pos)

Aggressive therapy
1) 2 month
Isonazid, rifampin, ethambutol, pyrazinamide
2) 4 months
isonazid and rifampin

845
Q

TB treatment children

A

Rifampin 4 months

Isoniazid plus rifampin 3 months

846
Q

Mild persistent asthma

A

day: > 2 per wk but not daily

night: > 2 per month

847
Q

Moderate persistent asthma

A

Day: daily

Night > 2 per week

848
Q

Intermittent astham

A

Day: <= 2 per week

Night <= 2 per month

849
Q

Severe asthma

A

Day: Daily

Night: freqent night symptoms

850
Q

Asthma treatment

A

Intermittent: SABA prn

Mild persistent: daily or prn ICS with inhaled SABA prn

Moderate to severe: ICS-formoterol therpay (SMART)

851
Q

LABA

A

Used with ICS in moderate to severe persistent asthma

NOT used as monotherapy

Formoterol
Salmeterol

852
Q

Formoterol

A

LABA

use w/ ICS
Do not use as monotherapy

Used in moderate/severe persistent asthma

853
Q

Symbicort

A

SMART
ICS/formoterol

854
Q

Tiotropium
- type of drug
- when used (2)
- Dont use (2)

A

LAMA

Used more in copd

Add on to ICS/LABA if uncontrolled severe persistent asthma

Dont use if urinary retention/ glaucoma

855
Q

How often to monitor asthma

A

At firts 2-6 weeks

COntrolled every 6 months

856
Q

Risk factors of death from asthma

A

Not on ICS

Overuse (>= 1 canister) per MONTH of inhaled SABA

Absense of Wheezing (silent chest)

857
Q

Trelegy Ellipta

A

Triple therapy for asthma/ copd overlap

ICS/ LAMA/ LABA

858
Q

Breztri

A

Triple therapy for asthma/ copd overlap

ICS/ LAMA/ LABA

COPD FDA approved only

859
Q

GOLD 1

A

FEV1 < 80

Mild

860
Q

Gold 2

A

50 <= FEV1 < 80

Moderate

861
Q

Gold 3

A

30<= FEV1 < 50

Severe

862
Q

GOld 4

A

FEV1 < 30

Very severe

863
Q

COPD Group A Tx

A

SABA or LABA

864
Q

COPD Group B treatment

A

LABA or LAMA

865
Q

COPD Group C tx

A

LAMA

866
Q

COPD Group D tx

A

LAMA

LAMA/LABA if CAT >= 20

LABA/ ICS if eos > 300

867
Q

FIrst line therapy for COPD

A

Beta agonist

LABA over SABA

Formeterol > Albuterol

868
Q

Stiolto

A

LAMA/LABA

869
Q

Anoro

A

LAMA/LABA

870
Q

Bilateral hilar infiltrates

A

Sarcoidosis

871
Q

Non caseating granuloma

A

Sarcoidosis

872
Q

Treatment rheumatoid arthritis

A

DEMARDS

  • non biologicals
    methotrexate
    Hydroxychloroquine
    sulfalazine
  • biologicals (anti-TNF biologics)
    Adalimumab
    Infliximab
    Etanercept
873
Q

Arthritis, back pain, and eye inflammation

marker
tx

A

Ankylosing sponylitis

HLA B27

Tx NSALDS

874
Q

Tx psoriatric arthritis

A

DEMARDS
- Etanercept
- Infliximab

875
Q

Psoriatric arthritis affects

A

Dactylitis
Nail changes
Asymmetric polyarthritis

876
Q

Reactve arthritis

A

Can see, pee, climb a tree

Urethritis
Conjunctiivits
Arthritis

877
Q

MCP with ulnar deviation

A

RA

878
Q

OA joints

A

DIP/ PIP

879
Q

OA nodules

A

DIP: Heberdens
PIP: Bouchards

B before H

880
Q

Negative birefringent crystal

A

Gout

881
Q

Medication bad for gout

A

Thiazides

882
Q

SLE ANA

A

Anti smith
Anti ds DNA

883
Q

RA ANA

A

RF

884
Q

Scleroderma ANA

A

Anti-centromete

885
Q

Polymyositis ANA

A

Anti-Jo1

886
Q

SJogren ANA

A

Anti- SSA/Ro
Anti- SSB/ La

887
Q

Wegner ANA

A

C-ANCA
P- ANCA

888
Q

Anti-smith

A

SLE

889
Q

Anti-dsDNA

A

SLE

890
Q

RF

A

RA

891
Q

Anti-centromere

A

Scleorderma

892
Q

Anti-Jo

A

Polymyositis

893
Q

Anti- SSA/Ro

A

Sjogen

894
Q

Anti- Ro

A

Sjogren

895
Q

C-ANCA, P-ANCA

A

Wegners

896
Q

Moderate risk procedures(4)

A

Head and neck
Abdominal
Ortho
Prostate

897
Q

Low risk procedures(4)

A

ENdoscopic
Cataract
Breast
Plastic

898
Q

MET score for climibing stair/ heavy work

A

MET 4-6

899
Q

Medications to continue day of surgery (12)

A

Antibiotics
Steroids
Long acting insulin
Anti-arrhythmic agents
HTN med
Statins
Neuro/psych meds
Anti-parkinson
ANtiseizure
Benzo
Non-biologic Demards (methotrexate)
HIV meds

900
Q

Aspirin pre surgery

A

Stop 5-7 days

901
Q

Warfarin presurgery low risk

A

Stop 5 days prior to surgery
STart day after or when taking PO

902
Q

Warfarin presurgery high risk

A

Stop warfarin 4 days preop
Start LMWH

Stop LMWH 12-18 hr preop

Restart LMWH 6 hr Postop

Restart warfarin when taking PO

903
Q

Ace/arb pre op

A

Day ofStop

904
Q

Clopidogrel pre op

A

stop 7 days

905
Q

Xarelto preop

A

stop 24 hr

906
Q

NSAIDS preop

A

stop `1-3 days

907
Q

DM meds preop

A

Stop all but Long acting insulin

908
Q

You remove a 5 mm basal cell cancer from the arm of a 60 year old and close the wound with 4 sutures. What should you apply to the wound?

A. Silver sulfadiazine
B. Triple antibiotic ointment
C. Povidone-iodine
D. Petrolatum
E. Mupirocin (Bactroban) Cream

A

D. Petrolatum

Antibiotics aggravate wounds

909
Q

Dental prophylaxis (4)

A
  1. Prosthetic heart valve
  2. Previous infective endocarditis
  3. Congenital heart disease
    - unrepaired cyanotic congenital heart disease
    - repaired heart defect w/ prosthetic devices < 6 month
    - repair heart defect with lead at prosthetic site
  4. Transplant with resultant valvulopathy
910
Q

67 y.o male with history of chronic LBP presents with 6 hr sudden severe diffuse abdominal pain. Takes celecoxib chronic use. Methylprednisolone (medrol dose pack) add 3 days ago. Abdomen is diffusely tender with guarding and rebound. Upright CXR shows free air under the diaphragm

A. Acute ischemic bowel
B. Perforated duodenal ulcer
C. Perforated appendicitis
D. Perforated diverticulitis

A

B. Perforated duodenal ulcer

911
Q

60 y.o amel present to the ED with 3 days of worsening RUQ pain. Pain radiates to the right scapula. Nausea and vomiting

He is febrile (T 102.2) P 122, BP 90/45.

He is jaundiced (total bili 7.8)
RUQ is tender

A. Acute cholecystitis
B. Ascending cholangitis
C. Gallstone pancreatitis
D. Gallstone ileus

A

B. Ascending cholangitis

due to occluded common bile duct

Charcot triad
- Fever
- abd pain
- Jaundice

912
Q

Fever, abdominal pain, jaundice

A

Charcots triad

Ascending cholangitis

913
Q

Rovsing sign

A

Appenditicits

LLQ palpation causes RLQ pain

914
Q

Psoas sign

A

Appendicitis

RLQ pain on thigh extension while lying in left lateral decubitus position

915
Q

Obturator sign

A

Appendicitis

RLQ pain with internal rotation of flex right thigh

916
Q

Carnett sign

A

Abdominal crunch (+) if pain persists/worsens –> abdominal wall pathology

917
Q

Appendicitis US age

A

age < 14 or pregnancy

918
Q

Most common cause of large bowel obstruction

A

Neoplasma

919
Q

Most common cause of small bowel obstruction

A

Adhesions

920
Q

Evaluation for small bowel obstrctuion

A

Non-contrast CT

921
Q

Large bowel obstruction signs(4)

A

Elderly
Crampy abd pain
Sudden onset
N/V
Distension

922
Q

Post pranial pain
Atheroscloerotic disease

A

Chronic mesenteric ischemia

923
Q

Pain out of proportion

A

Acute mesenteric arterial thrombosis

Afib not anticoagulated

usually SMA

924
Q

Crampy abdominal pain
Diarrhea
Heme + stool
> 60 y.o

A

Ischemic colitis

Left colon

925
Q

Pediatric abdominal surgery (4)

A

Intussuception
Meckels diverticulum
Hypertrophic pyloric stenosis
Necrotizing enterocollitis

926
Q

Current jelly stool
Abdominal pain

  • name
  • who
  • finding
  • tx
A

Intussusception

Male 5-10 month

Sausage like mass

Tx: barium enema

927
Q

Painless bleeding

  • name
  • description (3)
  • test
A

Meckels diverticulum

2% population
2x more likely male
2 ft from ileocecal valve

Technetium 99 scan

928
Q

Barium enema tx for

A

Intussusception

929
Q

Technetum 99 scan for

A

Meckels diverticulum

930
Q

Projective non bilious vomiting

-name
- who (2)
- finding

A

Hypertrophic pyloric stenosis

Male >female
3 wks

RUQ Olive mass

931
Q

Abdominal distension
Bloody stool
Pneumatosis intestinalis

  • name
  • who
  • finding
  • tx
A

Necrotizing enterocolitis

Increased risk with prematurity

pneumatosis interstinalis: gas in wall of intestines

NGT decompression, Abx

932
Q

Amblopia vs strabismus

A

Amblopia= lazy eye

Strabismus: cross eye

933
Q

A 64-year-old male presents with increasing dyspnea on exertion. He feels well otherwise and has no chronic medical problems. A physical examination is normal. Pulmonary function testing reveals normal spirometry, with no evidence of an obstructive or restrictive pattern. However, his lung carbon monoxide diffusing capacity (DLCO) is low.

Based on these results, which one of the following is the most likely diagnosis? (check one)
Asthma
Bronchiectasis
Chronic pulmonary emboli
COPD
Pulmonary fibrosis

A

Chronic pulmonary emboli

Low diffusing capacity of the lungs for carbon monoxide (DLCO) with normal spirometry indicates a disease process that disrupts gas transfer in the lungs without causing lung restriction or airflow obstruction. Common causes include chronic pulmonary emboli, heart failure, connective tissue disease with pulmonary involvement, and primary pulmonary hypertension. Asthma, bronchiectasis, COPD, and pulmonary fibrosis are associated with abnormalities on spirometry.

934
Q

A 46-year-old male with a prosthetic heart valve requests your advice regarding antibiotic prophylaxis for an upcoming dental procedure. The patient is allergic to penicillin.

Which one of the following would be most appropriate? (check one)
Ciprofloxacin (Cipro)
Clindamycin (Cleocin)
Doxycycline
Rifampin (Rifadin)
Trimethoprim/sulfamethoxazole (Bactrim)

A

Clindamycin (Cleocin)

Patients with an artificial heart valve should be given antibiotic prophylaxis prior to dental procedures to prevent infectious endocarditis. The organisms that most frequently cause infectious endocarditis include Staphylococcus aureus (31%), Streptococcus viridans (17%), coagulase-negative staphylococci (11%), Enterococcus (11%), Streptococcus bovis (7%), and other streptococci (5%). Amoxicillin is the preferred medication for prophylaxis. Clindamycin or azithromycin can be used in patients with a penicillin allergy. If the penicillin allergy is not associated with anaphylaxis, angioedema, or urticaria, then cephalexin would be an appropriate antibiotic choice. Ciprofloxacin, doxycycline, rifampin, and trimethoprim/sulfamethoxazole are not used for bacterial endocarditis prophylaxis.

935
Q

A 25-year-old gravida 1 para 1 presents for insertion of a levonorgestrel-releasing intrauterine
device (Mirena). She is on the last day of her menses, which began 5 days ago. A urine
pregnancy test in the office is negative. You insert the device without complications and she asks
how long she needs to use backup contraception.

Which one of the following would be the most appropriate advice? (check one)
Backup contraception is not necessary
She should use backup contraception for the next 48 hours
She should use backup contraception for the next 7 days
She should use backup contraception for the next 14 days
She should use backup contraception for the next month

A

Backup contraception is not necessary

The Centers for Disease Control and Prevention (CDC) provides specific recommendations for backup
contraception after IUD insertion. According to the CDC guidelines, this patient does not need to use
backup contraception if her IUD is inserted today because it was inserted within 7 days after menstrual
bleeding started. If the levonorgestrel IUD is inserted more than 7 days after menstrual bleeding starts,
the patient needs to abstain from sexual intercourse or use additional contraceptive protection for the next
7 days.

936
Q

A patient is admitted to the hospital with severe acute pancreatitis, based on diagnostic criteria for severity. After appropriate intravenous hydration, which one of the following is associated with shorter hospital stays and lower mortality? (check one)
Parenteral nutrition
Nothing by mouth until the pain has resolved
Clear liquids by mouth after 48 hours
Bolus nasogastric enteral nutrition
Continuous nasogastric enteral nutrition

A

Continuous nasogastric enteral nutrition

The American College of Gastroenterology recommends that patients with severe acute pancreatitis receive enteral nutrition. Enteral feedings help prevent infectious complications, such as infected necrosis, by maintaining the gut mucosal barrier and preventing translocation of bacteria that may seed pancreatic necrosis. Currently, continuous enteral feeding is preferred over bolus feeding. A meta-analysis has shown that continuous nasogastric enteral feeding started in the first 48 hours decreases mortality and the length of hospital stay.

Total parenteral nutrition is not recommended because of infectious and line-related complications. It should be avoided unless the enteral route cannot be used.

937
Q

A 20-year-old female presents to your office with questions about her contraceptive method. She has been using a combined oral contraceptive pill for the past 2 years without any complications. She has learned that several of her friends recently switched to an IUD. She is concerned about the efficacy of her current method and asks about the failure rate.

You tell her that with typical use, the annual failure rate of a combined oral contraceptive pill is (check one)
0.2%
2%
7%
18%
22%

A

7%

The annual failure rate of combined oral contraceptive pills with typical use is 7%. Typical failure rates for other contraceptive methods are 0.2% for the levonorgestrel IUD, 4% for injectable progestin, 13% for male condoms, and 22% for the withdrawal method.

938
Q

A 26-year-old male graduate student presents with concerns about having attention-deficit disorder (ADD). He reports inattentiveness and a lack of concentration for the last 2–3 years, resulting in poorer academic performance than when he was an undergraduate. He says his wife also complains of his lack of focus and attentiveness when attending to household responsibilities. He does not recall having similar symptoms in grade school or high school.

You explain that in order to diagnose adult ADD (check one)
evidence of symptoms of ADD must be present before age 6
evidence of symptoms of ADD must be present before age 12
a therapeutic trial of a short-acting dextroamphetamine/amphetamine combination (Adderall) is indicated
a therapeutic trial of an SNRI such as atomoxetine (Strattera) is indicated

A

evidence of symptoms of ADD must be present before age 12

Diagnosing attention-deficit disorder in adults requires symptoms that interfere with social, academic, or occupational functioning and are present in more than one setting. DSM-5 states that a history of symptoms before age 12 is required for the diagnosis. DSM-IVspecified that symptoms must have been present before age 7.

939
Q

A 7-month-old infant is hospitalized for the third time with lower-lobe bronchopneumonia. Findings include a weight and height below the 10th percentile. A sibling died of sudden infant death syndrome.

Laboratory testing reveals a hemoglobin level of 9.0 g/dL (N 10.5–14.0), a mean corpuscular volume of 85 μm3 (N 72–88), and a serum calcium level of 9.0 mg/dL (N 9.0–10.5). A sweat chloride level is 20 mEq/L (N <60).

Which one of the following is the most likely cause of this infant’s failure to thrive? (check one)
Cystic fibrosis
DiGeorge’s syndrome
Battered child syndrome
Gastroesophageal reflux
β-Thalassemia

A

Gastroesophageal reflux

Gastroesophageal reflux accounts for a significant number of cases of failure to thrive, crib death, and recurrent pneumonia. Features of gastroesophageal reflux include a history of recurrent pneumonia, a low growth curve, a family history of sudden infant death syndrome, and normocytic anemia. A sweat chloride level of 20 mEq/L rules out cystic fibrosis. Normal serum calcium excludes DiGeorge’s syndrome. The battered child generally presents with more than just a single recurring medical problem. β-Thalassemia would be indicated by a microcytic anemia.

940
Q

An 11-year-old female is brought to your office by her parent who is concerned that the child’s spine might be curved. The most appropriate evaluation for scoliosis at this point is (check one)
comparing the length from the pelvic brim to the pelvic floor on the left and the right
scoliometer measurement with the patient bent over to 90°
scoliometer measurement with the patient upright and arms to her side
determination of the Cobb angle with the patient bent over to 90°
determination of the Cobb angle with the patient upright and arms to her side

A

scoliometer measurement with the patient bent over to 90°

The forward bend test, combined with a scoliometer measurement, is the most appropriate initial test when evaluating for scoliosis. A scoliometer should be used with the patient’s spine parallel to the floor (bent over to approximately 90°), with the arms hanging down, palms together, and feet pointing forward. If 5°–7° of trunk rotation is assessed by the scoliometer or by a scoliometer app on a smartphone, radiography can be performed to assess the Cobb angle. This radiography should be performed with the patient upright. A Cobb angle >20° may signify scoliosis, which may benefit from bracing, depending on skeletal maturity. Comparing the length from the pelvic brim to the pelvic floor on the left and the right is not indicated in the evaluation for scoliosis.

The U.S. Preventive Services Task Force changed its recommendation for scoliosis screening from grade D to grade I in 2018. Bracing has been found to reduce by over 50% the chance that mild to moderate curvatures will progress to curvatures of greater than 50°.

941
Q

A 75-year-old male is admitted to the intensive-care unit with sepsis. His past medical history is significant for diabetes mellitus and coronary artery disease.

Which one of the following would be the most appropriate maximum blood glucose goal? (check one)
100 mg/dL
140 mg/dL
180 mg/dL
220 mg/dL

A

180 mg/dL

Sepsis is a severe life-threatening disorder that has a 25%–30% mortality rate. Early aggressive management has been shown to decrease the mortality rate. The initial step in the management of sepsis is respiratory stabilization. Fluid resuscitation should be started and followed by vasopressor therapy if there is an inadequate blood pressure response. Antibiotics should be initiated within 1 hour of presentation. Other interventions in early goal-directed therapy that have been shown to improve mortality rates include blood transfusions, low-dose corticosteroid therapy, and conventional (not intensive) glycemic control with a target glucose level of <180 mg/dL. Intensive management of glucose in critically ill adult patients (a target glucose level of 80–110 mg/dL) has been shown to increase mortality.

942
Q

A 43-year-old female is very distressed about symptoms of postprandial fullness and early satiety. She has seen several physicians over the last 6 months but said she was always told “nothing’s wrong.” You review her most recent workup, including normal blood tests, normal gallbladder testing, and upper endoscopy that shows no abnormalities, including negative testing for Helicobacter pylori . She has tried multiple antacid medications, including omeprazole (Prilosec), lansoprazole (Prevacid), and ranitidine (Zantac), with no success.

Which one of the following medications has the best chance of providing this patient with symptom relief? (check one)
Clonazepam (Klonopin)
Escitalopram (Lexapro)
Metoclopramide (Reglan)
Ondansetron (Zofran)
Sucralfate (Carafate)

A

Metoclopramide (Reglan)

This patient’s history fits the diagnosis of functional dyspepsia. Two subtypes of this disorder have been described. The first, epigastric pain syndrome, is described as intermittent pain and burning in the epigastrium. The second, postprandial distress syndrome, is more typical of the symptoms this patient describes: postprandial fullness and early satiety. Although there is considerable benefit from reassurance and “naming” a patient’s condition, empiric treatment is also warranted. Patients with epigastric pain syndrome are more likely to respond to proton pump inhibitors or H2-blockers. Patients with predominantly postprandial distress symptoms are more likely to improve with a motility agent such as metoclopramide. Sucralfate, antacids, and selective antidepressants have not been shown to be more effective than placebo in functional dyspepsia, whereas tricyclic antidepressants and buspirone have shown some benefit and are reasonable next steps for this patient.

943
Q

A 48-year-old female presents to the emergency department with chest pain. The evaluation, including CT angiography, reveals a pulmonary embolus.

Which one of the following initial findings would be the strongest indication for thrombolytic therapy? (check one)
Elevated troponin
Hypotension
Hypoxia
Bilateral pulmonary emboli
Right ventricular dysfunction on echocardiography

A

Hypotension

Recent guidelines have suggested that hypotension (a systolic blood pressure <90 mm Hg or a diastolic blood pressure <60 mm Hg, for 15 minutes or longer) should be treated with thrombolysis in patients who are not at high risk for bleeding. Patients who have other indicators of cardiopulmonary impairment without signs of hypotension should be given anticoagulation therapy and aggressive supportive care, but should not be treated with thrombolytic therapy. If the patient’s condition continues to deteriorate as evidenced by the development of hypotension or other clinical indicators of cardiopulmonary compromise, thrombolysis may be considered.

944
Q

You are discharging a mother and her infant after delivery. The infant was born at 35 weeks gestation, is exclusively breastfed, and has a weight that is appropriate for her gestational age. No blood transfusions were necessary and all evaluations have been unremarkable.

According to the guidelines of the American Academy of Pediatrics, at what age should you recommend initiation of iron supplementation for this infant? (check one)
Now
1 month
3 months
6 months
9 months

A

1 month

According to the guidelines of the American Academy of Pediatrics, elemental iron supplementation (2 mg/kg per day) should begin at 1 month of age for exclusively breastfed infants born before 37 weeks gestation and should continue until 12 months of age, unless the infant had multiple blood transfusions.

945
Q

HPV vaccination

A

2 shots 9-14
- 6 to months apartment
- if under 6 months then needs 3 shots

> 14 then 3 shots

946
Q

vaccine to not give late

A

rota
- dont give after 14 weeks

947
Q

Smudge Cells

A

CLL

948
Q

Flaccid bullae that leave painful erosions

-diagnosis
- Assoc w/
- tx
- test

A

Pemphigus Vulgaris

Associated with autoimmune disorder

Tx Glucocorticoids

Need skin biopsy

949
Q

Osteopenia score

A

T score: -1.1 to -2.4

950
Q

CKD effect on labs

A

Elevated parathyroid homrone (PTH)

GFR decreases –> increased phosphate due to reduced renal filtering and excretion of phosphate

Hyperphosphatemia interferes with renal calcitriol production –> decreased intestinal calcium absorption

–> Hypocalcemia

Hypocalcemia & hyperphosphatemia stimulate pTH release

TX: Limit phosphate and take VItamin D2 (ergocalciferol)
- also phosphate binders (sevelamer)

951
Q

bidirectional nystagmus

A

Central disease
- Cerebellar stroke
- Multiple sclerosis

952
Q

Peripheral nystagmus

A

Suppressible nystagmus that is unidirectional

Beats away from the lesion

Vestibular neuritis
Meniere disease

953
Q

Who to not give oral contraceptives in

A

> 35 who smoke

Migraine w/ aura

HTN >= 160/100

954
Q

Dont start stimulants until what age

A

6

955
Q

Clubbing of nails associated with

A

Lung malignancies
Idiopathic pulmonary fibrosis
Cystic fibrosis

956
Q

Turners syndrome heart condition

A

bicuspid aortic valve
Aortic dissection

957
Q

Hypocalcemia with elevated proteinuria

A

Hypoalbuminuria
Low albumin

958
Q

Acne tx steps

A

1) benzyl peroxide + topical tretinoin

2) Topical antibiotic (clindamycin/ erythromycin)

3) oral antibiotics

959
Q

Thyroid labs in pregnancy

A

Decreased TSH and increased T4

If hypothyroidism before pregnancy
- will need increased dose of levothyroxine

960
Q

Lead level for chelation therapy

A

> = 45

961
Q

Likihood ratio

A

Sensitivity/ 1- specificity

962
Q

Ototoxicity in htn med

A

Loop diuretic
- Furosemide

963
Q

Opioid withdrawal(4)

A

Dilated pupils
Nausea
Abdominal pain
Tremors

964
Q

child heel pain

A

severs disase

aka calcaneal apopphytitis

965
Q

Dont give with post partum hemorrage and asthma

A

carboprost tromethamine

966
Q

Tx croup

A

single dexamethasone dose

967
Q

A 32 y.o female present with heat intolerance, excessive weight loss, and anxiety. She hd an uncomplicated vaginal delivery 8 weeks ago adn recently stopped breast feeding. On examination, her thyroid gland is mildly enlarged without discrete nodules and is non tender. Lab studies reveal a decreased TSH level and elevated free T3 and T4 level.

Which of the following tests would be most helpful in making a diagnosis of portpartum thyroiditis in this patient?

A. Thyroid peroxidase antibody levels
B. Thyrotropin receptor antibody levels
C. Thyroid ultrasound
D. A radioactive iodine uptake scan.

A

B. Thyrotropin receptor antibody levels

Serum thyroid peroxidase antibodies may be elevated in both Graves and postpartum thyroiditis

Whereas thyrotropin receptor antibodies are elevated only in Graves.

968
Q

DDP-4
- examples
- Side effects

A

DDP-4 Inhibitors: Saxagliptin, sitagliptin, linagliptin, alogliptin cause headache URI, elevated liver enzymes

969
Q

canagliflozin

A

SGLT-2

UTI
Candiasis
Ketoacidosis

970
Q

Thiazolidinediones

A

Thiazolidinediones: Pioglitazone and rosiglitazone, weight gain, salt retention, edema

971
Q

Amantadine use

A

Amantadine can be used for patients under 65 years of age who are only experiencing tremors. Parkinson

972
Q

Sitagliptin

A

DDP

DDP-4 Inhibitors: Saxagliptin, sitagliptin, linagliptin, alogliptin cause headache URI, elevated liver enzymes

973
Q

Pioglitazone

A

Thiazolidinediones: Pioglitazone and rosiglitazone, weight gain, salt retention, edema

974
Q

Monoamine oxidase inhibitors used for

A

such as rasagiline and non-ergot dopamine agonists such as ropinirole are not as effective as carbidopa/levodopa for motor symptoms, but they do not cause the dyskinesias and motor fluctuations seen with levodopa. Monoamine oxidase inhibitors are considered first-line therapy for patients under age 65 with mild motor symptoms.

975
Q

First line treatment of acute mountain sickness

A

dexamethasone

976
Q

Tx hypertrophic cardiomyopathy

A

Beta blocker

CCB next verapamil

977
Q

Sulfonylureas

A

Glipizide, Glimepride

Weight Gain
Hypoglycemia

978
Q

Initial tx sarcoidosis

A

Steroids

979
Q

Liraglutide

A

GLP-4

N/V Diarrhea

980
Q

Ciprofloxacin coverage

A

Gram negative Rods

981
Q

Bactrim coverage

A

Gram positive and Gram Negatiev

Shigella
Salmonella

982
Q

MRSA Coverage(4)

A

Bactrim
Clindamycin
Vancomycin
Doxycycline

983
Q

Pseudomonas coverage(5)

A

Cefepime
Meropenem
Ciprofloxacin
Levofloxacin
Bactrim

984
Q

Doxycycline coverage

A

MRSA
Gram positive
and Ecoli

985
Q

First line COPD

A

Short acting anticholinergic or short acting beta 2 agonist

Short acting anticholinergic= ipratropium

Short acting beta 2 agonist= Albuterol

986
Q

Second line COPD treatment

A

A) SABA + LAMA or B) SAMA + LABA

Albuterol + Tiotropium (spiriva)

Ipratropium + salmeterol

987
Q

Third line COPD treatmnet

A

LAMA + LABA

988
Q

LAMA

A

umeclidinium (Ellipta)
Tiotropium (Spiriva)

989
Q

LABA

A

Serevent (salmeterol)
Foradil (formoterol)
Striverdi (olodaterol)

990
Q

Tiotropium

A

LAMA

991
Q

SABA

A

Albuterol

[Proair, Ventolin]

992
Q

Salmeterol

A

LABA

993
Q

Inhaled corticosteroids

A

Fluticasone
Budesonide

993
Q

Ipratropium

A

SAMA

994
Q

Anti smith

A

SLE

995
Q

Anti-JO

A

Polymyositis

996
Q

Keratoacanthomas

A

smooth dome shaped red papules that expand rapid overly weeks to sun damaged skin

Can have central keratin plug

Seen on elderly

997
Q

Iron treatment if have CKD

A. Ferric citrate
B. Ferrous Fumarate
C. Ferrous gluconate
D. Ferrous sulfte

A

A. Ferric citrate

998
Q

First line treatment of acute mountain sickness

A

dexamethasone

999
Q

Check aldosteronine/renin in

A

Primary hyperALDOSTERONISM

hypokalemia
HTN

1000
Q

Blood thinner for malignancy

A

Enoxaparin (LMWH) Lovenox

1001
Q

Blood thinner for PE in pregnancy

A

Xarelto (rivaroxaban)

1002
Q

V1-V4 Stemi

A

Anterior
LAD

1003
Q

Medication to add to statin if not tolerated

A

Ezetimibe

1004
Q

V5-V6 Stemi

A

Lateral
Circumflex

1005
Q

Antidepressant in children

A

Fluxetine
Escitalopram

1006
Q

Antidepressant in pregnancy (3)

A

Citalopram
Escitalopram
Sertraline

1007
Q

Bacterial meningitis

A

Increase pressure
> 1000 WBC
Protein > 200 (normal 15-45)
Low glucose (< 40)

1008
Q

Viral meningitis

A

Normal pressure
WBC elevated but < 300
Protein elevated but < 200
NORMAL glucose

1009
Q

Nerve for thumb and pointer finger

A

Median nerve

1010
Q

Tx depression in pregnancy first line

A

Sertraline

1011
Q

TTP vs HUS

A

TTP:
FEVER, neurologic signs, thrombocytopenia, anemia, renal dysfunction

HUS:
thrombocytopenia, kidney disease, BLOODY DIARRHEA, increased creatinine (ecoli)

1012
Q

Tx Von willebrand

A

Desmopressin

1013
Q

Itching after shower
-disease
- mutation
- tx

A

Polycythema vera
JAK2 mutation

hydroxyurea

1014
Q

Smudge cells

A

CLL

1015
Q

HIV and CML

A

CD4 < 100

1016
Q
A